Sie sind auf Seite 1von 560

MODULE 1

AUDITING CONCEPTS

PSA BASED QUESTIONS

1. The primary purpose of an independent audit of financial statements is to

A. provide a basis for assessing management‘s performance.


B. comply with laws and regulations.
C. assure management that the financial statements are unbiased and free from
material misstatements.
D. provide users with an unbiased opinion about the fairness of information
presented in the financial statements.

2. Which of the following statements best describes a review service?

A. A review engagement focuses on providing assurance on the assertions


contained in the financial statements of a public company.
B. A review engagement focuses on providing assurance on the internal controls
of a public company.
C. A review engagement focuses on providing limited assurance on financial
statements of a private company.
D. A review engagement focuses on providing advice in a three-party contract.

3. Professional skepticism dictates that when management makes a statement to


the auditor, the auditors should

A. disregard the statement because it ranks low of the evidence quality scale.
B. corroborate the evidence with other supporting documentation whenever
possible.
C. require that the statement be put in writing.
D. believe on the statement in order to maintain the professional client-auditor
relationship.

4. In performing a financial statement audit, which of the following would an auditor


least likely consider?

A. Internal control
B. Compliance with GAAP
C. Quality of managements‘ business decisions
D. Fairness of the financial statement amounts

5. The level of assurance provided by an audit of detecting a material misstatement


is referred to as:
A. Absolute assurance.
B. High assurance.
C. Negative assurance
D. Reasonable assurance.

6. An audit involves ascertaining the degree of correspondence between assertions


and established criteria. In the case of financial statement audit, which of the
following is not a valid criterion?

A. Accounting standards generally accepted in the Philippines.


B. International Accounting Standards.
C. Authoritative financial reporting framework.
D. Philippine Standards on Auditing.

7. Which of the following statements is (are) true regarding the provision of


assurance services?

I. The third party who receives the assurance generally pays for the
assurance received.
II. Assurance services always involve a report by one person to a third party
on which an independent organization provides assurance.
III. Assurance services can be provided either on information or processes.

A. I and II.
B. I and III.
C. III only.
D. I, II, and III.

8. Which of the following is least likely an objective of an assurance engagement?

A. The engagement is intended to prevent the issuance of materially misleading


information.
B. The engagement is intended to enhance the credibility of information about a
subject matter.
C. An assurance engagement is intended for a professional accountant to
express a conclusion that provides the intended users with a level of
assurance about the subject matter.
D. The engagement is intended to provide a level of assurance to be issued by a
professional accountant about the information of being in conformity, in all
material respects, with suitable criteria.

9. The broad range of assurance engagements includes all, but which of the
following?

1.) Engagements intended to provide high or moderate levels of assurance.


2.) Preparation of tax returns, though no conclusion is expressed.
3.) Attest and direct reporting engagements.
4.) Engagements to report externally, but not internally.
5.) Engagements in the private and public sector.
6.) Agreed-upon procedure engagement.

A. 2, 4, 5
B. 2, 4, 6
C. 2, 5, 6
D. 4, 6

10. Which statement does not accurately describe an assurance engagement?

A. The objective of an assurance engagement is for a professional accountant to


evaluate or measure a subject matter that is the responsibility of another party
against identified suitable criteria, and to express a conclusion that provides
the intended user with a level of assurance about that subject matter.
B. Not all engagements performed by professional accountants are assurance
engagements.
C. A particular engagement, to be an assurance engagement, depends upon
whether it exhibits all the following elements: a two-party relationship, a
subject matter, suitable criteria, and a conclusion.
D. An engagement in form of agreed-upon procedures result in the expressions
of factual findings.

11. Which of the following is not an element of an assurance engagement?

A. A subject matter.
B. Suitable criteria.
C. A conclusion.
D. A two-party relationship.

12. A draft of statement, studies or standards should be discussed by the Council en


banc. How many members of AASC are required to approve the draft for
exposure?

A. Majority
B. Ten
C. Eight
D. Twelve

13. Theoretically, it is possible to provide an infinite range of assurance from a very


low level of assurance to an absolute level of assurance in practice, the
professional accountants cannot provide absolute assurance because of the
following, except:

A. The professional accountants employ testing process.


B. The internal control has its inherent limitations.
C. The use of judgments in gathering evidence and drawing conclusions based
on that evidence.
D. The lack of expertise of the professional accountants in doing a systematic
engagement process.

14. Which is not true of the intended user?

A. The intended user is the person or class of persons for whom the professional
accountant prepares the report for a specific use or purpose.
B. The intended user(s) is (are) always limited to the addressee of the
professional accountant‘s report.
C. The responsible party may also be the one of the intended users.
D. The intended user(s) may not be the addressee of the professional
accountant‘s report.

15. Which of the following is least likely a subject matter of an assurance


engagement?

A. Data.
B. System and processes.
C. Compliance and regulations.
D. Degree of loyalty of employees to their employer.

16. The practitioner‘s report on an assurance engagement should always include the
following, except:

A. A description of the engagement and identification of the subject matter.


B. Identification of the standards under which the engagement was conducted.
C. Reference to the work of an expert.
D. Identification of the criteria.

17. Some or all of the following are planning considerations:

I. Criteria to be used.
II. Nature and extent of involvement of the experts.
III. Possible sources of evidence.
IV. Type of conclusion to be issued.
V. Preliminary judgment about materiality and engagement risk.
VI. Content of the management letter.

Which of the following are matters that need to be considered in planning an


assurance engagement?

A. All of them.
B. I, II, III, V
C. II, III, IV, VI
D. I, III, V, VI

18. How many members of the AASC are needed to approve the exposed draft as
Philippine Standards on Auditing?

A. Majority of the regular members.


B. At least eight.
C. At least ten
D. At least twelve.

19. Which of the following is required if the professional accountant uses experts
who are not professional accountants?

A. The ultimate responsibility or the professional service is assumed by the


expert who is not a professional accountant.
B. The professional accountant is discouraged to engage the services of experts
who are not a professional accountant.
C. The professional accountant must take steps to see that such experts are
aware of the ethical requirements of the profession.
D. Experts who are not professional accountants need not be informed of ethical
requirements because they are not members of the Accountancy profession.

20. Which of the following is expected of AASC to do?

A. AASC should normally expose a proposed interpretation of statements.


B. AASC should normally expose its opinion on specific queries from a
practicing CPA.
C. When it is deemed necessary to expose its statement for a comment on
proposed interpretations of statements, the exposure period is
understandably shorter than those of the regular drafts of standards.
D. To make the statements on Philippine Standards on Auditing operative, the
final statement shall be submitted to the Board of Accountancy for approval.

21. Which one of the following is not a key attribute that is essential to perform an
assurance service?

A. Subject matter knowledge


B. Independence
C. Established suitable criteria
D. Accounting skills

22. As it relates to an audit, materiality is

A. not taken into consideration.


B. related party to the sufficiency of procedures performed.
C. based upon audit fees.
D. determined based upon the importance to a user of the financial statement.

23. The suitability of the criteria to which the professional accountant will base his
evaluation of the subject matter partly depends on:

A B C D
Relevance YES YES YES NO
Reliability YES YES YES YES
Understandability YES NO YES NO
Neutrality NO NO YES YES

24. How did the framework of Philippine Standards on Auditing conceptually describe
assurance?

A. It refers to the auditor‘s satisfaction as to the reliability of an assertion being


made by one party for use by another party.
B. The level of assurance that may be provided is determined by the reporting
objective.
C. An assurance is expressed positively in the report.
D. Because of the inherent limitation in an audit, the assurance is of limited one.

25. It provides a threshold or cutoff point rather than being a primary qualitative
characteristic which information must have if it is to be useful.

A. Materiality
B. Reliability
C. Relevance
D. Misstatement
QUIZZERS

1. The difference between what the public expects to get from the audited financial
statements and what the public is actually getting is known as:

A. Credibility gap
B. Audit gap
C. Expectation gap
D. Level of assurance gap

2. Which of the following statements does not properly describe an element of the
theoretical framework of auditing?

A. The data to be audited can be verified.


B. Short-term conflicts may exist between the managers who prepare the data
and the auditors who examine them.
C. Auditors act on behalf of management.
D. An audit benefits the public.

3. An audit of financial statements is conducted in order to determine if the

A. organization is operation efficiently and effectively.


B. auditee is following specific procedures or rules set down by some higher
authority.
C. overall financial statements are stated in accordance with specified criteria.
D. client entity prescribes a good internal control system.

4. Which of the following does not describe a condition that creates a demand for
auditing?

A. Conflict between an information provider and a user can result in biased


information.
B. Information can have substantial economic consequences for a decision
maker.
C. Expertise is often required for information preparation and verification.
D. Users can directly assess the quality of information.

5. Why does a company choose to have an independent auditor report on its


financial statements?

A. Independent auditors will always detect management fraud.


B. The company‘s management preparing the statements may have a vested
interest in reporting certain results.
C. Independent auditors guarantee the accuracy of the financial statements.
D. An independent audit is designed to search for deficiencies in the company‘s
internal control.
6. Which of the following criteria is unique to the independent auditor‘s attest
function?

A. General competence
B. Familiarity with the particular industry in which each client operates
C. Due professional care
D. Independence

7. Which of the following best describes the main reason why the independent
auditor‘s report on an entity‘s financial statements?

A. A management fraud may exist, and it is likely to be detected by independent


auditors.
B. The management that prepares the statements and the persons who use the
statements may have conflicting interests.
C. Misstated account balances may be corrected as a result of an independent
audit work.
D. The management that prepares the statements may have overlooked a
poorly designed system of internal control.

8. Information risk refers to the risk that

A. the client‘s financial statements may be materially false and misleading.


B. the auditor may express an unqualified opinion on financial statements that
are materially misstated.
C. the client entity may not be able to remain in business.
D. errors and frauds would not be detected by the auditor‘s procedures.

9. Which of the following is responsible for an entity‘s financial statements?

A. The entity‘s management


B. The entity‘s audit committee
C. The entity‘s internal auditors
D. The entity‘s board of directors

10. A typical objective of an operational audit for the auditor to

A. determine whether the financial statements fairly present the client entity‘s
operations.
B. evaluate the feasibility of attaining the client entity‘s operational objectives.
C. make recommendations to client for improving its performance.
D. report on the entity‘s relative success in maximizing its profits.

11. Which of the following types of audit is performed in order to determine whether
an entity‘s financial statements are fairly stated, in all material respects in
conformity with the generally accepted accounting principles?
A. Operational audit
B. Financial statement audit
C. Compliance audit
D. Performance audit

12. An independent audit is important to the readers of financial statements


because it

A. provides a measure of management‘s stewardship function.


B. measures and communicates the financial data included in the financial
statements.
C. objectively examines and reports on management‘s financial statements.
D. reports on the accuracy of information in the financial statements.

13. Which of the following types of audit uses laws and regulations as its criteria?

A. Operational audit
B. Financial statements audit
C. Compliance audit
D. Financial audit

14. Which of the following types of auditing is performed most commonly by CPAs
on a contractual basis?

A. Internal auditing
B. Government auditing
C. BSP bank audit
D. External auditing

15. The primary goal of the CPA in performing the attest function is to

A. detect fraud.
B. examine individual transactions that the auditor may certify as to their
validity.
C. determine whether the client‘s assertions as embodied in the financial
statements are fairly stated.
D. assure the consistent application of correct accounting procedures.

16. An independent audit goal aids in the communication of economic data because
the audit

A. confirms the accuracy of management‘s financial representation.


B. lends credibility to financial statements.
C. guarantees that financial data are fairly presented.
D. assures the readers of financial statements that any fraudulent activity has
been detected and its effect has been corrected.

17. Which of the following best describes the attest process?

A. Providing accuracy of the books and records.


B. Gathering sufficient evidence about specific and known assertions.
C. Assisting management in the successful operations of the company.
D. Assembling and filing tax returns and related supplemental information.

18. The assumption underlying an audit of financial statements is that they will be
used by

A. the regulatory agencies to verify information that is relevant to their


supervisory functions.
B. the board of directors as basis of detecting cash dividends.
C. the general public in making investment decisions.
D. different groups for different purposes.

19. Which of the following is an example of an assertion made by the management


in an entity‘s financial statements?

A. The financial statements are prepared in an unbiased manner.


B. The reported inventory balances reflect all related transactions for the period.
C. The reported accounts receivable do not include any uncollectible accounts.
D. The scope of the auditor‘s investigation is not limited in any way by
management.

20. A CPA certificate is an evidence of

A. recognition of independence.
B. basic competence at the time the certificate is granted.
C. culmination of the education process.
D. membership in the PICPA.

21. An audit can have a significant effect on

A. information risk.
B. business risk.
C. the risk-free interest rate.
D. all of these.

22. Which of the following is a cause of information risk?

A. voluminous data.
B. biases and motives of the provider of information.
C. remoteness of the provider of the information.
D. each of these is a cause of information risk.

23. The main way(s) to reduce information risk is to have

A. the user verify the information.


B. the user share the information risk with management.
C. audited financial statements provided.
D. all of these.

24. The predominant type of attestation service performed by CPAs is

A. audit.
B. review.
C. compilation.
D. management consulting.

25. Upon completion of a typical audit, the auditor has

A. total assurance that all material errors and irregularities have been found.
B. high level of assurance that all material errors and irregularities have been
found.
C. a low level of assurance that all material errors and irregularities have been
found.
D. no assurance that all material errors and irregularities have been found.

26. The single feature that most clearly distinguishes auditing, attestation, and
assurance is the

A. type of service being rendered.


B. training required to perform the service.
C. scope of services.
D. CPA‘s approach to the service.

27. Which of the following attributes is more closely associated with assurance
services performed by a CPA firm than with other lines of professional work?

A. Integrity
B. Competence
C. Independence
D. Keeping informed on current professional developments.

28. An investor, while recording the financial statements of Silver Corporation,


learned that the statements are accompanied by an unqualified auditor‘s report.
From this the investor may conclude that:
A. Any disputes over significant accounting issues have been settled to the
auditor‘s satisfaction.
B. The auditor is satisfied that Silver‘s operationally efficient.
C. Informative disclosures in the financial statements but not necessarily in the
notes to financial statements are to be regarded as reasonably adequate.
D. The auditor has ascertained that Silver‘s financial statements have been
prepared accurately.

29. A CPA should maintain objectively and be free of conflicts of interest when
performing:

A. Audits, but not any other professional services


B. All attestation services, but not other professional services
C. All attestation and tax services, but not other professional services
D. All professional services

30. A summary of findings rather than assurance is most likely to be issued n which
engagement?

A. Agreed-upon
B. Compilation
C. Examination
D. Review

31. Which of the following professional has primary responsibility for the
performance of an audit?

A. The managing partner of the firm


B. The senior assigned t the engagement
C. The manager assigned to the engagement
D. The partner in charge of the engagement

32. Which of the following services provides the highest level of assurance to third
parties about a company‘s financial statements?

A. Audit
B. Review
C. Compilation
D. Each of the above provides the same level of assurance

33. The most common type of audit report contains a(n):

A. Adverse opinion
B. Disclaimer of opinion
C. Qualified opinion
D. Unqualified opinion
34. The auditor‘s judgment concerning the overall fairness of presentation of
financial position, results of operation, and changes in cash flow is applied within
the framework of

A. Quality control
B. Generally accepted auditing standards which include concept of materiality
C. The auditor‘s evaluation of the audited company‘s internal control
D. Philippine Financial Reporting Standards

35. In ―auditing‖ accounting data, the auditor is concerned with

A. determining whether recorded information properly reflects the economic


events that occurred during the accounting period.
B. determining if fraud as occurred.
C. determining if taxable income has been calculated correctly.
D. analyzing the financial information to be sure that it complied with
government requirement.

36. In all cases, audit reports must

A. be signed by the individual who performed that audit procedures.


B. certify the accuracy of the quantitative information which was audited.
C. inform readers of the degree of correspondence between the quantifiable
information and the established criteria.
D. communicate the auditor‘s findings to the general public.

37. Which one of the following is an example of management expectations from the
independent auditor?

A. An expert providing a written communication as the product of the


engagement.
B. Individuals who perform day-to-day accounting functions on behalf of the
company.
C. An active participant in management decision-making.
D. An individual source of expertise on financial and other matters.

38. When providing consulting services, the CPA acts primarily as a(n):

A. independent accountant.
B. expert on compliance with industry standards.
C. technology specialist.
D. objective advisor on how to use the information.

39. In performing attestation services, a CPA will normally:


A. improve the quality of information, or its context, for better use of the
decision makers
B. recommend how to use the information.
C. perform market analysis and cost estimates.
D. states a conclusion about a written assertion.

40. Which of the following best describes an operational audit?

A. It requires a constant review of the administrative controls by internal


auditors as they relate to operations of the company.
B. It concentrates on implementing financial and accounting control in a newly
organized company.
C. It attempts of verifying the fair presentation of a company‘s results of
operations.
D. It concentrates on seeking out aspects of operations in which waste would
be reduced by the introduction of controls.

41. Evidence is defined as any information used by the auditor to determine


whether the quantifiable information being audited is stated in accordance with
the established criteria. Evidence takes many different forms, including

A. oral representation (testimony) from the client management.


B. written communication (confirmation) with outsiders.
C. observations made by the auditor.
D. all of these.

42. Because the client company pays the external auditor a professional fee, he

A. is absolutely independent and may conduct an audit.


B. may be sufficiently independent to conduct an audit.
C. is never considered to be independent.
D. must receive approval of the Securities and Exchange Commission before
conducting an audit.

43. A typical objective of an operational audit is to determine whether an entity‘s

A. financial statements fairly present financial position and cash flows.


B. financial statements present fairly the results of operations.
C. financial statements fairly present financial position, results of operations,
and cash flows.
D. specific operating units are functioning efficiently and effectively.

44. Which of the following is more difficult to evaluate objectively?

A. Efficiency and effectiveness of operations.


B. Compliance with applicable government regulations.
C. Presentation of financial statements in accordance with the applicable
financial reporting criteria.
D. All the given criteria are equally difficult to evaluate objectively.

45. An audit which is undertaken in order to determine whether the auditee is


following specific procedures or rules laid down by somme higher authority is
classified as a(n)

A. audit of financial statements.


B. compliance audit.
C. operational audit.
D. production audit.

46. Assurance services involve which of the following?

A. Relevance as well as reliability.


B. Non-financial information as well as traditional financial statements.
C. Electronic databases as well as printed reports.
D. All of these.

47. Which of the following is a difference between attestation and auditing


standards?

A. Attestation standards cover attest engagements other than those involving


GAAP financial statement.
B. Attestation standards do not require independence in mental attitude.
C. Auditing standards apply only to CPAs while attestation standards apply to
all accountants.
D. Attestation standards do not include standards of reporting.

48. Which of the following pertains to the reliability of audit evidence?

A. The independence of the source of evidence.


B. The expertise level of the auditor who obtains the evidence.
C. Whether the audit client uses a manual or computerized accounting system.
D. The quantity of the evidence obtained.

49. The audit committee of the board of directors of a company is responsible for:

A. hiring the auditor.


B. preparing the financial statements.
C. the audit workpapers.
D. independence and obtaining evidence.

50. Which of the following statements is true concerning a compliance audit?


A. Compliance audits are only performed by government auditors.
B. Risks such as inherent risk, control risk, and detection risk are not
appropriate in planning and performance of a compliance audit.
C. Materiality is difficult to measure in a compliance audit.
D. A report on compliance can only include negative assurance.

51. Audits of financial statements include an expression of a conclusion about which


of the following financial statement characteristics?

A. Governance
B. Reliability
C. Relevance
D. Timeliness

52. A review of company‘s financial statements by a CPA firm:

A. is significantly less in scope than an audit and results in a report which


provides positive assurance, although not absolute assurance.
B. is similar in scope to an audit and adds similar credibility to the statements.
C. concludes with the issuance of a report expressing the CPA‘s opinion as to
the fairness of the statements.
D. is designed to provide only limited or moderate assurance.

53. The attest function:

A. is an essential part of every engagement performed by a CPA.


B. require a complete review of all transactions during the period under
examination.
C. requires a review of all transactions during the period under examination.
D. includes the preparation of a written report about the CPA‘s conclusion.

54. Broadly defined, the subject matter of any audit consists of

A. assertions.
B. operating data.
C. financial statements.
D. economic data.

55. The expertise that distinguishes auditors from accountants is in terms of the

A. ability to interpret generally accepted accounting principles.


B. requirement to possess education beyond the Bachelor‘s degree.
C. accumulation and interpretation of evidence.
D. ability to interpret accounting standards.
56. Most of the independent auditor‘s work in formulating an opinion on financial
statements consists of

A. studying and evaluating internal control.


B. obtaining and examining evidential matter.
C. examining cash transactions.
D. comparing recorder accountability with physical existence of property.

57. Attestation risk is limited to a low level in which of the following engagement(s)?

A. Both examination and review


B. Examination but not review
C. Review but not examination
D. Neither examination nor review

58. An engagement in which a CPA firm arranges for a critical review of its practices
by another CPA firm is referred to as a(n):

a. Peer review engagement


b. Quality control engagement
c. Quality assurance engagement
d. Attestation engagement

59. The review of a company‘s financial statements by a CPA firm

A. is substantially less in scope of procedures than an audit.


B. requires detailed analysis of major accounts.
C. has similar scope as an audit and adds similar credibility to the statements.
D. culminates in issuance of a report expressing the CPA‘s opinion as to the
fairness of the statements.

60. The risk associated with a company‘s survival and profitability is referred to as:

A. Business risk
B. Information risk
C. Detection risk
D. Control risk

61. An operational audit differs in many ways from an audit of financial statements.
Which of the following is the best example of these differences?

A. The unusual audit financial statement covers the four basic financial
statements whereas the operational audit is usually limited either the balance
sheet or the income statement.
B. The boundaries of an operation audit are often drawn from an organization
chart and are not limited to a single accounting period.
C. Operation audits do not ordinarily result in preparation of a report.
D. The operational audit deals with operating profit while financial audit
considers both the operating and net profits.

62. The audit of historical financial statements should be conducted by the CPA
professionals in accordance with

A. Philippine Financial Reporting Standards


B. Philippine Standards on Auditing
C. The auditor‘s judgment
D. The audit program

63. Whenever a CPA professional is engaged to perform an audit of financial


statements according to Philippine Standards on Auditing, he is required to
comply with those standards in order to

A. eliminate audit risk.


B. eliminate the professional judgment in resolving audit issues.
C. have a measure of the quality of audit performance.
D. to reduce the audit program to be prepared by the auditor.

64. What is the overall objective of internal auditing?

A. To attest to the efficiency with which resources used.


B. Ascertain that the cost of internal control is justified.
C. To ascertain that financial statements present accurately the financial
position, operating results, and changes in cash and stockholders‘ equity.
D. To help other members of the organization of effectively discharging their
responsibilities.

65. In determining the primary responsibility of external auditor for an audit if a


company‘s financial statements, the auditor owes primarily allegiance to:

A. the management of the audit client because the auditor is hired and paid by
management.
B. the audit committee to the audit client because that committee is responsible
for coordinating and reviewing all audit objectives within the company.
C. stockholders, creditors, and the investing public.
D. the Auditing and Assurance Standards Council, because it determines
auditing standards and auditor‘s responsibility.

66. Which of the following would not represent one of the primary problems that
would lead the users to demand for independent audits of a company‘s financial
statements?

A. Management bias in preparing financial statements.


B. The downsizing of business and financial markets.
C. The complexity of transactions affecting financial statements.
D. The remoteness of the user from the organization and thus the inability of the
user to directly obtain financial information from the company.

67. Assurance services involve all the following except:

A. Improving the quality of information for decision purposes.


B. Improving the quality of the decision model used.
C. Improving the relevance of information.
D. Implementing a system that improves the processing of information.

68. Which of the following is the broadest and most inclusive concept?

A. Audits of financial statements


B. Internal control audit
C. Assurance services
D. Compilation services

69. Which of the following is a correct statement?

A. An audit provides limited assurance by attesting to the fairness of the client‘s


assertions.
B. A review provides positive assurance by attesting the reliability of the client‘s
assertions.
C. Management consulting services provide attestation in all cases.
D. Accounting services do not provide attestation.

70. Unlike consulting services, assurance services:

A. Make recommendation to management


B. Report on how to use information
C. Report on the quality of information
D. Are two-party contracts

71. Financial statement audits:

A. Reduce the cost of capital


B. Report on compliance with laws and regulations
C. Assess management‘s efficiency
D. Overlook information risk

72. A summary of findings rather than assurance is most likely to be included in


a(n):

A. Agreed-upon procedures report


B. Compilation report
C. Examination report
D. Review report

73. The risk associated with a company‘s survival and profitability is referred to as:

A. Business risk
B. Information risk
C. Detection risk
D. Control risk

74. An engagement in which a CPA firm arranges for a critical review of its practices
by another CPA firm is referred to as a(n):

A. Peer review engagement


B. Quality control engagement
C. Quality assurance engagement
D. Attestation engagement

75. Attestation risk is limited to a low level in which of the following engagement(s)?

A. Both examination and reviews


B. Examination but not reviews
C. Review but not examinations
D. Neither examination nor reviews

76. An operational audit differs in many ways from an audit of financial statements.
Which of the following is the best example of these differences?

A. The unusual audit financial statement covers the four basic financial
statements whereas the operational audit is usually limited either the balance
sheet or the income statement.
B. The boundaries of an operation audit are often drawn from an organization
chart and are not limited to a single accounting period.
C. Operation audits do not ordinarily result in preparation of a report.
D. The operational audit deals with pre-tax income.

77. The review of a company‘s financial statements by a CPA firm

A. is substantially less in scope of procedures than an audit.


B. requires detailed analysis of major accounts.
C. is of similar scope as an audit and adds similar credibility to the statements.
D. culminates in issuance of a report expressing the CPA‘s opinion as to the
fairness of the statements.
78. When performing an engagement to review a nonpublic entity‘s financial
statements, an accountant most likely would:

A. Obtain an understanding of the entity‘s internal control.


B. Limit the distribution of the accountant‘s report.
C. Confirm a sample of significant accounts receivable balances.
D. Ask about actions taken a board of directors meetings.

79. Which of the following professionals has primary responsibility for the
performance of an audit?

A. The managing partner of the firm


B. The senior assigned to the engagement
C. The manager assigned to the engagement
D. The partner in charge of the engagement

80. Assurance services may include which of the following?

A. Attesting to financial statements


B. Examination of the economy and efficiency if governmental operations
C. Evaluation of a division‘s performance for management
D. All of the given choices

81. The auditor of financial statements must make very difficult interpretations
regarding authoritative literature. Additionally, the auditor must

A. Proceed beyond PFRS to assess how the economic activity is portrayed in


the financial statements.
B. Force management to make certain decisions regarding their financial
statements.
C. Disregard independence in order to find the underlying truth of the evidence.
D. Establish new criteria by which financial statements may be compared.

82. Which one of the following is not a part of the attest process?

A. Gathering evidence about assertions


B. Providing the accuracy of the books and records
C. Evaluating evidence against objective criteria
D. Communicating the conclusions reached

83. Which one of the following is not a reason why the users of financial statements
desire for an independent assessment of the financial statement presentation?

A. Complexity f transactions affecting the financial statements


B. Lack of criteria on which to base information
C. Remoteness of the user from the organization
D. All of them are potential reasons

84. Independent professional services that are provided on financial or other


information that improve the quality of decision making are known as

A. Internal auditing
B. Financial auditing
C. Assurance services
D. Attestation services

85. An audit which determines whether organizational policies are being followed nd
whether external mandates are being met is known as

A. A financial audit
B. A compliance audit
C. An operational audit
D. None of the above

86. Which of the following statements is correct regarding a review engagement if a


nonpublic entity?

A. An accountant must establish an understanding with the client in an


engagement letter.
B. An accountant must obtain an understanding if the client‘s internal control
when performing a review.
C. A review provides an accountant with a basis for expressing limited
assurance on the financial statements.
D. A review report contains an accountant‘s opinion that the financial
statements, taken as a whole, present fairly the assertions issued by the
management.

87. May a CPA hire for the CPA‘s public accounting firm a non CPA system analyst
who specializes in developing computer systems?

A. Yes, provided the CPA is qualified to perform each of the specialist‘s tasks.
B. Yes, provided the CPA is able to supervise the specialist and evaluate the
specialist‘s end product.
C. No, because non CPA professionals are not permitted to be associated with
CPA firms in public practice.
D. No, because developing computer systems is not recognized as a service
performed by public accountants.

88. Which of the following services may a CPA perform in carrying out a consulting
service for client?

I. Analysis of the client‘s accounting system


II. Review of the client‘s proposed business plan
III. Preparation of information for obtaining financing

A. I and II only
B. I and III only
C. II and III only
D. I, II, and III

89. Which of the following describes how the objective of a review of financial
statements differs from the objective of a compilation engagement?

A. The primary objective of a review engagement is to test the completeness of


the financial statements prepared, but a compilation tests for
reasonableness.
B. The primary objective of a review engagement is to provide positive
assurance that the financial statements are fairly presented, but a
compilation provides no such assurance.
C. In a review engagement, accountants provide limited assurance, but a
compilation expresses no assurance.
D. In a review engagement, accountants provide reasonable or positive
assurance that the financial statements are fairly presented, but a
compilation provides limited assurance.

90. Which of the following factors most likely would cause a CPA to decline a new
audit engagement?

A. The CPA does not understand the entity‘s operations and industry.
B. Management acknowledges that the entity has had recurring operating
losses.
C. The CPA is unable to review he predecessor auditor‘s working papers.
D. Management is unwilling to permit inquiry of its legal counsel.
MODULE 2

AUDITING STANDARDS

1. As guidance measuring the quality of the performance of an auditor, the auditor


should refer to

A. Statements of Financial Accounting Standards Board


B. Philippine Standards on Auditing
C. Interpretations of Rules of Conduct.
D. Statement on Quality Control Standards.

2. Generally Accepted Auditing Standards (GAAS) and Philippine Standards on


Auditing (PSA) should be looked upon by practitioners as

A. ideals to work for, but which are not achievable.


B. maximum standards which denote excellent work
C. minimum standards of performance which must be achieved on each audit
engagement.
D. benchmarks to be used on all audits, reviews, and compilations.

3. The auditor‘s responsibility for the detection of client's noncompliance with laws
and regulation is

A. Greater than for errors or fraud.


B. Less than for errors or fraud.
C. Restricted to information that comes to his attention.
D. The same as it is for errors or fraud.

4. Reasonable assurance means:

A. Gathering of all available corroborating evidence for the auditor to conclude


that there are no material misstatements in the financial statements, taken as
a whole.
B. Gathering of the audit evidence necessary for the auditor to conclude that
the financial statements, taken as a whole, are free from misstatements.
C. Gathering of the audit evidence necessary for the auditor to conclude that
the financial statements are free of material unintentional misstatements.
D. Gathering of the audit evidence necessary for the auditor to conclude that
there are no material misstatements in the financial statements, taken as a
whole.

5. Required auditor communication to the Audit Committee concerning


noncompliance with laws and regulations that were detected includes:
A. All material items.
B. All those which are not adequately addressed by management.
C. All those that constitute management fraud.
D. Any of such acts.

6. An auditor who accepts an audit engagement but does not possess the industry
expertise of the business entity should

A. Engage financial experts familiar with the nature of the business entity.
B. Obtain a knowledge of matters that relates to the nature of the entity's
business.
C. Refer a substantial portion of the audit to another CPA who will act as the
principal auditor
D. First inform the Client management that on unqualified opinion cannot be
issued.

7. Auditors focus on

A. areas where the risk of material errors and irregularities is least


B. areas where the risk of material errors and irregularities is greatest
C. all areas equally
D. a random selection of all areas.

8. The decision as to how much evidence to be accumulated for a given set of


circumstance is.

A. provided by following the generally accepted accounting principles.


B. one requiring professional judgment
C. determined by statistical analysis
D. provided in the Philippine Standards on Auditing.

9. Which of the following statements best describes the primary purpose of


Philippine Standards of Auditing?

A. They are guides intended to set forth auditing procedures that are applicable
to a variety of situations.
B. They are procedural outlines which are intended to narrow down the areas of
inconsistency and divergence of auditor‘s opinion.
C. They are authoritative statements, enforced through the Code of
Professional Conduct, that are intended to limit the degree of auditor's
judgment.
D. They are interpretations which are intended clarify the meaning of "generally
accepted auditing standards.‖
10. An auditor need not abide by, a Philippines Standard on Auditing if the auditor
believes that

A. The amount is insignificant.


B. the requirement of the PSA is impractical to perform
C. the requirement of the PSA is impossible to perform
D. any of the given three choices is correct.

11. Auditing standards are

A. statutory in nature
B. rules imposed by the securities exchange commission
C. rules imposed by the picpa
D. general guidelines to help the auditors.

12. Though PSAs do not provide ―hard and fast rules‖ they provide subjective
guidance which allow the auditors to:

A. Tailor their audit to procedures requested by management


B. Only apply those standards that are important to the audit
C. Accurately interpret the profession‘s Code of Professional Conduct.
D. Use adequate professional judgment when applying the standards.

13. Every independent audit engagement involves both auditing standards and
auditing procedures. The relationship between the two may be illustrated by how
they apply from management to engagement. The best representation of this
application is that, from one audit engagement to the next.

A. Both auditing standards and auditing procedures are applied uniformly


B. Auditing standards are applied uniformly, but auditing procedures may vary.
C. Auditing standards may vary but auditing procedures are applied uniformly.
D. Auditing standards are applied uniformly, but auditing procedures are
optional.

14. Philippine Financial Reporting Standards (PFRS] are distinguished from


generally accepted auditing standards (GAAS) is that:

A. PFRS are the principles for presentation of financial statements and


underlying transactions, while GAAS are the standards that the auditors
should follow when conducting an audit.
B. PFRS are the principles auditors follow when conducting an audit, while
GAAS are the standards for presentation of financial statements and
underlying transactions.
C. PFRS are promulgated by the SEC, while GAAS are promulgated by the
FRSC
D. When PFRS are violated sufficiently strong GAAS may make up for most
PFRS deficiencies

15. The Philippine Standards on Auditing issued by the Auditing and Assurance
Standards Council (AASC).

A. are interpretations of generally accepted auditing standards


B. are the equivalent of laws for audit practitioners.
C. must be followed in all situations.
D. are optional guidelines which an auditor may choose not to follow when
conducting an audit.

16. Competence as a certified public accountants includes all of the following except

A. Having the technical qualifications to perform an engagement.


B. Possessing the ability to supervise and evaluate the quality of staff work.
C. Warranting the infallibility of the work performed.
D. Consulting others if additional technical information is needed.

17. In any case in which the incoming accountant is not qualified to perform the work,
a professional obligation exists to

A. Acquire the required level of knowledge and skills.


B. Recommend someone else who is qualified to perform the work.
C. Decline the engagement.
D. Any of the given choices.

18. Ultimately, the decision about whether or not an auditor is independent must be
made by the

A. auditor
B. audit committee
C. client
D. public

19. To be independent, the auditor:

A. cannot place any reliance on the client's verbal and written assertion
B. is responsible only to third-party users of the financial statements.
C. cannot perform any other professional services for an audit client
D. must be impartial when dealing with the client
20. What is the meaning of the generally accepted auditing standard that requires
that the auditor the auditor be independent?

A. the auditor must be without bias with respect to the client entity.
B. the auditor must adopt a critical attitude during the audit.
C. the auditor‘s sole obligation is to third parties.
D. the auditor may have a direct ownership in the client‘s business if it is not
material.

21. A CPA, while performing on audit, strives to achieve independence in


appearance in order to

A. Reduce risk and liability.


B. Comply with the generally accepted standards of field work.
C. Become independent in fact.
D. Maintain public confidence in the profession.

22. Which of the following best describes why publicly-traded corporations follow the
practice of having the outside auditor appointed by the board of directors or
elected by the stockholders?

A. To comply with the regulations of the Financial Reporting Standards Council.


B. To emphasize the auditor‘s independence from the management of the client
entity.
C. To encourage a policy of rotation of the independent auditor.
D. To provide the corporate owners with an opportunity to voice their opinion
concerning the quality of the auditing firm selected by the directors.

23. Practitioner's independence:

A. minimizes risk.
B. helps achieve public confidence
C. defends against professional liability
D. achieves compliance with the standards of fieldwork

24. If the client refuses to accept an audit report that is qualified due to a known
existence of noncompliance to laws and regulation, the auditor should:

A. Issue on adverse opinion if management agrees lo fully disclose the matter


B. Withdraw from the engagement and communicate the reasons to the audit
committee in willing
C. Withdraw from the engagement and communicate the reasons to the
Securities and Exchange Commission or other regulatory body in writing.
D. Issue an unqualified opinion if management agrees to fully disclose the
matter.

25. Which of the following is not required by the Generally Accepted Auditing
Standard that states that due professional care is to be exercised in the
performance of the audit?

A. Observance of the standards of field work and reporting.


B. Critical review of the audit work performed at every level of supervision.
C. Degree of skill commonly possessed by others in the profession.
D. Responsibility for losses because of errors of judgment.

26. The standard of due audit care requires the auditor to

A. Apply judgment in a conscientious manner, carefully weighing the relevant


factors before reaching a decision.
B. Ensure that the financial statements are free from error
C. Make perfect judgment decision in all cases.
D. Possess skills clearly above the average for the profession.

27. Which of the following mostly describes the function of AASC?

A. To promulgate auditing standards practice and procedures that shall be


generally accepted by the accounting profession in the Philippines.
B. To monitor full compliance by all auditors to PSAs.
C. To assist the Board of Accountancy in conducting administrative proceedings
on erring CPAs in audit practice.
D. To undertake continuing research on both auditing and financial accounting
in order to make them responsive to the needs of the public.

28. The exercise of due professional care requires that an auditor

A. Examine all available corroborating evidence


B. Critically review the judgment exercised at every level of supervision.
C. Reduce control risk below the maximum.
D. Attain the proper balance of professional experience and formal education.

29. Which of the following best describes the reference to the expression ―taken as a
whole‖ in the fourth generally accepted auditing standard of reporting?

A. It applies only to a complete set of financial statements.


B. It applies equally to each item in each financial statement.
C. It applies equally to each material item in each financial statement.
D. It applies equally to a complete set of financial statements and to each
individual financial statement.

30. Philippine Standards on Auditing issued by AASC

A. apply to independent examination of financial statements of any entity when


such an examination is conducted for the purpose of expressing an opinion.
B. must not apply to other related activities of auditors.
C. need not to be applied on all audit-related engagements.
D. require that in no circumstances would an auditor may judge it necessary to
depart from a psa, even though such a departure may result to more
effective achievement of the objective of an audit.

31. Which of the following is required by the Generally Accepted Auditing Standard
that states that du professional care is to be exercised in the performance of an
audit?

A. Observance of the standards of fieldwork and reporting


B. Critical review of the audit work performed of every level of supervision.
C. Degree of skill commonly possessed by others in the profession.
D. Responsibility for losses because of errors of judgment.

32. A CPA who has been retained by a client that operates in an industry that is
totally new to him.

A. May not accept such engagement.


B. May accept the engagement only if the accounting firm specializes in the
audit of commercial banks.
C. May accept the engagement after attaining a suitable level of understanding
of the transactions and accounting practices unique to commercial banking.
D. May accept the engagement because his training as a CPA transcends
unique industry characteristics.

33. Which of the following is the best statement concerning the concept of
materiality?

A. materiality is determined by reference to PSA matrix.


B. materiality depends only on the peso amounts involves.
C. materiality depends on the nature of an item rather than on the peso amount.
D. materiality is o matter of professional judgment.

34. The first standard of field work, which states that the work is to be adequately
planned and assistants, if any, are to be properly supervised, recognizes that
A. Early appointment of the auditor is advantageous, both to the auditor and to
the client.
B. Acceptance of an audit engagement after the close of the client‘s fiscal year
is generally not permissible.
C. Appointment of the auditor subsequent to the physical count of inventories
requires a disclaimer of opinion
D. Performance of substantial parts of the engagement is necessary at interim
dates

35. Which of the following underlies the application of generally accepted auditing
standards, particularly the standards of fieldwork and reporting?

A. Elements of materiality and risk


B. Element of corroborating evidence
C. Element of internal control
D. Element of reasonable assurance

36. Which of the following is not an attestation‘s standard?

A. The engagement shall be performed by a practitioner having adequate


knowledge in the subject matter of the assertion.
B. Sufficient evidence stroll be obtained to provide a reasonable basis for the
conclusion that is expressed in the report.
C. The work shall be adequately planned, assistants, if any, shall be properly
supervised.
D. The report shall state whether the financial statements are presented in
accordance with generally accepted accounting principles.

37. Which of the following is a conceptual difference between the attestation


standards and generally accepted auditing standards?

A. The attestation standards provide a framework for the attest function beyond
historical financial statements.
B. The requirement that the practitioner be independent in mental attitude is
omitted from the attestation standard
C. The attestation standards do not permit an attest engagement to be part of a
business acquisition study or a feasibility study
D. None of the standards of fieldwork in generally accepted auditing standards
are included in the attestation standards.

38. The auditor‘s judgment concerning the overall fairness of the presentation of
financial position, results of operations, and changes in financial position is
applied within the framework of
A. Philippine Financial Reporting Standards
B. Generally accepted auditing standards
C. Internal Control
D. Information systems control

39. The auditor communicates the results of his or her work through the issuance of:

A. Engagement letter
B. Management letter
C. Audit report
D. Financial statements

40. The four major steps in conducting an audit are:

I. Testing internal control


II. Audit report
III. Planning
IV. Testing transactions and balances

The proper sequence in applying the above steps is:

A. III, I, IV, II
B. III, IV, I, II
C. II, III, IV, I
D. I, IV, III, II
MODULE 3

PROFESSIONAL AND LEGAL RESPONSIBILITY

PSA BASED QUESTIONS

1. The revised Code of Ethics is mandatory for all CPAs and is applicable to professional
services performed in the Philippines on or:

A. Before June 30, 2008


B. After June 30, 2008
C. Before January 1, 2008
D. After January 1, 2008

2. Which of the following is not explicitly referred to in the Code of Ethics as source of technical
standards?

A. Commission on Audit (COA)


B. Auditing and Assurance Standards Council (AASC)
C. Securities and Exchange Commission (SEC)
D. Relevant legislation

3. Immediate family includes:

A. Parent
B. Sibling
C. Non-dependent child
D. Spouse

4. Close family includes the following except:

A. Parent
B. Sibling
C. Non-dependent child
D. Spouse

5. Firm includes the following except

A. A sole practicing professional accountant.


B. An entity that controls a partnership of professional accountants.
C. An entity controlled by a partnership of professional accountants.
D. A sole practitioner, partnership or corporation of professional accountants.

6. Existing accountant, as defined in the Code of Ethics, means:

A. A professional accountant employed un industry, commerce, the public sector or


education.
B. A professional accountant in public practice currently holding an audit appointment or
carrying out accounting taxation, consulting or similar professional services for a client.
C. Those persons who hold a valid certificate issued by the Board of Accountancy
D. A sole proprietor or each partner or person occupying a position similar to that of a
partner and each staff in a practice providing professional services to a client
irrespective of their functional classification (e.g. audit, tax or consulting) and
professional accountants in a practice having managerial responsibilities.

7. The term professional accountant in public practice includes the following except:

A. A sole proprietor providing professional services to a client.


B. Each partner or person occupying a position similar to that of a partner staff in a practice
providing professional services to a client.
C. Professional accountants employed in the public sector having managerial
responsibilities.
D. A firm of professional accountants in public practice.

8. The term receiving accountant includes the following except:

A. A professional accountant in public practice to whom the existing has referred tax
engagement.
B. A professional accountant in public practice to whom the client of the existing accountant
has referred audit engagement.
C. A professional accountant in public practice who is consulted in order to meet the needs
of the client.
D. A professional accountant in public practice currently holding an audit appointment or
carrying out accounting, taxation, consulting or similar professional services for a client.

9. Related entity is an entity that has any of the following relationships with the client, except:

A. An entity that has direct or indirect control over the client provided that the client is
material to such entity.
B. An entity with a direct financial interest in the client even though such entity has no
significant influence over the client provided the interest in the client is material to such
entity.
C. An entity over which the client has direct or indirect control.
D. An entity which is under common control with the client (referred to as a ―sister entity‖)
provided the sister entity and the client are both material to the entity that controls both
the client and sister entity.

10. A primary purpose for establishing a code of ethics within a professional organization is to:

A. Demonstrate the acceptance of responsibility to the interest of those served by the


profession.
B. Reduce the likelihood that members of the profession will be sued for substandard work.
C. Ensure that all members of the profession posses approximately the same level of
competence.
D. Require the members of profession to exhibit loyalty in all matters pertaining to the
affairs of the organization.
11. Which statement is incorrect regarding the Code of Ethics for Professional Accountants in
the Philippines?

A. Professional accountants refer to persons who are registered in the PRC AS Certified
Public Accountants (CPA) and who hold a valid certificate issued by the Board of
Accountancy.
B. Where a national statutory requirement is in conflict with a provision of the IFAC Code,
the IFAC Code requirement prevails.
C. The Code of Ethics for Professional Accountants in the Philippines is mandatory for all
CPAs and is applicable to professional services performed in the Philippines on or after
June 30, 2008.
D. Professional accountants should consider the ethical requirements as the basic
principles, which they should follow in performing their work.

12. The communication to the public of facts about a professional accountant, which are not
designed for the deliberate promotion of that professional accountant.

A. Publicity
B. Indirect Promotion
C. Advertising
D. Solicitation

13. Advertising, as defined in the Code of Ethics, means

A. The communication to the public of facts about a professional accountant which are not
designed for the deliberate promotion of that professional accountant.
B. The approach to a potential client for the purpose of offering professional services.
C. The communication to the public of information as to the services or skills provided by
professional accountants in public practice with a view to procuring professional
business.
D. Any of the given choices.

14. The following bodies develop and or issue technical and professional standards for
implementation:

I. Board of Accountancy
II. II. National Economic Development Authority
III. Financial Reporting Standards Council
IV. Securities and Exchange Commission
V. Auditing and Assurance Standards Council
VI. Cooperative Commission of the Philippines

According to the revised code of ethics for CPAs, which of the foregoing are sources of
technical and professional standards in the Philippines?
A. I, III, IV, V
B. I, III, IV, V, VI
C. I, III, IV
D. All of them

15. The attainment of professional competence can be fulfilled by a combination of:

I. Period of work experience


II. High standard for professional education
III. High standard of general education
IV. Training and examination in professionally relevant subjects

What should be the logical pattern of the foregoing development for a professional accountant?

A. III, II, IV, I


B. III, I, II, IV
C. II, III, IV, I
D. II, III, I, IV

16. Which of the following is least likely the basis of determining audit fees?

A. The skill and knowledge required for the type of work involved.
B. The degree of responsibility ad urgency that the work entails.
C. The expected outcome of the engagement.
D. The required level of training and experience of the persons engaged on the work.

17. Which of the following is not allowed by the revised code of ethics?

A. A professional accountant in public practice may issue to client or, in response to an


unsolicited request, to a non-client a factual and objectively worded of the services
provided.
B. Booklets and other documents bearing the name of a professional and giving technical
information for the assistance of staff or clients may be issued to such persons, or other
professional accountants or other interested parties.
C. The use of the name of an international accounting firm affiliation/ correspondence is
generally allowed.
D. A firm or CPA practitioner can continue to use the term ―Accredited‖ or any similar words
or phrase calculated to convey the same meaning if the claimed accreditation has not
expired.

18. How frequent can a professional accountants have press and other media releases
commemorating their anniversaries in public practice by informing the public of their
achievements or accomplishments in contributing toward nation building or enhancing the
image r standards of the accounting profession?

A. 2 years
B. 3 years
C. 5 years
D. 6 years
19. Which of the following is not allowed to be included in a website of a firm of professional
accountants?

A. Names of partners/ principals with their educational attainment.


B. Membership in any professional body.
C. Awards received
D. Listing of the firm‘s clients.

20. In their fiduciary role, the professional accountants owe their primary loyalty to:

A. The accounting profession


B. The general public
C. The client
D. Government regulatory agencies

21. Which of the following is a distinguishing mark of the accountancy profession?

A. A drive to excellence
B. Acceptance of the responsibility to act in the public interest
C. Professional objectivity
D. Professional skepticism

22. Which statement is incorrect regarding the Code of Ethics for Professional Accountants in
the Philippines?

A. The objectives as well as the fundamental principles are of a general nature and are not
intended to be used to solve a professional accountant‘s ethical problems in a specific
case.
B. The code is divided in two parts, part A and part B.
C. Part A applies to all professional accountants unless otherwise specified.
D. Part B applies only to those professional accountants in public practice.

23. A professional accountant should comply with relevant laws and regulations and should
avoid any action that discredits the profession. This is a fundamental principle of:

A. Objectivity
B. Professional competence and due care
C. Professional behavior
D. Integrity

24. The IFAC Code of Professional Conduct will ordinarily be considered to have been violated
when the member represents that specific consulting services will be performed for a stated fee
and it is apparent at the time of the representation that the

A. Actual fee would be substantially higher.


B. Actual fee would be substantially lower than the fees charged by other members for
comparable services.
C. Fee was a competitive bid.
D. Member would not be independent.

25. Which of the following is not one of the fundamental principles of ethical conduct for
professional accountants?

A. Integrity
B. Confidentiality
C. Loyalty
D. Professional competence and due care

26. To what fundamental principle does the following statement best fit? A professional
accountant is likened to a prudent father to his son.

A. Professional competence and due care


B. Confidentiality
C. Integrity
D. Objectivity

27. Which fundamental principle is seriously threatened by an engagement that is compensated


based on the net proceeds on loans received by the client from a commercial bank?

A. Objectivity
B. Professional behavior
C. Confidentiality
D. Integrity

28. Which of the following is required to comply with the fundamental principle of professional
competence and due care?

A. A professional accountant should not allow bias, conflict of interest or undue influence of
others to override professional or business judgment.
B. A professional accountant should act diligently and in accordance with technical and
professional standards when providing professional services.
C. A professional accountant should comply
D. The accountant should observe fair dealings and truthfulness.

29. ―A professional accountant should be straight-forward and honest in all his professional and
business relationships.‖ This description appropriately describes the fundamental principle of:

A. Integrity
B. Objectivity
C. Confidentiality
D. Professional Behavior

30. It is essential that uses of the audited financial statements regard CPA firms as
A. Competent
B. Unbiased
C. Technically proficient
D. All of the given choices

31. The Code of Professional Ethics states, in part, that a CPA should maintain integrity and
objectivity. Objectivity refers to the CPA‘s ability to

A. Determine accounting practices that were consistently applied


B. Maintain an impartial attitude on all matters which come under his review
C. Determine the materiality of items
D. Insist on all matters regarding audit procedures

32. Which of the following values is not necessary for a professional accountant?

A. Honesty
B. Objectivity
C. Integrity
D. A primary commitment to self-interest

33. Which of the following is not a fundamental principle in codes of ethics for professional
accountants?

A. Act in the client‘s best interest


B. Objectivity and independence
C. Maintain the good reputation of the profession
D. Maintain confidentiality

34. Which of the following statements about conceptual framework of the code of ethics is
incorrect?

A. A conceptual framework that requires a professional accountant to identify, evaluate and


address threats to compliance with the fundamental principles, rather than merely with a
set of specific rules which may be arbitrary is in the public interest.
B. As a concern to public interests, the professional accountant should comply with a set of
specific rules rather than arbitrarily identify, evaluate and address threats to compliance
with fundamental principles.
C. If identifies threats are other than clearly insignificant, a professional accountant should
appropriately apply safeguards to eliminate the threats or reduce them to an acceptable
level.
D. The Code provides a framework to assist a professional accountant to identify, evaluate
and respond to threats to compliance with the fundamental principles.

35. Which of the following is true of the conceptual framework approach?


A. It is impossible to define every situation that creates specific threats and and specify the
appropriate mitigating action.
B. A professional accountant should take qualitative but not quantitative factors into
account when considering the significance of a threat.
C. A professional accountant should take quantitative but not qualitative factors into
account when considering the significance of a threat.
D. All inadvertent violations of the code of Ethics, irrespective of their nature and
significance, always compromise compliance with the fundamental principles

36. Which of the following appropriately describes an advocacy threat?

A. The professional accountant may be deterred from acting objectivity by threats, actual or
perceived.
B. Because of a close relationship, a professional accountant becomes too sympathetic to
the interest of others.
C. The professional accountant should provides a position or opinion to the point that
subsequent objectivity may be compromised.
D. The professional accountant needs to be reevaluate his previous judgment.

37. A threat that prevents the professional accountant from acting objectively by threats, actual
or perceived.

A. Self-interest
B. Familiarity
C. Intimidation
D. Advocacy

38. A form of threat which may occur when a previous judgment needs to be reevaluated by the
professional accountant who is responsible for that judgment.

A. Self-interest threat
B. Self-review threat
C. Familiarity threat
D. Advocacy threat

39. Advocacy threat may occur:

A. As a result of the financial or other interests of a professional accountant or an


immediate or close family member.
B. When, because of a close relationship, a professional accountant becomes too
sympathetic to the interests of others.
C. When a professional accountant promotes a position or opinion to the point that
subsequent objectivity may be compromised.
D. When a professional accountant may be deterred from acting objectively by threats,
actual or perceived.
40. It occurs when a firm or member of the assurance team could benefit from a financial
interest in, or other self-interest conflict with, an assurance client.

A. Self-interest threat
B. Self-review threat
C. Advocacy threat
D. Familiarity threat

41. A financial interest beneficially owned through a collective investment vehicle, estate, trust
or other intermediary over which the individual or entity has no control.

A. Indirect financial interest


B. Financial instrument
C. Direct financial interest
D. Clients‘ monies

42. Financial interest means:

A. Any bank account which is sued solely for the banking of clients‘ monies.
B. Any monies received by a professional accountant in public practice to be held or paid
out on the instruction of the person from whom or on whose behalf they are received.
C. A financial interest beneficially owned through a collective investment vehicle, estate,
trust or other intermediary over which the individual or entity has no control.
D. An equity interest or other security, debenture, loan or other debt instrument of an entity,
including rights and obligations to acquire such an interest and derivatives directly
related to such interest.

43. Direct financial interest is a financial interest:

A B C D

 Owned directly by and under


the control of an individual or
entity (including those Yes Yes Yes No
managed on a discretionary
basis by other)
 Beneficially owned through a
collective investment vehicle,
estate, trust or other
intermediary over which the Yes Yes No No
individual or entity has control
 Benecially owned through a
collective investment vehicle,
estate, trust or other
intermediary over which the Yes No No Yes
individual or entity has no
control

44. Occurs when any product or judgment of a previous assurance engagement or non-
assurance engagement needs to be re-evaluated in reaching conclusions on the assurance
engagement or when a member of the assurance team was previously a director or officer of
the assurance client, or was an employee in a position to exert direct and significance influence
over the subject matter of the assurance engagement.

A. Self-interest threat
B. Self-review threat
C. Advocacy threat
D. Familiarity threat

45. Intimidation threat

A. is not a threat to independence.


B. occurs when a member of the assurance team may be deterred from acting objectively
and exercising professional skepticism by threat, actual or perceived, from the directors,
officers or employees of an assurance client.
C. occurs when, by virtue of a close relationship with an assurance client, its directors,
officers or employees, a firm or a member of the assurance team becomes too
sympathetic to the client‘s interests.
D. occurs when a firm, or a member of the assurance team, promotes, or may be perceived
to promote, an assurance client‘s position or opinion to the point that objectivity may, or
may be perceived to be, compromised.

46. Safeguards created by the profession, legislation or regulation, include the following, except:

A. Educational, training and experience requirements for entry into the profession.
B. Continuing education requirements.
C. Legislation governing the independence requirements of the firm.
D. Policies and procedures that emphasize the assurance client‘s commitment to fair
financial reporting.

47. Which of the following is an example of engagement-specific safeguards?

A. Advising partners and professional staff of those assurance clients and related entities
from which the must be independent.
B. Consulting an independent third party, such as committee of independent directors, a
professional regulatory body or another professional accountant.
C. Policies and procedures that will enable the identification of interests or relationships
between the firm or members of engagement teams and clients.
D. External review by a legally empowered third party of the reports, returns,
communications or information produced by a professional accountant.

48. Which of the following is not a safeguard created by the profession, legislation or regulation?
A. Professional standards
B. Professional and procedures to implement and monitor quality control of engagements.
C. Continuing professional development requirements
D. Educational, training and experience requirements for entry into the profession.

49. Safeguards may eliminate or reduce threats to an acceptable level. The following are
examples of these safeguards:

I. Professional or regulatory monitoring and discipline procedures.


II. Documented internal policies and procedures requiring compliance with the
fundamental principles.
III. Policies and procedures to monitor and, if necessary, manage the reliance on
revenue received from a single client.

Which of the foregoing examples of safeguards is/ are classified firm-wide safeguards in the
work environment?

A. All of these
B. I and II
C. II and III
D. I and III

50. Which of the following fundamental principles is compromised when a professional


accountant is associated with reports or returns that are significantly misleading?

A. Integrity
B. Competence and due professional care
C. Objectivity
D. Professional behavior

51. Safeguards may eliminate or reduce threats to an acceptable level. The following are
examples of these safeguards:

I. Professional or regulatory monitoring and disciplinary procedures.


II. Documented internal policies and procedures requiring compliance with the
fundamental principles.
III. Policies and procedures to monitor and, if necessary, manage the reliance on
revenue received from a single client.
IV. Corporate governance regulations

Which of the foregoing examples of safeguards that can be applied is(are) created by the
profession, legislation or regulation?

A. I and III
B. II and Iv
C. I and IV
D. II and III

52. Which of the following examples of safeguards that may effectively reduce threats to
compliance with the fundamental of principles is created by the profession, legislation or
regulation?

A. Published policies and procedures to encourage and empower staff to communicate to


senior levels within the firm any issue relating to compliance with the fundamental
principles that concerns them.
B. Effective, well-publicized complaints system operated by the employing organization, the
profession or a regulator, which enable colleagues, employers and members of the
public to draw attention to unethical behavior.
C. Designating a member of senior management to be responsible for overseeing the
adequate functioning of the firm‘s quality control system.
D. Disclosing to those charged with governance of the client the nature of services provided
and the extent of fees charged.

53. Professional accountants may encounter problem in identifying unethical behavior or in


resolving an ethical conflict. When faced with significant ethical issues, professional accountants
should do the following except

A. Follow the established policies of the employing organization to seek a resolution of


such conflict.
B. Review the conflict problem with the immediate superior if the organization‘s policies do
not resolve the ethical conflict.
C. If the problem is not resolved with the immediate superior and the professional
accountant determines to go to the next higher managerial level, the immediate superior
need not be notified of the decision.
D. Seek counseling and advice on a confidential basis with an independent advisor or the
applicable professional accountancy body or regulatory body to obtain an understanding
of possible courses of action.

54. As a resolution of the conflict in the application of fundamental principles, the auditor, after
considering the ethical issues and relevant facts may do any of the following except:

A. Must immediately resign from the engagement or the employing entity.


B. Should weigh the consequences of each possible course of action.
C. Should consult with other appropriate persons within the firm or employing organization
foe help to finally resolve the matter.
D. The professional accountant may wish to obtain professional advice from the relevant
professional body without breaching confidentiality if significant conflict cannot be
resolved.

55. Which of the following is incorrect regarding integrity and objectivity?

A. Integrity implies not merely honesty but fair dealing and truthfulness.
B. The principle of objectivity imposes the obligation on all professional accountants to be
fair, intellectually honest and free of conflicts of interest.
C. Professional accountants serve in many different capacities and should demonstrate
their objectivity in varying circumstances.
D. Professional accountants should neither accept nor offer any gifts or entertainment.

56. If a professional accountant is billing an audit client a number of hours greater than those
actually worked, which of the following fundamental principles is likely violated?

A. Objectivity
B. Integrity
C. Professional due care
D. Confidentiality

57. Which of the following is incorrect regarding professional competence?

A. Professional accountants may portray themselves as having expertise or experience


they do not possess.
B. Professional competence may be divided into two separate phases.
C. The attainment of professional competence requires initially a high standard of general
education.
D. The maintenance of professional competence requires a continuing awareness of
development in the accountancy profession.

58. In which of the following circumstances may disclosure of confidential information be


appropriate?

A. Disclosure is necessary as required by legal proceedings.


B. The professional accountant volunteered to reveal information in order to help a faster
resolution of legal proceedings.
C. Working papers are returned over to other professional accountant who purchased the
accounting practice.
D. Detailed listing of inactive customers of one assurance client is passed on to other non-
assurance client.

59. The underlying reason for a code of professional conduct for any profession is

A. the need for public confidence in the quality of service of the profession.
B. that it provides a safeguard to keep unscrupulous people out.
C. that it is required by the congress.
D. that it allows Professional Regulation Commission to have a yardstick to measure
deficient performance.

60. A professional accountant may be associated with a tax return that

A. contains a false or misleading statement.


B. contains statements r information furnished recklessly or without any real knowledge of
whether they are true or false.
C. omits or obscures information required to be submitted and such omission or obscurity
would mislead the revenue authorities.
D. uses of estimates if such use is generally acceptable or if it is impractical under the
circumstances to obtain exact data.

61. There are fundamental principles that the professional accountant has to observe when
performing assurance engagements. The requirement of which principle is of particular
importance in an assurance engagement in ensuring that the conclusion of the professional
accountant has value to the intended user?

A. Integrity
B. Confidentiality
C. Professional competence
D. Objectivity

62. If a professional accountant is auditing a public company and he receives from his client its
shares of stock as payment for his audit services, he will be violating the fundamental principle
of:

A. Integrity
B. Professional due care
C. Objectivity
D. Confidentiality

63. Which of the following is least likely an indication that the CPA violates the Integrity
principle? The CPA is associated with reports or information that:

A. The CPA issues a qualified opinion due to scope limitation because he fails to arrive at a
clear-cut conclusion.
B. Contains a materially false or misleading statement.
C. Omits or obscures information required to be included when such omission or obscurity
would make the information misleading,
D. Contains statements or information furnished recklessly.

64. Which of the following is the least required in attaining professional competence?

A. High standard of general education


B. Specific education, training and examination in professionally relevant subjects.
C. Period of meaningful work experience.
D. Continuing awareness of development in the accountancy profession.

65. Which of the following is incorrect regarding confidentiality?


A. Professional accountants have an obligation to respect the confidentiality of information
about a client‘s or employer‘s affairs acquired in the course of professional services.
B. The duty of confidentiality ceases after the end of the relationship between the
professional accountant and the client or employer.
C. Confidentiality should always be observed by a professional accountant unless specific
authorization has been given to disclose information or there is a legal or professional
duty to disclose.
D. Confidentiality requires that a professional accountant acquiring information in the
course of performing professional services neither uses nor appear to use that
information for personal advantage or for the advantage of a third party.

66. A professional accountant is auditing Maiden Company and providing consulting services to
Widow Company. Both clients are in the same industry. IF the professional accountant uses
specific information from Maiden‘s audit to prepare a business plan for Widow, he will be
violating the principle of:

A. Integrity
B. Professional behavior
C. Objectivity
D. Confidentiality

67. Which of the following statements is incorrect about the principle of confidentiality?

A. The professional accountants must refrain from disclosing confidential information


acquired as a result of professional and business relationships to any party outside the
firm or employing organization unless there is a legal or professional right or duty to
disclose.
B. The professional accountants must never disclose confidential information obtained as a
result of professional business relationships.
C. The need to comply with confidentiality principle continues even after the end of
relationships between a professional accountant and a client or employer.
D. A professional accountant should consider that some information may be kept
confidential within the firm or employing organization.

68. The confidential relationship applies to

A. all services provided by CPAs.


B. only audit a d attestation services.
C. audit and tax services, but not MAS services.
D. audit and MAS services, but not tax services.

69. Which of the following may not be a professional duty to disclose confidential information?

A. In compliance with the quality review of a member body or professional body.


B. In compliance with technical standards and ethics requirements.
C. In response to specific inquiry from the major stockholder.
D. In protecting the professional interests of the professional accountant in case of
litigation.
70. The professional accountant has a professional duty or right to disclose confidential
information in each of the following, except

A. To comply with technical standards and ethics and requirements.


B. To disclose to BIR fraudulent scheme committed by the client on payment of income tax.
C. To comply with the quality of review of a member body or professional body.
D. To respond to an inquiry or investigation by a member body or regulatory body.

71. What kind of threat to noncompliance to fundamental principles is created if the professional
if the professional fees due from a financial statement audit client remain unpaid for a long time?

A. Self-interest review threat


B. Self-review threat
C. Familiarity threat
D. No threat is created

72. A CPA in public practice shall not disclose any confidential client information without the
specific consent of the client. The confidentiality rule is violated if the CPA discloses information
without a client‘s consent as a result of a

A. subpoena or summons
B. peer review
C. complaint filed with the trial board of the Board of Accountancy.
D. request from a client‘s largest stockholder.

73. The confidential relationship will be violated if, without the client‘s permission, the CPA
provides working papers about the client to

A. a court of law which subpoenas them.


B. another CPA firm as part of a peer review.
C. another CPA firm which has just purchased the CPA‘s entire practice.
D. an investigate or disciplinary body which is conducting a review of the CPA‘s practice.

74. Assurance team include

A B C D

 All professionals participating


In the assurance engagement Yes Yes Yes Yes
 All others within a firm who
can directly influence the
outcome of the assurance
engagement Yes Yes No No
 For the purposes of an audit
client, all those within a
network firm who can directly
influence the outcome of the
audit engagement Yes No No Yes

75. Examples of circumstances that may create self-interest threat include:

A. Contingent fees relating to assurance engagements.


B. A direct financial interest or material indirect financial interest in an assurance client.
C. A loans or guarantee to or from an assurance client or any if its directors or officers.
D. All of the given choices

76. Which of the following is least likely create ―self-interest threat‖?

A. Undue dependence on total fees from an assurance client.


B. Concern about the possibility of losing the engagement.
C. Having a close business relationship with an assurance client.
D. Pressure to reduce inappropriately the extent of work performed in order to reduce fees.

77. If the firm is involved in the preparation of accounting records or financial statements and
those financial statements are subsequently the subject matter of an audit engagement of the
firm, this will most likely create

A. self-interest threat
B. self-review threat
C. intimidation threat
D. familiarity threat

78. Examples of circumstances that may create self-review threat least likely include

A. preparation of original data used to generate financial statements or preparation of other


records that are the subject matter of the assurance engagement.
B. a member of the assurance team being, or having recently been, an employee of the
assurance client in a position to exert direct and significant influence over the subject
matter of the assurance engagement.
C. potential employment with an assurance client.

79. Family and personal relationships between a member of the assurance team and a director,
an officer or certain employees, depending on their role, of the assurance client, least likely
create

A. self-interest threat
B. self-review threat
C. intimidation threat
D. familiarity threat

80. A director, an officer or an employee of the assurance client in a position to exert direct and
significant influence over the subject matter of the assurance engagement has been a member
of the assurance team or partner of the firm. This situation least likely create
A. self-interest threat
B. advocacy threat.
C. intimidation threat.
D. familiarity threat.

81. A former officer, director or employee of the assurance client serves as a member of the
assurance team. This situation will least likely create

A. self-interest threat.
B. self-review threat.
C. intimidation threat.
D. familiarity threat.

82. Which of the following will least likely impair independence?

A. An immediate family member of a member of the assurance team is a director, an officer


or an employee of the assurance client in a position to exert direct and significant
influence over the subject matter of the assurance engagement.
B. A member of the assurance team participates in the assurance engagement while
knowing, or having reason to believe, that he or she is to, or may, join the assurance
client in the future.
C. A partner or employee of the firm serves as an officer or as a director on the board of an
assurance client.
D. A partner or employee of the firm or a network firm serves as Company Secretary for an
audit client, the duties and functions undertaken are limited to those of a routine and
formal administrative nature as such as the -preparation of minutes and. maintenance of
statutory returns.

83. The provision of services by a firm or network firm to an audit client that involve the design
and implementation of financial information technology systems that are used to generate
information forming part of a client's financial statements may most likely create

A. self-interest threat.
B. self-review threat.
C. intimidation threat.
D. familiarity threat.

84. Occurs when a firm, or a member of the assurance team, promotes, or may be perceived to
promote, an assurance client's position or opinion to the point that objectivity may, or may be
perceived to be, compromised. Such may be the case if a firm or a member of the assurance
team were to subordinate their judgment to that of the client.

A. Self-interest threat
B. Self-review threat
C. Advocacy threat.
D. Familiarity threat
85. A CPA-lawyer, acting as a legal counsel to one of his audit client, is an example of

A. Self-interest threat
B. Self-review threat
C. Advocacy threat
D. Familiarity threat

86. Which of the following is not likely a threat to independence?

A. Acting as an advocate on behalf of an assurance client in litigation or in resolving


disputes with third parties.
B. Long association of a senior member of the assurance team with the assurance client.
C. Threat of replacement over a disagreement with the application of an accounting
principle.
D. Owning immaterial indirect financial interest in an audit client.

87. Occurs when, by virtue of a close relationship with an assurance client, its directors, officers
or employees, a firm or a member of the assurance team becomes too sympathetic to the
client‘s interests.

A. Self-interest threat
B. Self-review threat
C. Advocacy threat
D. Familiarity threat

88. Examples of circumstances that may create familiarity threat least likely include

A. a member of the assurance team having an immediate family member or close family
member who is a director or officer of the assurance client.
B. a member of the assurance team having an immediate family member or close family
member who, as an employee of the assurance client, is in a position to exert direct and
significant influence over the subject matter of the assurance engagement.
C. a former partner of the firm being a director, officer of the assurance client or an
employee in a position to exert direct and significant influence over the subject matter of
the assurance engagement.
D. dealing in, or being a promoter of, share or other securities in an assurance client.

89. Consideration of the nature of the safeguards to be applied will be affected by matters such
as the

A B C D

 Significance of the threat Yes Yes Yes Yes


 Nature of the assurance engagement Yes Yes Yes No
 Intended users of the assurance report Yes Yes No Yes
 Structure of the firm Yes No No No

90. The safeguards available to eliminate the threats or reduce them to acceptable level include

A B C D

• Safeguards created by the


profession, legislation or
regulation YES YES YES YES
•Safeguards within the
assurance client YES YES NO NO
•Safeguards within the firm's
own systems and procedures YES NO NO YES

91. Safeguards within the firm's own systems and procedures, include the following, except:

A. Firm leadership that stresses the importance of independence and the expectation that
members of assurance teams will act in the public interest.
B. External review of a firm's quality control system.
C. Policies and procedures to implement and monitor quality control of assurance
engagements.
D. Policies and procedures that will enable the identification of interests or relationships
between the firm or members of the assurance team and assurance clients.

92. Safeguards within the assurance client, include the following, except

A. Professional standards and monitoring and disciplinary processes.


B. The assurance client has competent employees to make managerial decisions.
C. Internal procedures that ensure objective choices in commissioning non-assurance
engagements.
D. A corporate governance structure, such as an audit committee, that provides appropriate
oversight and communications regarding a firm's services.

93. In determining estimates of fees, an auditor may take into account each of the following,
except the:

A. Value of the service to the client.


B. Degree of responsibility assumed by undertaking the engagement.
C. Skills required in performing the service.
D. Attainment of specific findings.

94. The Code of Professional Conduct would be violated if a member accepted a fee for
services and the fee was

A. fixed by a public authority


B. based on price quotation submitted in competitive bidding.
C. based on the result of judicial proceedings.
D. payable after a specified finding was attained.

95. In the marketing and promotion of themselves and their Work professional accountants
should:

A. Not use means which brings the profession into disrepute.


B. Not make exaggerated claims for the services they are able to offer. the qualifications
they possess. or experience they have gained.
C. Not denigrate the work of other accountants.
D. All of the above.

96. Which of the following is incorrect regarding independence?

A. Independence consists of independence of mind and independence in appearance.


B. Independence of mind is the state of mind that permits the provision of an opinion
without being affected by influences that compromise professional judgment, allowing an
professional skepticism. individual to act with integrity', and exercise objectivity and
C. Independence in appearance is the avoidance of facts and circumstances that are so
significant a reasonable and informed third party, having knowledge of all relevant
information, including any safeguards applied, would reasonably conclude a firm‘s or a
member of the assurance team‘s integrity, objectivity or professional skepticism had
been comprmised.
D. Independence is a combination of impartiality, intellectual honesty and a freedom from
conflicts of interest.

97. The network firms are required to be independent of the client

A. for assurance engagements provided to an audit client the client.


B. for assurance engagements provided to clients that are not audit clients, when the report
is r:ioty expressly restricted for use by identified users.
C. for assurance engagements provided to clients that are not audit clients, when the
assurance report is expressly restricted for use by identified users.
D. All of the given choices

98. For assurance engagements provided to an audit client, the following should be
independent of the client:

A B C D

• The members of the assurance team YES YES YES YES


• The firm YES YES NO NO
• Network firms YES NO NO YES
99. Using partners who do not report to the audit partners for the provision of non-assurance
services to an audit client is an example of:

A. Safeguards reducing the risk of conflict of interest created by the profession, legislation,
or regulation.
B. Safeguards reducing the risk of conflict of interest within a client.
C. Safeguards reducing the risk of conflict of interest within a professional accounting firm.
D. All of these.

100. The recommendation for the appointment of the external auditors by the audit committee is
an example of:

A. Safeguards reducing the risk of conflict of interest created by the profession, legislation,
or regulation.
B. Safeguards reducing the risk of conflict of interest between an auditor and the
management.
C. Safeguards reducing the risk of conflict of interest within a professional accounting firm's
own systems and procedures.
D. All of these

101. For assurance engagements provided to clients that are not clients, when the report is not
expressly restricted for by identified users, the following should be independent of the client:

A B C D

• The members of the assurance team YES YES YES YES


• The firm YES YES NO NO
• Network firms YES NO NO YES
102. For assurance engagements provided to clients that are not audit clients, when the
assurance report is expressly restricted for use by identified users, the following should be
independent of the client:

A B C D

• The members of the assurance team YES YES YES YES


• The firm YES YES NO NO
• Network firms YES NO NO YES

103. Contingent fee pricing of public accounting services is:

A. Always strictly prohibited in public accounting practice.


B. Never restricted in public accounting practice.
C. Prohibited for clients for who m attestation services arc
D. Considered an act discreditable to the profession.

104. The firm should be independent of the client in the following engagements:
A B C D

 Assurance engagements provided


to an audit client YES YES YES YES
 Assurance engagements provided to
clients that are not audit clients,
when the report is not expressly
restricted for use by identified users YES YES NO NO
 Assurance engagements provided to
clients that are not audit clients,
when the assurance report is expressly
restricted for use by identified users YES NO NO YES

105. When the safeguards available are insufficient to eliminate the threats to independence or
to reduce them to an acceptable level, or when a firm chooses not to eliminate the activities or
interest creating the threat, the only course of action available will be the

A. Issuance of an adverse opinion.


B. Issuance of qualified opinion or disclaimer of opinion.
C. Issuance of unqualified opinion with explanatory paragraph.
D. Refusal to perform, or withdrawal from, the assurance engagement.

106. Which of the following is incorrect regarding engagement period?

A. The period of the engagement starts when the tesurance team begins to perform
assurance services and ends when the assurance report is issued, except when the
assurance engagement is of a recurring nature.
B. If the assurance engagement is expected to recur, the period of the assurance
engagement ends with the notification by either party that the professional relationship
has terminated or the issuance of the final assurance report, whichever is earlier.
C. In the case of an audit engagement, the engagement period includes the period covered
by the financial statements reported on by the firm.
D. When an entity becomes an audit client during or after the period covered by the
financial statements that the firm will report on, the firm should consider whether any
threats to independence may be created by previous services provided to the audit
client.

107. If a member of the assurance team, or their immediate family member, has a direct
financial interest, or a material indirect financial interest, in the assurance client, the self-interest
threat created would be so significant that the only safeguards available to eliminate the threat
or reduce it to an acceptable level would be to (choose the incorrect one)

A. dispose of the direct financial interest prior to the individual becoming a member of the
assurance team.
B. dispose of the indirect financial interest in total prior to the individual becoming a
member of the assurance team.
C. dispose of a sufficient amount of the indirect financial interest so that the remaining
interest is no longer material prior to the individual becoming a member of the assurance
team.
D. limit the participation of the member of the assurance team.

108. If a member of the assurance team, or their immediate family member receives, by way of,
for example, an inheritance, gift or, as a result of a merger, a direct financial interest or a
material indirect financial interest in the assurance client, a self-interest threat would he created.
The following safeguards should be applied to eliminate the threat or reduce it to acceptable
level:

A. Disposing of the financial interest at the earliest practical date.


B. Removing the member of the assurance team from the assurance engagement.
C. Either a or b
D. Neither a nor b

109. When a member of the assurance team knows that his or her close family member has a
direct financial interest or a material indirect financial interest in the assurance client, a self-
interest threat may be created. Safeguards least likely include:

A. The close family member disposing of all or a sufficient portion of the financial interest at
the earliest practical date.
B. Discussing the matter with those charged withgovernance, such as the audit committee.
C. Involving a professional accountant who took part in the assurance engagement to
review the work done by the member of the assurance team with the close family
relationship or otherwise advice as necessary
D. Removing the individual from the assurance engagement.

110. When a firm or a member of the assurance team holds a direct financial interest or a
material indirect financial interest in the assurance client as a trustee a self-interest threat tray
created by the possible influence of the trust over the assurance client. Accordingly, such an
interest cannot be held when:

A. The member of the assurance team, an immediate family member of the member of the
assurance team, and the firm are beneficiaries of the trust.
B. The interest held by the trust in the assurance client is not material to the trust.
C. The trust is not able to exercise significant influence over the assurance client.
D. The member of the assurance team or the Jinn does not have significant influence over
any investment decision involving a financial interest in the assurance client.

111. An inadvertent violation of the Independence rules as it relates to a financial interest in an


assurance client would not impair the independence of the firm, the network firm or a member of
the assurance team when:
A. The firm, and the network firm, has established policies and procedures that require all
professionals to report promptly to the firm any breaches resulting from the purchase,
inheritance or other acquisition of a financial interest in the assurance client.
B. The firm, and the network firm, promptly notifies the professional that the financial
interest should be disposed of.
C. The disposal occurs at the earliest practical date after identification of the issue, or the
professional is removed from the assurance team.
D. All of the given choices.

112. The following self-interest threat created would be so significant no safeguard could reduce
the threat to an acceptable level, except

A. If a firm, or a network firm, has a direct financial interest in an audit client of the firm.
B. If a fir, or a network firm, has a material indirect financial interest in an audit client of the
firm.
C. If a firm, or a network firm, has a material financial interest in an entity that has a
controlling interest in an audit client.
D. If the retirement benefit plan of a firm, or network firm, has a financial interest in an audit
client

113. If a firm, or a network firm, has a direct financial interest in an audit client of the firm, the
self-interest threat created would be so significant no safeguard could reduce the threat to an
acceptable level. The action appropriate to permit the firm to perform the engagement would be
to

A. dispose of the financial interest.


B. dispose of a sufficient amount of it so that the remaining interest is no longer material.
C. Either of the given choices
D. Neither of the given choices

114. If a firm, or a network firm, has a. direct financial interest in a financial statement audit
client of the firm, the appropriate safetuard against the self-interest threat created would be-

A. Dispose the entire financial interest.


B. Dispose of a sufficient amount of the financial interest so that the remaining interest is no
longer material.
C. Any of the two is appropriate.
D. None of the two is appropriate.

115. If a firm, or a network firm, has a material financial interest in an entity that has a controlling
interest in a financial statement audit client, the self interest threat created is so significant. The
audit firm can only perform the engagement if it:

I. Dispose of the entire financial interest.


II. Dispose of a sufficient amount of the financial interest so that the remaining interest
is no longer significant
A. Either I or II
B. Neither I or II
C. I only
D. II only

116. Which of the following safeguards is inappropriate if a firm has a material financial interest
in an entity that has a controlling interest in a financial statement audit client?

A. Discuss the presence of self-interest threat with the client's board of directors.
B. Dispose of the financial interest in total.
C. Dispose of a sufficient amount of the financial interest.
D. Either dispose of a sufficient amount of the financial interest or the financial interest in
total.

117. The retirement benefit plan of a firm, or a network firm, has a financial interest in a financial
statement audit client. If the self-interest threat that is created by the financial interest is
significant, the firm that intends to continue the engagement should:

A. Reduce the financial interest so that the remaining interest is no longer material
B. Discuss the matter with the audit committee of the financial statement audit client.
C. Refer the audit of the stockholders' equity of the financial statement audit client to other
CPA.
D. Either dispose of the financial interest in total or a sufficient amount so that the
remaining amount is no longer material.

118. The following loans and guarantees would not create a threat to independence, except:

A. A loan from, or a guarantee thereof by, an assurance client that is a bank or a similar
institution, to the firm, provided the loan is made under normal lending procedures, terms
and requirements and the loan is immaterial to both the firm and the assurance client.
B. A loan from, or a guarantee thereof by, an assurance client that is a bank or a similar
institution, to a member of the assurance team or their immediate family, provided the
loan is made under normal lending procedures, terms and requirements.
C. Deposits made 1* or brokerage accounts of, a firm or a member of the assurance team
with an assurance client that is a bank, broker or similar institution, provided the deposit
or account is held under normal commercial terms.
D. If the firm, or a member of the assurance team, makes a loan to an assurance client that
is not a bank or similar institution, or guarantees such an assurance clients‘ borrowings

119. Examples of close business relationships that may create self_ interest and intimidation
threat least likely include:

A. Having a material financial interest in director, officer or assurance client or a controlling


owner, director, officer or other individual who performs senior managerial functions for
that client.
B. Arrangements to combine one or more services or products of the firm with one or more
sees or products of the assurance client and to market the package with reference to
both parties.
C. Distribution or marketing arrangements under which the firm acts as a distributor or
marketer of the assurance client's products or services, or the assurance client acts as
the distributor or marketer of the products or services of the firm.
D. The purchase of goods and services from an assurance client by the firm (or from an
audit client by a network firm) or a member of the assurance team, provided the
transaction is in the normal course of business and on an arm's length basis.

120. When a firm or a member of the assurance team and the audit client or one of its officers
hold interest in a closely-held entity, a . threat to independence is not created, except:

A. The relationship is clearly insignificant to the member of the assurance team and the
audit client.
B. The relationship is other than insignificant acceptable for indirect financial interest.
C. The interest held is immaterial to the investors or group of investors.
D. The interest does not give the investor, or group the ability to control the closely-held
entity.

121. When an immediate family member of a member of the assurance team is a director or an
officer of the assurance client in a position to exert diret and siginficant infuence over the subject
matter information of the engagement, the threat to independence can only be reduced to an
acceptable level, aside from withdrawing from the engagement, by:

A. Removing the individual from the assurance team.


B. Reduce the participation of the professional.
C. Discuss the matter with the audit committee of the client entity.
D. Request the audit client management to require the immediate family member of the
professional to go on forced vacation leave.

122. Which of the following relationships is most likely to impair a CPA's independence with
respect to a particular audit client on which the CPA works as a member of the engagement
team?

A. A close relative has a material investment in that client of which the CPA is not aware.
B. A cousin has an immaterial investment in that client of which the CPA is not aware.
C. The CPA's father is the president of the audit client.
D. The CPA's spouse participates in a savings plan sponsored by the client.

123. An inadvertent violation of the rules on family and personal relationships would not impair
the independence of a firm or a member of the assurance team when:

A. The firm has established policies and procedures that require all professionals to report
promptly to the firm any breaches resulting from changes in the employment status of
their immediate or close family members or other personal relationships that create
threats to independence.
B. Either the responsibilities of the assurance team are re-structured so that the
professional does not deal with matters that are within the responsibility of the person
with whom he or she is related or has a personal relationship, or, if this is not possible,
the firm promptly removes the professional from the assurance engagement.
C. Additional care is given to reviewing the work of the professional.
D. All of the given choices.

124. If a member of the assurance team, partner or former partner of the firm has joined the
assurance client, the significance of the self-interest, familiarity or intimidation threats created is
least likely affected by.

A. The position the individual has taken at the assurance client.


B. The amount of any involvement the individual will have with the member of the
assurance team.
C. The length of time that the individual was a member of the assurance team or firm.
D. The former position of the individual within the assurance team or firm..

125. Using the same senior personnel on an assurance engagement over a long period of time
may create a familiarity threat. The significance of the threat will least likely depend upon.

A. The length of time that the individual has been a member of the assurance team.
B. The role of the individual on the assurance team.
C. The structure of the client.
D. The nature of the assurance engagement.

126. A small CPA firm provides audit services to a large local company. Almost 80 percent of
the CPA firm's revenues come from this client. Which statement is most likely to be true?

A. Appearance of independence may be lacking.


B. The small CPA firm does not have proficiency to perform a larger audit.
C. The situation is satisfactory if the auditor exercises due skeptical negative assurance
care in the audit.
D. The auditor should provide an "emphasis of a matter his audii report adequately-
disclosing information and then it may issue an unqualified opinion.

127. A professional accountant has been the partner-in-charge of a particular audit client for the
past eight years. This situation could result to the following threat to professional independence:

A. Self-review
B. Advocacy
C. Intimidation
D. Familiarity

128. Which statement is incorrect regarding long association of senior personnel with audit
clients that are listed entities?
A. Using the same lead engagement partner on an audit over a prolonged period may
create a familiarity threat.
B. The lead engagement partner should be rotated after a pre-defined period, normally no
more than six years.
C. A partner rotating after a pre-defined period should not participate in the audit until a
further period of time, normally two years, has elapsed.
D. When audit client becomes a listed entity the length of time the lead engagement partner
has served the audit client in that capacity should be considered in determining when the
partner should be rotated.

129. The professional accountant who has been the lead engagement partner for an audit
engagement for a prolonged period of time may continue to serve as the lead engagement
partner before rotating off the engagement for how many years after the audit client becomes a
listed entity?

A. One year
B. Three years
C. Two years
D. Four years

130. While the lead engagement partner should be rotated after such a pre-defined period,
some degree of flcxibihty over timing of rotation may be necessary in certain circumstances.
Examples of such circumstances include:

A. Situations when the lead engagement partner's continuity is especially important to the
audit client, for example, when there will be major changes to the audit client's structure
that would otherwise coincide with the rotation of the lead engagement partner.
B. Situations when, due to the size of the firm, rotation ip not possible or does not constitute
an appropriate safeguard.
C. Both choices are correct.
D. Both choices are incorrect.

131. A CPA can continue to be an engagement partner on the audit financial statements of
listed entities over a prolonged petioci Of engagement. In order to avoid a creation of familiarity
threat of subject to transitional provisions, how many years are prescribed by the as maximum
for the CPA to continue serving as engagement partner for a listed entity?

A. Five years
B. Three years
C. Seven years
D. Ten years

132. An engagement partner who is rotated in the audit of financial statements of listed entity
can only participate in the audit engagement for the same client after a period of:

A. Twelve months
B. two yen
C. Three years
D. Five years

133. While the engagement partner for an audit of financial statements of listed entities should
be rotated after a predefined period, some degree of flexibility over the timing of rotation maybe
necessary. How many years are allowed as transitional period for the rotation?

A. Six months
B. One year
C. Two years
D. Three years

134. The following activities would generally create self-interest or self-review threats
that are so significant and that only avoidance of the activity or refusal to perform the
assurance engagement would reduce the threats to an acceptable level, except

A. Authorizing, executing or consummating a transaction, or otherwise exercising authority


on behalf of the assurance client, or having the authority to do so.
B. Determining which recommendation of the firm should be implemented.
C. Reporting, in a management role, to those charged with governance.
D. Providing technical assistance and advice on accounting principles for audit clients.

135. Which of the following may not create a self-review threat?

A. Supervising assurance client employees in the performance of their normal recurring


duties.
B. Preparing source documents in electronic or other form evidencing a business
transaction.
C. Prolonged period of assignment as member of engagement team in one particular audit
engagement.
D. Performing corporate financial services for the audit client.

136. If firm, or network firm, personnel providing such assistance make management decisions,
the self-review threat created could not be reduced to an acceptable level by any safeguards.
Examples of such managerial decisions include the following, except

A. Determining or changing journal entries, or the classifications for accounts or


transactions or other accounting records without obtaining the approval of the audit
clients
B. Authorizing or approving transactions.
C. Preparing source documents or originating data (including decisions on evaluation
assumptions), or making changes to such documents or data.
D. Assisting an audit client in resolving account reconciliation problems.
137. The following services are considered to be a normal part of the audit process and do not,
under circumstances, threaten independence, except

A. Analyzing and accumulating information for regulatory reporting.


B. Assisting in the preparation of consolidated financial statements.
C. Drafting disclosures items.
D. Having custody of an assurance client's assets.

138. If the firm is involved in the preparation of accounting records or financial statements and
those financial statements are subsequently the subject matter of an audit engagement of the
firm, this will most likely create

A. Self-interest threat
B. Intimidation threat
C. Self-review threat
D. Familiarity threat

139. The firm, or a network firm, may provide an audit client that is not a listed entity with
accounting and bookkeeping services, including payroll services, of a routine or mechanical
nature, provided any self-review threat created is reduced to an acceptable level. Examples of
such services least likely include:

A. Recording transactions for which the audit client has determined or approved the
appropriate account classification.
B. Posting coded transactions to the audit client's general ledger.
C. Preparing financial statements based on information in the trial balance.
D. Determining and posting journal entries without obtaining the approval of the audit client.

140. The safeguards necessary to reduce the threat created by providing accounting and
bookkeeping services to an audit client that is not a listed entity to an acceptable level might
include the following, except:

A. Making arrangements so that such services are not performed by a member of the
assurance team.
B. Implementing policies and procedures to prohibit the individual providing such services
from making any managerial decisions on behalf of the audit client.
C. Requiring the source data for the accounting entries to be orignated by the assurance
team
D. Obtaining audit client approval for any proposed journal entries or other changes
affecting the financial statements.

141. The provision of accounting and bookkeeping senrs of a routine or mechanical nature to
divisions or subsidiariesice of listed audit clients would not be seen as impairing independence
with respect to the audit client provided that the following conditions are met, except:

A. The services do not involve the exercise of judgment.


B. The divisions or subsidiaries for which the service is provided are collectively immaterial
to the audit client.
C. The services provided are collectively immaterial to the division or subsidiary. D. The
fees to the firm, or network firm, from such services are collectively significant.

142. The provision of accounting and bookkeeping services to audit clients in emergency or
other unusual situations, when it is impractical for the audit client to make other arrangements,
would not be considered to pose an unacceptable threat to independence provided:

A. The firm, or network firm, does not assume any managerial role or make any managerial
decisions.
B. The audit client accepts responsibility for the results of the work.
C. Personnel providing the services are not members of the assurance team.
D. All of the given choices.

143. If the valuation services involves the valuation of matters material to the financial
statements and the valuation involves a significant degree of subjectivity, the self-review threat
created (choose the incorrect one)

A. Could not be reduced to an acceptable level by the application of any safeguard.


B. Could be reduced to an acceptable level by the application of safeguards.
C. Such valuation services should not be provided.
D. The assurance team should withdraw from the audit engagement, if the team opted to
perform the valuation services.

144. The following would generally create a significant threat to independence, except:

A. When a firm, or a network firm, performs a valuation service for an audit client for the
purpose of making a filing or return to a tax authority.
B. The firm provides formal taxation opinions and assistance in • the resolution of tax
disputes to an audit client.
C. The firm renders internal services involving an extension of the procedures required to
conduct an audit in accordance with Philippine Standards on Auditing to an audit client.
D. When a firm, or a network firm, provides assistance in the performance of a client's
internal audit activities or undertakes the outsourcing of some of the activities.

145. Which of the following may least likely create a self-review threat?

A. The lending of staff by a firm to the financial Statement audit client.


B. The firm provides internal audit services to the financial statement audit client. C. The
firm renders litigation support services to the financial statement audit.
C. Recruitment of senior manager for the financial statement audit client.

146. Which of the following is least likely considered to create a threat to independence?
A. The provision of services by a firm or network firm to an audit client which involve either
the design or the implementation of financial information technology systems that are
used to generate information forming part of a client's financial statements.
B. The provision of services in connection with the assessment, design and implementation
of internal accounting controls and risk management controls.
C. The lending of staff by a firm, or network firm, to an. audit client when the individual is in
a position to influence the preparation of a client's accounts or financial statements.
D. The provision of litigation support services to an audit client which include the estimation
of the possible outcome and thereby affects the amounts or disclosures to be reflected in
the financial statements.

147. The lending of staff by a firm to a financial statement audit client may be made only on the
understanding that the firm's personnel will not be involved in the following, except:

A. Assembling the annual financial statements based recorded transactions.


B. Making management decisions.
C. Approving or signing agreements or other similar documents.
D. Exercising discretionary authority to commit the client.

148. Which of the following is not a factor to evaluate the effect of litigation support services
rendered by a firm to an audit client?

A. The nature of the engagement.


B. The degree of subjectivity inherent in the matter concerned.
C. The materiality of the amount involved.
D. The manner of payment of professional fee on litigation service.

149. Legal services are defined as:

A. The making of assumptions with regard to future developments, the application of


certain methodologies and techniques, and the combination of both in order to compute
a certain value, or range of values, for an asset, a liability or for a business as a whole.
B. A broad range of services, including compliance, planning, provision of formal taxation
opinions and assistance in the resolution of tax disputes.
C. May include such activities as acting as an expert witness, calculating estimated
damages or other amounts that might become receivable or payable as the result of
litigation or other legal dispute, and assistance with document management and retrieval
in relation to a dispute or litigation.
D. Any services for which the person providing the services must either be admitted to
practice before the Courts of the jurisdiction in which such services are to be provided,
or have the required legal training to practice law.

150. The provision of legal services to financial statement auait clients most likely di edit a(an).

A. Familiarity threat
B. Self-interest threat
C. Advocacy threat
D. Intimidation threat
151. When the firm provides legal services to support a financial statement audit client in the
execution of corporate restructuring the threat created can be reduced to an acceptable level
provided that:

A. Members of the assurance team who are involved in providing the services are given
reduced participation in providing assurance service.
B. The advice provided was of the assurance team
C. The staff who makes ultimate decision is not a member of the
D. In relation to the advice provided, the audit client makes the ultimate decision.

152. Which of the following threats to independence can be eliminated or reduced to an


acceptable level?

A. Acting for an audit client in the resolution of a dispute or litigation in such circumstances
when the amounts involved are material in relation to the financial statements of the
audit client.
B. When a firm is asked to act in an advocacy role for an audit client in the resolution of a
dispute or litigation in circumstances when the amounts involved are not material to the
financial statements of the audit client.
C. The appointment of a partner or an employee of the firm or network firm as General
Counsel for legal affairs to an audit client.
D. None of them.

153. The recruitment of senior management for an such as those in a position to affect the
subject of the assurance engagement may least likely create:

A. Self-interest threat
B. Intimidation threat
C. Advocacy threat
D. Familiarity threat

154. Which of the following corporate finance services create advocacy or seif-review threats
cannot be reduced to an acceptable level?

A. Committing the assurance client to the terms of a transaction or consummating a


transaction on behalf of the client.
B. Assisting a client in developing corporate strategies.
C. Assisting in identifying or introducing a client to possible sources of capital that meet the
client specifications or criteria.
D. Providing structuring advice and assisting a client in analyzing the accounting effects of
proposed transactions.

155, Which of the following is not likely to create a threat to independence?

A. The total fees generated by an assurance client represent a large proportion of a firm's
total fees.
B. Fees due from an assurance client for professional services remain unpaid for a long
time.
C. A firm obtains an assurance engagement at a significantly lower fee level than that
charged by the predecessor firm, or quoted by other firms.
D. A court or other public authority is the one that established the fees.

156. A client company has not paid its 2008 audit fees. According to the Code of Professional
Conduct, for the auditor to be considered independent with respect to the 2009 audit, the 2008
audit fees must be paid before the

A. 2008 report is issued


B. 2009 report is issued
C. 2009 field work is started
D. 2010 field work is started

157. When a firm obtains an assurance engagement at a significantly lower professional fee
than that charged by the predecessor firm, or quoted by other firms, a(an):

A. threat to independence is not created.


B. intimidation threat is created.
C. advocacy threat is created.
D. self-interest treat is created.

158. Fees calculated on a predetermined basis relating to the outcome or result of a transaction
or the result of the work preformed.

A. Contingent fees
B. Flat sum fees
C. Retainer fees
D. Per diem fees

159. Which of the following is least likely to create a threat to independence?

A. The fees generated by the assurance client represent a large proportion of the revenue
of an individual partner.
B. The firm charges a contingent fee to an assurance client.
C. Accepting gifts or hospitality, the value of which is clearly insignificant, from an
assurance client.
D. When litigation takes place, or appears likely, between the firm or a member of the
assurance team and the assurance client.

160. Which of the following does not create a self-interest threat to independence?
A. An audit of an insurance company is engaged by the assurance client based upon the
instruction from the Office of Insurance Commission. The audit fee is contingent upon
the assessment by the Office of Insurance Commission of the liquidity of the company.
B. An audit fee on an assurance client that is outstanding for two years.
C. An assurance engagement with a fee significantly lower than the fee quoted by other
firm.
D. A litigation between the firm and the assurance client that relates to a prior assurance
engagement involving a breach of contract.

161. When litigation takes place between the firm and the assurance client, the firm and the
client management may be placed in adversarial positions and the firm may face a self-interest-
threat. Which of the following is least likely a factor in determining the Significance of the threat
created by this litigation?

A. The nature of the-assurance engagement.


B. Whether the litigation relates to a prior assurance engagement.
C. The materiality of the amount involved on litigation.
D. The likelihood of the firm winning the litigation.

162. When independence is threatened by litigation between the member of the assurance
team and the client management, the following safeguards that can reduce the effect to an
acceptable level may be applied, except:

A. Involve an additional professional accountant in the firm who is not a member of the
assurance team to review the work done.
B. Disclose to the audit committee, or others charged with governance, the extent and the
nature of the litigation.
C. Remove the particular member of the assurance team who is involved in litigation hum
cite engagement.
D. Submit a new engagement letter.

163. Which of the following threats to independence is least likely considered a result of the
firm's service of recruiting senior managers for an assurance client?

A. Self-interest threat
B. Familiarity threat
C. Intimidation threat
D. Self-review threat

164. Which of the following combination of threats to independence is most likely to occur as a
result of the provision of corporate finance services advice or assistance to an assurance client?

A. Advocacy and self-review threats


B. Self-review and familiarity threats
C. Familiarity and advocacy threats
D. Self-review and self-interest threats
165. Which of the following is not allowed to be included in a website of a firm of professional
accountants?

A. Names of partners/principals with their educational attainment.


B. Membership to any professional body.
C. Awards received.
D. Listings of the firm's clients.

166. The set of rules and regulations promulgated in 2004 for the "supervision, control and
regulation" of the practice of Accountancy in the Philippines.

A. Philippine Financial Reporting Standards.


B. The Code of Ethics for Professional Accountants.
C. Philippine Standards on Auditing.
D. The IRR of the Philippine Accountancy Act of 2004.

167. The objectives of the Philippine Accountancy Act of 2004 are the following except:

A. The standardization and regulation of accounting education.


B. Examination for registration of certified public accountants.
C. Supervision, control, and regulation of the practice of accountancy.
D. Integration of accountancy profession.

168. A document under seal issued to an individual by the Professional Regulation Commission
signifying that he has complied with all the legal and procedural requirements for such issuance
including the passing of the licensure examination for Certified Public Accountants.

A. Certificate of Accreditation
B. Professional Identification Card
C. Certificate of Registration
D. Professional Seal

169. The following is deemed a practice of accountancy, except:

A. Appointment to a position in the government that requires a CPA license as a


prerequisite.
B. Employment as budget officer in a local government unit regardless of the officer being a
holder of a CPA license or not.
C. Teaching professional subjects in a collegiate program leading to the degree of Bachelor
of Science in Accountancy.
D. Representing his clients before government agencies on tax and other matters related to
accounting.

170. The following statements relate to RA 9298. Which statement is true?


A. The Professional Regulation Commission has the authority to remove any member of
the Board of Accountancy for negligence, incompetence, or any other just cause.
B. Insanity is not a ground for proceeding against a CPA.
C. A person shall be considered to be in the professional practice of accounting if, as an
officer in a private enterprise, he makes decisions requiring professional accounting
knowledge.
D. After three years, subject to certain conditions, the Board of Accountancy may order the
reinstatement of a CPA whose certificate of registration has been revoked.

171. The president of the Philippines appoints the members of the Board of Accountancy based
on the recommendation submitted to the office of the president. Which of the following is an
incorrect statement about the submission of nominations?

A. The Accredited National Professional Organization of CPAs shall submit the names of its
nominees to the PRC not later than 60 days prior to the expiry of the term of an
incumbent chairman or member.
B. There should be an adequate documentation to show the qualifications and primary field
of professional activity of each nominee
C. The Accredited National Professional Organization of CPAs shall submit the names of its
nominees to the PRC not later than days prior to the expiry of the term of an incumbent
chairrn or member.
D. If the Accredited National Professional Organization of CPAs failsto submit its own
nominee(s) to the PRC within the prescribed period, the PRC, in consultation with the
Board of Accountancy shall submit to the president of the Philippines a list of three
nominees for each position.

172. Which of the following is not a qualification of a member of the Board of Accountancy?

A. He must be a natural-born citizen and a citizen of the Philippines.


B. He must not be a director or officer of the Accredited National. Professional Organization
of CPAs at the time of his appointment.
C. He must be of good moral character and must not have been convicted of crimes
involving moral turpitude.
D. He must be a duly registered certified public accountant with at least ten years of
experience in public accounting.

173. The following statements relate to the term of office of the chairman and members of the
Board of Accountancy. Which of them is incorrect?

A. The chairman and members of the Board of Accountancy shall hold office for a term of
three years.
B. Any vacancy occurring within the term of a member shall be filled up for the unexpired
portion of the term only.
C. Appointment to fill up an expired term is not to be considered as a complete term.
D. The Board of Accountancy member who has served two successive complete terms as
chairman or member shall be eligible for reappointment until the lapse of three years.

174. No person shall serve the Professional Regulatory Board of Accountancy for more than
A. 3 years
B. 6 years
C. 9 years
D. 12 years

175. Which of the following is not a function of the Board of Accountancy as specified in the
Philippine Accountancy Act of 2004?

A. To investigate violations of the Accountancy Law and the rules and regulations
promulgated therewith.
B. To look from time to time into the conditions affecting the practice of the accountancy
profession.
C. To create and direct accrediting agencies that are entrusted the functions of reviewing
higher educational institutions' policies and practices leading to
accreditation/reaccreditation of BSA program.
D. To determine and prescribe minimum requirements leading to the admission of
candidates to the CPA licensure examination.

176. The following are represented both to the Financial Reporting Standards Council (FRSC)
and Auditing and Assurance Standards Council (AASC), except:

A. Bangko Sentral ng Pilipinas


B. Securities and Exchange Commission
C. Bureau of Internal Revenue
D. Board of Accountancy

177. All of the following are represented to the Financial Reporting Standards Council, except:

A. Commission on Higher Education


B. Board of Accountancy
C. Securities and Exchange Commission
D. Bureau of Internal Revenue

178. The Financial Reporting Standards Council which is the accounting standards setting body
is composed of a chair and:

A. Fourteen members
B. Fifteen members
C. Sixteen members
D. Seventeen members

179. The chairman and the members of both Financial Repo standards Council and Auditing
and Assurance Standard: Council have a renewable term of:

A. 4 years
B. 2 years
C. 3 years
D. 5 years

180. Which of the following is not a requisite in applying for the CPA licensure examinations?

A. Natural-born citizen of the Philippines


B. Good moral character
C. Holder of the degree of Bachelor of Science in Accountancy
D. Has not been convicted of any criminal offers e.2 involving moral turpitude

181. Which of the following is incorrect?

A. Candidates who fails to obtain a general average of 75% but obtains a rating of at least
75% in at least four subjects shall receive a conditional credit for the subjects passed.
B. To successfully pass the licensure examination, the candidates should obtain a general
weighted average of at least 75% with no rating lower than 65% in any subject.
C. Conditional candidates shall take an examination in the conditional subject(s) within two
years from the preceding examination.
D. Candidates who failed in three complete examinations must enroll in refresher course
consisting of twenty-four units of the subjects given in the licensure examination.

182. Which of the following is one of the reasons for not issuing a certificate of registration to a
successful examinee? The individual:

A. Is of unsound mind.
B. Had been guilty of immoral and dishonorable conduct.
C. Had been convicted by a court of a criminal offense involving moral turpitude. D. All of
the given choices.

183. A CPA whose certificate of registration has been revoked:

A. Can no longer be reinstated.


B. Is automatically reinstated as a CPA by the PRC after two years if he has acted in an
exemplary manner.
C. May be reinstated by the Professional Regulation Commission after two years if he has
acted in an exemplary manner.
D. May be reinstated as a CPA by the Board of Accountancy after two year if he has acted
in an exemplary manner.

184. The Philippine Accountancy Act of 2004 provides that all working papers made during an
audit shall be the property of the auditor. These working papers shall include the following,
except:

A. Working papers prepared by the CPA and his staff.


B. Analysis and schedule prepared and submitted to the auditor by his client's staff.
C. Excerpts or copies of documents furnished to the auditor.
D. Any report submitted by the auditor to his client.

185. Who are required to apply for accreditation with the Professional Regulation Commission if
the applicant is a partnership of Professional Accountants?

A. Managing partner only


B. All partners only
C. Partners and staff members
D. Partners, principals, and staff members

186. Which of the following is not included in the seal of a professional accountant?

A. Tax identification number


B. Name of the professional accountant
C. Title of the profession
D. Registration number

187. The body mandated by law to promulgate rules and regulations affecting the practice of
Accountancy.

A. Professional Regulation Commission


B. Philippine Institute of Certified Public Accountants
C. Professional Regulatory Board of Accountancy
D. Commission on Higher Education

188. Individual CPAs, Finns or Partnerships of CPAs, including partners and staff members
thereof shall register with the 130A and the PRC. If the accreditation of Alano and Co. CPAs,
was renewed on September 30, 2008, the next renewal must be on or before:

A. September 30, 2010


B. September 30, 2011
C. December 31, 2010
D. December 31, 2011

189. How many CPE credit units must be accumulate-1 by a registered accounting professional
within the 3-year period?

A. 15 credit units
B. 45 credit units
C. 60 credit units
D. 90 credit units
190. The APO shall renew its Certificate of Accreditation once every how many years after the
date of the Resolution granting the petition for re-accreditation and the issuance of the said
certificate upon submission of the requirements?

A. 2 years
B. 3 years
C. 4 years
D. 6 years

191. Engagement letters are widely used in practice for professional engagements of all types.
The primary purpose of the engagement letter is to

A. remind management of its primary responsibility over the financial statements. B. satisfy
the requirements of the Code of Professional Conduct for CPAs.
B. provide a starting point for the auditor's preparation of the peliwirtaly audit program.
C. provide a written record of the agreement with the client as to the services to be
provided.

192. The accuracy of information included in the footnotes that accompany the audited financial
statements of a company whose shares are traded on a stock exchange is the primary
responsibility of the:

A. stock exchange officials.


B. company's management.
C. independent auditor.
D. Securities and Exchange Commission.

193. Which of the following is not likely a quality control procedure on consultation?

A. Identifies areas and specialized situations where consultation is required and


encourages personnel to consult with or in use authoritative sources on other complex
matters.
B. Designates individuals as specialists to serve as authoritative sources and define their
authority in consultative situations.
C. Assigns an appropriate person or persons to be responsible for assigning personnel to
audits.
D. Specifies the extent of documentation to be provided for the result of consultation in
those areas and specialized situations where consulthtion is required.

194. According to Philippine Standards on Auditing, because there are inherent limitations in an
audit that affect the auditor's ability to detect material misstatements, the auditor is:

A. neither a guarantor nor an insurer of financial statements.


B. a guarantor but not an insurer of the statements.
C. an insurer but not a guarantor of the statements.
D. both a guarantor and an insurer of the financial statements.
195. The objective of an ordinary examination by the independent auditor is the expression of
an opinion on the:

A. accuracy of the financial statements.


B. balance sheet and income statement.
C. fairness of the presentation of the financial statements.
D. quality of the decision process of the management.

196. When a CPA expresses an opinion on the financial statements, his responsibilities extend
to

A. the underlying wisdom of the client's management decision.


B. active participation in the implementation of the advice given to the client.
C. an ongoing responsibility for the client's solvency.
D. whether the results of the client's operating decisions are fairly presented in the financial
statements.

197. The working papers prepared by a CPA in connecction with an audit engagement are
owned by the CPA, subject to certain limitations. The rationale for this rule is to

A. protect the working papers from being subpoenaed.


B. provide the basis for excluding admission of the working papers as evidence because of
the privileged communication rule.
C. provide the CPA with evidence and documentation which may be helpful in the event of
a lawsuit.
D. establish a continuity of relationship with the client whereby indiscriminate replacement
of CPAs is discouraged.

198. The responsibility for adopting sound accounting policies, maintaining adequate internal
control, and making fair representations in the financial statements rests

A. with the management.


B. with the independent auditor.
C. equally with management and the auditor.
D. with the internal audit department.

199. Fraudulent financial reporting is often called:

A. Management fraud
B. Defalcation
C. Theft of assets
D. Employee Fraud
200. The ordinary examination of financial statements is not primarily designed to disclose
defalcations and other irregularities although their discovery may result. Normal audit
procedures are more likely to detect a fraud arising from

A. collusion on the part of several employees.


B. failure to record cash receipts for services rendered.
C. forgeries on company checks.
D. theft of inventories.

201. The factor that distinguishes an error from an irregularity is

A. materiality.
B. intent.
C. whether it is peso amount or a process.
D. whether it is a caused by the auditor or the client.

202. Audit standards require an auditor to:

A. Perform procedures that are designed to detect all instances of fraud.


B. Provide reasonable assurance that the financial statements are not materially misstated.
C. Issue an unqualified opinion only when the auditor is satisfied that no instances of fraud
have occurred.
D. Design the audit program to meet financial statement users' expectations concerning
fraud.

203. If specific information comes to an auditor's attention that implies the existence of possible
noncompliance with laws and regulations that could have a material, but indirect effect on the
financial statements, the auditor should next

A. apply audit procedures specifically directed to ascertaining whether a noncompliance


with laws and regulations has occurred.
B. seek the advice of an informed expert qualified to practice law as to possible contingent
liabilities.
C. report the matter to an appropriate level of management at least one level above those
involved.
D. discuss the evidence with the client's audit committee, or others with equivalent
authority.

204. A principal purpose of a letter of representation from management is to

A. serve as an introduction to company personnel and an authorization to examine the


records.
B. discharge the auditor from legal liability for his examination.
C. confirm in writing management's approval of limitations the scope of the audit. D. remind
management of its primary responsibility for fin statements.
205. The auditor should not assume that management is but the possibility of dishonesty must
be considered." This is an example of

A. expectation gap.
B. an attitude of professional skepticism.
C. due diligence.
D. an ethical requirement.

206. Which of the following statements is true?

A. It is usually easier for the auditor to uncover irregularities than errors.


B. It is usually easier for the auditor to uncover errors than irregularities.
C. It is usually equally difficult for the auditor to uncover errors or irregularities.
D. It is usually, none of the given statements is true.

207. Should the auditor uncover circumstances that may cause suspicions of management
fraud, the auditor must

A. issue an adverse opinion.


B. issue a disclaimer of opinion.
C. evaluate their implications and consider the need to modify audit evidence.
D. withdraw from engagement.

208. Generally, the decision to notify parties outside te client‘s organization regarding a
noncompliance with laws and regulations is the responsibility of the

A. independent auditor.
B. management.
A. c. outside legal counsel.
C. internal auditors.

209. An audit made in accordance with Philippine Standards on Auditing generally should

A. be expected to provide assurance that noncompliance with laws and regulations will be
detected if the internal control is effective.
B. be relied upon to disclose indirect-effect noncompliance with laws and regulations.
C. encompass a plan to search actively for noncompliance with laws and regulations which
relate to operating aspects.
D. not be relied upon to provide assurance that all noncompliance with laws and regulations
will be detected.

210. An auditor who believes that a material irregularity may exist should initially

A. discuss the matter with those believed to be involved in the perpetration of material
irregularity.
B. discuss the matter with a higher level of management.
C. withdraw from the engagement.
D. consult legal counsel.

211. When management refuses to disclose in the financial statements noncompliance to laws
and regulations which are identified by the independent auditor, the CPA may be charged with
unethical conduct for

A. withdrawing from the engagement.


B. issuing a disclaimer of opinion.
C. failure to uncover the noncompliance to laws and regulations during the prior audits.
D. reporting these activities to the audit committee.

212. In discovering material management fraud and an equally material error, the audit plan

A. should be expected to provide the same degree of assurance


B. cannot be expected to provide the same degree of assurance.
C. provide no assurance of detecting either.
D. should provide complete assurance of detection.

213. An auditor who finds that the client has committed noncompliance with laws and
regulations would most likely withdraw from the engagement when the

A. noncompliance with laws and regulations affects the auditor's ability to rely on
management representations.
B. noncompliance with laws and regulations has material financial statement implications.
C. noncompliance with laws and regulations has received widespread publicity.
D. auditor cannot reasonably estimate the effect of the noncompliance with laws and
regulations on the financial statements.

214. When the auditor knows that a noncompliance with laws and regulations has occurred, the
auditor must

A. issue an adverse opinion.


B. withdraw from the engagement.
C. consider the effects on the financial statements, including the adequacy of disclosure.
D. report it to the proper government authorities.

215. When an independent auditor's examination of financial statements discloses special


circumstances that make the auditor suspects that fraud may exist, the auditor's initial course of
action should be to

A. recommend that the client pursue the suspected fraud to a conclusion that is agreeable
to the auditor.
B. extend normal audit procedures in an attempt to detect the full extent of the suspected
fraud.
C. reach an understanding with the proper client representative as to whether the auditor or
the client is to make th e investigation necessary to determine if a fraud has in Met
occurred.
D. determine whether the fraud, if .in fact it does exist, might.be of such a magnitude as to
affect the auditor's report on the financial statements
QUIZZERS

1. A procedure n which a quality control partner periodically tests the application of quality
control procedures is most directly related to which quality control element?

A. Engagement performance
B. Independence, integrity, and objectivity
C. Monitoring
D. Personnel management

2 The work of each assistant needs to be reviewed by personnel of at least equal competence.
Which of the following is not one of the objectives of this requirement?

A. The conclusions expressed are consistent with the result of the work performed and
support the opinion.
B. The work performed and the results obtained have been adequately documented.
C. The audit objectives have been achieved.
D. All available evidences have been obtained, evaluated and documented.

3. Which of the following acts is prohibited by the Code of Professional Ethics for CPAs?

A. The use of a firm name which includes the name of a retired partner.
B. An announcement in a newspaper of the opening of a public accounting office. C.
Engaging in civic activities during business hours.
C. Accepting an engagement or employment which one cannot reasonably expect to
complete or discharge with professional competence.

4. Which of the following is a violation of the code of professional ethics for certified public
accountants?

A. A CPA permits his name to be used in a client's advertising as having verified financial
data and/or statistical facts with respect to client‘s products.
B. Based on information obtained in an audit, a CPA reports a noncompliance with laws
and regulations of his client to government authorities.
C. Three years after a partner has retired, the remaining partners continue to practice
under a firm name that includes the name of the retired partner. The retired partner has
severed all connections with the CPA firm.
D. A CPA running for public office uses the professional designation "CPA" after his name
on posters employed in connection with his election campaign.

5. Which of the following is incorrect regarding the professional accountants' tax practice?

A. A professional accountant rendering professional tax services is entitled to put forward


the best position in favor of a client, or an employer.
B. Doubt may be resolved in favor of the client or the employer if there is a reasonable
support for the position.
C. A professional accountant may hold out to a client or an employer the assurance that the
tax return prepared and the tax advice offered by him are beyond challenge.
D. Professional accountants should ensure that the client or the employer is aware of the
limitations attaching to tax advice and services so that they do not misinterpret an
expression-of opinion as an assertion of fact.

6. Which of the following is least likely an application of maintaining an attitude of professional


skepticism?

A. The auditor does not consider representations from management as substitute for
obtaining sufficient appropriate audit evidence to be able to draw reasonable
conclusions on which to base the audit opinion.
B. In planning and performing an audit, the auditor assumes that management is dishonest.
C. The auditor is alert to audit evidence that contradicts or brings into question the reliability
of documents or management representations.
D. The auditor makes a critical assessment, with a questioning mind, of the validity of audit
evidence obtained.

7. Prior to beginning the field work on a new audit engagement in which a CPA does not
possess expertise in the industry in which the client operates, the CPA should

A. reduce audit risk by lowering the preliminary levels of materiality. special substantive
tests to compensate for the lack of
B. design industry expertise.
C. engage financial experts who are familiar with the nature of the industry.
D. obtain a knowledge of matters that relates to the nature of the entity's business.

8. Which of the following statements is true when the CPA has been engaged to do an
attestation engagement?

A. The CPA firm is engaged and paid by the client; therefore, the firm has primary
responsibility to be an advocate for the client.
B. The CPA firm is engaged and paid by the client, but the primary beneficiaries of the audit
are the statement users.
C. Should a situation arise where there is no convincing authoritative standard available,
and there is a choice of actions, which could impact client's financial statements either
positively or negatively, the CPA is free to endorse the choice which is best in the client's
interest.
D. As long as CPA firms are competent, it is not required that they remain unbiased.

9. One difference between auditors and other professionals is that most professionals

A. need not be concerned about remaining independent.


B. do not have requirements for continuing education beyond the baccalaureate program.
C. do not have to pass a rigorous examination.
D. are not expected to act in public interest.
10. Independence of a CPA with respect to a client is not impaired if

A. The CPA has a loan to an officer of the client.


B. The 1 CPA has an immaterial direct financial interest in the client
C. The CPA is a trustee for the client's pension plan.
D. The CPA has an immaterial joint, closely held tin investment with the client.

11. For which of the following services is a CPA professional not required to be independent?

A. Tax returns preparation


B. Audits of historical financial statements
C. Review engagement
D. Examination of a forecast

12. Which of the following will impair the independence of a CPA in public practice?

A. He has his name and address listed on a one-page section of the telephone book.
B. He obtained a loan from a bank under the normal lending procedures, terms and
requirements of the bank.
C. He holds one share of the client's capital stock.
D. He failed to disclose a client's departure from GAAP.

13. When CPAs are able to maintain an independent attitude in fulfilling their responsibility, it is
referred to as independence in

A. fact.
B. appearance.
C. conduct.
D. total.

14. When the users of financial statements have confidence in the independence of the CPA, it
is referred to as in independence in

A. fact.
B. appearance.
C. conduct.
D. total.

15. Which of the following statements is incorrect? CPAs lose their independence if they:

A. acquire any direct financial interest in a client.


B. have a material direct financial interest in a client.
C. acquire any indirect financial interest in a client.
D. acquire a material indirect financial interest in a client.
16. When determining whether independence is impaired because of an ownership interest in
client company, materiality will affect Whether ownership is a violation of rule of independence

A. in all circumstances.
B. only for direct ownership.
C. only for indirect ownership.
D. under no circumstances.

17. A successor auditor is required to communicate with the previous auditor. The primary
concern in this communication is

A. information which will help the successor auditor determine whether the client
management has integrity.
B. to learn about client by examining predecessor's working papers.
C. to enable successor auditor to perform a more efficient audit.
D. to save successor auditor time and money in gathering data.

18. When a. CPA firm is requested to provide a written or oral opinion on the application of
accounting principles or the type of audit opinion that would be issued for a specific or
hypothetical transaction relating to an audit client of another CPA firm, primary among the
requirements set forth is that

A. client is entitled to confidentiality, so the consulting CPA firm is forbidden from


communicating with the CPA firm which does the audit.
B. the consulted CPA firm should communicate with the entity's existing auditors to
ascertain all the available facts relevant to forming a professional judgment on the
matters the firm has been requested to report on.
C. client is entitled to confidentiality; so the CPA firm which does the audit, should refuse to
share any information with the consulting CPA firm under any circumstances.
D. client is not entitled to confidentiality under these circumstances, so the existing auditors
should share an information with the consulting CPA firm.

19. A professional accountant has a professional duty or right to disclose confidential


information in each of the following, except:

A. To comply with technical standards and ethics requirements.


B. To disclose to the Bureau of Internal Revenue any fraudulent scheme committed by the
client on payment of income tax.
C. To comply with the quality review of a member body or professional body.
D. To respond to an inquiry or investigation by a member body or regulatory body.

20. Which of the following best describes the paccing of confidential information from a client to
its auditor? The information:

A. Should in no circumstances be conveyed to third parties.


B. Is not legally protected and can be subpoenaed by a competent court.
C. Can only be released for peer reviews after receiving permission from the client.
D. Should be conveyed to the public if it affects the "correctness" of the financial
statements.

21. The CPA must not subordinate his or her professional judgment to that of others in every

A. engagement.
B. audit engagement.
C. engagement except tax services.
D. engagement except management advisory services.

22. Which of the following is an indication of lack of objectivity of an auditor?

A. The auditor believes that accounts receivable may not be collectible, but accepts
management's opinion without an independent evaluation.
B. In preparing client:s. tax return, the CPA encourages client to take a deduction which tile
CPA believes is valid, but for which there is some but not complete support.
C. Both are violations.
D. Neither would be a violation

23. Several months after an unqualified audit report was issued, the auditor discovers that the
financial statements were materially misstated. The client's chief executive officer agrees that
the statements are misstated, but refuses to issue a correction, and claims that "confidentiality"
prevents the CPA from informing anyone.

A. CEO is correct; the auditor must maintain confidentiality.


B. CEO is wrong, but since auditor's report is issued, it is too late to retract.
C. CEO is wrong; and the auditor has an obligation to issue a revised correct audit report,
even if CEO will not revise and correct the financial statements.
D. CEO is correct, but to be ethically correct the auditor should violate the confidentiality
rule and disclose the error.

24. A member in public practice may perform for a contingent fee any professional services for a
client for whom the member or member's firm performs

A. an audit.
B. a review.
C. a compilation used only by management.
D. an audit of prospective financial information.

25. Which one of the following contingent fee is allowed?

A. All services performed by a CPA firm.


B. Non-attestation services.
C. Non-attestation services, unless the CPA firm was also performing attestation services
for the same client.
D. Attestation services.

26. Solicitation consists of the various means that CPA firms use to engage new clients. Which
one of the following would not be an example of solicitation?

A. Accepting new clients that approach the firm.


B. Taking prospective clients to lunch.
C. Offering seminars on current tax law changes clients.
D. Advertisements in the yellow pages.

27. If requested to perform a review engagement for a nonpublic entity in which an accountant
has an immaterial direct financial interest, the accountant is

A. independent because the financial interest is immaterial and therefore, may issue a
review report.
B. not independent and, therefore, may not he associated with the financial statements.
C. not independent and, therefore, may not issue a review report.
D. not independent and, therefore, may issue c review report, but may not issue an
auditor's opinion.

28. Which of the following most completely describes how independence has been defined by
the CPA profession?

A. Performing an audit from the viewpoint of the public.


B. Avoiding the appearance of significant interests in the affairs of an audit, client.
C. Possessing the ability to act with integrity and objectivity.
D. Possessing the ability to act professionally and accordance with a professional code of
ethics.

29. To emphasize auditor independence from management, many corporations follow the
practice of

A. appointing a partner of the CPA firm to the corporation's audit committee.


B. establishing a policy of discouraging social contact between employees of the
corporation and the staff of the independent auditor.
C. requesting that a representative of the independent auditor be on hand at the annual
stockholders' meeting.
D. having the independent auditor report to an audit committee of outside members of the
board of directors.

30. In determining independence with respect to any engagement, the ultimate decision as to
whether or not the auditor is independent must be made by the

A. auditor.
B. client.
C. audit committee.
D. public.

31. When a CPA who is not independent is associated with financial statements, he would be
precluded from expressing an opinion because any audit engagement, the auditor is
independent

A. the public would be aware of his lack of independence and would place little or no faith
on his opinion.
B. he would place himself in the position of suffering an adverse decision in a possible
liability suit.
C. he would be in the position of auditing his own work.
D. any auditing procedures he might perform would not be in accordance with generally
accepted auditing standards.

32. Which of the following statements best describes why the profession of certified public
accountants has deemed it essential to promulgate a code of ethics and to establish a
mechanism for enforcing observance of the code?

A. A distinguishing mark of a profession is its acceptance of responsibility to the public.


B. A prerequisite to success is the establishment of an ethical code that stresses primarily
the professional's responsibility to clients and colleagues.
C. The law requires an establishment of a code of ethics.
D. An essential means of self-protection for the profession is the establishment of flexible
ethical standards by the profession.

33. In which of the following circumstances would a CPA be bound by ethics to refrain from
disclosing any confidential information obtained during the course of a professional
engagement?

A. The CPA is issued a summons enforceable by a court that orders the CPA to present
confidential information.
B. A major stockholder of a client company seeks accounting information from the CPA
after the management declined to disclose the requested information.
C. Confidential client information is made available as part of a quality review of the CPA's
practice by a peer review team authorized by the PICPA.
D. An inquiry by a in disciplinary body of PICPA requests confidential client information.

34. Which of the following best describes why publicly-traded corporations follow the practice of
having the outside auditor appointed by the board of directors or elected by the stockholders?

A. To comply with the regulations of the FRSC.


B. To emphasize the auditor's indeper c. once from the management of the corporation.
C. To encourage a policy of rotation of the independent auditors.
D. To provide the corporate owners with an opportunity to voice their opinion concerning
the quality of the auditing firm selected by the directors.
35. A violation of the ethical standards would most likely have occurred when a CPA

A. made arrangement with a bank to collect notes issued by a client in payment of fees
due.
B. joined an accounting firm made up of three non-CPA practitioners.
C. issued an unqualified opinion on the 2009 financial statements when fees for the 2008
audit were unpaid.
D. purchased a bookkeeping firm's practice of monthly write-ups for a percentage of fees
received over a three-year period.

36. The concept of materiality would be least important to an auditor when considering the

A. decision whether to use positive or negative confirmations of accounts receivable.


B. adequacy of disclosure of a client's noncompliance with laws and regulations.
C. discovery of weaknesses in a client's internal control structure.
D. effects of a direct financial interest in the client upon the CPA's independence.

37. Which of the following is a violation of Confidentiality rule of the 37. code of Professional
Conduct?

A. The CPA, in response to a court subpoena, submits auditor-preparedworking papers as


evidence of possible noncompliance with laws and regulations perpetrated by the client.
B. The CPA discloses to the board of directors a scheme concocted by top management to
intentionally inflate earnings.
C. The CPA warns Client B as to the inadvisability of acquiring Client A. The CPA bases
this warning on knowledge of Client A's financial condition and a belief that the
management of Client A lacks integrity. This knowledge was obtained by the CPA as a
result of auditing Client A during the past several years.
D. The CPA, when questioned in court, admits of having a knowledge of certain
noncompliance with laws and regulations perpetrated by the client.

38. An auditor who accepts an audit engagement and does not possess the industry expertise
of the business entity, should

A. engage financial experts familiar with the nature of the business entity.
B. obtain a knowledge of matters that relates to the nature of the entity's business.
C. refer a substantial portion of the audit to another CPA who will act as the principal
auditor.
D. first inform management that an unqualified opinion cannot be issued.

39. A CPA, while performing an audit, strives to achieve independence in appearance in order
to

A. reduce risk and liability.


B. comply with the generally accepted standards of fieldwork.
C. become independent in fact.
D. maintain public confidence in the profession

40. In which of the following instances would the independence of the CPA not be considered to
be impaired? The CPA has been retained as the auditor of a brokerage firm

A. which owes the CPA audit fees for more than one year.
B. in which the CPA has a large active margin account.
C. in which the CPA's brother is the controller.
D. which owes the CPA audit fees for services in the current year and has just filed a
petition for bankruptcy.

41. Which of the following fee arrangements is in violation of the Code of Professional Conduct?

A. A fee based on whether the CPA's report on the client's financial statements results in
the approval of a bank loan.
B. A fee based on the outcome of a bankruptcy proceeding.
C. A fee based on the nature of the service rendered and the CPA's particular expertise
instead of the actual time spent on the engagement.
D. A fee based on the fee charged by the prior auditor.

42. The auditor is not liable to his client for

A. negligence.
B. bad faith.
C. dishonesty.
D. errors of judgment.

43. When the auditor issues an erroneous opinion as a consequence of an underlying failure to
comply with the requirements of generally accepted auditing standards, it results to

A. business failure.
B. audit failure.
C. audit risk.
D. all of them

44. The reason why an auditor accumulates evidence is to

A. defend himself in the event of a lawsuit.


B. justify the conclusions he has otherwise reached.
C. satisfy the requirements of the Bureau of Internal Revenue.
D. enable him to reach conclusions about the fairness of the financial statements and issue
an appropriate audit report.
45. The auditor gives an audit opinion on the fair presentation of the financial statements and
associates his or her name with them when, on the basis of adequate evidence, the auditor
concludes that the financial statements are unlikely to mislead

A. a prudent user.
B. management.
C. the reader.
D. investors.

46. When preparing the financial statements, it is acceptable .for the auditor to prepare

A. the statements for the client.


B. the footnotes for the client.
C. a draft of the statements for the client.
D. a draft of the statements and footnotes for the client.

47. Which of the following statements best describes the auditor's responsibility regarding the
detection of material errors and frauds?

A. The auditor is responsible for the failure to detect material errors and frauds only when
such failure results from the misapplication of generally accepted accounting principles.
B. The audit should be designed to provide reasonable assurance that material error and
frauds are detected.
C. The auditor is responsible for the failure to detect material errors and frauds only when
the auditor fails to confirm receivables or observe inventories.
D. Extended auditing procedures are required to detect unrecorded transactions even if
there is no evidence that material errors and frauds may exist.

48. The auditor has considerable responsibility for notifying users as to whether or not the
financial statements are properly stated. This imposes upon the auditor a duty to

A. be an insurer of the fairness of the presentation of the financial statements.


B. be a guarantor of the fairness in the statements.
C. be equally responsible with management for the preparation of the financial statements.
D. provide reasonable assurance that material misstatements will be detected.

49. Which of the following statements best distinguishes ordinary negligence from gross
negligence?

A. Failure to detect material errors, whether internal control is strong or weak, suggests
gross negligence.
B. Failure to exercise reasonable care denotes ordinary negligence, whereas failure to
exercise minimal care indicates gross negligence.
C. Gross negligence is most probable when the auditor fails to detect errors that occurred
under conditions of strong internal control.
D. The more material the undetected error is, the greater the likelihood of ordinary
negligence being committed.
50. The auditor's responsibility for failure to detect fraud arises

A. when such failure clearly results from non-compliance to generally accepted auditing
standards.
B. whenever the amounts involved are material.
C. only when the examination was specifically designed to detect fraud.
D. only when such failure clearly results from negligence so gross as to sustain an
inference of fraud on the part of the auditor.

51. Which of the following statements is correct concerning the auditor‘s responsibility with
respect to noncompliance with laws and regulations? An auditor must design tests to:

A. obtain reasonable assurance of detecting material direct-effect,noncompliance with laws


and regulations.
B. detect both immaterial and material direct-effect noncompliance with laws and
regulations.
C. detect both direct-effect and indirect-effect noncompliance with laws and regulations.
D. detect both material direct-effect and material indirect-effect noncompliance with laws
and regulations.

52. Most accounting and auditing professionals agree that when an . audit has failed to uncover
material misstatements, and the wrong type of audit opinion is issued, the audit firm.

A. has failed to follow Philippine standards on auditing (PSAs).


B. deserves to lose the lawsuit.
C. should be asked to defend the quality of the audit.
D. should not be held responsible for the financial loss suffered by others.

53. What is the independent auditor's responsibility prior to the completion of fieldwork when he
believes that a material fraud may have occurred?

A. Notify the appropriate law enforcement authority.


B. Investigate the persons involved, the nature of the fraud, and the amounts involved.
C. Reach an understanding with the appropriate client representatives as to the desired
nature and extent of subsequent audit work.
D. Continue to perform normal audit procedures and write the audit report in such a way to
disclose adequately the suspicion of material fraud.

54. The risk that an audit will fail to uncover a material misstatement is eliminated

A. if client has good internal control.


B. if client follows generally accepted accounting principles.
C. when the auditor has complied with genrally accepted auditing standards.
D. under no circumstances.
55. The auditor's evaluation of the likelihood of material employee fraud is normally done initially
as a part of

A. the assessment of whether to accept the audit engagement.


B. understanding the entity's internal control structure.
C. the tests of controls.
D. the tests of transactions.

56. A CPA establishes quality control policies and procedures for deciding whether to accept a
new client or continue to perform services for a current client. The primary purpose for
establishing such policies and procedures is to

A. enable the auditor to attest to the integrity or reliability of a client.


B. comply with the quality control standard established by regulatory bodies.
C. lessen the exposure to litigation resulting from failure to detect material misstatements
due to irregularities in client's financial statements.
D. minimize the likelihood of association with clients whose management lacks integrity.

57. In which circumstance is a CPA firm's independence most likely to be impaired?

A. A member of the engagement team has a close relative who is a receptionist for the
client.
B. The father of the audit senior holds a material financial interest in the client of which the
senior is unaware.
C. The spouse of a member of the audit team has an immaterial common stock investment
in the audit client.
D. The partner in charge of the office's compensation is affected by office profitability, a
portion of which arises from this audit.

58. While performing services for their clients, professionals have always had a duty to provide
a level of care which is

A. reasonable.
B. greater than average.
C. superior
D. guaranteed to be free from error.

59. The existence of extreme or unusual negligence, even though 0. there was no intent to
deceive or do harm, is a(n)

A. fraud.
B. gross fraud.
C. constructive fraud.
D. ordinary fraud.

60. The failure of the auditor to meet generally accepted auditing standards is
A. an accepted practice.
B. a suggestion of negligence.
C. an evidence of negligence.
D. tantamount to criminal behavior.

61. Which of the following statement(s) is (are) true?

A. Gross negligence may constitute constructive fraud.


B. Constructive fraud is also termed recklessness.
C. Fraud requires the intent to deceive.
D. All the responses are true.

62. Which of the following, if present, would support a finding of constructive fraud on the part of
a CPA?

A. Privity of contract
B. Intent to deceive
C. Reckless disregard
D. Ordinary negligence

63. In rare cases auditors have been held liable for criminal acts. A criminal conviction against
an auditor can result only when it is demonstrated that the auditor

A. was negligent.
B. was grossly negligent.
C. intended to deceive or harm others.
D. caused financial loss to an innocent third party.

64. The principal issue to be resolved in cases involving alleged negligence is usually

A. the amount of the damages suffered by the users of the financial statements.
B. whether to impose punitive damages on defendant.
C. the level of care required to be exercised.
D. whether defendant was involved in fraud.

65. "Absence of reasonable care that can be expected of a person in a set of circumstances" is
the description of

A. ordinary negligence.
B. constructive fraud.
C. gross negligence.
D. fraud.

66. A CPA firm is considered independent when it performs which of the following services for a
publicly-traded audit client?
A. Serving as a member of the client's board of directors.
B. Determining which accounting policies will be adopted by the client.
C. Accounting information system design and implementation.
D. Tax return preparation as approved by the board of directors.

67. The limitation of an auditor's liability under contract law is known as

A. privity of contract.
B. contributory liability.
C. statutory liability.
D. common law liability.

68. As a consequence of his failure to adhere to generally accepted auditing standards in the
course of his examination of the s Corporation, Herman, CPA, did not detect the embezzlement
of a material amount of funds by the company's controller. As a matter of common law, to what
extent would Herman be liable to Leis Corporation for losses attributable to the theft?

A. He would have no liability, since the ordinary examination A' cannot be relied upon to
detect defalcations.
A. B.He would have no liability because privity of contract is lacking.
B. He would be liable for losses attributable to his negligence.
C. He would be liable only if it could be proven that he was grossly negligent.

69. In connection with the examination of financial statements, an independent auditor could be
responsible for failure to detect a material fraud if

A. statistical sampling techniques were not used on the audit engagement.


B. the auditor planned the work in a hasty and inefficient manner.
C. accountants performing important parts of the work failed to discover a close relationship
between the treasurer and the cashier.
D. the fraud was perpetrated by one client employee, who circumvented the existing
internal control.

70. A CPA is criminally liable if he

A. refuses to turn over the schedules or working papers prepared by the client staff to the
client.
B. performs an audit in a negligent manner.
C. intentionally allows an omission of a material fact required to be stated in a financial
statement.
D. was not able to submit the audited financial statements on time.

71. The auditor's defense of contributory negligence is most likely to prevail when

A. third party injury has been minimal.


B. the auditor fails to detect fraud resulting from management override of the control
structure.
C. the client has privately held as contrasted with a public company.
D. undetected errors have resulted in materially misleading financial statements.

72. Ana and Associates, CPAs, issued an unqualified opinion on the financial statements of
Seral Corp. for the year ended December 31, 2010. It was determined later that Seral's
treasurer had embezzled P300,000 from Seral during 2010. Seral sued Ana because of Ana's
failure to discover the embezzlement. Ana was unaware of the embezzlement. Which of the
following is Ana‘s best defense?

A. The audit was performed in accordance with PSAs.


B. The treasurer was Seral's agent and, therefore, Seral was responsible for preventing the
embezzlement.
C. The financial statements were presented in conformity with PFRS.
D. Ana had no actual knowledge of the embezzlement.

73. The factor that distinguishes constructive fraud from actual fraud is

A. materiality.
B. quality of internal control.
C. type of error or irregularity.
D. intent.

74. If a CPA recklessly abandons standards of due care and diligence while performing an
audit, he or she may be held liable to unknown third parties for:

A. Fraudulent misconduct.
B. Gross misconduct.
C. Gross negligence.
D. Contributory negligence.

75. Salve Corp. orally engaged Rex & Co., CPAs, to audit its financial statements. The
management of Salve informed Rex that it suspected that the accounts receivable were
materially overstated. Although the financial statements audited by Rex did, in fact, include a
materially overstated accounts receivable balance, Rex issued an unqualified opinion. Salve
relied on the financial statements in deciding to obtain a loan from City Bank to expand its
operations. City Bank relied on the financial statements in making the loan to Salve. As a result
of the overstated accounts Salve has defaulted on the loan and has incurred a substantial loss.
If Salve sues Rex for negligence in failing to discover the overstatement, Rex‘s best defense
would be that

A. no engagement letter had been signed by Rex.


B. the audit was performed by Rex in accordance with generally accepted auditing
standards.
C. Rex was not in privity of contract with Salve.
D. Rex did not perform the audit recklessly or with an intent to deceive.

76. In a common law action against an accountant, the lack of privity is a viable defense if the
plaintiff

A. bases his action upon fraud.


B. is the accountant's client.
C. is a creditor of the client who sues the accountant for negligence.
D. can prove the presence of gross negligence which amounts to a reckless disregard for
the truth.

77. Which of the following conditions suggests an auditor's negligence?

A. Failure to detect material errors under conditions of weak internal control.


B. Failure to detect collusive fraud perpetrated by members of middle management.
C. Failure to detect collusive fraud perpetrated by members of top management.
D. Failure to detect errors occurring outside the internal control structure.

78. Marcia Corporation orally engaged Legaspi and Lopez, CPAs, to audit its year-end financial
statements. The engagement was to be completed within two months after the close of Marcia's
fiscal year for a fixed fee of PI25,000. Under these circumstances, what obligation is assumed
by Legaspi and Lopez?

A. None. The contract is unenforceable since it is not in writing.


B. An implied promise to exercise reasonable standards of competence and care.
C. An implied obligation to take extraordinary steps to discover all defalcations. D. The
obligation of an insurer of its work, which is liable without fault.

79. A third party sues a public accounting firm for negligence under common law on the basis of
materially false financial statements. Which of the following is the firm's defense?

A. Lack of privity
B. Lack of reliance
C. Lack of intent
D. Contributory negligence

80. What type(s) of liability do CPA's have in the Philippines?

Common Law Liability Statutory Law Liability

A. YES YES
B. YES NO
C. NO YES
D. NO NO
81. A CPA firm issues an unqualified opinion on financial statements that were not prepared in
accordance with GAAP. The CPA firm would have acted with fraud or its equivalent in all the
following circumstances except where the firm

A. intentionally disregards the truth.


B. has actual knowledge of fraud.
C. negligently performs auditing procedures.
D. intends to gain monetarily by concealing the fraud.

82. Conflict between financial statement users and auditors often arises because of the

A. high cost of performing an audit.


B. technical vocabulary which the auditor uses in the report.
C. placement of the auditor's report at the back of the client's annual report where it is hard
to locate
D. expectation gap.

83. A CPA will most likely be negligent when he fails to:

A. Correct errors discovered in the CPA's previously issued audit reports.


B. Detect all of a client's fraudulent activities.
C. Include a: negligence disclaimer in the CPA's engagement letter.
D. Warn a client's customers of embezzlement that may be perpetuated by the client's
employees.

84. A CPA should not be liable to any party if he performs his services with:

A. Ordinary negligence
B. Regulatory providence
C. Due professional care
D. Good faith

85. If a CPA recklessly departs from the standards of due care when conducting an audit, the
CPA will be liable to third parties who are

A. Ordinary negligence
B. Gross negligence
C. Strict liability
D. Criminal deceit
MODULE 4
AUDIT REPORT

PSA BASED QUESTION

1. General purpose financial statements are financial statements prepared in


accordance with a financial reporting framework that is designed to:

A. meet the particular information needs of a wide range of users


B. meet the particular information needs of a group of users
C. meet the common information needs of a wide range of users
D. meet the common information needs of a group of users

2. PSA 700 provides guidance on the:

A. audit report that includes an unqualified opinion


B. audit report that includes an unqualified opinion
C. audit report that includes an unqualified opinion, through the auditor‘s report is
modified due to an emphasis of matter
D. audit report, irrespective of the type of opinion issued by the auditor

3. PSA 701 provides guidance on the:

A. auditor‘s report issued as a result of an audit of a complete set of general


purpose financial statements
B. auditor‘s report issued as a result of performing a special purpose audit
engagement|
C. auditor‘s report that contains an opinion other than unqualified and/ or a
modified report due to an emphasis of matter
D. auditor‘s report issued as a result of an audit of a single statement or specified
account(s)

4. PSA 800, ―The Independent Auditor‘s Report on Special Purpose Audit


Engagements‖ least likely applies to auditor‘s report issued as a result of an audit of:

A. compliance to royalty engagement


B. a component of a complete set of general purpose financial statements
wherein the opinion of the auditor is unqualified
C. complete set of general purpose financial statements where the auditor is
prohibited from observing the inventory count
D. summarized financial statements

5. The auditor‘s judgment regarding whether the financial statements give a ―true and
fair view‖ or ―are presented fairly‖ on all material respects, is made in the context of:

A. generally accepted auditing standards


B. standards of reporting of generally accepted auditing standards
C. applicable financial reporting framework
D. applicable Philippine Standards on Auditing (PSAs)

6. In forming an opinion on the financial statements,

A. the auditor should evaluate the conclusion drawn from the audit evidence
obtained during the course of the audit
B. the auditor evaluates whether there is a reasonable assurance about whether
the financial statements are free from any misstatements
C. the auditor evaluates whether sufficiently appropriate audit evidence has been
obtained to eliminate the risk of material misstatements
D. the auditor verifies that all errors that misstate the financial statements have
been corrected by the client

7. In evaluating whether the financial statements have been prepared and presented in
accordance with the specific requirements of the applicable financial reporting
framework for particular classes of transactions, account balances and disclosures, the
auditor should consider:

A. that the accounting estimates made by the management are reasonable in the
circumstances
B. that the information presented in the financial statements, including accounting
policies id relevant, reliable, comparable, and understandable
C. that the accounting policies selected and applied are consistent with the
financial reporting framework
D. All of the choices given are to be considered

8. Which of the following is least likely considered by the auditor when he has to
evaluate the fair presentation of the financial statements?

A. Whether the financial statements, after any adjustments made by the


management as a result of audit process, are consistent with the auditor‘s
understanding of the entity and its environment
B. Whether the financial statements, including the disclosures faithfully represent
the underlying transaction and events in a manner that gives a true and fair view
of, in all material respects, the information conveyed in the financial statements
within the context of the financial reporting framework
C. Whether the results of analytical procedures performed at or near the end of
the audit help to corroborate conclusion formed during the audit
D. Whether the financial statements are approved by the client‘s board of
directors

9. If the auditor encounters circumstances that lead him to conclude that the compliance
with a specific requirement results to financial statements that are misleading, the
auditor:

A. considers the need to appropriately modify the auditor‘s report


B. does not need to modify the report
C. needs to issue qualified opinion
D. needs to disclaim his opinion

10. What is the overriding benefit of having consistency in the report?

A. Consistency promotes credibility in the global marketplace by making more


readily identifiable those audits that have been conducted in accordance with
globally recognized standards
B. Consistency in the form promotes the expression of unqualified opinion
C. Consistency lessens the auditor‘s legal and civil liabilities
D. The audit report eliminates some disclosures required in the financial
statements

11. The auditing profession recognizes the need for uniformity in reporting as means of

A. defending against third party litigations


B. systematic and easier review of the audit report by the lead engagement
partner
C. standardizing the policies of various CPA firms
D. avoiding confusion

12. What is the descriptive word in the title of an audit report issued on a complete set
of general purpose financial statement which affirms that the auditor has met all of the
relevant ethical requirements?
A. Audit
B. Opinion
C. Independent
D. Report

13. The auditor‘s standard report should always include in its tittle the word:

A. Standard
B. Independent
C. Opinion
D. Audit

14. The auditor‘s report may be addressed to any of the following, except the client‘s

A. Stockholders
B. Chief executive officer
C, Board of directors
D. Partners

15. The introductory paragraph of the standard audit report may include the following:

A) Identification of the entity whose financial statements have been audited


B) Statement that the financial statements have been audited
C) Title of each of the financial statements that comprise the complete set of financial
statements
D) Reference to ―basic financial statements‖ without indicating the title of each financial
statements
E) Reference to the summary of significant accounting policies and other explanatory
notes
F) Specific date and period covered by the financial statements
G) Reference to Philippine Standards on Auditing

Which of the foregoing are specifically required by the applicable standards on auditing
to be included or referred to in the introductory paragraph of the standard audit report?

A. A, B, D, F, G
B. A, B, C, E, F, G
C. A, B, C, E, F
D. B, C, E, F
16. The purpose of the introductory paragraph in the standard unqualified report is

A. to clarify the responsibilities of the auditor


B. to identify the financial statements which were audited, and the dates and time
periods covered by the report
C. to communicate the responsibilities of management in preparing the financial
statements, and to clarify the respective roles of management and the auditor
D. all of the responses are correct

17. The complete set of general purpose financial statement that are prepared in
accordance with PFRS comprise of:

A. Balance Sheet, Income Statement, Statement of Changes in Equity, Cash


Flow Statement
B. Statement of Assets and liabilities, Income statements, Statement of Changes
in Retained Earnings, Cash Flow Statement
C. Balance Sheet, Income Statement, Cash Flow Statement, Summary of
Significant Accounting Policies and Other Explanatory Notes
D. Balance Sheet, Income Statement, Cash Flow Statement, Statement of
Changes in Equity, Summary of Significant Accounting Policies and Other
Explanatory Notes

18. When an entity presents, together with the financial statements, supplementary
information that cannot be clearly differentiated from the financial statements because
of its nature and how it is presented, such supplementary information

A. must be specifically referred to in the introductory paragraph of the auditor‘s


report
B. is covered by the auditor‘s opinion
C. is referred by adding an emphasis of matter paragraph
D. is not covered by the auditor‘s opinion

19. Which of the following is an incorrect statement about supplementary information?

A. The auditor‘s opinion may or may not cover the supplementary information
B. It is important for the auditor to be satisfied that any supplementary
information that is not covered by the financial statements
C. The supplementary information that cannot be differentiated from the financial
statements is covered by the auditor‘s opinion
D. Supplementary information that is presented as an integral part of the financial
statement always needs to be specifically referred to in the introductory
paragraph of the auditor‘s report

20. Which of the following is not specifically referred to in the second paragraph of the
standard audit report as management‘s responsibilities?

A. Designing, implementing, and maintaining internal control relevant to the


preparation and fair presentation of financial statements
B. Effectiveness and efficiency of operating decisions
C. Selecting and applying appropriate accounting policies
D. Making accounting estimates that are reasonable in the circumstances

21. Management‘s responsibility for the financial statements is

A. Implicitly represented in the auditor‘s standard report


B. Explicitly represented in the opening paragraph of the auditor‘s standard
report
C. Explicitly represented in the scope – responsibility of the management
paragraph of the auditor‘s standard report
D. Explicitly represented in the opinion paragraph of the auditor‘s standard report

22. Which paragraph(s) of the standard auditor‘s report affirms the responsibility of the
management with respect to the entity‘s financial statements?

A. First paragraph
B. Second paragraph
C. Second and Third paragraphs
D. Third paragraph

23. How are management‘s responsibility and the auditor‘s responsibility represented in
the applicable paragraph(s) of the standard auditor‘s report?

Management Auditor
A. Explicit Explicit
B. Implicit Implicit
C. Implicit Explicit
D. Explicit Implicit

24. The auditor‘s responsibility in an audit engagement is limited to:


A. expression of an opinion on the financial statements
B. expression of an opinion on the financial statements and adequacy of
summary of accounting policies and other notes
C. opinion issued and the fairness of presentation of the financial statements
D. expression of opinion and an inclusion of supplementary information if
necessary

25. The existence of audit risk is recognized by the statement in the scope paragraph -
auditor‘s responsibility of the auditor‘s standard report that the

A. auditor is responsible for expressing an opinion on the financial statements


B. financial statements are presented fairly, in all material respects, in conformity
with PFRS
C. audit includes examining, on a test basis, evidence supporting the amounts
and disclosure in the financial statements
D. auditor obtains reasonable assurance about whether the financial statements
are free of material misstatements

26. The standard audit report explains that a financial audit includes all of the following
except

A. Examining support for the amounts and disclosure in the financial statements
B. Assessing the level of control risk
C. Assessing the accounting principles used and significant estimates made by
the management
D. Evaluating the overall financial statement presentation

27. Which of the following statements is a basic element of the standard audit report?

A. The disclosures provide reasonable assurance that the financial statements


are free from material misstatements
B. The auditor tested compliance to internal control by the client
C. An audit includes assessing significant estimates made by the management
D. The financial statements are consistent with those of the prior period

28. The auditor‘s report should describe an audit by addressing some concerns that
may include:

A) An audit includes evaluating the appropriateness of the accounting policies used


B) An audit includes evaluating the reasonableness of the accounting estimates made
by management
C) Evaluating the overall presentation of the financial statements
D) An audit involves performing procedures to obtain audit evidence about the amounts
and disclosures in the financial statements
E) The procedures selected depend on the auditor‘s judgment

Which of the foregoing concerns are stated in the audit report?

A. A, B, C, D, E
B. A, B, C
C. A, C, E
D. A, B, D

29. The audit report date is important to users of financial statements because it
indicates

A. the last day of the fiscal period


B. the date on which the financial statements were filed with the SEC
C. the last date on which users may institute a lawsuit against the client or the
auditor
D. the last day of the auditor‘s responsibility for the review of significant events
that occurred after the date of financial statements

30. The standard audit report refers to GAAS and PFRS in which paragraphs?

GAAS PFRS
A.Scope only Opinion only
B.Introductory only Scope and opinion only
C.Introductory and scope only Opinion
D.Introductory only All paragraphs

31. The auditor‘s judgment of whether financial statements are fairly presented is made
within the context oof:

A. Philippine financial reporting standrads


B. control risk
C. attestation standards
D. auditing standards
32. The auditor‘s standard report states that the financial statements are presented
fairly
A. with reasonable assurance
B. in all material respects
C. without significant errors
D. on a consistent basis

33. How are other reporting responsibilities addressed within the auditor‘s report?

A. They should be addressed in a separate section that follows the opinion


paragraph
B. They should be addressed within the introductory paragraph
C. They should be addressed within the scope paragraph
D. They should be addressed within the introductory paragraph and separately
described in a separate paragraph

34. Which of the following is incorrect regarding the auditor's signature?

A. The auditor's signature is either in the name of the audit firm, the personal
name of the auditor, or both, as appropriate
B. The auditor's signature is either in the name of the audit firm or the personal
name of the auditor, but not both
C. In addition to the auditor's signature, the auditor may be required to declare
the auditor's professional accountancy designation
D. The auditor's report filed with the Securities and Exchange Commission (SEC)
must be manually signed
35 Which of the following information is(are) required when an auditor's report is issued
on financial statements to be filed with the Securities and Exchange Commission?

1. Audit report is manually signed.


2. Certifying partner to sign his name.
3. Partner's Tax Identification Number.
4. PRC registration number
5. Accreditation with SEC

A. 1, 2, 3, 4, 5
B. 2, 4, 5
C. 1, 3, 1, 5
D. 2, 3, 4, 5
36. An audit report should be dated as of the

A. date the stockholders approve the audited financial statements


B. date of management approving the audited financial statements
C. balance sheet date of the latest period reported on
D. date a letter of audit inquiry is received from the entity's attorney.

37. Why is the date of the auditor's report important?

A. To have a basis of determining the audit fees to be paid to the auditor


B. The date of the auditor's report informs the readers that the auditor has
considered the effect of events and transactions of which the auditor became
aware and that occurred up to that date
C. To emphasize completeness assertion
D. To inform the users of the financial statements that the auditor complied with
the applicable Philippine Standards on Auditing.

38. How is the auditor's report on the financial statements that require final approval by
stockholders before such financial statements are issued publicly dated?

A. The auditor's report should be dated coinciding the date of approval of the
financial statements by the stockholders
B. The auditor's report should be dated after the approval of the financial
statements by the stockholders
C. The date of the auditor's report coincides the date of approval of the financial
statements by the board of directors
D. The audit report should be dual dated, the first date coinciding the approval by
the board of directors and the second date to coincide with the approval by the
stockholders

39. The auditor's address is indicated in the auditor's report by:

A. naming the location in the country where the auditor practices his profession
B. including the complete mailing address of the auditor
C. identifying the country from where the auditor had secured his professional
license
D. the auditor's address is omitted in the report

40. Which of the following is ordinarily true of a modification of the audit report by
adding an emphasis of matter paragraph?
A. The modification by adding an emphasis of matter paragraph is an "except for"
qualification of opinion
B. The emphasis of matter paragraph is a "subject to qualification of opinion
C. The emphasis of matter paragraph would ordinarily refer to the fact that the
auditor's opinion is not qualified
D. The emphasis of matter paragraph is presented before the opinion paragraph

41. When additional language is added to the auditor's report without - modifying the
opinion, the additional language should be included in:
A. the introductory paragraph
B. the scope paragraph
C. the opinion paragraph
D. one or more additional paragraphs that follow the opinion paragraph.

42. Which of the following-statements is not true?

A. A one-paragraph report is generally used when the auditor is not independent


B. A modification of the audit report that involves modified wordings may contain
an unqualified opinion
C. An addition of another paragraph to an otherwise standard audit report always
requires a modification of an unqualified opinion
D. An unqualified opinion may be issued though the audit report requires an
additional explanatory paragraph
43. An auditor includes a separate paragraph in an otherwise unmodified report to
emphasize that the entity being reported on had significant transactions with related
parties. The inclusion of this separate paragraph

A. is considered an "except for" qualification of the opinion


B. violates generally accepted auditing standards if this information is already
disclosed in the footnotes to the financial statements
C. necessitates a revision of the opinion paragraph to include the phrase "with
the foregoing explanation."
D. is appropriate and would not negate the unqualified opinion.

44. An auditor concludes that there is substantial doubt about an entity's ability to
continue as a going concern for a reasonable period of time. If the entity's disclosures
concerning this matter are adequate, the audit report should include a(an)

Adverse opinion "Except for" qualified opinion


A. Yes Yes
B. No No
C. No Yes
D. Yes No

45. Under certain circumstances, the CPA may wish to emphasize specific matters
regarding the financial statements even though he or she intends to express an
unqualified opinion. Normally, such an explanatory information should be included in

A. the introductory paragraph


B. a separate paragraph following the opinion paragraph in the report
C. the opinion paragraph
D. A separate paragraph preceding the opinion paragraph

46. Salmon Company's financial statements adequately disclose uncertainties that


concern future events, the outcome of which cannot reasonably be estimated. The
auditor's report should include a(an)

A. unqualified opinion
B. except for qualified opinion
C. "subject to" qualified opinion
D. adverse opinion

47. The paragraphs of the report which is modified for uncertainties are the same as the
standard unqualified report. The explanatory Paragraph as a form of the modification to
describe the uncertainty is added as the

A. first paragraph
B. last paragraph
C. third paragraph with the opinion paragraph last
D. second paragraph with the opinion paragraph last

48. An explanatory paragraph following an opinion paragraph that describes an


uncertainty is as follows:

As discussed in Note X to the financial statements, the company is a defendant in a


lawsuit alleging infringement of certain patent rights and claiming damages. Discovery
proceedings are in progress. The ultimate outcome of the litigation cannot presently be
determined. Accordingly, no provision for any liability that may result upon adjudication
has been made in the accompanying financial statements.
What type of opinion should the auditor express in this circumstance?
A. Unqualified
B. Disclaimer
C. Qualified
D. adverse

49. The audit report issued by Lozano and Co., CPAs, included the following paragraph
that followed the opinion paragraph: Without qualifying our opinion we draw attention to
Note 11 to the financial statements. The Company is the defendant in a lawsuit alleging
infringement of certain patent rights
This paragraph is considered:

A.an inappropriate reporting practice


B. an additional information to be a part of the notes to financial statements
C. an emphasis of matter regarding uncertainty which is considered an
acceptable reporting practice
D. inappropriate because it contradicts the unqualified opinion issued by the
auditor

50. In extreme cases such as situations involving multiple uncertainties that are
significant to the financial statements, the auditor

A. may consider to express a disclaimer of opinion


B. may qualify his opinion instead of issuing an unqualified opinion with emphasis
of matter paragraph
C. may issue an adverse opinion because of their significance
D. may issue a "subject to" opinion because the situations related to
uncertainties

51. A client company has issues that cause substantial doubt regarding the entity's
ability to continue as a going concern. If this is the only major audit issue, which type of
opinion will the auditor usually refrain from issuing?

A. Adverse
B. Unqualified with explanatory language
C. Clean opinion
D. Disclaimer of opinion

52. Which of the following situations, the effect of which is significant, least likely require
a decision of whether to issue a qualified or adverse opinion?
A. Any disagreement with entity management regarding the acceptability of the
accounting policies selected by the management
B. Limitation on the scope of the auditor's work
C. Inadequate disclosures of financial information
D. Unjustified changes in accounting policies

53. The auditor may continue to express unqualified opinion though there are
modifications made in the audit report. Which of the following situations, would the
auditor likely modify his opinion?

A. The existence of multiple uncertainties that are adequately described in the


notes to financial statements
B. The prior year's financial statements were audited by other CPAs
C. An important subsidiary whose financial statements were included in the
consolidated financial statements were audited by other CPAs
D. A substantial doubt about the client's ability to continue as a going concern
that is adequately disclosed in the financial statements.

54. In which of the following situations would qualified opinion be inappropriate?

A. Financial statements are materially misstated


B. A doubt that is more than substantial about the ability of the company to
continue as a going concern
C. A significant scope limitation
D. The management insisted of not attaching the statement of cash flows

55. Which of the following is not a reason to issue a modified audit report with opinion
other than unqualified opinion?

A. The scope of the auditor's work is restricted by the client


B. The amount of inventories at cost as presented in the balance sheet
significantly exceeded their market values
C. Certain significant matter is omitted from either the financial statements or
notes to financial statement
D. An adequately disclosed significant uncertainty, the resolution of which is
dependent upon future events and which may affect the financial statements.

56. Which of the following situations may likely require a modified audit report witn
modified wordings or an emphasis of matter paragraph?
A. A significant uncertainty, not adequately disclosed in the financial statements
B. An audit of inventory is restricted by the client. The auditor was satisfied about
the balance of the inventory by doing alternative audit procedures
C. A change in the application of generally accepted accounting principle that is
justified.
D. A less than substantial doubt regarding the ability of the entity to continue as a
going concern.

57. Which of the following circumstances may not result- to a disclaimer of opinion?

A. A significant scope limitation in auditing the existence of inventories. The


inventory amount comprises 75 percent of the total assets of the client
B. The auditor believes that there are multiple, uncertainties that are significant to
the financial statements
C. The accounts receivable of the client comprises 80 percent of the total assets.
The auditor was instructed by the client not to confirm account balances. The
auditor, however, was satisfied by the results of alternative audit procedures
D. The auditor's wife owns very a few number of common shares of the client.

58. An auditor may express a qualified opinion because of

Departure from PFRS Lack of Consistency Scope Limitation


A. YES YES YES
B. NO YES NO
C. YES NO NO
D. NO YES YES

59. Whenever an auditor issues a qualified report, he or she

A. must use the term "subject to" in the opinion paragraph


B. may use either the terms "subject to" or " except for" in the opinion paragraph,
depending on the nature of the qualification
C. must use the term "except for" in the opinion paragraph
D. must not use the terms "subject to" or "except for" in the opinion paragraph

60. An explanatory paragraph may be added to the audit report while at the same time
issuing an unqualified opinion in all cases except when:

A. the client has changed an accounting principle with the agreement of the
auditor
B. there is an immaterial departure from GAAP to ensure fair presentation with
the agreement of the auditor
C. the audit opinion is partly based on the work of another auditor
D. the audit work has been significantly limited by management. .

61. Under which of the following sets of circumstances might an auditor disclaim an
opinion?

A. The financial statements contain a departure from PFRS, the effect of which is
material
B. The principal auditor decides to make reference to the report of another
auditor who audited a subsidiary
C. There has been a material change between periods in the method of the
application of accounting principles
D. There were significant limitations on the scope of the audit.

62. If an auditor is engaged to audit a client's financial statements after the annual
physical inventory count was made and the accounting records are not sufficiently
reliable to enable the auditor to become satisfied as to the year-end inventory balances,
the opinion to be expressed is

A. either an "except for" qualified opinion or an adverse opinion


B. either a disclaimer or opinion or an "except for" qualified opinion
C. either an adverse opinion or disclaimer of opinion
D. an unqualified opinion.

63. An adverse opinion is issued when the auditor believes

A. some parts of the financial statements are materially misstated or misleading


B. the financial statements investigation be found to be misleading or misstated,
if an adequate investigation is performed
C the overall financial statements are so materially misstated or misleading as a
whole that they do not present fairly the financial position or results of operations,
changes in cash and stockholders' equity in conformity with PFRS
D. the audit firm is not independent

64. If the scope of the auditors procedures in conducting an audit is significantly


restricted by the client management, the audit opinion will most likely be a(n):
A. Adverse opinion
B. Qualified opinion
C. Unqualified with explanatory paragraph
D. Disclaimer of opinion.

65. The auditor would most likely disclaim his opinion because of

A. the client's failure to present supplementary information required by the FRSC


B. inadequate disclosure of material information
C. the qualification of an opinion by the other auditor of a subsidiary where there
is a division of responsibility
D. a client-imposed scope limitation.

66. Whenever the client imposes restrictions on the scope of the audit, the auditor
should be concerned about the possibility that management is trying to prevent
discovery of misstated information. In such cases, PSA 701 has encouraged the auditor
to issue a:

A. disclaimer of opinion in all cases


B. qualification of both scope and opinion in all cases
C. disclaimer of opinion, whenever materiality is in question
D. qualification of both the scope and opinion paragraphs whenever materiality is
in question

67. An explanatory paragraph or modified wordings may be added to the audit report
while at the same time issuing an unqualified opinion in all cases except when:

A. the client has changed an accounting principle with the agreement of the
auditor
B. there is an immaterial departure from PFRS to ensure fair presentation with
the agreement of the auditor
C. the audit opinion is partly based on the work of another auditor
D. the audit work has been materially limited by management.

68. The most common case in which conditions beyond the client's and auditor's control
cause a scope restriction is an engagement

A. agreed upon after the client's balance sheet date


B. where client will not allow the auditor to confirm receivables for fear of
offending its customers
C. where auditor does not have enough staff to audit all of client's foreign
subsidiaries satisfactorily
D. where client is going through a bankruptcy

69. An audit report contains the following paragraph: "Because of the inadequacies in
the company's accounting records during the year ended June 30, 2008, it was not
practicable to extend our auditing procedures to the extent necessary to enable us to
obtain certain evidential matter as it relates to classification of certain items in the
consolidated statements of operations." This paragraph most likely describes

A. A material departure from PFRS requiring a qualified audit opinion


B. An uncertainty that should not lead to a qualified opinion
C. A matter that the auditor wishes to emphasize and that does not lead to a
qualified audit opinion
D. A material scope restriction requiring a qualification of the audit opinion.

70. Which of the following circumstances least likely result to either a qualified opinion
or an auditor disclaiming his opinion?

A. The auditor is unable to carry out an audit Procedure believed to be desirable;


the auditor carried out alternative audit procedures to support the management's
assertion
B. The auditor believed the client's accounting records are inadequate
C. A client-imposed scope limitation with respect to the audit of inventory
D. Circumstances did not permit the auditor to perform certain required audit
procedure.

71. Addition of an "emphasis of a matter" paragraph to what remains an unqualified


opinion is least likely required for which of the following situations?

A. Related party transactions


B. Scope limitation
C. A significant subsequent event
D. An uncertainty.

72. When the scope of the auditor's work has been limited, the audit report should
contain a(n):

A. unqualified opinion if the scope limitation was unavoidable


B. indication that the financial statements are materially misstated because of a
departure from PFRS
C. estimate of the financial impact of the scope limitation on the financial
statements
D. emphasis of matter paragraph that refers to the particular note to the financial
statements.

73. When there is a limitation on the scope of the auditor's work that requires a
modification of the audit report:

A. The auditor's report should either contain a qualified or adverse opinion


B. The auditor's report may contain an unqualified opinion with an emphasis of
matter paragraph that follows the opinion paragraph
C. The auditor's report should describe the limitation and indicate the possible
adjustments to the financial statements that might have been determined to be
necessary had the limitation not existed
D. Should always contain a disclaimer of opinion

74. Which of the following least likely requires an expression of unqualified opinion with
modified wordings or an emphasis of the matter paragraph?

A. The financial statements of prior period, which are presented for comparative
purposes, were audited by as 'Gillet CPAs
B. The auditors have substantial doubt about the ability of the entity to continue
as a going concern
C. The entity changed the measurement of certain significant transaction from
one GAAP to another GAAP
D. The auditors failed to observe physical inventory count. however, the auditor
was satisfied that the inventory amount was fairly presented by doing alternative
audit procedures.

75. When there is a limitation in the scope of the audit that results to a disclaimer of
opinion, the following paragraphs are modified, except:

A. Introductory paragraph
B. Management's responsibility for the financial statements
C. Auditor's responsibility
D. Auditor's opinion.

76. The expression of a qualified opinion means that the financial statements, taken as
a whole, in all material respects, are
A. materially misstated
B. materially misleading
C. presented fairly
D. do not present fairly

77. When the auditor cannot perform certain required procedures and the amounts are
so material that a disclaimer of opinion rather than a qualified opinion is required

A. the opinion paragraph will state "do not present fairly"


B. the opinion paragraph will state "present fairly"
C. the scope - auditor's responsibility paragraph will be deleted
D. the scope paragraph will be unchanged from the standard unqualified opinion.

78. Which of the following indicates a qualified opinion?

A. The financial statements do not present fairly in all material respects, the
financial position, results of operations, and cash flows in conformity with PFRS
B. The auditor does not express an opinion on the financial statements
C. The financial statements present fairly, in all material respects, the financial
position, results of operations, and cash flows in conformity with PFRS
D. Except for the effects of a matter, the financial statements present fairly, in all
material respects, the financial position, results of operations, and cash flows in
conformity with PFRS

79. An auditor who qualifies an opinion because of his inability to obtain sufficient
evidential matter should describe the limitation in an explanatory paragraph that is
inserted between the scope -responsibility of the auditor and opinion paragraphs. The
auditor should also refer to the limitation in the:

Scope Opinion Notes to financial


(Responsibility Notes to the Paragraph statements
of the auditor)
A. YES NO YES
B. NO YES NO
C. YES YES NO
D. YES YES YES

80. Which of the following circumstances requires the auditor to omit the sentence
stating the responsibility of the auditor in the auditor's report?
A. Limitation on the scope of the audit, resulting to qualified opinion
B. Limitation on the scope of the audit resulting to disclaimer of opinion
C. The management refuses to issue a representation letter that prompts the
auditor to qualify his opinion
D. A material misstatement that requires an adverse opinion

81. When the client is not following PFRS, and the auditor believes that adherence to
PFRS would result to misleading statements, the opinion paragraph of the audit report

A. must express an adverse opinion


B. must express a qualified opinion
C. should be unqualified with a required explanatory paragraph
D. should be the standard unqualified opinion

82. In which of the following conditions is an unqualified audit opinion least likely
issued?

A. The auditor believes that a substantial doubt about the entity's ability to
continue as a going concern exists
B. The auditor believes that inventory is valued at market values that accurately
reflect market conditions and materially exceed cost
C. The audit is conducted with no circumstance or imposed scope limitation
D. PFRS are not consistently applied from year to year

83. Once the auditor has determined that an exception is material enough to warrant a
qualification of his auditor's report, he must then determine if the exception is sufficiently
material to negate an overall opinion. If the auditor is applying this decision process to
an exception based on a departure from Philippine financial reporting standards, he is
deciding
A. Whether to issue an adverse opinion rather than a disclaimer of opinion
B. Whether to issue a disclaimer of opinion rather than a qualified opinion
C.Whether to issue an adverse opinion rather than a qualified opinion
D. Nothing because such a decision process is not applicable to this type of
exception.

84. An auditor who is reporting on financial statements that contain a material departure
PFRS should include in his audit report a separate explanatory paragraph and

A. not modify the opinion paragraph as long as the departure is adequately


disclosed in a footnote
B. disclaim an opinion on the financial statements
C. express a qualified or adverse opinion
D. express a qualified opinion or disclaim an opinion

85. When the auditor knows that the financial statements may be misleading because
they are not prepared in conformity with Philippine financial reporting standards, he
must issue a(n)

A. a qualified opinion
B. an adverse opinion
C. a disclaimer of opinion
D. a qualified or an adverse opinion, depending on the materiality of the item in
question.

86. An auditor should disclose the substantive reasons for expressing an adverse
opinion in an explanatory paragraph

A. preceding the scope - responsibility of the auditor paragraph


B. between the scope - responsibility of the auditor paragraph, and the opinion
paragraph
C. following the opinion paragraph
D. within the notes to the financial statements.

87. A qualified opinion report can be used only when the auditor believes that the
overall financial statements are

A. fairly stated
B. not fairly stated
C. materially misstated
D. materially misleading.

88. If the auditor believes that a required material disclosure is omitted from the financial
statements, the auditor should decide between issuing a(n)

A. qualified opinion or an adverse opinion


B. disclaimer of opinion or a qualified opinion
C. adverse opinion or a disclaimer of opinion
D. unqualified opinion or a qualified opinion
89. An auditor is confronted with an exception sufficiently material to warrant departing
from the standard wording of an unqualified report. it the exception relates to a
departure from the Philippine financial reporting standards, the auditor must decide
between a(n)

A. adverse opinion and an unqualified opinion


B. adverse opinion and a qualified opinion
C. adverse opinion and a disclaimer of opinion
D. disclaimer of opinion and a qualified opinion

90. In which of the following situation would a decision of selecting between a qualified
or adverse opinions be inappropriate?

A. limitation in the scope of the audit


B. The financial statements are significantly misleading
C. A disagreement between the auditor and the client arose because of
capitalization of research and development costs
D. A required disclosure that is significant is omitted from the financial
statements.

91. When the auditor qualifies his opinion due to his disagreement with the client in
applying accounting policies, the auditor modifies:

A. The introductory paragraph and opinion paragraph


B. Management's responsibility for the financial statements, and opinion
paragraphs.
C.The opinion paragraph only
D. Management's responsibility for the financial statements, auditor's
responsibility, and auditor's opinion paragraphs.

92. Which of the following circumstances requires the modification of both the auditor's
responsibility, and the auditor's opinion paragraphs of the auditor's report?

A. Limitation on the scope of audit that results to qualified opinion


B. Auditor's disagreement with the client management on accounting policies that
requires qualified opinion
C. Inadequate disclosures that requires qualified opinion
D. Disagreement with the client management regarding accounting policies that
requires adverse opinion
93. In which of the following situations would an auditor ordinarily choose between
expressing a qualified opinion or an adverse opinion?

A. The auditor did not observe the entity's physical inventory and is unable to be
satisnea about its balance by other auditing procedures
B. Conditions that cause the auditor to have substantial doubt about the entity's
ability to continue as a going concern are not disclosed
C. There has been a change in accounting principles, the material effect on the
comparability of the entity's financial statements has been properly disclosed in
the financial statements
D. The auditor is unable to apply necessary procedures concerning an investor's
share on an investee's earnings recognized on the equity method

94. How should the auditor address the comparatives that are presented as
corresponding figures?

A. The comparatives are specifically identified in the audit report because the
auditor's opinion on the current period financial statements applies also to the
corresponding figures
B. The comparatives are specifically identified in the introductory paragraph and
in the opinion paragraph
C. The comparatives are not specifically identified because the auditor's opinion
applies to the current period financial statements as a whole, including the
corresponding figures
D. The comparatives are referred to in the opinion paragraph as the auditor
applies to both the current year's financial statements and the corresponding
figures.

95. Which of the following circumstances requires an issuance of unqualified opinion


with modified wordings?

A. A significant uncertainty that may affect the financial statements of the future
period is adequately disclosed in the financial statements
B. The auditor agrees with the client for a change in accounting policy that
significantly affects the financial statements
C. An insignificant scope limitation in the work of the auditor
D. The successor auditor reports on the current year's financial statements. The
prior-years financial statements that were presented as comparatives were
audited by another CPA
96. When management prepares financial statements on the basis of a going concern
and the auditor believes the company may not continue as a going concern, the auditor
should issue a(n)

A. qualified opinion
B. unqualified opinion with an explanatory paragraph
C. disclaimer of opinion
D. adverse opinion

97. If an amendment is necessary in the other information and the entity refuses to
make the amendment, the auditor, depending on particular circumstance, may do any of
the following, except:

A. Describe the material inconsistency as an emphasis of matter in a paragraph


following the opinion paragraph
B. The auditor may not issue the auditor's report.
C. The auditor may withdraw from the engagement
D. The auditor to issue either a qualified or adverse opinion.

98. An auditor concludes that there is a substantial doubt about an entity's ability to
continue as a going concern for a reasonable period of time. If the entity's financial
statements adequately disclose these financial difficulties, the auditor's report is
required to include an explanatory paragraph that specifically mentions

"Management's plans" "Going concern"


A. Yes Yes
B. Yes No
C. No Yes
D. No No

99. When there is uncertainty about a company's ability to continue as a going concern,
the auditor's concern is the possibility that the client may not be able to continue its
operations or meet its obligations for a "reasonable period of time." For this purpose, a
reasonable period of time is considered not to exceed

A. six months from the date of the financial statements


B. six months from the date of the audit report
C. one year from the date of the financial statements
D. one year from the date of the audit report.
100. When the auditor concludes that there is a substantial doubt the entity's ability to
continue as going concern, he should issue a(n)

A. unqualified opinion with an explanatory paragraph provided that there is an


adequacy of disclosures in the financial statements
B. qualified opinion with an explanatory paragraph, regardless of the adequacy of
disclosures in the financial statements
C. adverse opinion, regardless of the adequacy of disclosures in the financial
statement D. standard unqualified report, provided that client has made adequate
disclosures in the financial statements
QUIZZERS

1. Which of the following representations does an auditor make explicitly and implicitly
when issuing an unqualified opinion?

Conformity with PFRS Adequacy of Disclosure


A. Explicitly Explicitly
B. Implicitly Implicitly
C. Implicitly Explicitly
D. Explicitly Implicitly

2. Materiality is:

A. addressed within a practitioner's attestation am: audit reports.


B. expressed in terms of pesos.
C. measured using guidelines established by the FRSC.
D. not applicable to attestation engagements.

3. Which one of the following is an example of the contents of an opinion paragraph


found in a disclaimer of opinion?

A. "except for..."
B. "nothing came to our attention..."
C. The financial statements do not present fairly...'
D. None of these represents a disclaimer of opinion.

4. An auditor issued an audit report that was dual dated for a subsequent event
occurring after the management's approval of the financial statements but before
issuance of the auditor's report. The auditor's responsibility for events occurring
subsequent to the completion of fieldwork was

A. Extended to subsequent events occurring through the date of issuance of the


report.
B. Extended to include all events occurring since the date of approval of the
financial statements by the management.
C. Limited to the specific event referenced.
D. Limited to include only events occurring up to the date of the last subsequent
event referenced.
5. An auditor has been asked to report on the balance sheet of Jane company but not
on the other basic financial statements. The auditor will have access to all information
underlying the basic financial statements. Under these circumstances, the auditor

A. May accept the engagement but should disclaim an opinion because of his
inability to apply the procedures that are considered necessary in the
circumstance.
B. May accept the engagement because such engagements merely involve
limited reporting objective.
C. Should refuse the engagement because there is a client-imposed scope
limitation.
D. Should refuse the engagement because of a departure from generally
accepted auditing standards.

6. The use of a negative assurance in the audit reports for historical financial statements
is

A. a violation of the standards of reporting.


B. supported by PSAs and their interpretations.
C. allowed if the auditors' opinion is unqualified.
D. properly located in the a opinion paragraph of the unqualified report.

7. Which of the following is true of the notes to financial statements that are prepared in
accordance with PFRS?

A. Notes are not required, but are typically included by all companies.
B. Notes are not required, since they only give additional information contained in
the financial statements.
C. Notes are an integral part of the financial statements.
D. Notes are not encompassed in the auditors' opinion of the financial statements
since they are supplementary information.

8. If the balance sheet of a publicly-held company is dated December 31, 2010, the
audit report is dated March 6, 2011, and both are released to the public on March 15,
2011, this indicates that the auditor has searched for material subsequent transactions
and events that occurred up to

A. December 31, 2010


B. March 15, 2011
C. March 6, 2011
D. None of these

9. The three main types of audit opinion other than the unqualified report are the

A. adverse opinion, disclaimer of opinion, and qualified opinion.


B. adverse opinion, reports on unaudited financial statements, and disclaimer.
C. disclaimer, the qualified opinion, and reports on unaudited financial
statements.
D. special audit reports, reports on unaudited financial statements, and adverse
opinion.

10. In the scope - responsibility of the auditor paragraph of the audit report, the use of
the term "material misstatements" conveys that auditors are responsible to search for

A. minor misstatements.
B. significant misstatements.
C. fraudulent misstatements.
D. all misstatements.

11. The guidelines which enable auditors to decide when something is immaterial,
material, or highly material are provided by

A. the AASC through its Philippine Standards on Auditing.


B. the FRSC through its Statements on Financial Accounting Standards.
C. the Securities and Exchange Commission.
D. auditor's judgment.

12. When financial statements of prior period are presented on a comparative basis with
financial statements of the current period, the continuing auditor is responsible for

A. expressing dual dated opinions.


B. updating the report on the previous financial statements only if there has not
been a change in the opinion.
C. updating the report on the previous financial statements only if the previous
report was qualified and the reasons for the qualification no longer exists.
D. updating the report on the previous financial statements regardless of the
opinion previously issued.

13. If a principal auditor decides to assume responsibility for another auditor's work, the
principal auditor should consider performing all of the following procedures except:
A. Performing a peer review of the other auditor.
B. Reviewing the audit program of the other auditor.
C. Reviewing the working papers of the other auditor .
D. Discussing the audit procedures and the results of the audit with other auditor.

14. When the audited financial statements of the prior year are presented together with
those of the current year, the continuing auditor's report should cover

A. both years.
B. only the current year.
C. only the current year, but the prior year's report should be : presented.
D. only the current year, but the prior year's report should be referred to.

15. As a further attempt to appear that the auditor is independent, the addressee of the
audit report is usually the:

A. client company.
B. board of directors of client company.
C. President and/or CEO of client company.
D. stockholders of client company.

16. Which of the following is not a true statement? "In the opinion paragraph of the
standard unqualified report, required to state

A. an opinion about the financial statements taken as whole


B. a conclusion about whether the company follows Philippine financial reporting
standards.‖
C. whether the management has or has not made adequate disclosure.
D. That the financial statements are presented fairly."

17. In the auditor's responsibility - scope paragraph of the audit report, the use of the
term "reasonable assurance" is intended to indicate that

A. no misstatements exist in the financial statements.


B. no material misstatements exist in the statements.
C. there is a possibility that material misstatements still exist in the financial
statements.
D. there is a possibility that immaterial statements still exist in the financial
statements.
18. An audit opinion that is considered a "good" opinion is on that is

A. A disclaimer of opinion.
B. Adverse.
C. Qualified.
D. Unqualified.

19. Most auditors believe that the financial statements are "presented fairly" when the
statements are in accordance with Philippine financial reporting standards, but it is also
necessary to

A. determine that they are not in violation of FRSC statements.


B. examine the substance of transactions and balances for possible
misinformation.
C. review the statements using the financial reporting standards that are
promulgated by the Accounting Standards council.
D. assure investors that the net income reported this year willequal or exceed the
prior year's. an unqualified opinion, the implication is that the auditor

20. Whenever an auditor issues an unqualified opinion, the implication is that the auditor

A. does not know if the statements are presented fairly.


B. does not believe the statements are presented fairly.
C. is satisfied that the statements are presented fairly except for a specific aspect
of them.
D. is satisfied that the statements are presented fairly.

21. The most common type of audit report contains a(n):

A. adverse opinion.
B. disclaimer of opinion.
C. qualified opinion.
D. unqualified opinion.

22. When a misstatement in the financial statements exists, but is unlikely to affect the
decisions of a reasonable user, it would be appropriate to issue a(n)

A. unqualified opinion.
B. qualified opinion.
C. disclaimer of opinion.
D. adverse opinion.

23. A misstatement in the financial statements can be considered material if

A. it overshadows the financial statements as a whole.


B. knowledge of the misstatement would affect a decision of a reasonable user of
the statements.
C. it affects more than one account on the statements.
D. it affects only one account on the statements.

24. If a misstatement is immaterial relative to the financial statements of the entity for
the current period and is not expected to have a. material effect in future periods, it is
appropriate to issue a(n)

A. unqualified opinion.
B. qualified opinion.
C. adverse opinion.
D. disclaimer of opinion.

25. The only unqualified reports which use modified wording are those involving

A. the use of other auditors.


B. material uncertainties.
C. substantial doubt about going concern.
D. lack of consistent application of PFRS.

26. When the principal auditor decides to refer to another auditor in


his/her report, the report should always include:

A. a qualified or adverse opinion.


B. a disclaimer of opinion regarding the financial statements audited by the other
auditor.
C. the percentage and 'monetary amounts of the portion of the financial
statements examined by the other auditor.
D. reference to a footnote where the division of responsibility between the
principal auditor and the other auditor is described in detail.

27. A principal auditor decides not to take responsibility for the work of another CPA
who audited a wholly-owned subsidiary of the principal auditor's client. The total assets
and revenues of the subsidiary represent 30% and 24% of the related con s3lidated
totals. What type of opinion should the auditor generally issue?

A. Unqualified opinion.
B. Adverse opinion.
C. Qualified opinion.
D. Disclaimer of opinion.

28. When a principal auditor decides to make reference to the examination of another
auditor, the principal auditor's report should clearly indicate the

A. Principal auditor's qualification on the overall fairness of the financial


statements, taken as a whole, "subject to" the work and report of the other
auditor.
B. Procedures that were performed by the other auditor in connection with the
oilier auditor's examination.
C. Division of responsibility between that portion of the financial statements
covered by the examination made by the principal auditor and that covered by
the examination made by the other auditor.
D. Procedures that were performed by the principal auditor to obtain satisfaction
as to the reasonableness of the examination made by the other auditor.

29. The principal auditor is satisfied with the independence and professional reputation
of the other auditor who has audited a subsidiary but wants to indicate the division of
responsibility. The principal auditor should modify

A. only the introductory paragraph.


B. only the scope - auditor's responsibility paragraph.
C. the introductory, scope - auditor's responsibility and opinion paragraphs.
D. only the opinion paragraph.

30. Francis and Company, CPAs, acted as the principal auditor. However, since Francis
and Company, CPAs, did not have the resources, it hired other CPA firm to audit a
subsidiary of the client located in Bukidnon. If Francis is willing to take the responsibility
for the work of other CPA firm, which type of audit report is Francis and Company most
likely to issue?

A. Standard unqualified report.


B. Unqualified with explanatory language.
C. Qualified with explanatory language.
D. Disclaimer of opinion.

31. When the report of a principal auditor makes reference to the examination made by
another auditor, the other auditor may be named if an expressed permission to do so is
given and:

A. The report of the principal auditor names the other auditor in both the scope
and opinion paragraphs. B. The principal auditor accepts responsibility for the
work of the other auditor.
C. The report of the other auditor is presented together with the report of the
principal auditor.
D. The other auditor is not an associate or correspondent firm whose work is
done at the request of the principal auditor.

32. When a client declines to disclose essential information in the financial statements
or their accompanying notes, the auditor should?

A. Provide the information in the audit report, if practicable, and qualify his
opinion because of a scope limitation.
B. Provide the information in the audit report, if practicable, and quality his
opinion because of a departure from PFRS.
C. Issue a disclaimer of opinion because the client has interfered with the
auditor's function of assessing the adequacy of disclosure.
D. Issue an unqualified opinion but inform the readers by including the omitted
information in an emphasis of matter paragraph.

33. Magsombol, CPA, is the principal auditor for a multi-national corporation. Another
CPA has examined and reported on the financial statements of a significant subsidiary
of the corporation. Magsombol is satisfied with the independence and professional
reputation of the other auditor, as well as the quality of his examination. With respect to
Magsombol's report on the consolidated financial statements, taken as a whole,
Magsombol

A. must not refer to the examination of the other auditor.


B. must refer to the examination of the other auditor.
C. may refer to the examination of the other auditor.
D. may refer to the examination of the other auditor, in which case Magsombol
must include in the auditor's report on the consolidated financial statements a
qualified opinion.
34. If a company's financial statements violate PFRS for an immaterial item which is
expected to become material in the future, the auditor is likely to issue:

A. Unqualified standard report.


B. Unqualified with explanatory language.
C. Qualified.
D. Adverse.

35. In performing an audit, the auditor found that the client had changed the estimated
useful life of its assets. The auditor believed that the change in useful lives of the assets
is realistic. The appropriate report is:

A. Unqualified standard report.


B. Unqualified with explanatory language.
C. Qualified.
D. Disclaimer.

36. In which of the following circumstances would the auditor likely issue an unqualified
opinion?

A. Client company's financial statements show a significant net loss for each of
the last three years, including the current fiscal period.
B. The financial statements have not been prepared in accordance with
Philippine financial reporting standards.
C. The auditor is not independent during the fiscal period under audit.
D. The scope of the auditor's examination has been restricted due to
circumstances which are beyond the control of either the auditor or the client.

37. Comparative financial statements include the prior-year statements that were
audited by a predecessor auditor whose report is not presented. If the predecessor's
report was unqualified, the successor should:

A. express an opinion on the current year statements alone and make no


reference to the prior year's statements.
B. Indicate in the auditor's report that the predecessor auditor expressed an
unqualified opinion.
C. Obtain a letter of representation from the predecessor concerning any matters
that might affect the successor's opinion.
D. Request that the predecessor auditor reissue the prior report.
38. The "unqualified report with explanatory paragraph" and the 'unqualified report with
modified wording'

A. arise as a result of an incomplete audit.


B. arise when the financial statements are not quite "presented fairly".
C. meet the criteria of a complete audit with satisfactory results.
D. meet the criteria of a complete audit but with unsatisfactory results.

39. Which of the following is not a cause of an explanatory paragraph or modified


wordings to be added to the standard unqualified report?

A. Emphasis of a matter.
B. Reports involving other auditors.
C. Auditor disagrees with client's departure from PFRS.
D. Lack of consistent application of PFRS.

40. When determining whether an exception is highly material, the extent to which the
exception affects different parts of the financial statements must be considered. This is
referred to as

A. materiality.
B. pervasiveness.
C. financial analysis.
D. ratio analysis.

41. In order to make materiality decisions when a condition requiring a departure from
an unqualified report exists, the auditor must evaluate

A. the magnitude of the error on the account involved.


B. the effect on the financial statement which contains the erroneous account.
C. the effect of the error on both the income statement and the balance sheet.
D. all effects on the financial statements.

42. In which of the following situations would an auditor ordinarily express an


unqualified audit opinion with an explanatory paragraph?

A. The auditor wishes to emphasize that the entity had significantly lost a market
share.
B. The auditor decides not to refer to the report of another auditor as a basis, in
part, for the auditor's opinion.
C. The entity issues financial statements that present financial position and
results of operations, but omits the required notes to financial statements.
D. At the client's request, the client's attorney has refused to respond to the
auditor's inquiries about ongoing litigation.

43. A report other than an unqualified report must be issued whenever any of the three
conditions requiring a departure from an unqualified report

A. exists
B. exists and is material
C. exists, is material, and is within management's control
D. exists, is material, and is within either management's or the auditor's control

44. When a material uncertainty exists, the auditor must

A. disclose it in the audit report


B. first determine whether adequate disclosure is included by the client in the
financial statements
C. issue a disclaimer
D. issue a qualified opinion

45. Which of the following opinion is appropriate if a company has a probable and
material loss contingency, and the company has accrued the loss in the financial
statements?

A. Adverse opinion.
B. Qualified opinion.
C. Standard unqualified opinion.
D. Unqualified opinion with explanatory language.

46. Which of the following conditions most likely requires the auditor to consider issuing
a going concern modification?

A. A decrease in profitability as compared to the previous year.


B. A loss contingency related to a lawsuit.
C. Default on a loan agreement.
D. A material related-party transaction.
47. Which of the following, by itself, would not cause uncertainty about the ability of a
company to continue as a going concern?

A. A significant net loss.


B. inability to nay its obligations as they come due.
C. The occurrence of uninsured catastrophes.
D. Legal proceeding that might jeopardize the entity's ability to continue
operating.

48. JJ, CPA, has performed most of the audit of Macmood Company's financial
statement and qualifies as the principal auditor, RT, CPA did the remainder of the work.
JJ wishes to assume f responsibility for RT's work. Which of the following correct?

A. In such circumstance, when appropriate requirements h been met, JJ should


issue a standard unqualified opinion on the financial statements.
B. Such assumption of responsibility violates the profession's standards.
C. In such circumstance, when appropriate requirements have been met, JJ
should issue an unqualified opinion but should make appropriate reference to RT
in the audit report.
D. JJ should normally qualify his audit report on the basis of the scope limitation
as there is another auditor involved in the audit.

49. Which of the following is a change which does not affect consistency and therefore
does not require an explanatory paragraph?

A. Change in accounting principle, such as a change from LIFO to FIFO.


B. Change in reporting entity, such as the inclusion of an additional company in
combined financial statements.
C. Change in an accounting estimates, such as a decrease in the life of an asset
for depreciation purposes.
D. Correction of errors by changing from non-PFRS to PFRS.

50. If an auditor is not independent of a client, the auditor should issue a(n):

A. unqualified opinion with an explanatory paragraph describing the lack of


independence.
B. explanatory letter accompanying the financial statements.
C. disclaimer of opinion.
D. qualified opinion because of the lack of independence.
51. Whenever an auditor issues an adverse opinion, the implication is that the auditor

A. does not know if the statements are presented fairly.


B. does not believe the statements are presented fairly.
C. is satisfied that the statements are presented fairly except for a specific aspect
within we financial statements.
D. is satisfied that the statements are presented fairly.

52. When a misstatement in the financial statements would affect a user's decision but
the overall statements are fairly stated, it would be appropriate to issue a(n):

A. unqualified opinion.
B. qualified opinion.
C. adverse opinion.
D. disclaimer of opinion.

53. Which of the following is most accurate with respect to a CPA's responsibility in
considering a going concern question on financial statement audits?

A. Based on audit procedures performed, assess whether there is substantial


doubt about the entity's ability to continue as a going concern.
B. Perform analytical procedures aimed particularly at assessing whether
bankruptcy is probable.
C. Issue a report with a "going concern" modification when failure is at least
reasonably probable.
D. Determine that related uncertainties are properly disclosed and make no
mention in the audit report.

54. The distinction between an adverse opinion and a disclaimer is

A. the lack of PFRS versus lack of GAAS.


B. knowledge versus lack of knowledge.
C. the audit report versus the review report.
D. AASC statements versus the FRSC Standards.

55. The auditor's best course of action with respect to "other financial information"
included in an annual report containing the auditor's report is to

A. indicate hi die auditor's report, that the "other financial information" is


unaudited.
B. Consider whether the "other financial information" is accurate uy pert-waling a
limited review.
C. Obtain written representations from management as to the material accuracy
of the 'other financial information.
D. Read and consider the manner of presentation of the "other financial
information."

56. A CPA is not able to confirm a large account receivable, but he has satisfied himself
as to the proper statement of the receivable by means of alternative auditing
procedures. The auditor's report on the financial statements should include

A. A description of the limitation on the scope of his examination and the


alternative auditing procedures used, but an opinion qualification is not required.
B. An opinion qualification, but reference to the use alternative auditing
procedures is not required.
C. Both a scope qualification and an opinion qualification.
D. Neither a comment on the use of alternative auditing procedures nor an
opinion qualification.

57. An auditor expressed a qualified opinion on a company's prior year financial


statements because of a material departure from PFRS in the prior year. The company
has properly restated last
of year's financial statements and presents them in comparative form with the current
year's financial statements. The auditor's updated report on the prior year's financial
statements should:

A. express an unqualified opinion on the restated financial statements of the prior


year.
B. continue to express a qualified opinion on the prior year's financial statements.
C. include the auditor's qualified opinion from the prior year.
D. not mention the opinion on the prior year's financial statements.

58. An auditor would express an unqualified opinion with an explanatory paragraph


added to the audit report for:

An Unjustified Accounting Material Weakness in


The internal Control
A. YES YES
B. YES NO
C. NO YES
D. NO NO

59. Which of the following statements appropriately distinguishes a disclaimer of


opinion from an adverse opinion?

A. A disclaimer of opinion indicates that the auditor is not able to gather enough
evidence to render an opinion on the financial statements, while an adverse
opinion indicates that the financial statements are materially misstated.
B. A disclaimer of opinion indicates that the financial statements are materially
misstated, while an adverse opinion indicates that, the auditor is not able to
gather enough evidence to render an opinion on the financial statements.
C. The opinions are generally equivalent, except on adverse opinion includes a
going concern paragraph.
D. Adverse opinion indicates that the financial statements are materially
misstated, while a disclaimer indicates that the financial statement are "so
pervasively erroneous" that no opinion can be given.

60. A departure from PFRS with a material effect on the financial statements is most
likely to result in a (n):

A. Disclaimer of opinion.
B. Qualified opinion.
C. Standard unqualified opinion.
D. Unqualified opinion with explanatory language.

61. When a CPA has concluded that an action should be taken to prevent future
reliance on his report, he should

A. Advise his client to make appropriate disclosure of the newly discovered facts
and their impact on the financial statements to persons who are known to be
currently relying or who are likely to rely on the financial statements and the
related report.
B. Recall the financial statements and issue revised statements and include an
appropriate opinion.
C. Advise the client and others not to rely on the financial statements and make
appropriate disclosure of the correction in the statements of a subsequent period.
D. Recall the financial statements and issue a disclaimer of opinion which should
generally be followed by revised statements and a qualified opinion.
62. In which circumstance would an auditor be most likely to express an adverse
opinion?

A. The chief executive officer refuses the auditor's access to minutes of the board
of directors' meetings.
B. Tests of controls show that the client's internal control is so ineffective that it
cannot be relied upon.
C. The financial statements are not in conformity with the FRSC Statements
regarding the capitalization of leases.
D. Information comes to the auditor's attention than raises substantial doubt
about the client's ability to continue as a going concern.

63. An auditor's report would be designated as a special report when it is issued in


connection with financial statements that are:

A. for an interim period and are subject to a limited review.


B. unaudited and are prepared from a client's accounting records.
C. prepared in accordance with a comprehensive basis of accounting other than
Philippine financial reporting standards.
D. purported to be in accordance with generally accepted accounting principles
but do not include a presentation of the Statement of Cash Flows.

64. The opinion paragraph of a CPA's report states: "In our opinion, with the exception
of the effects of not observing inventory in one of the client's Mactan warehouses, as
discussed in the preceding paragraph, the financial statements present fairly, in all
material respects,..." This paragraph expresses a (n)

A. unqualified opinion.
B. adverse opinion due to scope limitation
C. qualified opinion due to scope limitation.
D. opinion modified because of an uncertainty.

65. Which of the following subsequent events will be least likely to result to an
adjustment on the financial statements?

A. Culmination of events affecting the realization value of accounts receivable


owned as of the balance sheet date.
B. Culmination of events affecting the realization of inventories owned as of the
balance sheet date.
C. Material changes in the settlement of liabilities which were estimated as of the
balance sheet date.
D. Material changes in the quoted market prices of listed investment securities
since the balance sheet date.

66. An auditor was unable to obtain sufficient competent evidential matter concerning
certain transactions because a fire burned down the client's office building destroying all
the company's records. Given these circumstance, the auditor would choose between a
(n)

A. qualified opinion and an unqualified opinion with an explanatory paragraph.


B. unqualified opinion with an explanatory paragraph and an adverse opinion.
C. adverse opinion and a disclaimer of opinion.
D. disclaimer of opinion and a qualified opinion.

67. The necessity to issue a disclaimer of opinion may arise because of

A. a severe limitation on the scope of the audit examination


B. a nonindependent relationship between auditor and client
C. either of the two choices
D. neither of the two choices

68. In his Letter to Stockholders in the annual report, the president of Better Vision's
states that this year was the most profitable year in the company‘s history Actually the
company did better, profit-wise, last year according to the audited financial statements.
What type of opinion should the auditor issue?

A. An unqualified opinion with an emphasis of a matter paragraph noting the


inconsistency.
B. A disclaimer of opinion because the additional information accompanying the
financial statements wasn't audited.
C. An adverse opinion because the annual report does not present fairly the
financial condition of the company.
D. An "except for " qualification because the President's letter is not part of the
audited financial statements.

69. Both disclaimers and adverse opinions are used

A. only when the condition is highly material.


B. whether the condition is material or not.
C. regardless of the auditor's independence.
D. regardless of client's choice of a non-PFRS accounting method.

70. Soon after Patricia's audit report was issued, Patricia learned of certain related party
transactions that occurred during the year under audit. These transactions were not
disclosed in the notes to the financial statements. Patricia should

A. plan to audit the transactions during the next engagement.


B. recall all the copies of the audited financial statements.
C. determine whether the lack of disclosure would affect the auditor's report.
D. ask the client to disclose the transactions in subsequent interim statements.

71. When an auditor mentions consistency in the audit report, a reader of the financial
statements may infer

A. that PFRS have not been consistently observed in the current period in
relation to the preceding period.
B. that a material departure from PFRS has been detected.
C. that a reclassification of items or change in classifications has occurred.
D. nothing about application of accounting principles within the period.

72. As a result of management's refusal to permit the auditor to physically examine


inventory, the auditor has not accumulated sufficient evidence to conclude whether
financial statements are stated in accordance with PFRS. The auditor must depart from
the unqualified audit report because

A. the financial statements have not been prepared in accordance with PFRS.
B. the scope of the audit has been restricted by circumstances beyond either the
client's or auditor's control.
C. the auditor has lost independence.
D. the scope of the audit has been restricted.

73. A disclaimer of opinion is issued whenever the auditor

A. is unable to satisfy himself that the overall financial statements arc presented
fairly.
B. believes that the overall financial statements are not presented fairly.
C. believes that some material parts of the financial statements are not presented
fairly.
D. has determined that the financial statements are presented fairly.
74. Management of Blue Company has decided not to account for a material transaction
in accordance with the provisions of an FRSC standard. In setting forth its reasons in a
note to the financial statements, management has clearly demonstrated that due to
unusual circumstances had the financial statements been presented in accordance with
the PFRS, they would be misleading. The auditor's report should include an explanatory
separate paragraph and contain a(an)

A. adverse opinion.
B. unqualified opinion.
C. "except for" qualified opinion.
D. "subject to" qualified opinion.

75. In the ―management discussion and analysis (MD&A)‖ contained in the 2008 annual
report of Concorde Corporation, the management started that the total sales were P4.95
billion and net profit was P500 million. The audited sales and net profit, however,
wereP3.8 billion and P450 million respectively. The financial statements, contained in
the annual report, reflected the audited figures and the CPA planned to issue an
unqualified opinion. Upon noting the inconsistencies between the data in the annual
report and the audited financial statements, however, the CPA should

A. refer to the inconsistency in the audit report and issue a qualified audit
opinion.
B. issue an unqualified opinion without an explanatory paragraph, because the
MD&A is not covered in the audit report.
C. issue an unqualified audit opinion with an explanatory paragraph describing
the inconsistency.
D. render an adverse opinion on the basis that management had intentionally
misrepresented reported sales and net profit.

76. The adverse opinion report will be issued by the independent auditors when they

A. suspect that the client has not followed Philippine financial reporting
standards.
B. suspect that the client's financial statements are not in conformity with
generally accepted auditing standards.
C. have knowledge that the financial statements are not in conformity with
Philippine financial reporting standards (PFRS).
D. have knowledge that generally accepted auditing standards (GAAS) were not
followed.
77. A post-audit review, conducted by another audit partner, revealed that the audit
team had failed to examine or confirm securities held in safekeeping. The amounts
involved were material in relation to reported net assets. The unqualified audit report,
along with the audited financial statements, had been released two-months earlier.
Based on this information, the audit team should

A. request the client for permission to examine or confirm the securities.


B. notify persons known to be relying on the audit report that the report can no
longer be relied upon.
C. draft a revised audit report containing a qualified opinion due to a scope
restriction.
D. ignore the finding inasmuch as the financial statements and audit report have
already been released.

78. Which of the following statements is true?

A. The auditor is required to issue a disclaimer of opinion in the event of a


material uncertainty.
B. The auditor is required to issue a disclaimer of opinion in the event of a going
concern problem.
C. The auditor is required to issue a disclaimer of opinion for a material
uncertainty and for a going concern problem.
D. The auditor has the option, but is not required, to issue a disclaimer of opinion
for a material uncertainty or for a going concern problem.

79. The least severe type of report for disclosing departures from an unqualified report
is the

A. adverse opinion
B. qualified opinion
C. disclaimer of opinion
D. report on unaudited financial statements

80. Which of the following statements is correct with respect to explanatory paragraphs
in report on an audit of financial statements?

A. They always precede the opinion paragraph.


B. The always follow the opinion paragraph.
C. They always precede the scope paragraph.
D. Sometimes they precede and sometimes they follow the opinion paragraph.

81. Of the two major categories of scope restrictions, (1) those caused by client and (2)
those caused by conditions beyond the control of either the client or auditor; the effect
on the auditor's judgment is

A. the same for either.


B. more serious for i than for 2.
C. more serious for 2 than for 1.
D. negligible.

82. Whenever an auditor issues a qualified opinion, the implication is that the auditor

A. does not know if the statements are presented fairly.


B. does not believe the statements are presented fairly.
C. is satisfied that the statements are presented fairly 'except for a specific
aspect of them.
D. is satisfied that the statements are presented fairly.

83. Whenever an auditor issues a disclaimer of opinion, the implication is that the
auditor

A. does not know if the statements are presented fairly.


B. does not believe the statements are presented fairly.
C. is satisfied that the statements are presented fairly exc ot for a specific aspect
of them.
D. is satisfied that the statements are presented fairly.

84. Which of the following best describes the auditor's responsibility for "other
information" that is, together with the audited financial statements and the auditor's
report, included in the annual report to stockholders?

A. The auditor has rio obligation to read the "other information."


B. The auditor has no obligation to corroborate the "other information," but should
read the "other information" to determine whether it is materially inconsistent with
the financial statements.
C. The auditor should extend the examination to the extent necessary to verify
the "other information."
D. The auditor must modify the auditor's report to state that the 'Other infoz-
mation is unaudited" or "not covered by the auditor's report."
85. If the inventory comprises the largest balance on the financial statements, a large
misstatement that is so material would cause the auditor to issue a(n):

A. unqualified opinion.
B. adverse opinion.
C. qualified opinion.
D, disclaimer of opinion.

86. When a known or suspected misstatement with highest level of materiality exists on
the financial statements, the auditor must issue

A. an adverse opinion.
B. a disclaimer of opinion.
C. either a qualified opinion or an adverse opinion, depending on which condition
exist.
D. either an adverse opinion or a disclaimer of opinion, depending on which
conditions exist.

87 When a disclaimer of opinion is issued because the auditor lacks independence,

A. no report title is included in the report.


B. a one-paragraph audit report is issued.
C. the only reason cited for issuing the disclaimer of opinion is the lack of
independence.
D. all of the choices are required.

88. Under which of the following circumstances would an unqualified audit opinion,
followed by an explanatory paragraph, not be appropriate?

A. The auditor wishes to emphasize that the client has entered into material
transactions with related parties. The substance of the related party transactions
is properly disclosed in the audited financial statements.
B. The client has completed material transactions with related parties and the
auditor is unable to persuade management to properly reflect the economic
substance of the transactions in the financial statements.
C. The client has used a method of revenue recognition that is at variance with
promulgated accounting standards. The auditor, however, agrees with the
departure on the basis that the use of the promulgated standard would make the
financial statements materially misleading.
D. The auditor believes that substantial doubt exists concerning the ability of the
client to continue as a going concern.

89. If the auditor is determined to lack independence, a disclaimer of opinion must be


issued

A. in all cases.
B. Only if it is highly material.
C. Only if it is material.
D. If the client requests it.

90. Any deviation from the independence rule is considered

A. immaterial.
B. Slightly material.
C. Material.
D. Highly material.

91. When comparing misstatements with a measurement base, the auditor must
consider the pervasiveness of the misstatement. An example of a pervasive
misstatement would be

A. an understatement of inventory, caused by an oversight.


B. an understatement of retained earnings, caused by a miscalculation of
dividends payable.
C. a misclassification of notes payable as a long-term liability when it should be
current.
D. a misclassification of salary expense as a selling expense when it should be
allocated equally to both selling and administrative expense.

92. Kim, an independent auditor, was engaged to perform an examination of the


financial statements of Sue Incorporated one month after its fiscal year had ended.
Although the count was not observed by Kim, and accounts receivablein:ternet riot
inventory confirmed by direct communication with debtors, Kim was able to gain
satisfaction by applying alternative auditing procedures. Kim's auditor's report will
probably contain

A. a standard unqualified opinion.


B. an unqualified opinion and an explanatory middle paragraph.
C. either a qualified opinion or a disclaimer of opinion.
D. an "except for" qualification.

93. The peso amount of some e misstatements cannot 9 measured. If, for example, the
client is unwilling existing lawsuit, the materiality question that the evaluate in such a
situation is

A. what effect will it have on net income.


B. how will it affect management's future decisions.
C. does it increase the auditor's exposure to lawsuits.
D. what effect will it have on statement users.

94. Whenever there is a scope restriction, the appropriate response is to issue


be accurately to disclose an auditor must

A. a disclaimer of opinion.
B. an adverse opinion.
C. a qualified opinion.
D. an unqualified report, a qualification of scope and opinion, or a disclaimer,
depending on materiality.

95. The adverse effects of events causing an auditor to believe there is substantial
doubt about an entity's ability to continue as a going concern would most likely be
mitigated by evidence relating to the

A. ability to expand operations into new product lines in the future.


B. feasibility of plans to purchase leased equipment at less than market value.
C. marketability of assets that management plans to sell.
D. committed arrangements to convert preferred stock to long-term debt.

96. After an audit report containing an unqualified opinion on a non-public client's


financial statements was issued, the client decided to sell the shares of a subsidiary that
accounts for 30% of its revenue and 25% of its net income. The auditor should

A. determine whether the information is determined to be reliable, request that


statements be issued.
B. notify the entity that the auditor's report associated with the financial
statements.
reliable and, if revised financial may no longer be
C. describe the effects of this subsequently discovered information in a
communication with persons knovyn to be relying on the financial statements.
D. take no action because the auditor has no obligation to make any further
inquiries.

97. A client company has changed its method of inventory, valuation from an
unacceptable one to one in conformity with Philippine financial reporting standards. The
auditor's report on the financial statements in the year of the change should include

A. no reference to consistency.
B. a reference to a prior period adjustment in the opinion paragraph.
C. an explanatory paragraph explaining the change.
D. a justification for making the change and the impact of the change on reported
net income.

98. The client has presented all the required financial statements with the exception of
the statement of cash flows. The auditor has completed the audit and is satisfied that
everything, with the exception of the missing statement, is presented fairly. Accordingly,
the auditor

A. must issue a qualified opinion.


B. must issue an adverse opinion.
C. may issue an unqualified opinion.
D. may issue either an unqualified or qualified opinion.

99. An audit report contained the following wording: "In our opinion, except for the
omission of the segment information referred to in the preceding paragraph…‖ The
excerpt was taken from a(n)

A. Unqualified audit opinion with an explanatory paragraph added to emphasize


a matter.
B. Unqualified audit opinion with an explanatory paragraphto describe a material
uncertainty.
C. Audit opinion qualified due to a departure from PFRS
D. Adverse audit opinion

100. For the report containing a disclaimer for lack of independence, the disclaimer is in
the

A. fourth or opinion paragraph.


B. Scope -responsibility of the auditor paragraph.
C. first and only paragraph.
D. explanatory paragraph.

101. Should a situation arise where all audit procedures considered " necessary in the
circumstances were performed and the auditor would otherwise issue an unqualified
report, and then it was discovered that the auditor has not fulfilled the independence
requirements specified by the Code of Ethics, the audit report issued

A. may still be the unqualified opinion.


B. must be a disclaimer of opinion.
C. may be either an unqualified or disclaimer of opinion.
D. must be an adverse opinion.

102. An auditor's examination reveals a misstatement in segment information that is


material in relation to the financial statements taken as a whole. If the client refuses to
make modifications to the presentation of segment information, the auditor should issue
a (n)

A. "Except for" opinion.


B. Adverse opinion.
C. Unqualified opinion.
D. Disclaimer of opinion.

103. In a qualified, adverse, or disclaimer report, the auditor

A. has not per a satisfactory audit.


B. is not satisfied that the financial statements are presented
fairly.
C. either of the two responses.
D. none of the two responses.

104. When there is a significant change in accounting principle, an auditor‘s report


should refer to the lack of consistency in

A. the scope paragraph.


B. an explanatory paragraph between the second paragraph and the opinion
paragraph.
C. the opinion paragraph.
D. an explanatory paragraph following the opinion paragraph.
105. If client has been inconsistent in applying PFRS from year one to year two and the
auditor does not concur with the appropriateness of the change, the auditor will issue a
(n)

A. disclaimer.
B. qualified opinion.
C. adverse opinion.
D. unqualified opinion.

106. When a principal auditor decides to make reference to another auditor's


examination, the principal auditor's report should always indicate clearly, in the
introductory, scope, and opinion paragraphs, the

A. Magnitude of the portion of the financial statements examined by the other


auditor.
B. Disclaimer of responsibility concerning the portion of the financial statements
examined by the other auditor.
C. Name of the other auditor.
D. Division of responsibility.

107. Which of the following requires recognition in the auditor's opinion as to


consistency?

A. The correction of an error in the prior year's financial statements resulting from
a mathematical mistake in capitalizing interest.
B. The change from the cost method to the equity method in accounting for
investment in common stock.
C. A change in the estimate of provisions for warranty costs.
D. A change in depreciation method which has no effect on current year's
financial statements but is certain to affect future years.

108. Items that materially affect the comparability of financial statements generally
require disclosure in the footnotes. If the client refuses to properly disclose the item, the
auditor may require to issue

A. the disclaimer.
B. a qualified opinion.
C. an unqualified opinion.
D. an adverse opinion.
109. A statement in a report such as "Nothing came to our attention that would lead us
to question the fairness of the presentations" is referred to as

A. the unqualified opinion.


B. a disclaimer of opinion.
C. negative assurance.
D. positive assurance.

110. An auditor may issue the standard audit report when the

A. auditor refers to the findings of an expert.


B. financial statements are derived and condensed from complete audited
financial statements that are filed with a regulatory agency.
C. financial statements are prepared on the cash receipts and disbursements
basis of accounting.
D. principal auditor assumes responsibility for the work of another auditor.

111. Several types of "special audit reports" are issued by CPAs. Which one of the
following circumstances would not require the issuance of such a special report?

A. Client's financial statements are prepared using the cash basis.


B. Client's financial statements are prepared using the accrual basis.
C. The CPA has been retained to audit only the current assets.
D. The CPA has been retained to review the internal control system, not the
financial statements.

112. In which of the following situation would an auditor ordinarily, issue an unqualified
audit opinion without an explanatory paragraph?

A. The auditor wishes to emphasize that the entity had significant related party
transactions.
B. The auditor decides to make reference to the report of another auditor as a
basis, in part, for the auditor's opinion.
C. The entity issues financial statements that present financial position, the
results of operations, and the changes in stockholders' equity but intentionally
omits the statement of cash flows.
D. The auditor has substantial doubt about the entity's ability to continue as a
going concern, but the circumstances are fully disclosed in the financial
statements.
113. Comparative financial statements include the financial statements of the prior year
that were audited by a predecessor auditor whose report is not presented. If the
predecessor's report was qualified, the successor should

A. indicate the substantive reasons for the qualification in the predecessor


auditor's opinion.
B. request the client to reissue the predecessor's report on the prior year's
statements.
C. issue an updated comparative audit report indicating the division of
responsibility.
D. express an opinion only on the current year's statements and make no
reference to the prior year's statements.

114. Which of the following events occurring after the issuance of an auditor's report
most likely would cause die auditor to make further inquiries about the previously issued
financial statements?

A. A technological development that could affect the entity's future ability to


continue as a going concern.
B. The discovery of information regarding a contingency that existed before the
financial statements were issued.
C. The entity's sale of a subsidiary that accounts for 30 per cent of the entity's
consolidated sales.
D. The final resolution of a lawsuit that was fully explained in a footnote.

115 A principal auditor decides not to refer to the audit by another CPA who audited a
subsidiary of the principal auditor's client. After making inquiries about the other CPA's
professional reputation and independence, the principal auditor most likely would

A. add an explanatory paragraph to the auditor's report indicating that the


subsidiary's financial statements are not material to the consolidated financial
statements.
B. document in the engagement letter that the principal auditor assumes no
responsibility for the other CPA's work and opinion.
C. obtain written permission from the other CPA to omit the reference in the
principal auditor's report.
D. contact the other CPA and review the audit programs and working papers
pertaining to the subsidiary.
116. The Fragile Company's financial statements contained a departure from Philippine
financial reporting standards because, due to unusual circumstances, the statements
would have otherwise been misleading. The auditor should express an opinion that is

A. unqualified but not mention the departure in the auditor's report.


B. unqualified and describe the departure in a separate paragraph.
C. qualified and describe the departure in a separate paragraph.
D. qualified or adverse, depending on materiality, and describe the departure in a
separate paragraph.

117. When a predecessor auditor reissues the report on the prior period's financial
statements at the request of the former client, the predecessor auditor should

A. indicate ill tile introductory paragraph of the reissued report that the financial
statements of the subsequent period were audited by another CPA.
B. obtain an updated management representation letter and compare it to that
obtained during the prioi period audit.
C. add an explanatory paragraph to the reissued report stating that the
predecessor has not performed additional auditing procedures concerning the
prior period's financial statements.
D. Review the current year's financial statements for reasonableness.

118. Miller Company uses the first-in, first-out method of costing for its international
subsidiary's inventory and the last-in, first-out method of costing for its domestic
inventory. Under these circumstances, Miller should issue an auditor's report with an

A. "except for" qualified opinion.


B. unmodified opinion.
C. explanatory paragraph as to consistency.
D. opinion modified as to consistency.

119. When the audited financial statements are presented in a client's document
containing other information, the auditor should

A. perform inquiry and analytical procedures to ascertain whether the other


information is reasonable.
B. add an explanatory paragraph to the auditor's report without changing the
opinion on the financial statements.
C. perform the appropriate substantive auditing procedures to corroborate the
other information.
D. read the other information to determine that it is consistent with the audited
financial statements.

120. When an auditor qualifies an opinion because of inadequate disclosure, the auditor
should describe. the nature of the omission in a separate explanatory paragraph and •
modify the respective paragraphs

Introductory Scope Opinion


A. YES NO NO
B. YES YES NO
C. NO YES YES
D. NO NO YES

121. If an accounting change has no material effect on the financial statements in the
current year but the change is reasonably certain to have a material effect in later years,
the change should be

A. treated as a consistency modification in the auditor's report for the current


year.
B. disclosed in the notes to the financial statements of the current year.
C. disclosed in the notes to the financial statements and referred to in the
auditor's report for the current year.
D. treated as a subsequent event.

122. When comparative financial statements are presented, the fourth reporting
standard, which refers to financial statements "taken as a whole", should be considered
to apply to the financial statements of the

A. periods presented plus one preceding period.


B. current period only.
C. current period and those of the other periods presented.
D. current and immediately preceding period only.

123. An auditor's report includes a statement that "the financial statements do not
present fairly the financial position in conformity with Philippine financial reporting
standards." This auditor's report was probably issued in connection with financial
statements that were

A. prepared on a comprehensive basis for accounting other than PFRS.


B. restricted for use by management.
C. misleading.
D. condensed.

124. In relation to the "The Auditor's Consideration of an Entity's Ability to Continue as a


Going Concern", an independent auditor is responsible to

A. predict whether the entity will be in business one year from the balance sheet
date.
B. evaluate whether there is substantial doubt about the entity's ;ability to
continue as a going concern.
C. weigh mitigating factors against contrary information about the entity's ability
to continue as a going concern.
D. report the entity's ability to continue as a going concern to senior management
and to the board of directors.

125. An auditor is unable to determine the amounts associated with illegal acts
committed by a client. The auditor would most likely issue

A. either a qualified opinion or a disclaimer of opinion.


B. an adverse opinion.
C. either a qualified opinion or an adverse opinion.
D. a disclaimer of opinion.
MODULE 5

AUDIT PLANNING

PSA-BASED QUESTIONS

1. Given that an audit in accordance with generally accepted auditing standards is


influenced by the possibility of materials errors and fraud, the auditor should
conduct the audit with an attitude of

a. Professional responsiveness
b. Conservative advocacy
c. Objective judgment
d. Professional skepticism

2. With respect to errors and fraud, which of the following should be a part of an
auditor‘s planning in an audit engagement?

a. Planning to search for error or fraud that would have a material or immaterial
effect on the financial statements
b. Planning to discover errors or fraud that are either material or immaterial
c. Planning to discover errors or fraud that are material
d. Planning to consider factor affecting the risk of material misstatements both at
the financial statement and the account balance level

3. The risk that the auditor may unknowingly fail to appropriately modify the
unqualified opinion on financial statements that are materially misstated is
referred to as

a. Audit risk
b. Detection risk
c. Information risk
d. Business risk

4. The risk the financial statements are likely to be misstated materially without
regard to the effectiveness of internal control is which type of risk?

a. Inherent risk
b. Audit risk
c. Client risk
d. Control risk
5. The type of transactions that ordinarily have a high inherent risk because they
involve management judgment or assumptions are referred to as

a. Estimation transactions
b. Nonroutine transactions
c. Routine transactions
d. Related-party transactions

6. Inherent risk is defined as the susceptibility of an accountant balance or class of


transactions to error that could be material assuming that there were no related
internal controls. Of the following conditions, which one does not increase
inherent risk?

a. The client has entered into numerous related party transactions during the
year under audit.
b. Internal control over shipping, billing, and recording of sales revenue is weak
c. The client has lost a major customer accounting for approximately 30% of
annual revenue
d. The board of directors approved a substantial bonus for the president and
chief executive officer, and also approved an attractive stock option plan for
themselves

7. Which of the following descriptions best describes inherent risk?

a. Auditors fail to discover a material misstatement in the course of their audit


and do not modify their audit opinion.
b. A company‘s internal control fails to identify a material misstatement on a
timely fashion
c. Auditing procedures fail to find a material misstatement
d. The possibility that a material misstatement will occur in any given account
before considering internal control

8. The risk that a material misstatement in an assertion will not be prevented or


detected on a timely basis by internal control is

a. Detection risk
b. Control risk
c. Inherent risk
d. Audit risk
9. The risk that the audit will fail to uncover a material misstatement is eliminated

a. If a client has a strong internal controls


b. If a client follows generally accepted accounting principles (GAAP)
c. When the auditor has complied with the generally accepted auditing
standards (GAAS)
d. Under no circumstances

10. Audit risk components consist of inherent, control and detection risks. Which of
them is (are) the dependent variable(s)?

a. Inherent risk
b. Control risk
c. Detection risk
d. Inherent and control risks

11. The probability that an auditor‘s procedures leading to the conclusion that a
material error does not exist in an account balance when, in fact, such error does
exist is referred to as

a. Prevention risk
b. Inherent risk
c. Control risk
d. Detection risk

12. Which of the following is the best definition of detection risk?

a. The auditor will compute audit materiality incorrectly


b. The auditor will fail to detect material misstatements that exist
c. The auditor will apply more audit procedures than are required in the
circumstances
d. The auditor will fail to modify the audit opinion on financial statement that are
materially misstated.

13. Which of the following is not a component of audit risk?

a. Detection risk
b. Business risk
c. Control risk
d. Inherent risk
14. Which of the following types of risk is significantly affected by the nature, amount
and timing of substantive auditing procedures?

a. Inherent risk
b. Control risk
c. Detection risk
d. Sufficiency risk

15. The understanding between the client and the auditor as to the degree of
responsibilities to be assumed by each is normally set forth in a(an)

a. Representation letter
b. Engagement letter
c. Management letter
d. Comfort letter

16. After an auditor had been engaged to perform the first auditor for a non-public
entity, the client requested to change the engagement to a review.in which of the
following situations would there be a reasonable basis for the auditor to comply
with the client‘s request?

a. the client‘s bank required an audit before committing to a loans, but the client
subsequently acquired alternative financing
b. the auditor is prohibited by the client from corresponding with the client‘s legal
counsel
c. management refuses to sign the client representation letter
d. the audit is substantially complete and the auditor determined that an
unqualified opinion is warranted but there is a disagreement concerning the
audit fee

17. Which of the following statements is correct with respect to obtaining an


understanding with a client?

a. Auditors are not required to obtain an understanding with their clients.


b. Auditors must obtain an understanding only if an audit is to be conducted
c. Auditors must document their understanding of the engagement
d. Auditors must obtain an engagement letter

18. An engagement letter is best described as:


a. A letter from company management to the auditors specifying management‘s
expectations for completion of the audit on a timely basis and the fees
b. A letter from the auditors to company management specifying that
management is responsible for the financial statements and the auditors will
issue an opinion on the financial statement.
c. A letter from the auditors to company management that specifies the
responsibilities of both the company and the auditors in completing the audit
and the timing for its completion
d. A letter from the Board of Directors audit committee to the auditor that
indicates that the auditor has been engaged to perform the audit and the fees
to be paid.

19. The primary reason why an engagement letter is submitted by audit fism prior to
starting the work is that it

a. Clarifies the responsibilities of the management and those of the audit firm
b. Defines the firm‘s policies and procedures regarding new clients
c. Provides an insurance policy for both the firm and its client
d. Communicated the type of opinion that will be rendered on the engagement

20. Which of the following best describes the purpose of an engagement letter?

a. The engagement letter relieves the auditor of some responsibility for the
exercise of due care
b. By clearly defining the nature of the engagement, the engagement letter helps
avoid and resolve misunderstandings between the CPA and the client
regarding the precise nature of the work to be performed and the type of
report to be issued
c. The engagement letter conveys to the management the detailed steps to be
applied in the audit process
d. The engagement letter should be signed by both the client and the CPA and
should be used only for independent audits

21. Which of the following is not included in an audit engagement letter?

a. Objectives of the engagement


b. Representations that the financial statements were prepared in accordance
with PFRS
c. Management responsibilities
d. A clear explanation of the services to be performed on the engagement
22. Which of the following statement would least likely appear in an auditor‘s
engagement letter?

a. The fees for our serviced are based on our regular per diem rates, plus travel
and other out-of-pocket expenses
b. During the course of our audit, we may observe opportunities for economy in,
or improved controls over, your operations
c. Our engagement is subject to the risk that material errors, fraud, and
defalcations, if they exist, will not be detected
d. After performing our preliminary analytical procedures we will discuss with
you the other procedures we consider necessary to complete the
engagement

23. Which of the following is not done during the client selection and retention phase
of planning?

a. Obtain an understanding of internal controls


b. Obtain and review financial information
c. Consider the need for special skills
d. Ensure that the firm has sufficient resources to complete the engagement on
a timely manner

24. Professional skepticism

a. Neither assumes that the management is dishonest nor of unquestioned


honesty
b. Assumes that management is either dishonest or of unquestioned honesty
c. Either assumed that management is hones or dishonest
d. None of the given choices is a correct statement
25. Which of the following is not required by PSA No.315, ―consideration of fraud in a
financial statement audit‖?
a. Conduct a continuing assessment of the risks of material misstatement due
fraud throughout the audit
b. Conduct a discussion by the audit team of the risks of material misstatement
dues to fraud
c. Conduct the audit with professional skepticism, which includes an attitude that
assumes balances are incorrect until verified by the auditor
d. Conduct inquiries of the audit committee as to their views about the risks of
fraud and their knowledge of any fraud or suspected fraud
26. The primary difference between financial statement errors and fraud is that

a. Errors are intentional misstatement by management, while fraud involves


unintentional mistakes or omissions, while fraud involves intentional
misstatements
b. There is no difference as error and fraud have the same meaning
c. Errors are more likely to provide an indication that an illegal act may have
occurred

27. The risk of fraudulent financial reporting increases in the presences of

a. Incentive systems based on operating income


b. Improved control systems
c. Substantial increases in sales
d. Frequent changes in suppliers

28. Which of the following is least likely included in an auditor‘s inquiry of


management while obtaining information to identify the risks of material
misstatement due to fraud?

a. Are financial reporting operations controlled by and limited to one location


b. Does it have knowledge of fraud or suspect or fraud
c. Does it have programs to mitigate fraud risks
d. Has it reported to the audit committee the nature of the company‘s internal
control

29. Which of the following should the auditors normally interview as part of their
assessment of fraud risk?

a. Senior management
b. Audit committee
c. Various employees whose duties do not include normal financial reporting
responsibilities
d. All of the given choices

30. Which of the following characteristics most likely would heighten an auditor‘s
concern about the risk of intentional manipulation of financial statements?

a. Turnover of senior accounting personnel is low


b. Insiders recently purchase additional shares of the entity‘s stock
c. Management places substantial emphasis on meeting earning projections
d. The rate of change in the entity‘s industry is low

31. When planning the audit, if the auditor has no reason to believe that
noncompliance to laws and regulations exist, he should

a. Make inquiries of management regarding their policies and their knowledge of


violations, and then rely on normal audit procedures to detect error,
irregularities, and illegalities
b. Still include some audit procedures designed specifically to uncover illegal
acts
c. Ignore the possibility of illegal acts to occur
d. Include audit procedures which have a strong probability of detecting illegal
acts

32. An audit plan is a

a. Detailed plan of analytical procedures and all substantive tests to be


performed in the course of the audit
b. Document that provides an overview of the company and a general plan for
the audit work to be accomplished, timing of the work, and other matters of
concern to the audit
c. Generic document that auditing firms have developed to lead the process of
the audit through a systematic and logical process
d. Budge of the time that should be necessary to complete each phase of the
audit procedures

33. What will an auditor who has been proposed for an audit engagement usually do
prior to accepting a new client?

a. Draft the financial statement of the client as a measure of goodwill


b. With the permission from the prospective client, contact the predecessor
auditor to determine if there are any disagreements between the client and
the audit firms
c. Obtain the potential client‘s permission to talk to the former auditor and review
work papers
d. Perform a peer review on the potential client in accordance with professional
standards
34. Philippine Standards on Auditing reacquire auditors to assess the risk of material
misstatements due to fraud

a. For every audit


b. For first-time audits
c. Sufficient to find any frauds which may exist
d. Whenever it would be appropriate

35. If, when performing analytical procedures, an auditor observes that operating
income has declined significantly between the preceding year and current year,
the auditor should next

a. Require that the decline be disclosed in the financial statements


b. Consider the possibility that the financial statements may be materially
misstated
c. Inform management that a qualified opinion on the financial statement will be
necessary
d. Determine managements responsibility for the decline and discuss the issue
with the audit committee

36. Which of the following statements best describes materiality?

a. Materiality is typically measures as a fixed percentage of assets


b. Materiality is typically measure as a fixed percentage of net income
c. Materiality does not depend on the company being audited but is solely
dependent on the auditor‘s discretion
d. Materiality provides a cutoff point at which judgment, based on the financial
statements, may be altered

37. If the auditor sets the preliminary judgment about materiality level at a relatively
low peso amount

a. More evidence will be required that for a high level


b. Less evidence will be required that for a high level
c. The same amount of evidence will be required as for a high level
d. The amount of evidence required will not be affected

38. Which of the following statement is true with regard to the relationship among
audit risk, audit evidence, and materiality?
a. The lower the inherent risk and control risk, the lower the aggregated
materiality threshold
b. Under conditions of high inherent and control risk, the auditor should place
more emphasis on obtaining external evidence and should reduce reliance on
internal evidence
c. Where inherent risk is high and control risk is low, the auditor may safely
ignore inherent risk
d. Aggregated materiality thresholds should not change under conditions of
changing risk levels

39. Which of the following is most likely to be an overall response to fraud risks
identifies in an audit?

a. Supervise members of the audit team less closely and rely more upon
judgment
b. Use less predictable audit procedures
c. Use only certified public accountants on the engagement
d. Place increased emphasis on the audit of objective transactions rather than
subjective transactions

40. When must an auditor perform analytical review procedures in a financial


statement audit?

a. Testing controls over financial cycles


b. Performing tests of substantiate balances
c. Planning the nature, timing and extent of procedures
d. Performing tests to substantiate transactions

41. The purpose of analytical procedures during the audit planning stage is to

a. Aid in planning the observation of physical inventory


b. Identify unusual circumstances that the auditor may need to investigate
further
c. Flag individual transactions for further review
d. Determine whether scales transactions are approved

42. Which of the following represents a procedure that the auditor may use because
plausible relationships among financial statement balances are expected to
exist?
a. Attributes testing
b. Enterprise risks assessment
c. Inherent tests of control
d. Analytical review

43. The main purpose of risk assessment procedures is to

a. Obtain an understanding of the entity and its environment, including its


internal control, to assess the risks of material misstatement at the financial
statement and assertion levels
b. Test the operating effectiveness of controls in preventing, or detecting and
correcting, material misstatements at the assertions level
c. Detect material misstatement at the assertion level

44. Which of the following statements is incorrect regarding obtaining an


understanding of the entity and its environment?

a. Obtaining an understanding of the entity and its environment is an essential


aspect of performing an audit in accordance with PSAs
b. Understanding of the entity and its environment establishes a frame of
reference within which the auditor plans the audit and exercises professional
judgment about assessing risks of material misstatement in the financial
statement and responding to those risks throughout the audit
c. The auditor‘s primary consideration is whether the understanding that has
been obtained is sufficient to assess the risks of material misstatement in the
financial statements and to design and perform further an audit procedures
d. The depth of the overall understanding that is required by the auditor in
performing the audit is at least equal to that possessed by management in
managing the entity

45. Which statement is incorrect regarding analytical procedures?

a. Analytical procedures may be helpful in identifying the existence of unusual


transactions or events, and amounts, ratios, and trends that might indicate
matters that have financial statement and audit implications
b. In performing analytical procedures as risk assessment procedures, the
auditor develops expectations about plausible relationships that are
reasonably expected to exist
c. When comparison of those expectation with recorded amounts or ratios
developed from recorded amounts yields unusual or unexpected
relationships, the auditor considers those results in identifying risks of
material misstatement
d. When such analytical procedures use data aggregated at a high level (which
is often the situation), the result of those analytical procedures provide a
clear-cut indication about whether a material misstatement may exist

46. Which statement is correct regarding business risk?

a. The risk of material misstatements in the financial statements is broader than


business risk, through it includes the latter
b. The auditor should identify or assess all business risks
c. All business risks give risk to risks of material misstatement
d. A business risk may have an immediate consequence of the risks of
misstatement for classes of transactions, account balances, and disclosures
at the assertion level or the financial statements as a whole

47. Inquiries directed towards those charged with governance may most likely

a. Related to their activities concerning the design and effectiveness of the


entity‘s internal control and whether management has satisfactorily
responded to any finding from these activities
b. Help the auditor understand the environment in which the financial statements
are prepared
c. Relate to changes in the entity‘s marketing strategies, sales trends, or
contractual arrangement with its customers
d. Help the auditor in evaluating the appropriateness of the selection and
application of certain accounting policies

48. Which statement is incorrect regarding significant risks that require special audit
consideration?

a. The auditor should determine which of the identified risks are, in the auditor‘s
judgment, require special audit consideration
b. The auditor excludes the effect of identified controls related to the risks to
determine whether the nature of the risk, the likely magnitude of the potential
misstatement including the possibility that the risk may give risk to multiple
misstatements, and the likelihood of the risk occurring are such that they
require special audit consideration
c. Routing, non-complex transactions that are subject to systematic processing
are more likely to give rise to significant risks because they have higher
inherent risks
d. Significant risks are often derived from business risks that may result in a
material misstatement

49. The assessment of the risks of material misstatement at the financial statement
level is affected by the auditor‘s understanding of the control environment.
Weaknesses in the control environment ordinarily will lead the auditor to

a. Have more confidence in internal control and the reliability of audit evidence
generated internally within the entity
b. Conduct some audit procedures at an interim date rather than at period end
c. Modify the nature of audit procedures to obtain more persuasive audit
evidence
d. Decrease the number of locations to be included in the audit scope

50. The auditor should determine overall responses to address the risks of material
misstatement at the financial statement level. Such responses least likely include

a. Emphasizing to the audit team the need to maintain professional skepticism in


gathering and evaluating audit evidence
b. Assigning more experienced staff or those with special skills or using experts
c. Incorporating additional elements of unpredictability in the selection of further
audit procedures to be performed
d. Performing substantive procedures at an interim date instead of at period end

51. Which statement is incorrect regarding the nature of further audit procedures?

a. The nature of further audit procedures refers to their purpose and their type
b. Certain audit procedures may be more appropriate for some assertions then
others
c. The higher the auditor‘s assessment of risk, the less reliable and relevant is
the audit evidence sought by the auditor from substantive procedures
d. The auditor is required to obtain audit evidence about the accuracy and
completeness of information produced by the entity‘s information system
when that information is used in performing audit procedures

52. Which statement is incorrect regarding the extent of further audit procedures?
a. Extent included the quantity of a specific audit procedures to be performed
b. The extent of an audit procedure is determined by the judgment of the auditor
after considering the materiality, the assessed risk, and the degree of
assurance the auditor plans to obtain
c. The auditor ordinarily decreases the extent of audit procedures as the risk of
material misstatement increases
d. Increasing the extent of an audit procedure is effective only if the audit
procedure itself is relevant to the specific risk

53. The auditor should design and perform further audit procedures whose nature,
timing and extent are responsive to the assessed risks of material misstatement
at the assertion level. Which of the following is the most important consideration
in responding to the assessed risk?

a. The nature of the audit procedures


b. The timing of the audit procedures
c. The extent of the audit procedures
d. All of these are equally important

54. While assessing the risk of material misstatement, the auditor identify risks,
relate risk to what could go wrong, consider the magnitude of risks and:

a. Assess the risk of misstatements due to noncompliance to laws and


regulations
b. Consider the complexity of the transactions involved
c. Consider the likelihood that the risks could result in material misstatements
d. Determine the materiality level

55. Which of the following would heighten an auditor‘s concern about the risk of
fraudulent financial reporting?

a. Inability to generate positive cash flows from operations while reporting large
increases in earning
b. Management‘s lack of interest in increasing the dividend paid on common
stock
c. Large amount of liquid assets that are easily convertible into cash
d. Inability to borrow necessary capital without obtaining waivers on debt
covenants
56. Which of the following is least likely considered a financial statement audit risk
factor?

a. Management operating and financing decisions are dominated by top


management
b. A new client with no prior audit history
c. Rate of change in the entity‘s industry is rapid
d. Profitability of the entity relative to its industry is inconsistent

57. Which of the following is most likely to be considered a risk factor relating to
fraudulent financial reporting?

a. Low turnover of senior management


b. Extreme degree of competition within the industry
c. Capital structure including various operating subsidiaries
d. Sales goals in excess of any of the preceding three years

58. Which of the following is correct concerning requirement about auditor‘s


communications about fraud?

a. Fraud that involves senior management should be reported directly to the


audit committee regardless of the amounts involved
b. All fraud with a material effect on the financial statements should be reported
directly by the auditor to the Securities and Exchange Commission
c. Fraud with a material effect on the financial statement should ordinarily be
disclosed by the auditor through the use of an emphasis of a matter
paragraph added to the audit report
d. The auditor has no responsibility to disclose fraud outside the entity under
any circumstances

59. Which of the following factors most likely would heighten an auditor‘s concern
about the risk of fraudulent financial reporting?

a. Large amounts of liquid assets that are easily convertible into cash
b. Low growth and profitability as compared to other entity‘s in the same industry
c. Financial management‘s participation in the initial selection of accounting
principles
d. An overly complex organizational structure involving unusual lines of authority
60. Which of the following is most likely to be an overall response to fraud risks
identified in an audit?

a. Only use certified public accountants on the engagement


b. Place increased emphasis on the audit of objective transactions rather than
subjective transactions
c. Supervise members of the audit team less closely and rely more upon
judgment
d. Use less predictable audit procedures
QUIZZERS

1. As part of audit planning, CPAs should design audit programs for each individual
audit and should include audit steps and procedures to

a. Detect and eliminate fraud


b. Increase the amount of management information available
c. Provide assurance that the objectives of the audit are met
d. Ensure that only material items are audited

2. Preplanning the audit involves several key activities. Which of the following would
not be included in preplanning an audit?

a. Investigating the client‘s background


b. Determining the likelihood of issuing an unqualified audit opinion on the
client‘s financial statements
c. Communicating with the prospective client‘s prior auditor to inquire about any
disagreements with the client
d. Understanding the client‘s reasons for obtaining an audit

3. During audit planning, which of the following is not a factor that affects the
auditor‘s judgment as to the quantity, type and content of working papers?

a. The auditor‘s prelimary assessment of control risk


b. The auditor‘s prelimary evaluation of inherent risk based on discussions with
the client
c. The nature of the client‘s business
d. They type of report to be issued by the auditor

4. Which one of the following is not considered a valid source of information about
the client‘s processes?

a. Confirmation from third-parties


b. Review of the client‘s budget
c. A tour of the client‘s plant facility
d. Management inquiry

5. The element of the audit planning process most likely to be agreed upon with the
client before the implementation of audit strategy is the determination of the

a. Method of statistical sampling to be used in confirming accounts receivable


b. Pending legal matters to be included in the inquiry of the client‘s attorney
c. Evidence to be gathered to provide a sufficient basis for the auditor‘s opinion
d. Schedules and analyses to be prepared by the client‘s staff

6. Which of the following is not a component of audit planning?

a. Observing the client‘s annual physical inventory taking and making test
counts of selected items
b. Making arrangement with the client concerning the timing of audit fieldwork
and use of the client‘s staff in completing certain phases of the examination
c. Obtaining an understanding of the business
d. Developing audit program

7. R & O, CPAs, have been retained as the auditors of City Corporation. What are
the R & O‘s responsibilities with regards to contracting City Corporation‘s
predecessor auditors?

a. If City Corporation had a disagreement with the predecessor auditors, R & O


should not contact the predecessor auditors
b. R& O is not required to attempt communication with the predecessor auditors
under any circumstances
c. R&O should attempt communications with the predecessor auditor and ask if
they had any accounting policy disagreements with City Corporation
d. It would be unethical for R & O to ask the predecessor auditors about the
integrity of City Corporation‘s management

8. An initial audit requires more audit time to complete than a recurring audit. One
of the reasons for this is that

a. The new auditors are usually assigned to an initial audit


b. The predecessor auditors need to be consulted
c. The client‘s business, industry, and internal control are unfamiliar to the
auditor and he needs to carefully study them
d. A large proportion of customer accounts receivable need to be confirmed on
an initial audit

9. Prior to beginning the fieldwork on a new audit engagement in which he does not
possess industry expertise, the CPA should

a. Reduce audit risk by lowering the preliminary levels of materiality


b. Design special substantive tests to compensate for the lack of industry
expertise
c. Engage financial experts who are familiar with the nature of the industry
d. Obtain a knowledge of matters that relates to the nature of the entity‘s
business and industry

10. Which of the following will an auditor least likely discuss with the former auditors
of a potential client prior to acceptance of an audit engagement?

a. Integrity of the management


b. Fees charges for the services
c. Disagreements between the predecessor auditor and the management
regarding accounting principles
d. Reasons for changing audit firms

11. What is the most likely course of action to be taken by an auditor in assessing
management integrity?

a. Tour the plant


b. Review the minutes of the board of directors
c. Research the background and histories of officers
d. Review the bank reconciliation statements

12. An engagement letter should be written before the start of an audit because

a. It may limit the auditor‘s legal liability by specifying the auditor‘s


responsibilities
b. It specifies the client‘s responsibility for preparing schedules and making the
record available to the auditor
c. It specifies the basis for billing the audit for the upcoming year
d. All of the choices given are correct

13. When a CPA is approached to perform an audit for the first time, the CPA should
make inquiries of the predecessor auditor. This is a necessary procedure
because the predecessor may be able to provide the successor with information
that will assist the successor in determining whether:

a. The predecessor‘s work should be utilized


b. The company follows the policy of rotating its auditors
c. In the predecessor‘s opinion, internal control of the company is satisfactory
d. The engagement should be accepted

14. A written understanding between the auditor and the client concerning the
auditor‘s responsibility for the discovery of noncompliance to laws is usually set
forth in a(an):

a. Client representation letter


b. Letter of audit inquiry
c. Management letter
d. Engagement letter

15. Prior to acceptance of an audit engagement with a client who has terminated the
services of the predecessor auditor, the CPA should

a. Contact the predecessor auditor without advising the prospective client and
request a complete report of the circumstances leading to the termination of
the engagement with an understanding that all information disclosed will be
kept confidential
b. Accept the engagement without contacting the predecessor auditor since the
CPA can include audit procedures to verify the reason given by the client for
the termination of the engagement
c. Not communicate with the predecessor auditor because this would in effect
be asking the auditor to violate the confidential relationship between an
auditor and the client
d. Advise the client of the intention to contact the predecessor auditor and
request a permission for the contact

16. Before accepting an audit engagement, a successor auditor should make specific
inquiries of the predecessor auditor regarding the predecessor‘s

a. Opinion of any subsequent event occurring since the predecessor‘s audit


report was issued
b. Understanding as to the reasons for the change of auditors
c. Awareness of the consistency in the application of PFRS between periods
d. Evaluation of all matters of continuing accounting significance

17. A successor auditor would most likely make specific inquiries of the predecessor
auditor regarding

a. Specialized accounting principle being used by the client‘s industry


b. The competency of the client‘s internal audit staff
c. The uncertainty inherent in applying sampling procedures
d. Disagreements with management as to auditing procedures

18. Which of the following statement concerning materiality thresholds is incorrect?

a. Aggregate materiality threshold are a function of the auditor‘s preliminary


judgment concerning audit risk
b. In general, the more misstatements the auditor expects, the higher should be
the aggregate materiality threshold
c. The smallest aggregate level of errors or fraud that could be considered
material to any of the financial statements is referred to as a ―materiality
threshold‖
d. Materiality thresholds may change between the planning and review stages of
the audit. These changes may be due to quantitative and/or qualitative factors

19. Which of the following concepts about materiality is incorrect?

a. Materiality is directly related to the acceptable level of detection risk


b. Materiality does not apply if internal control is highly effective
c. Materiality is a matter of professional audit judgment
d. Materiality is more closely related to fieldwork and reporting standards than to
general standards

20. Which of the following would not be a source of information about the risk of a
potential new audit client?

a. The predecessor auditor


b. Management
c. The internet
d. The new auditor‘s permanent file

21. In comparing management fraud with employee fraud, the auditor‘s risk of failing
to discover the fraud is greater for:

a. Employee fraud because of the larger number of employees in the


organization
b. Employee fraud because of the higher crime rate among blue collar workers
c. Management fraud because of management‘s ability to override existing
internal controls
d. Management fraud because managers are inherently smarter than
employees

22. Management‘s integrity affects all of the following risks except:

a. Enterprise risk
b. Financial reporting risk
c. Engagement risk
d. All of the above risks are affected

23. The auditor is most likely to presume that a high risk of irregularities exists if

a. The client is a multinational company that does business in numerous foreign


countries
b. The client does business with several related parties
c. Inadequate segregation of duties places an employee in a position to
perpetrate and conceal thefts
d. Inadequate employee training results in lengthy EDP exception reports each
month

24. Which of the following audit risk components may be assessed in nonquantitaive
terms?

Inherent Risk Control Risk Detection Risk


a. Yes Yes No
b. Yes No Yes
c. No Yes Yes
d. Yes Yes Yes

25. Which of the following combinations of engagement risk, audit risk, and
materiality would lead the auditor to most audit work?

Engagement Risk Audit Risk Materiality

a. Low High High


b. Moderate Low Low
c. Low Moderate Low
d. High High High
26. Which of the following conditions justifies an auditor‘s decision of raising the
materiality level?

a. Internal control over revenue and receipts cycle is excellent


b. Application of analytical procedures reveals a significant increase in sales
revenue in December, the last month of the fiscal year
c. Internal control over shipping, billing and recording of sales revenue is weak
d. Study of the business reveals that the client recently acquired a new company
in an unrelated industry

27. Which of the following does an auditor least likely perform in assessing audit
risk?

a. Gather audit evidence in support of recorded transactions


b. Obtain an understanding of the client‘s system of internal control
c. Understand the economic substance of significant transactions completed by
the client
d. Understand the entity and the industry in which it operates

28. Which type of risk does the management of a company have the most control
over in the short term?

a. Inherent risk
b. Control risk
c. Detection risk
d. Sufficiency risk

29. In which of the following order would the auditors perform the following steps?

a. Determine audit risk; assess control risk; determine detection risk; set
materiality
b. set materiality; determine audit risk;assess control risk; determine detection
risk
c. set materiality; assess control risk;determine detection risk; determine audit
risk
d. Determine audit risk;set materiality;assess control risk;determine detection
risk

30. If the results of the auditor‘s tests of controls induce the auditor to change the
assessed level of control risk for inventory from 0.2 to 0.4 and audit risk and
inherent risk remain constant, what is the effect on the acceptable level of
detection risk?

a. A change in detection risk cannot be calculate because audit risk and


inherent risk values are not given
b. Detection risk would increase from 0.3 to 0.6
c. Detection risk would decrease from 0.4 to 0.2
d. Detection risk would not change since audit risk and inherent risk do not
change

31. Which of the following may cause the management to intentionally understate
profits?

a. Management wants to create ―cookie jar‖ reserves fir a rainy day


b. The company is under scrutiny by tax authorities
c. The company is suffering a large loss and wants to take a ―big bath‖
d. All of the given choices

32. Which of the following is true?

a. Auditors are responsible for detecting all fraudulent financial reporting


b. Auditors must specifically consider fraud risk from overstating liabilities
c. Auditor must specifically consider fraud risk from management override of
controls
d. All of them are true

33. Why should the auditor plan more work on individual accounts as lower
acceptable levels of both audit risk and materiality are established?

a. To find smaller errors


b. To find larger errors
c. To increase the tolerable error in the accounts
d. To decrease the risk of overreliance

34. With respect to error and fraud, the auditor should plan to

a. Search for errors or fraud that would have a material effect on the financial
statements
b. Discover errors or fraud that would have a material effect on the financial
statement
c. Search for errors that would have a material effect and for fraud that would
have either material or immaterial effects on the financial statement
d. Search for fraud that would have a material effect and for errors that would
have either material or immaterial effects on the financial statements

35. The auditor‘s responsibility for identifying ―direct-effect‖ non-compliance to laws


and regulations differs from their responsibility for detecting

a. Errors
b. Indirect-effect non-compliance to laws and regulations
c. Fraud
d. Management fraud

36. Which of the following might be considered a ―red flag‖ that may indicate possible
fraud in a large manufacturing company with several subsidiaries?

a. The existence of a financial subsidiary


b. A consistent record of above average return on investment for all subsidiaries
c. Complex sales transactions and transfers of funds between affiliated
companies
d. Use of separate bank accounts for payrolls by each subsidiary

37. Experience has shown that certain conditions in an organization are symptoms of
possible management fraud. Which of the following conditions would not be
considered an indicator of possible fraud?

a. Managers are regularly assuming subordinates ‗duties


b. Managers are dealing in matters outside their profit center‘s scope
c. Manager are not complying with corporate directive and procedures
d. Manager are subject to formal performance reviews on a regular basis

38. Warning signs that cause the auditor to question management integrity must be
taken seriously and pursued vigorously. Which of the following may lead the
auditor to suspect management dishonesty?

a. The president/CEO of the client corporation has held numerous meetings with
the controller for the purpose of discussing accounting practices that will
maximize reported profits
b. The client has been names as a defendant in a product liability suit
c. The client has experienced a decrease in revenue from increased import
competition
d. A new statutory regulation making customer licenses more difficult to obtain
may adversely affect the client‘s operations

39. Which of the following methods may be used to commit fraudulent financial
reporting?
a. Overstate revenues
b. Understate liabilities
c. Fail to provide adequate disclosure
d. Each of the given choice can be used to commit fraudulent financial reporting

40. Which of the following internal control policies, when absent, would increase the
opportunity for fraud?

a. Appropriate segregation of duties or independent checks


b. Job applicant screening for employees with access to assets
c. Mandatory vacations for employees with access to assets
d. The absence of any of the given choices increases the opportunity for fraud.

41. Whom should the auditors contact when they suspect a fraud?

a. Senior management
b. Expected perpetrators of the fraud
c. Audit committee of the board of directors
d. Either the senior management or the audit committee

42. Analytical procedures performed in the planning stage of an audit suggest that
several accounts have unexpected relationships. The results of these procedures
most likely would indicate that:

a. Irregularities exist among the relevant account balances


b. Additional tests of details are required
c. Internal control activities are not operating effectively
d. The communication with the audit committee should be revised

43. Which of the following statements identifies a potential weakness when


comparing client fata with the industry?

a. Industry data may not be representative of the client‘s business


b. Other companies in the industry could use accounting principles different from
what the client is using
c. Data bases are comprised of data from thousands of companies of various
sizes, which may limit the effectiveness of the comparisons
d. All the given choices are weaknesses

44. Which of the following statements is correct with respect to the auditor‘s use of
analytical procedures?

a. Analytical procedures are time saving procedures that auditors may employ at
their discretion
b. Analytical procedures are powerful tools that are required to be used during
the planning and testing phases of the audit
c. Analytical procedures may be used to identify misstatements in a client‘s
accounts
d. Analytical procedures are required to be used during the planning and
completion phases of the audit.

45. Which of the following is not an information source for developing analytical
procedures used in the audit?

a. Relationships among financial statement elements


b. Relationships between financial and relevant nonfinancial data
c. Comparison of financial data with anticipated results (e.g., budgets and
forecasts)
d. Comparison of current year financial data with projections for next year‘s
financial results

46. Which of the following results from analytical procedures might indicate obsolete
inventory?

a. A decline in inventory turnover


b. A decline in day‘s sales in inventory
c. A decline in the gross margin ratio
d. An increase in operating margin

47. Auditors try to identify predictable relationship when using analytical procedures.
Which of the following accounts would most likely yield the highest level of
evidence regarding relationships that involve transactions?

a. Accounts payable
b. Accounts receivable
c. Payroll expense
d. Advertising expense

48. Analytical procedures are performed in the following order:


a. Calculate predictions and compare them to the recorded amount; define a
significant difference; develop an expectation; investigate significant
differences
b. Calculate predictions and compare them to the recorded amount;investigate
significant differences;define a significant difference; develop an expectation
c. develop an expectation;define a significant difference;calculate predictions
and compare them to the recorded amount;investigate significant differences
d. develop an expectation;calculate predictions and compare them to the
recorded amount;define a significant difference;investigate significant
differences

49. An auditor compares experience as a percent of sales to expectations. This is


an example of:

a. Ratio analysis
b. Trend analysis
c. Internal control analysis
d. Vertical analysis

50. How is the audit program best described at the beginning of the audit process?

a. Temporary
b. Conclusive
c. Confirmed
d. Optional

51. After discovering that a related-party transaction exists, the auditor should be
aware that the

a. Substance of the transaction could be significantly different from its form


b. Adequacy of disclosure of the transaction is secondary to its legal form
c. Transaction is assumed to be outside the ordinary course of business
d. Financial statements should recognize the legal form of the transaction rather
than its substance

52. In which of the following would the auditor most likely find information about
compensation of corporate officers?

a. Corporate charter
b. Corporate by-laws
c. Corporate minutes
d. Audit engagement letter

53. The existence of a related-party transaction may be indicated when another


entity
a. Sells real estate to the corporation at a price that is comparable to its
appraised value
b. Absorbs the expenses of the corporation
c. Borrow from the corporation at a rate of interest which equals the current
market rate
d. Lends to the corporation at a rate of interest, which equals the current market
ratio

54. An auditor judges an item to be immaterial when planning an audit. . However,


the auditor may still include the item if it is subsequently determined that

a. sufficient number of staff is available


b. Adverse effects related to the item are likely to occur.
c. Related evidence is reliable.
d. Miscellaneous income is affected.

55. Which of the following is least likely required in an audit?

a. Test appropriateness of journal entries and adjustment


b. Review accounting estimates for biases
c. Evaluate the business rationale for significantly unusual transactions
d. Make a legal determination of whether fraud has occurred

56. Of the following procedures, which one is not considered a purl "obtaining an
understanding of the client's environment?"

a. Reading trade publications to gain a better understanding of the client's


industry
b. Confirming customer accounts receivable for existence and valuation
c. Touring the client‘s manufacturing and warehousing facilities to gain a clearer
understanding of the entity's operations
d. Studying the internal controls over cash receipts disbursements

57. The element of the audit planning process most likely to be agreed upon with the
client before the implementation of the audit strategy is the determination of the

a. Timing of inventory observation procedure to be performed


b. Evidence to be gathered to provide a sufficient basis for the auditor‘s opinion
c. Procedures to be undertaken to discover litigation, claims, and assessments.
d. Pending legal matters to be included in the inquiry of the client's attorney.

58. Which of the following concepts is most useful in assessing the scope of an
auditor's program relating to various accounts?

a. Attribute sampling
b. Materiality
c. The reliability of information
d. Management fraud

59. With respect to the auditors planning of a year-end examination, which of the
following statements is always true?

a. An engagement proposed after the fiscal year ends should not accepted.
b. An inventory count must be observed at the balance sheet date
c. The client's audit committee should not be told of the specific audit
procedures that will be performed.
d. It is an acceptable practice to-carry out substantial parts of the examination at
interim dates

60. Which of the following is not a consideration in the development of audit


programs?

a. Internal control over the recording of plant asset additions and repairs and
maintenance expenditures is found to be weak.
b. The client constructed a major addition to its central manufacturing facility
during the year under audit.
c. The client is a private university located in Southern Philippines.
d. The members of the board of directors are elected by the stockholders during
the annual meeting.

61. An audit program provides a proof that

a. Sufficient competent evidential matter is obtained.


b. The work is adequately planned.
c. There is compliance with generally accepted standards of reporting.
d. There is a proper study and evaluation of internal control.

62. The principal reason for developing a written audit program is to assure that the

a. audit work is properly supervised


b. Audit work is properly planned and documented.
c. audit report contains only significant-findings
d. Work of different auditors is properly coordinated.

63. One of the primary uses of an audit program is to

a. Serve as a tool for planning, directing, and controlling the audit work.
b. Document an auditors understanding of the internal control
c. Provide for a standardized approach to the audit engagement
d. Delineate the audit risk accepted by the auditor.

64. An audit program is


a. the detailed plan of audit procedures to be performed in the course of the
audit.
b. an overview of the company and a general plan for the work to be
accomplished
c. a generic document that auditing firms have developed to lead the process of
the audit through a. systematic logicalprocess
d. a budget of the time that should be necessary to complete each phase of the
audit procedures.

65. In deciding whether to use the work of internal auditors, external auditors must
evaluate the internal auditors‘
a. Objectivity and competence.
b. Independence and professionalism.
c. education and certification
d. age and gender

66. Which of the following is not true regarding planning in an electronic


environment?

a. The definition of auditing is not changed


b. The purposes of auditing is not changed
c. The procedures used are not changed
d. Auditing standards are not changed

67. For which of the following judgments may on independent auditor share
responsibility with an entity's internal auditor who is assessed to be both
competent and objective?

Assessment of inherent risk assessment of control risk


a. Yes Yes
b. No No
c. No Yes
d. No No

68. After studying and evaluating a client's existing internal control, an auditor has
concluded that the policies and procedures are well designed and functioning as
intended. Under these circumstances, the auditor would most likely

a. Perform further control tests to the extent outlined in the audit program.
b. Determine the control policies and procedures that should prevent or detect
errors and fraud.
c. Setacceptable detection, risk at a higher level than would be set under
conditions of weak internal control.
d. set acceptable detection risk at a lower level than would be set under
conditions of weak internal control.

69. Which of the following matters is generally included in an auditor's engagement


letter?

a. Limitations of the engagement


b. Factors to be considered in establishing preliminary judgment about
materiality
c. Management's liability for non-compliance to laws committed by its
employees
d. The auditor's responsibilityto obtain negative assurance relating to the
occurrence of non-compliance to laws

70. Which of the following factors most likely would lead a CPA to conclude that a
potential audit engagement should not be accepted?

a. There are significant related party transactions that management claims


occurred in the ordinary course of business.
b. Internal control activities requiring the segregation of duties are subject to
management override.
c. Management continues to employ an inefficient system of information
technology to record financial transactions
d. It is unlikely that sufficient evidence is available to support an opinion on the
financial statements.

71. Which of the following is an example of fraudulent financial reporting?

a. The company management falsifies inventory count tags thereby overstating


ending inventory and understating cost of goods sold.
b. An employee diverts customer payments to his personal use, concealing his
actions by debiting an expense account, thus overstating expenses.
c. An employee steals inventory and the shrinkage is recorded in cost of goods
sold.
d. An employee borrows tools from the company and neglects to return them the
cost is reported as a miscellaneous operating expense.

72. Which of the following statements is accurate about "fraud risk factors"
considered when conducting an audit?

a. Factors whose presence indicates that fraud exists.


b. Factors whose presence often have been observed in circumstances where
frauds have occurred.
c. Factors whose presence will require modification to planned audit
procedures.
d. Factors obtained during the audit which leads to required communications
with the audit committee.

73. Which of .the following is not an example of a likely adjustment in the auditor‘s
overall audit approach when significant risk is found to exist?

a. Apply increased professional skepticism about material transactions


b. Increase the assessed level of detection risk
c. Assign personnel with particular skill to areas of high risk.
d. Obtain increased evidence about the appropriateness of management's
selection of accounting principles.

74. Which of the following conditions identified during the audit increases the risk of
employee fraud?

a. Large amount of cash in the bank.


b. Existence of a mandatory vacation policy for employees performing inventory
key functions.
c. Inventory items of small size, but high value
d. Presence of reconciling items on a client prepared year-end proof of cash.

75. Which of the following is not ordinarily considered a factor indicative of increased
financial reporting risk when an auditor is considering a client‘s risk assessment
policies?

a. fixed monthly salaries for sales personnel


b. implementation of a new information system
c. Rapid growth of the organization
d. Corporate restructuring
MODULE 6

CONSIDERATIONS OF INTERNAL CONTROL

PSA-BASED QUESTIONS

1. An auditor considers internal control in order to

a. Determine whether assets are safeguarded


b. Suggest improvements in internal control.
c. Plan audit procedures
d. Express an opinion

2. The primary purpose of the auditor's consideration of internal control is to provide


a basis for

a. Determining whether procedures and records that are concerned with the
safeguarding of assets are reliable.
b. Constructive suggestions to clients concerning deficiencies in internal control
c. Determining the nature, timing, and extent of audit tests to be applied
d. Expressions of an opinion.

3. The primary objective of procedures to be performed in obtaining an


understanding of internal control is to provide an auditor with

a. Evidential matter for use in reducing detection risk


b. Knowledge necessary to plan the audit
c. A basis for which to modify tests of controls
d. Information necessary to prepare flowcharts.

4. A reason to establish internal control is to

a. Have a basis of planning the audit.


b. Provide reasonable assurance that the objectives of the organization are
achieved
c. Encourage compliance with organizational objectives.
d. Ensure the accuracy, reliability, and timeliness of information

5. What is the primary purpose of effective internal control organization?

a. Achievement of certain organizational gods.


b. Completion of a successful audit for the entity.
c. Shareholders' involvement in the company‘s success
d. Obtaining profitability and financial strength.

6. An effective internal control is not expected to provide. a reasonable assurance


regarding the achievement of objectives concerning

a. Reliability of financial reporting.


b. Compliance to applicable laws and regulations.
c. Elimination of material misstatements.
d. Effectiveness and efficiency of operations.

7. Internal control is primarily establishedwithin a companyto accomplish which of


the following objectives?

a. Prevent irregularities
b. Provide reasonable assurance that the company's objective be achieved
c. Catch all errors that may occur in the company
d. Aid in the effective auditing of the company

8. Which of the following is not an element-of an entity's internal control?

a. Control risk
b. Control activities
c. The information System
d. The control environment

9. One of the major components of an organization‘s internal control structure


includes:

a. Audit control risk


b. The cost-benefit ratio
c. Risk assessment
d. Communication

10. Which of the following best describes the interrelated components of internal
control?

a. Organizational structure, management philosophy, and planning.


b. Control environment, risk assessment, control activities, information and
communication systems, and monitoring
c. Risk assessment backup facilities, responsibility accounting and natural laws
d. Legal environment of the firm, management philosophy, and organizational
structure.

11. Which of the following is not typically one of management's concerns in


designing an effective internal control structure?

a. Reliability of financial reporting.


b. Obtaining the best internal control system possible.
c. Compliance with applicable laws and regulations
d. Efficiency and effectiveness of operations.

12. The auditor‘s review of the client's internal control is documented in order to
substantiate

a. Conformity of the accounting records, with Philippine Financial Reporting


Standards.
b. Representation as to adherence to requirements of management.
c. Representation as to compliance with Philippine Standards on Auditing
d. The fairness of the financial statement presentation.

13. An auditor would most likely .be concerned with internal control policies and
procedures that provide reasonable assurance about:
a. The efficiency of management's decision-making process
b. Appropriate prices thatthe entity should charge for its products.
c. Methods of assigning production tasks to employees
d. The entity's ability to process and summarize financial data.

14. When considering the effectiveness of internal control, the auditor should
recognize that inherent limitations do exist. Which following is an example of
inherent limitations in a client‘s internal control?

a. The effectiveness of procedures depends or the segregation of employee


duties.
b. Procedures are designed to assure the execution and of transactions in
accordance with management's authorization
c. In the performance of most control procedures, there are possibilities of errors
arising from mistakes in judgment
d. Procedures for handling large numbers of transactions are processed by
electronic data processing equipment.
15. Which of the following is least likely to be an evidence of effectiveness of
controls?

a. Cancelation of supporting document.


b. The policy of documenting the usage of computer program
c. Confirmation of bank balances.
d. Signatures on authorization forms.

16. Which of the following is an inherent limitation of any client‘s internal control?

a. The benefits expected to be derived from effective internal control should not
exceed the costs of such control.
b. The competence and integrity of client personnel provide an environment
conducive to control and provide assurance that effective control will be
achieved.
c. The procedures that are designed to assure the execution and recording of
transactions in accordance with proper authorizations are effective against
frauds perpetrated by management
d. The procedures whose effectiveness depends on segregation of duties can
be circumvented by collusion.

17. A system of internal control, regardless of how carefully designed and


implemented, contains certain inherent limitations. Which of the following errors
or irregularities is not caused by an inherent limitation?

a. The president and chief executive officer, with the assistance of the corporate
controller, inflated earnings by recording fictitious sales at year-end.
b. A newly-installed electronic data processing system failed to provide for a
comparison of sales order amount with prior customer balance and credit
limit. This resulted in numerous sales to customers who had already
exceeded their credit limits.
c. Numerous recording errors occurred because persons analyzing and
recording transactions did not have the necessary accounting background.
d. A computer programmer and a computer operator conspired to divert funds
from the company to an account controlled by dishonest employees.

18. Corporate directors, management, external auditors, and internal auditors all play
important roles in creating a proper control environment. Top management is
primarily responsible for
a. Establishing a proper environment and specifying overall internal control.
b. Reviewing the reliability and integrity of financial information and the means
used to collect and report such information.
c. Ensuring that external and internal auditors adequately monitor the control
environment.
d. Implementing and monitoring controls that are designed by the board of
directors

19. The primary responsibility for establishing and maintaining internal controls rests
with the

a. internal auditors
b. management
c. Securities and Exchange Commission
d. External auditors.

20. Which of the following is not a part of the control environment?

a. Management philosophy and operating style


b. Organizational structure
c. Information rind communication systems
d. Assignment of authority and responsibility

21. Internal control is a function of management, and effective control is based upon
the concept of charge and discharge of responsibility and duty. Which of the
following is an important feature of effective internal control?

a. Responsibility for accounting and financial duties should be assigned to one


responsible officer.
b. Responsibility for the performance of each duty must be fixed
c. Responsibility for the accounting duties must be borne by the auditing
committee of the company.
d. Responsibility for accounting activities and duties must be assigned only to
employees who are bonded.

22. The control environment includes which of the following?

a. Control activities
b. Management philosophy and operating style
c. Assessing activity level risks
d. Application level controls

23. Which of the following is not a major emphasis in the design of effective internal
accounting control?

a. Assets are properly protected


b. Duties are segregated.
c. Transactions are authorized
d. Processes are efficient.

24. A proper understanding of the client's internal control is an integral part of the
audit planning process. The results of the understanding

a. Must be reported to the major stakeholders.


b. always require the auditor in testing the control policies and procedures.
c. are not reported to client management.
d. May be used as the basis for withdrawing from an audit engagement

25. The auditor who becomes aware of reportable conditions is required to


communicate such a weakness to the

a. Audit committee and client‘s legal counsel


b. Board of directors and internal directors.
c. Senior management and board of directors
d. Internal auditors and senior management.

26. As part of understanding internal controls, an auditor is not required to

a. Consider factors that affect the risk of material misstatement


b. Ascertain whether internal control policies and procedures have been placed
in operation
c. Identify the types of potential misstatements that may occur
d. Obtain knowledge about the operating effectiveness of internal control

27. The purpose of tests of controls is to provide reasonable assurance that the

a. Accounting treatment of transactions and balances is valid and proper


b. Control procedures are functioning as intended
c. Entity has complied with disclosure requirements of PFRS
d. Entity has complied with requirements of quality control.
28. Tests of controls are used to test whether controls are

a. Operating effectively
b. Placed in operation (implemented)
c. Properly accumulated into balance sheet totals
d. Properly documented by the client

29. After documenting internal control in an audit engagement, the auditor may
perform tests on

a. Those controls that the auditor plans to rely on


b. Those controls in which deficiencies were identified.
c. Those controls that have a material effect on the financial statement balances
d. A random sample of the controls that were reviewed.

30. Which of the following audit techniques would most likely provide an auditor with
the most assurance about the effectiveness of the operation of an internal control
procedure?

a. Inquiry of client personnel


b. Recomputation of an accountbalance
c. Observation of client personnel
d. Confirmation of balances or transactions with outside parties

31. After the study and evaluation of a client's internal control policies and
procedures has been completed, an auditor might decide to

a. increase the extent of substantive testing in areas where the internalcontrol


policies and procedures are strong
b. Reduce the extent of control testing in areas where the in control policies and
procedures are strong.
c. Reduce the extent of both substantive and control testing in areas where the
internal control policies and procedures are strong
d. Increase the extent of substantive testing in areas where internal controls are
weak.

32. The Philippine Standards on Auditing require the auditor to obtain an


understanding of the client's internal controls
a. for every audit.
b. for first-time audits.
c. sufficient to find any frauds which may exist.
d. whenever it would be appropriate.

33. Control testing is performed in order to determine whether or not

a. The assessed level of control risk can be reduced


b. Necessary controls are absent.
c. Incompatible functions exist.
d. material peso errors exist

34. The auditor is studying internal control policies and procedures within the sales,
shipping, and billing subset of the revenue cycle. Which of the following
conditions suggests a need for additional testing of controls?

a. Internal control is found to be weak with regard to shipping and billing


b. Internal control over sales, billing; and shipping appears strong but 80% of
sales revenue is attributable to three major customers
c. Internal control over billing and shipping is thought to be strong and the
auditor considers additional testing of selected controls will result in a major
reduction in substantive testing
d. Internal control over the recording of sales is found to be weak and the sales
are evenly divided among a large number of customers

35. To obtain an understanding of the relevant policies and procedures of internal


control, the auditor performs all of the following except:

a. Make inquiries
b. Make observations
c. Inspect documents and records
d. Design substantive tests

36. It is most appropriate that tests of controls be applied to transactions and


controls

a. At the balance sheet date


b. At each quarterly interim period.
c. For the entire period under audit
d. At the beginning of the fiscal period.
37. To obtain evidence about control risk, an auditor ordinarily selects tests from a
variety of techniques, including

a. Analysis
b. Confirmation
c. Reperformance
d. Comparison

38. An auditor wishes to perform tests of controls on a client's cash disbursements


procedures. If the controls leave no audit trail of documentary evidence; the
auditor most likely will test the procedures by

a. confirmation and observation


b. analytical procedures and confirmation
c. observation and inquiry
d. inquiry and analytical procedures

39. Management's attitude toward aggressive financial reporting and its emphasis on
meeting projected profit goals most likely would significantly influence an entity's
control environment when

a. The audit committee is active in overseeing the entity's financial reporting


policies.
b. external policies established by parties outside the entity affect its accounting
practices
c. Management is dominated by one individual who is also a shareholder.
d. Internal auditors have direct access to the board of directors and entity
management.
QUIZZERS

1. Of the following control environment characteristics, identify the one that


contributes most to effective internal control.

a. The audit committee consists of the president, two vice-dents, and the
corporate controller.
b. The company does not have a centralized human resource function
c. The company has an effective internal audit staffs that monitors controls on a
continuous basis.
d. The company routinely transacts business with related parties.

2. Effective internal control

a. Reduces the need for management to review exception reports on a day-to-


day basis.
b. Eliminates risk and potential loss to the organization.
c. Cannot be circumvented by management.
d. Is unaffected by changing circumstances arid conditions encountered by the
organization.

3. Effective internal control requires organizational independence of different


departments. Organizational, independence would be impaired in which of the
following situations?

a. The internal auditors report to the audit committee of the board of directors
b. The controller reports to the vice president of production.
c. The payroll accounting department reports to the chief accountant
d. The cashier reports to the treasurer

4. Internal controls are designed to provide reasonable assurance that:

a. Control policies have not been circumvented through management‘s joint


effort.
b. The internal auditing department's guidance and oversight of management‘s
performance is accomplished economically and efficiently.
c. Management is planning, organizing, and directing processes are properly
evaluated.
d. Material errors or fraud would be prevented or detected and corrected within
a timely period by employees in the course of performing their assigned
duties.
5. Which of the following is not an assurance to be provided by an effective internal
control system?

a. Management is responsible for knowledge and authorization of transactions.


b. Transactions are recorded to maintain account accountability for assets
c. Access to assets is limited to members of management
d. Transactions are recorded to permit the preparation of reliable financial
statements.

6. An entity's internal control consists of policies and procedures established to


provide reasonable assurance that specific entity objectives will be achieved.
Only some of these objectives, policies and procedures are relevant to a financial
statement audit. Which of the following would most likely be considered in an
audit of financial statements?

a. Timely reporting and review of quality control


b. Maintenance of control over unused checks.
c. Marketing analysis of sales generated by advertising projects
d. Maintenance of statistical production analyses.

7. When an organization has strong internal control, management can expect


various benefits. The benefit least likely to occur is

a. a reduced cost of an external audit


b. an elimination of employee fraud
c. the availability of reliable data for decision-making purposes and protection of
important documents and records.
d. an assurance of compliance to applicable laws and regulations

8. Which of the following statements about internal control is correct?

a. Effectively designed internal control reasonably ensures that collusion among


employees cannot occur
b. The establishment and maintenance of internal control are important
responsibilities of the internal auditor
c. Exceptionally strong internal control is enough for the auditor to eliminate
substantive tests on a significant account balance.
d. The cost benefit relationship is a primary criterion that should be considered
in designing internal control.
9. Internal control can only provide reasonable and not an absolute e of achieving
entity's control objectives. One of the factors limiting the likelihood of achieving
those objectives is that

a. The auditor's primary responsibility is the detection of fraud.


b. The board of directors is activeand independent.
c. The cost of internal control should not exceed its benefits.
d. Management monitors internal control.

10. A proper segregation of duties requires that an individual who is:

a. Authorizing a transaction records it.


b. Authorizing a transaction maintains custody of the asset that results from the
transaction
c. Maintaining a custody of an asset be entitled to have access to the
accounting records for the asset
d. Recording a transaction not compares the accounting record of the asset with
the asset itself.

11. External factors can either strengthen or weaken an entity's internal control.
Which of the following conditions supports strong internal control?

a. Strict monitoring by the Bureau of Internal Revenue.


b. The existence of related parties and related-party transactions.
c. Pressure imposed by the financial community to improve earnings
d. An economic downturn.

12. Proper segregation of functional responsibilities in an effective system control


calls for separation of the functions of:

a. Authorization, execution and payment


b. Authorization, recording and custody
c. Custody, executionand reporting.
d. Authorization, payment, and recording.

13. For good internal control, which of the following functions should not be the
responsibility of the treasurer's department?

a. Data processing.
b. Handling of cash
c. Custody of securities.
d. Establishing credit policies.

14. In general, a material internal control weakness may be defined as a condition in


which material errors or fraud would ordinarily not be detected within a timely
period by

a. an auditor during the normal study and evaluation of the system of internal
control
b. a controller when reconciling accounts in the general ledger
c. Employees in the normal course of performing their assigned functions.
d. The chief financial officer when reviewing interim financial statements.

15. In general, material irregularities perpetrated by which of the following are most
difficult to detect?

a. Internal auditor
b. Computer operator
c. Cashier
d. Controller

16. Controls that enhance the reliability of the financial statements may be classified
as prevention controls and detection controls. Which the following is primarily a
detection control?

a. Separation of duties between recording cash receipts and cash custody.


b. Bank accounts are reconciled monthly by persons who are independent of
cash recording and cash custody.
c. The human resource department authorizes the hiring of only those persons
for accounting positions that meet the written job requirements specified by
the corporate controller.
d. An accounting manual, accompanied by a detailed chart of accounts,
carefully and clearly describes each type of transaction affecting the entity.

17. One aspect of internal control requires companies to maintain adequate


documentation and records. Which of the following statements is not correct with
respect to an entity's maintenance of documents and records?

a. Documents should be pre-numbered only if the client has no other means to


maintain records of which documents have been used
b. Documents should be sufficiently simple to ensure that they are clearly
understood.
c. Documents should be prepared at the time a transaction occurs or as soon
thereafter as possible.
d. Documents should be pre-numbered consecutively to facilitate control over
missing documents

18. Which of the following factors are included in an entity's control environment?

Audit Committee Internal Audit Function Management Style


a. YES YES NO
b. YES NO YES
c. NO YES YES
d. YES YES YES

19. Emir is responsible for the custody of finished goods in the warehouse. If his
company wishes to maintain strong internal control, which of the following
responsibilities are incompatible with his primary job?

a. He is also responsible for the company's fixed asset control ledger.


b. He is also responsible for receiving of goods into the warehouse.
c. He is responsible for the accounting records for all receipts and shipments of
goods from the warehouse.
d. He is responsible for issuing goods for shipment.

20. As part of a periodic planning exercise, a company discovers that an Eastern


European political dispute may interfere with supply sources. This is an example
of:

a. Control environment
b. Risk assessment
c. Control activities
d. Monitoring

21. Infernal control procedures are not designed to provide reasonable assurance
that
a. Transaction are executed in accordance with management‘s authorization
b. Irregularities will be eliminated
c. Access to assets is permitted only in accordance with management's
authorization
d. The recorded accountability for assets is compared with the existing assets at
reasonable intervals.

22. Transaction authorization within on organization may be either specific or


general. An example of specific transaction authorization is the

a. Setting of automatic reorder points.


b. Approval of a construction budget for a new warehouse
c. Establishment of a customer's credit limits
d. Establishment of sales prices.

23. Which of the following statements best describes a weakness often associated
with computers?

a. Computer equipment is more subject to a systems error than manual


processing which is subject to human error
b. Computer equipment processes and records similar transactions in a similar
manner.
c. Control activities for detecting invalid and unusual transactions are less
effective than manual control activities
d. Functions that would normally be separated in a manual system are
combined in a computer system.

24. The financial statements are not likely to correctly reflect Philippine Financial
Reporting Standards if

a. The controls affecting the reliability of financial reporting are inadequate.


b. The company's controls do not promote efficiency
c. The company‘s control do not promote effectiveness
d. All them are true.

25. The basic concept of internal control which recognizes that the cost of internal
control should not exceed the benefits expected to be derived is known as:
a. Management by exception
b. Management responsibility
c. Limited assurance
d. Reasonable assurance

26. Internal control should provide reasonable (but not necessarily absolute)
assurance which means that:

a. Internal control is a management‘s, not auditor‘s responsibility


b. An attestation engagement about management‘s internal control assertion
may not necessarily detect all reportable conditions
c. The cost of control activities should not exceed the benefits
d. There is always a risk that reportable conditions may result in material
misstatements

27. When considering internal control, an auditor must be aware of the concept of
reasonable assurance which recognizes that

a. The employment of competent personnel provides assurance that the


objectives of internal control will be achieved
b. The establishment of and maintenance of a system of internal control is an
important responsibility of the management and not of the auditor
c. The cost of internal control should not exceed the benefits expected to be
derived from internal control
d. The segregation of incompatible functions is necessary to obtain assurance
that the internal control is effective.

28. To qualify as an "outside director" in an audit committee, one must

a. Not own any stock in client company.


b. Not be a part of management.
c. Not receive any remuneration or expense reimbursement.
d. Work for CPA firm.

29. Adequate separation of duties. within an EDP department would ensure that
a. the programmer does not have an access to computer operation
b. the librarian has no physical control over the computer programs
c. the computer operator does not have access to computer run instructions.
d. the data control group hove as little independence as possible.

30. Which of the following duties would indicate a weakness in the internal control
system?

a. The accounting function is under the controller


b. The custodianship of cash is the responsibility of the treasurer's function.
c. The internal auditor reports to the board of directors.
d. The custodianship of buildings and equipment is the responsibility of the
controller's function.

31. Control risk is a measure of the auditor's expectation that the internal control
structure

a. Will prevent material misstatements from occurring.


b. Will detect and correct material misstatements.
c. Will either prevent material misstatements or detect and correct them
d. Will neither prevent material misstatements nor detect and correct them.

32. Of the following statements about an internal control system, which one is not
valid?

a. No one person should be responsible for the custodial responsibility and the
recording responsibility for an asset.
b. Transactions must be properly authorized before such transactions are
processed.
c. Because of thecost benefit relationship, a client may apply control procedures
on a test basis.
d. Control procedures reasonably ensure that collusion among employees
cannot occur.

33. An entity should consider the cost of a control in relation to the risk. Which of the
following controls best reflects this philosophy for a large peso investment in
heavy machine tools?
a. Conducting a weekly physical inventory
b. Placing security guards at every entrance 24 hours a day.
c. Imprinting a controlled identification number on each tool.
d. Having all dispositions approved by the vice president of sales

.
UNDERSTANDING AND DOCUMENTATION OF INTERNAL CONTROL

34. In an audit of financial statements, an auditor's primary consideration regarding


internal control is whether it

a. incorporates management's philosophy and operating style.


b. affects management's financial statement assertions.
c. provides adequate safeguards over access to assets.
d. supports management's decision-making processes.

35. The procedures to test effectiveness of control policies and procedures in


support of a reduced assessed control risk are called

a. tests of transactions.
b. analytical tests.
c. tests of controls.
d. a walk-through.

36. The actual operation of an internal control system may be most objectively
evaluated by

a. Completing a questionnaire and flowchart related to the accounting system in


the year under audit.
b. A review of the previous year's audit work papers to update the report of the
internal control evaluation.
c. A selection of items processed by the system and determination of the
presence or absence of errors and compliance deviations.
d. Substantive tests of accounts balances based on the auditor's assessment of
internal control strength.

37. The auditor places, primary emphasis on controls over


I. Classes of transactions
II. Account balances

a. I only.
b. II only.
c. both I and II, because they are equally weighted.
d. both I and II, because they vary from client to client

38. The three key concepts that underlie the study of an internal control structure and
the assessment of control risk would not include a criterion that

a. the control risk may range from zero to 100%.


b. Management, not the auditor, must establish and maintain the entity's
controls.
c. the internal control structure provides reasonable, but not absolute,
assurance that the financial statements are fairly sated.
d. the internal control structure can never be regarded as completely effective.

39. Even with the most effectively designed internal control structure, the auditor
must obtain audit evidence, beyond testing the controls, for every

a. Financial statement account.


b. Material financial statement account.
c. Financial statement account that will be relied upon by third parties.
d. Transaction.

40. The primary purpose of performing controls is to provide reasonable assurance


that:

a. The internal control is effective.


b. The accounting system isdocumented accurately.
c. Transactions are recorded at the amounts executed.
d. All control activities leave visible evidence,

41. A consideration of internal control made during an audit is usually not sufficient to
express an opinion on an entity's controls because
a. weaknesses in the system may go unnoticed during the audit engagement.
b. a consideration of internal control is not necessarily made during an audit
engagement.
c. only those controls on which an auditor intends to rely are reviewed, tested,
and evaluated.
d. controls can change each year.

42. An auditor's report on internal control of a publicly held company would ordinarily
be of least use to

a. shareholders.
b. directors.
c. officers.
d. regulatory agencies.

43. The accountant's report that expresses an opinion on an entity's internal controls
should state that the

a. establishment and maintenance of internal control is the responsibility of


management.
b. objectives of the client's internal controls are being met.
c. consideration of the internal controls is conducted in accordance with
generally accepted auditing standards.
d. inherent limitations of the client's internal controls are examined.

44. Reportable conditions are matters that come to an auditor's attention and that
should be communicated to an entity's audit committee because they represent

a. material irregularities or illegal acts perpetrated by management.


b. significant deficiencies in the design or operation of internal control.
c. flagrant violations of the entity's documented conflict-of-interest policies.
d. intentional attempts by client personnel to limit the scope of the auditor's
work.

45. The accountant's report that expresses an opinion on an entity's internal controls
would not include a
a. description of the scope of the engagement.
b. specific date that the report covers rather than a period of time.
c. brief explanation of the broad objectives and inherent limitations of internal
control.
d. statement that the entity's internal controls are consistent with that of the prior
year after giving effect to subsequent changes.

46. A CPA's consideration of internal control in an audit

a. is generally more limited than that made in connection with an engagement to


express an opinion on internal control.
b. is generally more extensive than that made in connection with an
engagement to express an opinion on internal control
c. will generally be identical to that made in connection with an engagement to
express an opinion on internal control.
d. will generally result in the CPA expressing an opinion on the internal control.

47. The understanding of internal control that relates to a financial statement


assertion should be used to do all of the following except:

a. Determine inherent risk for that assertion.


b. identify types of potential misstatements for that assertion.
c. Consider factors that affect the risk of material misstatement for that assertion
and assess control risk.
d. Design substantive tests that correspond with the assessment of control risk.

48. The sequence of steps in gathering evidence as the basis of the auditor‘s opinion
is:

a. Substantive tests, documentation of control structure, and tests of controls.


b. Documentation of control structure, substantive tests, and tests of controls
c. Documentation of control structure, tests of controls, and substantive tests.
d. Tests of controls, documentation of control structure, and substantive tests

49. Which of the following procedures is essential in determining whether necessary


control activities are prescribed and are being followed?
a. Development of questionnaires and checklists
b. Evaluation of the entity's procedures for risk assessment
c. Documentation of and testing controls.
d. Observing employees and making inquiries

50. Which of the following is the logical order of performing the following auditing
procedures?

a. Tests of internal control procedures.


b. Preparation of a flowchart depicting the client's internal control system.
c. Substantive tests.

a. ABC
b. ACB
c. BAC
d. BCA

51. A secondary purpose of the auditor's consideration of internal control is to


provide a(n)

a. basis for constructive suggestions about improvements in internal control


structure.
b. basis for assessing control risk
c. assurance that the records and documents have been maintained in
accordance with existing company policies and procedures.
d. basis for the determination of the resultant extent of the tests to which
auditing procedures are to be restricted

52. Which of the following statements with respect to the independent auditor‘s
evaluation of internal control is correct?

a. The auditor should decrease control testing when weaknesses in


cashreceipts are mitigated by strong controls in cash disbursement
procedures.
b. The auditor should increase control testing when weaknesses in billing
procedures are mitigated by strong controls in collection procedures.
c. The auditor generally should not evaluate the overall effectiveness of internal
control, but should separately evaluate each of the transaction cycles.
d. The auditor should evaluate all internal control weaknesses before
determining the, control procedures that should prevent or detect errors or
irregularities.

53. The auditor concludes that a public company has significant deficiencies in its
internal controls over financial reporting. Which of the following is not a proper
response to this finding?

a. Report the deficiencies to management and the audit committee.


b. Report the deficiencies to the shareholders.
c. Expand the planned testing of account balances to consider the types of
errors that might occur because of the deficiency.
d. Any of the responses

54. The development of constructive suggestions to clients for improvements in


internal accounting control is

a. a requirement of the auditor‘s study and evaluation of internal accounting


control.
b. a desirable by-product of an audit engagement.
c. only addressed by the auditor during a special engagement.
d. as important as establishing a basis for reliance on internal accounting control
system.

55. Which of the following statements concerning the independent auditor's required
communication of material weaknesses in internal control is correct?

a. Weaknesses reported at interim dates must be repeated in the final


communication
b. If the auditor is not aware of any material weaknesses during the
examination, that fact must be communicated
c. Weaknesses that had been reported in prior years' communications and have
not been corrected need not be repeated in thecurrent year‘s communication.
d. Although written communication is preferable; the auditor may communicate
the findings orally.
56. After obtaining a sufficient understanding of internal control, the auditor assesses

a. The need to apply PSAs


b. detection risk to determine the acceptable level of inherent risk
c. Detection risk and inherent risk to determine the acceptable level control risk.
d. Control risk to determine the acceptable level of detection risk.

57. The ultimate purpose of assessing control risk is to contribute to the auditor‘s
evaluation of the risk that

a. Tests of controls may fail to identify controls relevant to assertions.


b. Material misstatements may exist in the financial statements.
c. Specified controls requiring segregation of duties may circumvented by
collusion.
d. Entity policies may be circumvented by senior management

58. It is important for the auditor to consider the competence of the audit employees
because their competence bears directly and importantly upon the

a. Cost-benefit relationship of internal control


b. achievement of the objectives of internal control
c. comparison of recorded accountability with assets
d. timing of the tests to be performed

59. in gaining an understanding of internal control the auditor may trace several
transactions through the control process, including how the interface with any
service organizations whose services are part of the information system. The
primary purpose of this task is to

a. replace substantive tests.


b. Determine whether the controls have been placed in operation.
c. determine the effectiveness of the control procedures.
d. Detect fraud.
60. When obtaining an understanding of an entity's internal control, the auditor
should concentrate on the substance of controls rather than on their form
because

a. The controls may be operating effectively but may not be documented.


b. Management may establish appropriate controls but not act on them.
c. the controls may be so inappropriate that no reliance is contemplated by the
auditor:
d. Management may implement controls with costs in excess of benefits.

61. In obtaining an understanding at an entity's internal control in a financial


statement audit, an auditor is not required to

a. determine whether the controls have been placed in operation


b. perform procedures to understand the design of controls.
c. document the understanding of the entity's internal control components.
d. search for significant deficiencies in the operation of internal control

62. Which of the following tasks should be performed prior to the final audit?

a. Determining the fairness of property, plant, and equipment


b. Confirming accounts receivable.
c. Understanding internal control.
d. Collecting and evaluating evidence supporting the fairness of inventory
values.

63. A conceptual approach to the auditor's evaluation of internal accounting control


consists of the following four steps:

I. Determine whether the necessary procedures are prescribed and are


being followed satisfactorily.
II. Consider the types of errors and fraud that could occur.
III. Determine the internal control policies and procedures that should
prevent or detect errors and fraud
IV. Evaluate any weakness to determine its effect on the nature, timing or
extent of auditing procedures to be applied and suggestions to be made
to the client.
What should be the logicalorder in which these four steps are performed?

a. III, IV, I, II
b. III, I, II, IV
c. II, III, I, IV
d. II, I, III, IV

64. An auditor evaluates the existing internal control in order to

a. determine the extent of substantive tests which must be performed


b. determine the extent of control tests which must be performed.
c. ascertain whether irregularities are probable.
d. ascertain whether any employees have incompatible functions.

65. The auditor‘s study and evaluation of internal control may be done for all but
which of the following reasons?

a. To provide a basis for offering the client, other value-added services designed
to increase efficient and effective operations.
b. To determine the nature, timing, and extent of other audit tests.
c. To establish a basis for reliance on internal controls.
d. To provide training and development for staff accountants.

66. Regardless of the assessed level of control risk, an auditor would perform some

a. tests of control policies and procedures in order to determine their operating


effectiveness.
b. analytical procedures to verify the design of controls.
c. substantive tests to restrict detection risk for significant transaction classes
d. dual-purpose tests to evaluate both the risk of monetary misstatement and
preliminary control risk

67. A procedure that involves tracing a transaction from its origination through the
company's information systems is referred to as a (n):

a. Inquiry/analysis approach.
b. Re-analysis approach.
c. Remediation
d. Walkthrough

68. Auditors frequently use flowcharts in connection with which of the following?

a. Preparation of generalized computer audit programs.


b. Review of the client's internal control procedure
c. Use of statistical sampling in performing on
d. Performance of analytical review procedures for account balances.

69. A well-prepared flowchart should make it easier for the auditor to

a. prepare audit procedure manuals.


b. prepare detailed job descriptions.
c. trace the origin and disposition of documents.
d. assess the degree of accuracy of financial data.

70. Flowcharting as a means of internal control evaluation provides the following


advantage over the use of questionnaires and descriptive narratives:

a. Ease of preparation
b. Comprehensive coverage of controls.
c. Simplicity.
d. Ease in following information flow.

71. As part of the evaluation of a client's system of internal control, to determine


whether the necessary procedures are prescribed and have been implemented
satisfactorily, an auditor must

a. develop questionnaires and checklists


b. obtain an understanding of internal control.
c. perform tests of internal control procedures
d. evaluate administrative policies
72. Which of the following is an appropriate form of documenting the auditor's
understanding of a client's internal controls?

a. Narratives
b. Flowcharts
c. Internal control questionnaires
d. Each of the three documentation techniques is appropriate to do

73. Which of the following is of least concern to an auditor regarding a client‘s


internal controls?

a. Efficiency and effectiveness of operations


b. Controls related to the reliability of financial reporting
c. Controls over classes of transactions
d. Auditors are equally concerned with each issue

74. An auditor is least likely to test for on internal control that provides for

a. segregation of the functions of recording disbursements and reconciling the


bank account.
b. comparison of receiving reports and vendors‘ invoices with purchase orders
c. approval of the purchase and sale at marketable securities.
d. classification of revenue and expense transactions by product

75. An internal control questionnaire (ICQ) contains the following question: ―Does a
single individual receive and list cash receipts and perform posting tosales and
general ledgers?" What action should an auditor take if the accounting manager
responds "yes to the question?

a. No action is required because "yes" responses on an ICQ indicate the


presence of good control
b. Statistically sample the response along with all other "yes‖ responses to verify
their accuracy.
c. Treat it as a potential control weakness and perform appropriate testing
d. Include it with other reportable findings in the next audit report.
76. Which of the following would be considered an advantage of using an internal
control questionnaire in understanding and documenting the controls for an
important accounting application? The questionnaires can be:

a. computerized to provide linkages of weaknesses to particular type of errors


that might occur in the account balances
b. used for many years without updating.
c. easily understood and provide easy identification of potential control
deficiencies through ―no‖responses to questions.
d. Adopted to bothe large and small businesses as well as to different industries

77. Which method provides the auditor with the best visual grasp of a system and a
means for analyzing complex operations?

a. A flowcharting approach
b. A questionnaire approach.
c. A matrix approach
d. A detailed narrative approach.

78. In addition to gaining an understanding of the internal controls, anexternal


auditor, at the minimum would be expected to

a. evaluate the internal auditor‘s work as an important part of the accounting


system element of the internal controls
b. observe client employees to determine the extent of their compliance with
quality control standards
c. trace a few transactions through the control process to obtain evidence that
the controls have been placed in operation.
d. study organizational charts to obtain an understanding of the informal lines of
communication

79. An advantage of an internal control questionnaire is:

a. Flexibility in design and application


b. Its strict adherence to a yes/no format
c. That it provides sufficient data for the assessment of control risk.
d. Ease of completion.
80. The auditors‘understanding of their clients‘ internal control provides a basis for:

Assessing Sufficiency Risk Planning the audit Assessing control risk

a.Yes Yes Yes


b.Yes Yes No
c.No Yes Yes
d.No Yes No

81. With respect to the client's system of internal control, the auditor is concerned
that the existing policies, and procedures provide reasonable assurance that

a. operational efficiency can be achieved in accordance with management


plans.
b. Errors and fraud can be prevented or detected.
c. fraud cannot be committed through collusion.
d. management cannot override the internal controls.

82. Which of the following best describes the primary reason why an auditor uses
flowcharts during on audit engagement?

a. To comply with the requirements of Philippine Standards on Auditing


b. To classify the client's documents and transactions by major transaction
Cycles, e.g., cash receipts, cash disbursements, etc.
c. To record the auditor's understanding of the client's internal control 'policies
and procedures
d. To interpret the operational effectiveness of the client's existing organizational
structure.

83. Regardless of whether the auditor decides to test or not the controls for operating
effectiveness, he/she must fully document his or her understanding of the internal
control policies and procedures obtained through whatever means. Which of the
following does not describe on appropriate means for documenting his/her
understanding of the control system?

a. internal control flowchart


b. Internal control implementation
c. internal control memorandum
d. internal control questionnaire

84. Which of the following questions would an auditor most likely include in an
internal control questionnaire for notes payable?

a. Are assets that collateralize notes payable critically needed for the entity‘s
continuedexistence? B
b. Are two or more authorized signatures required on checks that repay notes
payable?.
c. Are the proceeds from notes payable used for the purchase of noncurrent
assets?
d. Are direct borrowings on notes payable authorized by the board of directors?

85. In an auditor‘s consideration of internal control, the completion of a questionnaire


is most closely associated with which of the following?

a. Separation of duties
b. Flowchart accuracy
c. Understanding the system
d. Tests of controls

86. During the review of the client's system of internal control, the auditor observes
theclient employees as they apply the operating controls in order to

a. prepare flowchart.
b. update information contained in the organization and procedure manuals
c. corroborate the information obtained during the initial review of the system.
d. determine the extent of compliance with quality control standards.

87. An auditor's flowchart of a client‘s internal controls is a diagram depicting the


auditor‘s

a. understanding of the internal controls


b. program for tests of controls.
c. documentation of consideration of internal controls.
d. understanding of the types of irregularities that are probable.
88. Which of the following statements regarding the auditor's documentation of the
client‘s internal control structure is correct?

a. Documentation must include flow chart.


b. Documentation must include procedural write-ups
c. No documentation is necessary although it is desirable.
d. No one particular form of documentation is necessary and the extent of
documentation may vary

TESTS OF CONTROLS

89. Which of the following the auditor's purpose of further testing the control
procedures?

a. Provide a basis for reducing the assessed level of control risk.


b. Reduce the risk that error or fraud that has not been prevented or detected by
the internal control system is not detected by the independent audit.
c. Provide assurance that transactions are executed in accordance with
management's authorization and access to assets is limited by a segregation
of functions.
d. Provide assurance that transactions are recorded as necessary to permit the
preparation of the financial statements in conformity with PFRS.

90. Tests of controls are concerned primarily with each of the following questions
except:

a. How were the controls applied?


b. Why were the controls applied?
c. Were the necessary controls consistently performed?
d. By whom were the controls applied?

91. The objective of tests of details of transactions that are being performed as tests
of controls procedures is to

a. monitor the design and use of entity documents such as pre-numbered


shipping form.
b. determine whether controls have been placed in operation.
c. detect material misstatements in the account balances in the financial
statements.
d. evaluate whether controls operate effectively.

92. Which of the following is ordinarily considered a test of internal control


procedures?

a. Send confirmation letters to banks.


b. Count and list cash on hand.
c. Examine signatures on checks.
d. obtain or prepare reconciliation of bank accounts as of the balance sheet
date.

93. Auditors can use several types of audit procedures to test controls. Which of the
following type of audit procedures is least likely to be used during tests of
controls?

a. Physical examination of assets


b. Inquiries of client personnel
c. Examination of documents, records, and reports
d. Observation of control-related activities.

94. The objective of dual-purpose tests is to:

a. Evaluate whether internal controls are operating effectively.


b. Detect material misstatements in the financial statements
c. Identify unusual trends or patterns in comparative financial statements
d. Test internal controls as well as transactions and balances using the same
test procedures

95. Which of the following types of evidence will be gathered in order to test internal
controls?

a. Confirmations of accounts receivable with customers.


b. Observation of client personnel receiving inventory shipments.
c. Observation of inventory counts.
d. Inquiry of management regarding significant litigation.
96. Tests of controls least likely include:

a. Inquiries of appropriate client vendors.


b. Reperformance of a control.
c. Observation of the application of an accounting procedure.
d. Inspection of documents

97. A procedure that would most likely be used by on auditor in performing tests of
control regarding segregation of functions on which no audit trail is available:

a. inspection.
b. Observation
c. Reprocessing
d. Reconciliation

98. The primary purpose of performing further control tests is to provide

a. A basis for the assessed level of control risk below the maximum level
b. a basis for understanding the flow of transactions through the accounting
system.
c. assurance that transactions are properly recorded.
d. all accounting control procedures leave visible evidence.

99. Which of the following procedures most likely would be included as part of an
auditor's tests of control procedures?

a. Inspection
b. Reconciliation
c. Confirmation
d. Analytical procedures

100.Which of the following audit tests would be a test of controls?

a. Tests of the specific items making up the balance in a financial statement


account
b. Comparing inventory prices to vendors' invoices.
c. Tracing signatures on cancelled checks to board of directors' authorizations
d. Tests of the additions to property plant and equipment by physical
inspections.

101. Which of the following statements is true?

a. Tests of controls are necessary if the auditor plans to use the primarily
substantive approach
b. Tests of controls are necessary if the auditor plans to assess the level of
control risk at maximum
c. The auditor can simultaneously obtain an understanding of internal control
and perform tests of controls.
d. After performing tests of controls, the auditor will always assess control risk at
maximum.

102. In considering the evidence needed to assess control risk during the period from
interim to year-end, all of the following should be considered except the:

a. Significance of the assertion being tested.


b. Specific internal control policies and procedures tested during the interim
period.
c. Degree to which the policies and procedures weretested and the test results.
d. Control risk on other assertions.

103. After obtaining an understanding of a client's controls, an auditor may decide to


omit tests of controls. Which of the following is not an appropriate reason to omit
tests of controls?

a. The controls appear adequate


b. The controls duplicate other controls.
c. Reportable conditions preclude the auditor of assessing control risk below the
maximum.
d. The effort to test controls exceeds the effort saved by not performing
substantive tests.
104. The auditor is examining copies of sales invoices only for the initials of the
person responsible for checking the extensions. This is an example of

a. test of controls.
b. dual purposetest.
c. substantive test.
d. test of balances

105. Which of the following statements about tests of controls is not valid?

a. The auditor mayperform inquiry and observation and gathering audit evidence
about the operating effectiveness of the control
b. Ordinarily, making inquiries provides more reliable audit evidence than doing
observation when testing segregation of functional responsibilities
c. Audit evidence obtained by doing observation pertains only to the point in
time at which the procedure was applied.
d. Observation of who applies a control procedure is useful as a test of control
procedures when evaluating control effectiveness of both cornpuferized and
manual system

106. Which of the following is appropriate about risk assessment?

a. The assessed level of inherent and control risk can be sufficiently low, thus
resulting to eliminating the need for substantive tests.
b. Audit risk may be more effectively determined by assessing inherent and
control risk separately.
c. There is an inverse relationship between detection risk and the combined
level of inherent and control risk
d. Detection risk is eliminated if an auditor were to examine 100 percent of the
account balance or class of transactions.

107. Tests of controls are directed toward the control's

a. Efficiency
b. efficiency and effectiveness.
c. effectiveness.
d. cost benefit ratio.
108. Audit evidence concerning segregation of duties ordinarily is best obtained by

a. performing tests of transactions that corroborate management's financial


statement assertions.
b. observing the employees as they apply specific controls.
c. obtaining a flowchart of activities performed by available personnel
d. developing audit objectives that reduce control risk.

ASESSMENT OF CONTROL RISK

109. Which of the following is a step in an auditor's decision to assess control risk at
below the maximum?

a. Apply analytical procedures to both financial data and nonfinancial


information to detect conditions that may indicate weak controls.
b. Perform tests of details of transactions and account balances to identify
potential errors and fraud.
c. Identify specific controls that are likely to detect or prevent misstatements
d. Document that the additional audit effort to perform tests of controls exceeds
the potential reduction in substantive testing

110. Which of the following statements concerning control risks correct?

a. Assessing control risk and obtaining an understanding of an entity‘s internal


control maybe performed concurrently.
b. When control risk is at the maximum level an auditor is required to document
the basis for that assessment.
c. Control risk may be assessed sufficiently low to eliminate substantive testing
for significant transaction classes
d. When assessing control risk, an auditor should not consider evidence
obtained in prior audits about the operation of controls

111. After obtaining an understanding of internal control and assessing control risk,
an auditor decides not perform additional tests of controls. The auditor most likely
concludes that the
a. additional evidence to support a further reduction in control risk is not cost-
beneficial to obtain
b. Assessed level of inherent risk exceeds the assessed level of control risk.
c. internal control is property designed and can be relied on
d. Evidence obtainable through tests of controls would not support an increased
assessment of control risk.

112.Which of the following models expresses the general relationship of risks


associated with the auditor's evaluation of internal control (CR), study of the
business and application of analytical procedures (IR), and overall audit risk (AR),
that would lead the auditor to conclude that additional substantive tests of details of
on account balance are not necessary?

IR CR AR
a. 20% 40% 10%
b. 20% 60% 5%
c. 10% 70% 4.5%
d. 30% 40% 5.5%

113. In studying internal control and assessing control risk, the auditor applies the
following steps:

A. Determine the internal control policies and procedures necessary to


prevent or detect errors or fraud that could occur in case of the
absence of controls
B. Identify control weaknesses
C. Determine whether the necessary policies and procedures have been
designed and whether they have been placed in operation
D. Design substantive audit programs
E. Consider the types of errors or fraud that could occur in the absence of
necessary controls.

The proper sequence in applying these steps is:

a. CDEAB
b. CBAED
c. EACBD
d. AECBD
114. After obtaining an understanding of the client's internal control, the auditor
should consider whether

a. the projected degree of effectiveness of internal control is justified.


b. the evidential matter obtained from the study of the internal control system
can provide a reasonable basis for an opinion.
c. further testing of internal control procedures is likely to permit further
reduction in the assessed-level of control risk.
d. sufficient knowledge has been obtained about the entity's entire system of
internal control.

115. An independent auditor has concluded that the client's records, procedures and
representations can be relied upon based on tests made during the year when
internal control was found to be effective. The aucitor should test the records,
procedures, and representations again at year-end if:

a. Inquiries and observations lead the auditor to believe that conditions have
changed significantly
b. Comparisons of year-end balances with like balances at prior dates revealed
significant fluctuations.
c. Unusual transactions occurred subsequent to the completion at the interim
audit work
d. Client records are in a condition that facilitates effective and efficient testing.

116. In assessing control risk, The auditor must, as a minimum,

a. perform tests of all significant controls


b. obtain an understanding of the design and implementation of the client's
internal control.
c. obtain an understanding of the design of the client's internal control
d. obtain an understanding of the design, implementation, and operating
effectiveness of the client's internal control.

117. In the assessment of control risk, the auditor is basically concerned that the
client‘s internal control provides reasonable assurance that

a. management cannot override the system


b. operational efficiency has been achieved in accordance with management
plans.
c. errors and fraud have been prevented or detected.
d. controls have not been circumvented by collusion.

118. Which of the following is one of the most fundamental and effective controls?

a. Increased use of computers for recording accounting transactions.


b. Increased reliance on internal auditors to monitor accounting systems
c. Segregation of incompatible duties across several people
d. Having internal auditors report only tothe Board of Directors.

119. Which circumstance would be consistent with both the planned assessed level
and the assessed level of controlrisk being low?

a. No tests of controls have been performed.


b. Tests of controls have been performed.
c. Externally generated evidence supports management's contentions relating to
internal control.
d. The results of the consideration of internal control suggest that controls are
not operating effectively.

120. The results of the considerations of internal control least likely affect the
auditor‘s decisions pertaining to:

a. The use of analytical procedures.


b. The assessment of control risk.
c. The assessment of inherent risk.
d. Detailed tests of ending balances.

121. A material weakness in the design and the operation of controls that had been
discovered in an audit of internal controls results in:

a. A management letter
b. An unfavorable opinion
c. Firing of the auditors
d. Adjusting audit journal entries
122. Which of the following requirements of internal control is expected of
management of a public entity?

a. Accept responsibility for the effectiveness of internal control.


b. Evaluate the effectiveness of internal control using a written assertion
c. Prepare a detailed flowchart that summarizes the company's internal control.
d. Provide a monthly updated report on-internal control.
123. The definition of which type of control deficiency explicitly includes an amount
that is more than inconsequential?

a. Material weakness.
b. Reportable condition.
c. Significant deficiency
d. Strong control deficiency.

124. Which of the following terms includes the concept that least likely to have a
likelihood of occurrence?

a. Material Weakness–Yes; Reportable Condition—Yes


b. Material Weakness–Yes; Reportable Condition—No
c. Material Weakness–No; Reportable Condition—Yes
d. Material Weakness–No; Reportable Condilion—No

125. When performing an audit of internal control for a public company, the auditors
will consider which of the following types of controls?

a. Preventive–Yes: Detective—Yes
b. Preventive–Yes: Detective—No
c. Preventive--No; Detective—Yes
d. Preventive--No; Detective—No

126. Which of the following is not a strong indicator of the existence of a material
weakness in internal control?

a. An ineffective control environment.


b. An ineffective internal audit function.
c. Inadequate controls over the selection of the best investment opportunities.
d. Ineffective oversight of external reporting by the audit committee.

127. A control that reduces the risk that on existing or potential control weakness will
result in a failure to meet a control objective is referred to as a:

a. Compensating control.
b. Conditional control.
c. Non-routine control.
d. Walkthrough control.

128. Which of the following is not a characteristic of the primarily substantive


approach?

a. The auditor usually gathers all or most of the evidence with substantive tests.
b. Usually little or no reliance is placed on controls.
c. The assessment of control risk is usually at or near maximum level.
d. Extensive tests of controls are performed.

129. Which Of the following is not a characteristic of the lower control risk approach?

a. Control risk is usually assessed at maximum level.


b. Substantive tests are usually restricted.
c. The auditor usually plans to place considerable reliance on the controls.
d. The auditor plans to perform extensive tests of controls.

130. If the auditor uses the primarily substantive approach instead of the lower
control risk approach

a. A higher level of understanding of internal control is required.


b. The auditor plans to assess control risk at a lower level.
c. The auditor plans a heavier reliance on substantive tests
d. The auditor plans to restrict substantive tests

131. After obtaining an understanding of an entity's internal control, an auditor may


assess control risk at the maximum level for some assertions because he
a. believes the internal control policies and procedures are unlikely to be
effective.
b. determines that the pertinent internal control components are not well
documented.
c. performs tests of controls to restrict detection risk to an acceptable level.
d. identifies internal control policies and procedures that are likely to prevent
material misstatements.

132. An auditor may decide to assess control risk at the maximum level for certain
assertions because he believes

a. sufficient evidential matter to support the assertions is likely to be available


b. Evaluating the effectiveness of policies and procedures is inefficient.
c. more emphasis on tests of controls than substantive tests is warranted,
d. Considering the relationship of assertions to specific account bc4ances is
more efficient.

133. The assessment of control risk can be made at any of the following times
except:

a. Immediately after obtaining an understanding of internal control.


b. After some tests of controls are performed concurrently with obtaining an
understanding.
c. After the performance of additional tests of controls designed to further lower
the assessment of control risk.
d. After performing all the necessary substantive tests.

134. Which of the following statements is true? If control risk is assessed of the
maximum, the

a. nature of related substantive tests should be changed from more to less


effective
b. nature of related substantive tests should be changed from less to more
effective.
c. timing of related substantive tests should be changed from year-end to an
interim date.
d. extent of related substantive tests should he changed from a larger to a
smaller sample.

135. After obtaining an understanding of an entity's internal controls and assessing


control risk, an auditor may next:

a. Perform tests of controls to verify management's assertions that are


embodied in the financial statements
b. Consider whether he can reduce the assessed level of control risk further
c. Discontinue searching for reportable condition.
d. Evaluate whether control activities can detect material misstatements.

136. After considering a client's internal control, the auditor has concluded that the
system is well designed and is functioning as anticipated. Under these
circumstances, the auditor would most likely

a. cease to perform further substantive tests.


b. not increase the extent of planned substantive tests
c. increase the extent of anticipated analytical procedures.
d. perform all tests of controls to the extent outlined in the pre-planned audit
program.

137. Which of the following statements is correct concerning reportable conditions in


an audit?

a. An auditor is required to search for reportable conditions during an audit.


b. All reportable conditions are also considered to be material weaknesses.
c. An auditor may communicate reportable conditions during an audit or after
the audit's completion.
d. An auditor may report that no reportable conditions were noted during an
audit.

138. As a result of tests of controls, an auditor assessed control risk too low d
decreased substantive testing. This assessment occurred because the true
deviation rate in the population was
a. Less than the risk of assessing control risk too low, based on the auditor's
sample.
b. Less than the deviation rate in the auditor's sample.
c. More than the risk of assessing control risk too low, based on the auditor's
sample.
d. More than the deviation rate in the' auditor's sample.

139. After obtaining an understanding of internal control and assessing control risk,
an auditor decided to perform tests of controls. The auditor most likely decided
that:

a. It would be efficient to perform tests of controls that would result in a


reduction in planned substantive tests.
b. Additional evidence to support a further reduction in control risk is not
available
c. An increase in the assessed level of control risk is justified for certain financial
statement assertions.
d. There were many internal control weaknesses that could allow errors to enter
the accounting system.

140. Once an understanding of the internal control structure that is sufficient for
audit planning is obtained, then the auditor must first assess

a. whether a lower level of control risk could be supported


b. whether the financial statements are auditable
c. the level of control risk supported by the understanding obtained
d. the level of control risk to use.
Module 7

Computerized Environment

1. Which of the following is not an advantage of a computerized accounting system?

A. Computers process transactions uniformly


B. Computers help alleviate human errors
C. Computers can process many transactions quickly
D. Computers leave a thorough audit trail which can be easily followed

2. A common difficulty in auditing a computerized accounting system is:

A. Data can be erased from the computer with no visible evidence


B. Because of the lack of an audit trail, computer systems have weaker controls
and more substantive testing is required
C. Because of the uniform nature of transaction processing, computer systems
have strong controls and less substantive testing is required
D. The large dissemination of the entity points into the computer system leads to
weak overall reliance on information generated by a computer

3. Which of the following most likely represents a disadvantage for an entity that
maintains computer data files rather than manual files?

A. It‘s usually more difficult to detect transposition errors.


B. Transactions are usually authorized before they are executed and recorded.
C. It‘s usually easier for unauthorized persons to access and alter the files.
D. Random error is more common when similar transactions are processed in
different ways.

4. Which of the following statements best describes a weakness often associated with
computers?

A. Computer equipment is more subject to a systems error than manual


processing is subject to human error.
B. Computer equipment processes and records similar transactions in a similar
manner.
C. Control activities for detecting invalid and unusual transactions are less
effective than manual control activities.
D. Functions that would normally be separated in a manual system are
combined in a computer system.
5. How have Electronic Data Interchange (EDI) systems affected audits?

A. Since orders and billing transactions are done over the computer, source
documents cannot be obtained/
B. Auditors often need to plan ahead to capture information about selected
transactions over the EDI.
C. There is no audit trail in an EDI systems, so controls are typically assessed as
weak.
D. Since all transactions occur over the computer, reliability is high and little
substantive testing is needed.

6. Since the computer can do many jobs simultaneously, segregation is not as defined
as it is in a manual system. How can a computer system be modified to compensate
for the lack of segregation of duties?

A. The computer system should be under the direction of the internal audit
department.
B. The computer system should be accessible to various competent parties so
they can check on each other‘s work.
C. Strong controls should be built into both the computer software and hardware
to limit access and manipulation.
D. Many companies run complete parallel manual and automated accounting
systems for a cross check on input and output.

7. Internal control is ineffective when the computer personnel

A. Participates in computer software acquisition decisions.


B. Designs documentation for computerized systems.
C. Originates changes in master files.
D. Provides physical security for program files.

8. An auditor‘s consideration of a computer‘s control activities has disclosed the


following four circumstances. Which circumstance constitutes a significant deficiency
in internal control?

A. Computer operators do not have access to the complete software support


documentation.
B. Computer operators are closely supervised by the programmers.
C. Programmers are not authorized to operate computers.
D. Only one generation of backup files is stored in an off-premises location.
9. Matthews Corp. has changed from a system of recording time worked on time cards
to a computerized payroll system in which employees record time in and out with
magnetic cards. The IT system automatically updates all payroll records for the time
worked. Because of this change,

A. Generalized audit software must be used.


B. Part of the audit trail is altered.
C. The potential for payroll-related fraud is diminished.
D. Transactions must be processed in batches.

10. The increased presence of the microcomputer in the workplace has resulted in an
increasing number of persons having access to the computer. A control that is often
used to prevent unauthorized access to sensitive programs is:

A. Backup copies of the diskettes.


B. Passwords for each of the users.
C. Disaster-recovery procedures.
D. Record counts of the number of input transactions in a batch being
processed.

11. Checklists, systems development methodology, and staff hiring are examples of
what type of controls?
A. Detective
B. Preventive
C. Subjective
D. Corrective

12. When an on-line, real-time (OLRT) computer-based processing system is used,


internal control can be strengthened by

A. Providing for the separation of duties between keypunching and error listing
operations.
B. Attaching plastic file protection rings to reels of magnetic tape before new
data can be entered on the file.
C. Making a validity check of an identification number before a user can have
access to the computer files.
D. Preparing batch totals to provide assurance that file updates are made for the
entire input.

13. One of the features that distinguishes computer processing from manual processing
is
A. Computer processing virtually eliminates the occurrence of computational
error normally associated with manual processing.
B. Errors or fraud in computer processing will be detected soon after their
occurrences.
C. The potential for systematic error is ordinarily greater in manual processing
than in a computerized processing.
D. Most computer systems are designed so that transaction trails useful for audit
do not exits.

14. What type of computer-based system is characterized by data that are assembled
from more than one location and records that are updated immediately?

A. Microcomputer system
B. Minicomputer system
C. Batch processing system
D. Online real-time system

15. For the accounting system of Acme Company, the amounts of cash disbursements
entered into on CBIS terminal are transmitted to the computer that immediately
transmits the amounts back to the terminal for display on the terminal screen. This
display enables the operator to

A. Establish the validity of the account number.


B. Verify that the amount was entered accurately.
C. Verify the authorization of the disbursement.
D. Prevent the overpayment of the account.

16. The technique that lends an effective control for protecting confidential information
from unauthorized access by transforming programs and information into an
unintelligible form:

A. Defragmentation.
B. Cryptography.
C. Trojan horse.
D. Recycle.
17. Which of the following is not a major reason why an accounting audit trail should be
maintained for a computer system?
A. Query answering
B. Deterrent to fraud
C. Monitoring purposes
D. Analytical review

18. Adequate control over access to data processing is required to


A. Prevent improper use or manipulation of data files and programs.
B. Ensure that only console operators have access to program documentation.
C. Minimize the need for backup data files.
D. Ensure that hardware controls are operating effectively and as designed by
the computer manufacturer.

19. In studying a client‘s internal controls, an auditor must be able to distinguish


between prevention controls and detection controls. Of the following data controls;
which is the best detection control?

A. Use of data encryption techniques


B. Review of machine utilization logs
C. Policy requiring password security
D. Backup and recovery procedure

20. Which of the following is classified as general computerized information system


controls that relates to segregation of duties?

A. Reconciliation of record counts.


B. Authorization of modifying the operating system.
C. A system of transaction logs.
D. Follow up all errors detected during processing.

21. For good internal control over computer program changes, a policy should be
established requiring that

A. The programmer designing the change adequately test the revised program.
B. All program changes be supervised by the CBIS control group.
C. Superseded portions of programs be deleted from the program run manual to
avoid confusion.
D. All proposed changes should be approved in writing by a responsible
individual.

22. Which of the following would result to an ineffective control in a computer-based


information system?

A. The computer librarian maintains custody of computer program instructions


and detailed listings.
B. Computer operators have access to operator instructions and detailed
program listings.
C. The control group is solely responsible for the distribution of all computer
output.
D. Computer programmers write and debug programs which perform routines
designed by the systems analyst.

23. What is the computer process called when data processing is performed
concurrently with a particular activity and the results are available as soon enough to
influence the particular course of action being taken or the decision being made?

A. Batch processing
B. Real time processing
C. Integrated data processing
D. Random access processing

24. Online real-time systems and electronic data interchange systems have the
advantages of providing more timely information and reducing the quantity of
documents associated with less automated systems. The advantages, however,
may create some problems for the auditor. Which of the following characteristics of
these systems does not create an audit problem?

A. The lack of traditional documentation of transactions creates a need for


greater attention to programmed controls at the point of transaction input.
B. Hard copy may not be retained by the client for long periods of time, thereby
necessitating more frequent visits by the auditor.
C. Control testing may be more difficult give the increased vulnerability of the
client‘s files to destruction during the testing process.
D. Consistent on-line processing of recurring data increases the incidence of
errors.

25. To ensure that goods received are the same as those shown on the purchase
invoice, a computerized system should:

A. Match selected fields of the purchase invoice to goods received.


B. Maintain control totals of inventory value.
C. Calculate batch totals for each input.
D. Use check digits in account numbers.

26. Errors in data processed in a batch computer system may not be detected
immediately because

A. Transaction trails in a batch system are available only for a limited period of
time.
B. There are delays in processing transactions in a batch system.
C. Errors in some transactions cause rejection of other transactions in the batch.
D. Random errors are more likely in a batch than in an on-line system.

27. An integrated group of programs that supervised and support operations of a


computer system as if executes users‘ application programs is called a(n)

A. Operating system
B. Utility programs
C. Data base management system
D. Language program

28. A compiler is

A. A procedure oriented language.


B. A machine that translates the assembler program to machine language.
C. A program that converts procedure oriented language to machine language.
D. A program that translates symbolic language to machine language.

29. An operating system is

A. The assembler program including the source and object program.


B. All hardware and software needed to operate the computer system.
C. The programs that manage the processing operations of the computer.
D. Only the hardware of the computer system.

30. Computer systems are becoming more vulnerable to unauthorized access because

A. Hardware design consideration have declined.


B. Software cannot be readily written to control access.
C. Systems documentation must be available to all users.
D. Access can be gained electronically without physical entry to the facilities.

31. A partial set of standard characteristics of a real-time system is

A. Batched input, on-line files, and an extensive communication network.


B. Reliance upon sequential files, prompt input from users, and interactive
programs.
C. On-line files, prompt input users, and an extensive communication network.
D. The use of high-level language and the major need being for historical
reports.

32. A system flowchart


A. Is synonymous with a program flowchart
B. Is necessary for only computer processes
C. Shows general flow and sequence but not processing details
D. Is necessary for only manual processes

33. An advantage of decentralizing data processing facilities is

A. Economy of scale obtainable through the use of microcomputer.


B. That all similar activities are better handled at a local level.
C. That system failure is of lesser significance.
D. The virtual elimination of the need for communication capability.

34. Which of the following may not be a purpose of using computer-audit software?

A. Add transactions or balances in the data files for comparison with control
account balance.
B. Select accounts or transactions for detailed testing.
C. To evaluate the collectability of accounts receivable.
D. To examine data bases for unusual items.

35. While systems analysis focuses on information needs and objectives, system design
concentrates on

A. Writing programs
B. Providing for controls
C. Testing completed modules
D. What to do and how to do it

36. All activities related to a particular application in a manual system are recorded in a
journal. The name of the corresponding item in a computerized system is a

A. Master file.
B. Transaction file.
C. Year-to-date file.
D. Current balance file.

37. The most important factor in planning for system change is

A. Having an auditor as member of the design team.


B. Using state-of-the-art techniques.
C. Involving top management and people who use the system.
D. Selecting the user to lead the design team.

38. Specialized programs that are made available to users of computer systems to
perform routing and repetitive functions are referred to as

A. Operating system.
B. Utility programs.
C. Database management system.
D. Language program.

39. An interactive system environment is best characterized by

A. Data files with records that are arranged sequentially.


B. Sorting the transaction file before processing.
C. Processing data immediately upon input.
D. The overlapping of input/output and processing operations.

40. Mainframe computer systems include several advanced processing procedures.


Two of the most common processing procedures are multiprocessing and
multiprogramming. Which one of the following statements about these processing
procedures is false?

A. Multiprogramming allows multiple programs to be executed at exactly the


same time.
B. Multiprogramming switches back and forth between programs during
processing.
C. Multiprocessing allows the sharing of a central memory during processing.
D. Multiprocessing allows multiple programs to be executed at exactly the same
time.

41. A system with several computers that are connected for communication can also
process its own data, is known as

A. Distributed data processing network.


B. Multidrop network.
C. Centralized network.
D. Decentralized network.

42. Which of the following represents a lack of internal control in a computer-based


information system?
A. The design and implementation is performed in accordance with
management‘s specific authorization.
B. Any and all changes in application programs have the authorization and
approval of management.
C. Provisions exist to ensure the accuracy and integrity of computer processing
of all files and reports.
D. Both computer operators and programmers have unlimited access to the
programs and data files.

43. The most critical aspect regarding separation of duties within the information
systems is between

A. Project leaders and programmers.


B. Programmers and computer operators.
C. Data control and file librarians.
D. Management and users.

44. Which of the following most likely represents a significant deficiency in the internal
control structure?

A. The systems analyst review applications of data processing and maintains


systems documentation.
B. The systems programmer design systems for computerized application and
maintains output control.
C. The control clerk establishes control over data received by the EDP
department and reconciles control totals after processing.
D. The accounts payable clerk prepares data for computer processing and
enters the data into the computer.

45. Which of the following activities would most likely be performed in the EDP
department?

A. Initiation of changes to master records.


B. Conversion of information to machine-readable form.
C. Correction of transactional errors.
D. Initiation of changes to existing applications.

46. An arrangement where two or more personal computers are linked together through
the use of special software and communication lines.

A. A stand-alone work station.


B. Extension.
C. Local area network.
D. Internet.

47. Which of the following controls most likely would assure that an entity can
reconstruct its financial records?

A. Hardware controls are built into the computer by the computer manufacturer.
B. Backup diskettes or tapes of files are stored away from originals.
C. Personnel who are independent of data input and output operations.
D. System flowcharts provide accurate description of input and output
operations.

48. Mill Co. uses a batch processing method to its sales transactions. Data on Mill‘s
sales transactions tape are electronically checks in preparing its invoice, sales
journals, and updated customer account balance. One of the direct outputs of the
creation of the tape most likely would be a

A. Report showing exceptions and control totals.


B. Printout of the updated inventory records.
C. Report show overdue accounts receivable
D. Printout of the sakes price master file

49. An auditor anticipates assessing control risk at a low level in a computerized


environment. Under these circumstances, on which , of the following procedires
would be the auditor initially focus?

A. Programmed control procedures


B. Application control procedures
C. Output control procedures
D. General control procedures.

50. A computer system that enables users to access data and programs directly
through workstations.

A. On-line computer systems


B. Data-base system
C. Flat-file system
D. Computer line system

51. Which of the following client electronic data processing (EDP) systems generally
can be audited without examining or directly testing the EDP computer programs of
the systems?
A. A system that performs relatively uncomplicated processes and produces
detailed output.
B. A system that affects a number of essential master filers and produces a
limited output.
C. A system that updates a few essential master files and produces no printed
output other than final balances.
D. A system that performs relatively complicated processing and produces very
little detailed output.

52. Computer systems are typically supported by a variety of utility software packages
that are important to an auditor because they

A. May enable unauthorized changes to data files if not properly controlled.


B. Are very versatile programs that can be used on hardware of many
manufacturers.
C. May be significant components of a client‘s application programs.
D. Are written specifically to enable auditors to extract and sort data.

53. To obtain evidence that online access controls are properly functioning an auditor
most likely would

A. Create checkpoints of periodic intervals after live data processing to test for
unauthorized use of the system.
B. Examine the transaction log to discover whether any transactions were lost or
entered twice due to a system malfunction
C. Enter invalid identification numbers or passwords to ascertain whether the
system rejects them.
D. Vouch a random sample of processed transactions to assure proper
authorization.

54. Whether or not a real time program contains adequate controls is most effectively
determined by the use of

A. Audit software
B. A tracing routine
C. An integrated test facility
D. A traditional test deck

55. An auditor would least likely use computer software to:

A. Access client date fails


B. Prepare spreadsheets
C. Assess EDP controls
D. Construct parallel simulations

56. Which of the following is not a basis of classifying an on-line system?

A. How information is processed


B. How information is entered into the system
C. When the results are available to the user
D. The type of information to be processed

57. Which of the following is not a characteristic of a batch processed computer


system?

A. The collection of like transactions which are sorted and processed


sequentially against a master file
B. Keypunching of transactions, followed by machine processing
C. The production of numerous printouts
D. The posting of a transaction, as it occurs, to several files, without immediate
printouts.

58. In a computerized system, procedure or problem-oriented language is converted to


machine language through a(an):

A. Interpreter
B. Verifier
C. Compiler
D. Converter

59. Adequate technical training and proficiency as an auditor encompasses an ability to


understand an IT system sufficiently to identify and evaluate:

A. The processing and imparting of information.


B. All accounting control features.
C. The degree to which programming conforms with application of generally
accepted accounting principles.
D. Essential control procedures

60. The computer process whereby data processing is performed concurrently with a
particular activity and the results are available soon enough to influence the course
of action being taken or the decision being made is called:

A. Random access sampling.


B. Integrated data processing.
C. On-line, real-time system.
D. Batch processing system.

61. A type of computer system whereby individual transactions are entered on line and
are added to a transaction file that contains other transactions entered during the
period. Later, this transaction file, on a periodic basis, updates the master file.

A. On-line, batch system.


B. On-line, real-time system.
C. On-line, memo update system.
D. On-line, shadow update system.

62. When a database administrator‘s position exists within an organization, the auditor
must be aware of the:

A. Output effectiveness/efficiency considerations


B. Need for coded program files
C. Use of encrypted dialog in a two-way authentication process
D. Inherent violation of the principle of separation of duties

63. Which of the following functions would have the least effect on an audit if they are
not properly segregated?

A. The systems analyst and the programmer functions.


B. The computer operator and programmer functions.
C. The computer operator and the user functions.
D. The applications programmer and the systems programmer.

64. Which of the following is the least significant characteristic of an on-line computer
system?

A. Adequately visible transaction trail


B. Potential access to the system by the programmer
C. On-line access to the system by users
D. On-line data entry and validation

65. When software or files can be accessed from on-line servers, users should be
required to enter a(n)

A. Parity check.
B. Self-diagnosis test.
C. Personal identification code.
D. Echo check.

66. Controls which are designed to assure that the information processed by the
computer is valid, complete, and accurate are called

A. Input controls.
B. Output controls.
C. Processing controls.
D. General controls.

67. Input controls include controls over:

A. Conversion of input data to machine-readable form.


B. Visual scanning.
C. Comparison to source documents.
D. Process tracing data.

68. Which of the following is mostly a potential threat to an on-line computer system?

A. A user may have the potential ability to make unauthorized changes to the
data and program.
B. Validation checks of data entered is usually ignored.
C. Most of transactions may be omitted because of the absence of adequate
audit trail.
D. Most of documents that will be processed may lack proper authorization.

69. In a computer system, hardware controls are designed to

A. Arrange data in a logical sequence for processing.


B. Correct errors in software.
C. Monitor and detect errors in source documents.
D. Detect and control errors arising from use of equipment.

70. Which of the following represent examples of general, application and user control
activities, respectively, in the computer environment?

A. Control over access to programs, computer exception reports, and manual


checks of computer output.
B. Manual checks of computer output, control over access to programs, and
computer exception reports.
C. Computer exception reports, control over access to programs, and manual
checks of computer output.
D. Manual checks of computer output, computer exception reports, and control
over access to programs.

71. Which of the following is not a type of general information technology controls?

A. Processing controls.
B. Systems development.
C. Physical and on-line security
D. Segregation of IT duties

72. Suppose that the computer operators are also programmers and have access to
computer programs and data files, in which of the following general controls will the
auditor most likely conclude to have a weakness?

A. Processing controls.
B. Back-up and contingency planning
C. Systems development
D. Segregation of computer professionals‘ duties

73. The possibility of losing a large amount of information stored in computer files most
likely would be reduced by the use of

A. Back-up files.
B. Check digits.
C. Completeness tests.
D. Conversion verification.

74. In considering a client‘s internal control structure in a computer environment, the


auditor will encounter general controls and application controls. Which of the
following is an application control?

A. Organizations charts
B. System flowcharts
C. Hash total
D. Control over program changes

75. Which of the following control policies would potentially prevent the programmer to
make unauthorized changes to programs?

A. An unscheduled rotation of duties of the different programmers.


B. Having a system programmer, a person different form an application
programmer.
C. A thorough checking and testing of the application program before they are
finally installed.
D. A strict implementation of a policy that prevents the programmer to have
access to program used in the computer operations

76. Totals of amounts in the computer-record data fields which are not usually added
for other purposes but are used only for data processing control purposes are called

A. A record totals.
B. Hash totals.
C. Processing data totals.
D. Field totals.

77. A hash total of employee numbers is part of the input to a payroll master file update
program. The program compares the hash total to the total computed for
transactions applied to the master file. The purpose of this procedure is to:

A. Verify that employee numbers are valid.


B. Verify that only authorized employees are paid.
C. Detect errors in payroll calculations.
D. Detect the omission of transaction processing.

78. An accounts payable program posted a payable to a vendor that is not included in
the on-line vendor master file. A control which would prevent this error is a

A. Validity check
B. Range check
C. Reasonableness test
D. Parity check

79. In a computerized sales processing system, which of the following controls is most
effective in preventing sales invoice pricing errors?

A. Sales invoices are reviewed by the product managers before being mailed to
customers.
B. Current sales prices are stored in the computer, and, as stock numbers are
entered from sales orders, the computer automatically prices the orders.
C. Sales prices, as well as product numbers, are entered as sales orders are
entered at remote terminal locations.
D. Sales prices are reviewed and updated on a quarterly basis.
80. A computer report which is designed to create an audit trail for each on-line
transaction.

A. Transaction file
B. Master file
C. Transaction edit report
D. Transaction log

81. If a control total were to be computed on each of the following data items, which
would best be identified as a hash total for a payroll computerized application?

A. Net pay
B. Department numbers
C. Hours worked
D. Total debits and total credits

82. A control to verify whether the peso amounts for all debits and credits for incoming
transactions are posted to a receivables master file is the:

A. Generation number check.


B. Master reference check.
C. Hash total.
D. Control total.

83. Which of the following would not be an appropriate procedure for testing the general
control activities of an information system?

A. Inquiries of client personnel.


B. Inspecting computer logs
C. Testing for the serial sequence of source documents.
D. Examination of the organizational chart to determine the segregation of
duties.

84. While entering data into the cash receipts transaction file, an employee transposed
two numbers in a customer code. Which of the following controls could prevent input
of this type of error?

A. Sequence check
B. Record check
C. Self-checking digit
D. Field-size check
85. Reconciling the processing control totals is an example of

A. An input control
B. An output control
C. A processing control
D. A file management control

86. The completeness of computer-generated sales figures can be tested by comparing


the number of items listed on the daily sales report with the number of items billed
on the actual invoices. This process uses

A. Check digits
B. Control totals
C. Validity tests
D. Process tracing data

87. If an auditor is using test data in a client‘s computer system to test the integrity of the
systems output, which of the following types of controls is the auditor testing?

A. General controls
B. User controls
C. Quantitative test controls
D. Application controls

88. Which of the following controls would be most efficient in reducing common data
input errors?

A. Keystroke verification
B. A set of well-designed edit checks
C. Balancing and reconciliation
D. Batch totals

89. EDP accounting control procedures are referred to as general or application


controls. The primary objective of application controls in a computer environment is
to

A. Ensure that the computer system operates efficiently.


B. Maintain the accuracy of the inputs, files and outputs for specific applications.
C. Ensure the separation if incompatible functions in the data processing
departments.
D. Plan for the protection of the facilities and backup for the systems.
90. Which of the following is an incorrect statement regarding testing strategies related
to auditing through the computer?

A. The test data approach involves processing the client‘s data on a test basis to
determine the integrity of the system
B. The test data approach involves processing the auditor‘s test data on the
client‘s computer system to determine whether computer-performed controls
are working properly
C. Test data should include all relevant data conditions that the auditor is
interested in testing
D. When the auditor uses the embedded audit module approach, an audit
module is inserted in the client‘s system to capture transactions with certain
characteristics

91. An internal auditor noted the following points when conducting a preliminary survey
in connection with the audit of an EDP department. Which of the following would be
considered a safeguard in the control system on which the auditor might rely?

A. Programmers and computer operators correct daily processing problems as


they arise
B. The control group works with user organizations to correct rejected input
C. New systems are documented as soon as possible after they begin
processing live data
D. The average tenure of employees working in the EDP department is ten
months

92. The employees in a manufacturing are made many errors as they wrote their clock
numbers on time sheets and cost distribution forms. An effective control technique
would have been the use of

A. Batch totals.
B. Turn around documents.
C. Hash totals.
D. Record counts.

93. An on-line access control that checks whether the user‘s code number is authorized
to initiate a specific type of transaction of inquiry us referred to as

A. Password
B. Computability test
C. Limit check
D. Reasonableness test
94. A control procedure that could be used in an on-line system to provide an
immediate check on whether an account number has been entered on a terminal
accurately is a

A. Compatibility test
B. Self-checking digit
C. Hash total
D. Sequence check

95. When erroneous data are detected by computer program controls, such data may
be excluded from processing and printed on an error report. The error report should
most probably be reviewed and followed up by the

A. IT control group
B. Supervisor of computer operations
C. System analyst
D. Computer programmer

96. An advantage of having a computer maintain an automated error log in conjunction


with a computer edit process is that

A. Reports can be developed that summarize the errors by type, cause and
person responsible.
B. Less manual work is required to determine how to correct errors.
C. Better editing techniques will result.
D. The audit trail is maintained.

97. Program documentation is a control designed primarily to ensure that

A. Programmers have access to the tape library or information on disk files.


B. Programs do not make mathematical errors.
C. Programs are kept up to date and perform as intended.
D. No one has made use of the computer hardware of personal reasons

98. In order to control purchasing and accounts payable, an information system must
include

A. Purchase order, receiving reports, and vendor invoices.


B. Receiving reports, and vendor invoices.
C. Purchase requisitions, purchase orders, receiving reports of goods needed,
and vendor invoices.
D. Purchase orders, receiving reports, and inventory reports of goods needed.
99. Which of the following is not an inherent characteristic of customize application
software?

A. They are typically used without modifications of the programs.


B. The programs are tailored-made according to the specific needs of the user.
C. They are developed by software manufacturer according to a particular user‘s
specifications.
D. It takes a longer time of implementation.

100.Payroll systems should have elaborate controls to prevent, detect, and correct
errors and authorized tampering. The best set of controls for a payroll system
includes

A. Batch and hash totals, record counts of each run, proper separation of duties,
special control over unclaimed checks, and backup copies of activities and
master files.
B. Passwords and user codes, batch totals, employee supervision and record
count for each run.
C. Sign tests, limit tests, passwords and user codes on-line edit check, and
payments by check.
D. Batch totals, record counts, user codes, proper separation of duties, and on-
line edit checks.

101.Some of the more important controls that relate to automated accounting


information systems are validity checks, limit checks, field checks, and sign tests.
These are classified as

A. Control total validation routines.


B. Output controls.
C. Input validation routines.
D. Data access validation routines.

102.In an automated payroll processing environment, a department manager


substituted the time card for a terminated employee with a time card for a fictitious
employee. The fictitious employee had the same pay rate and hours worked as the
terminated employee. The best control technique to detect this action using
employee identification numbers would be a

A. Batch total
B. Hash total
C. Record count
D. Subsequent check
103.An employee in the receiving department keyed in a shipment from a remote
terminal and inadvertently omitted the purchase order number. The best systems
control to detect this error would be

A. Batch total
B. Sequence check
C. Compatibility test
D. Completeness test

104.The reporting of accounting information plays a central role in the regulation of


business operations. Preventive controls are an integral part of virtually all
accounting processing systems, and much of the information generated by the
accounting system is used for preventive control purposes. Which one of the
following is not a preventive essential element of a sound preventive control system?

A. Separation of responsibilities for the recording, custodial, and authorization


functions.
B. Sound personnel policies
C. Documentation of policies and procedures
D. Implementation of state-of-the-art software and hardware

105.Which one of the following input validation routines is not likely to be appropriate in
a real time operation?

A. Field check
B. Sequence check
C. Redundant data check
D. Reasonableness check

106.Which of the following controls is a processing control designed to ensure the


reliability and accuracy of data processing?

Limit test Validity check

A. Yes Yes
B. No No
C. No Yes
D. Yes No

107.For control purposes, which of the following should be organizationally segregated


from the computer operations functions?
A. Data conversion
B. Surveillance of CRT messages
C. Systems development
D. Minor maintenance according to a schedule

108.When EDP programs or files can be accessed from terminals, users should be
required to enter a(n)

A. Parity check
B. Personal identification code
C. Self-diagnostic test
D. Echo check

109.An auditor would most likely be concerned with which of the following controls in a
distributed data processing system?

A. Hardware controls
B. Systems documentation controls
C. Access controls
D. Disaster recovery controls

110.Which of the following standards or group of standards is(are) mostly affected by a


computerized information system environment?

A. General standards
B. Standards of fieldwork
C. Reporting standards
D. All of these are equally affected.

111.A control feature in an electronic data processing system requires the central
processing unit (CPU) to send signals to the printer to activate the print mechanism
for each character. The print mechanism, just prior to printing, sends signal back to
the CPU verifying that the proper print position has been activated. This type of
hardware control is referred to as

A. Echo check
B. Validity control
C. Signal control
D. Check digit control

112.Which of the following is an example of a check digit?


A. An agreement of the total number of employees to the total number of
checks printed by the computer
B. An algebraically determined number produced but the other digits of the
employee number
C. A logic test that ensures all employee numbers are nine digits
D. A limit check that an employee‘s hours do not exceed 50 hours per work
week

113.A customer erroneously ordered item No. 86321 rather than item No. 83621. When
this order is processed, the vendor‘s EDP department would identify the error with
what type of control?

A. Key verifying
B. Self-checking digit
C. Batch total item inspection
D. Item inspection

114.In an automated payroll system, all employees in the finishing department were
paid at the rate of P75 per hour when the authorized rate was P70 per hour. Which
of the following controls would have been most effective in preventing such an error?

A. Access controls which would restrict the personnel department‘s access to


the payroll master file data
B. A review of all authorized pay rate changes by the personnel department
C. The use of batch control totals by department
D. A limit test that compares the pay rate per department with the maximum rate
for all employees

115.Which of the following errors would be detected by batch controls?

A. A fictitious employee as added to the processing of the weekly time cards


by the computer operator
B. An employee who worked only 5 hours in the week was paid for 50 hours
C. The time card for one employee was not processed because it was lost in
transit between the payroll department and the date entry functions
D. All of the above

116.After preliminary phase of the review of a client‘s EDP controls, an auditor may
decide not to perform tests of controls (compliance tests) related to the control
procedures within the EDP portion of the client‘s internal control structure. Which of
the following would not be a valid reason for choosing to omit such tests?
A. The controls duplicate operative controls existing elsewhere in the
structure
B. There appear to be major weaknesses that would preclude reliance on the
stated procedure
C. The time and costs of testing exceed the time and costs in substantive
testing if the tests of controls show the controls to be operative
D. The controls appear adequate

117.To obtain evident that user identification and password control procedures are
functioning as designed, an auditor would most likely

A. Attempt to sign on to the system using invalid user identifications and


passwords
B. Write a computer program that simulates the logic of the client‘s access
control software
C. Extract a random sample of processed transactions and ensure that the
transactions were appropriately authorized
D. Examine statements signed by employees stating that they have not
divulged their user identifications and passwords to any other person

118.A well prepared flowchart should make it easier for the auditor to

A. Prepare audit procedures manuals


B. Prepare detailed job descriptions
C. Trace the origin and disposition of documents
D. Assess the degree of accuracy of financial data

119.Testing controls without the use of the computer is possible when the:

A. Computer generates visible evidence of compliance with the control


B. Auditor does not fully understand the computer system
C. Controls appear to be adequate
D. Input/output is done in batches

120.Which of the following audit techniques most likely would provide an auditor with
the most assurance about the effectiveness of the operation of an internal control
procedure?

A. Inquiry of client personnel


B. Recomputation of account balance amounts
C. Observation of client personnel
D. Confirmation with outside parties
121.Creating simulated transactions that are processed through a system to generate
results that are compared with predetermined results, is an auditing procedure
referred to as:

A. Desk checking
B. Use of test data
C. Completing outstanding jobs
D. Parallel simulation

122.Which of the following is least likely a risk characteristic associated with CIS
environment?

A. Error embedded in an application‘s program logic maybe difficult to manually


detect on a timely basis
B. The separation of functional responsibilities diminishes in a computerized
environment
C. Initiation of changes in the master file is exclusively handled by respective
users
D. The potential unauthorized access to data or to alter them without visible
evidence maybe greater

123.Which of the following is an incorrect statement regarding testing strategies related


to auditing through the computer?

A. The test data approach involves processing the client‘s data on a test basis to
determine the integrity of the system
B. The test data approach involves processing the auditor‘s test data on the
client‘s computer system to determine whether computer performed controls
are working properly
C. Test data should include all relevant data conditions that the auditor is
interested in testing
D. When the auditor uses the embedded audit module approach, an audit
module is inserted in the client‘s system to capture transactions with certain
characteristics

124.Which of the following is not one of the auditor‘s major concerns when he has to
make a documentation of the internal control in a computerized environment?

A. The organizational structure of the clients CIS activities


B. The access controls
C. The significance and complexity of computer processing in each significant
accounting application
D. The use of software packages instead of customized software
125.An auditor is preparing test data for use in the audit of a computer-based accounts
receivable application. Which of the following items would be appropriate to include
as an item in the test data?

A. A transaction record which contains an incorrect master file control total


B. A master file record which contains invalid customer identification number
C. A master file record which contains an incorrect master file control total
D. A transaction record which contains an invalid customer identification number

126.If an auditor is using test data in a client‘s computer system to test the integrity of
the systems output, which of the following types of controls is the auditor testing?

A. General controls
B. User controls
C. Quantitative test controls
D. Applications controls

127.Which of the following is not a function of generalized audit software?

A. To aid in the random selection of transactions for substantive testing


B. To run in parallel with the client‘s application software and compare the output
C. To test the mathematical accuracy by footing and cross-foot items in the
accounting system
D. To keep an independent log of access to the computer applications software

128.Which of the following is not a computer-assisted audit technique?

A. Test data
B. Tagging and lagging
C. Integrated test facility
D. Program analysis

129.When auditing ―around the computer‖, the auditor performs tests of

A. General computer controls but does not test application computer controls
B. Application computer controls but does not test general computer controls
C. Neither general nor application computer controls
D. Both general and application computer controls

130.A common difficulty in auditing a computerized accounting system is:


A. Data can be erased from the computer with no visible evidence
B. Because of the lack of audit trail, computer systems have weaker controls
and more substantive testing is required
C. Because of the uniform nature of transaction processing, computer systems
have strong controls and less substantive testing is required
D. The large dissemination of entry points into the computer system leads to
weak overall reliance on information generated by a computer

131.In auditing through a computer, the test data method is used by auditors to test the

A. Accuracy of input data


B. Validity of the output
C. Procedures obtained within the program
D. Normalcy of distribution within the program

132.An integrated test facility (ITF) would be appropriate when the auditor needs to

A. Trace a complex logic path through an application system


B. Verify processing accuracy concurrently with processing
C. Monitor transactions in an application system continuously
D. Verify load module integrity for production programs

133.The auditor‘s objective to determine whether client‘s computer programs can


correctly handle valid and invalid transactions as they arise is accomplished through
the:
A. test data approach
B. Generalized audit software approach
C. Microcomputer-aided auditing approach
D. Generally accepted auditing standards

134.Which of the following is a disadvantage of the integrated test facility approach?

A. In establishing fictitious entities, the auditor may be compromising audit


independence
B. Removing the fictitious transactions from the system is somewhat difficult
and, if not carefully done, may contaminate the client‘s files
C. ITF is simple an automated version of auditing ―around‖ the computer
D. The auditor may not always have a current copy of the authorized version of
the client‘s program

135.When testing a computerized accounting system, which of the following is not true
of the test data approach?
A. The test data need consist pf only those valid and invalid conditions in which
the auditor is interested
B. Only one transaction of each type need be tested
C. Test data are processed by the client‘s computer programs under the
auditor‘s control
D. The test data must consist of all possible valid and invalid conditions

136.Which of the following procedures is an example of auditing ―around‖ the


computer?

A. The auditor traces adding machine tapes of sales order batch totals to a
computer printout of the sales journal
B. The auditor develops a set of hypothetical sales transactions and, using the
client‘s computer program, enters the transactions into the system and
observes the processing flow
C. The auditor enters hypothetical transactions into the client‘s processing
system during client processing of live data
D. The auditor observes client personnel as they process the biweekly payroll.
The auditor is primarily concerned with the computer rejection of data that
fails to meet reasonableness limits

137.Auditing by testing the input and output of a computer-based system instead of the
computer program itself will

A. Not detect program errors which do not show up in the output sampled
B. Detect all program errors regardless of the nature of the output
C. Provide the auditor with confidence in the results of the auditing procedures
D. Not provide the auditor with confidence in the results of the auditing
procedures

138.Which of the following is an acknowledged risk of using test data when auditing
computerized records?

A. The test data may not include all possible types of transactions
B. The computer may not process a simulated transaction in the same way it
would an identical actual transaction
C. The method cannot be used with simulated master records
D. Test data may be useful in verifying the correctness of account balances, but
not in determining the presence of processing controls

139.An auditor used test data to verify the existence of controls in a certain computer
program. Even though the program performed will on the test, the auditor may still
have a concern that
A. The program tested is the same one used in the regular production runs
B. Generalized audit software may have been a better tool to use
C. Data entry procedures may change and render the test useless.
D. No documentation exists for the program

140.An auditor most likely would introduce test data into a computerized payroll system
to test internal controls related to the

A. Existence of unclaimed payroll checks held by supervisors


B. Early cashing of payroll checks by employees
C. Discovery of invalid employee I.D. numbers
D. Proper approval of overtime by supervisors

141.When an auditor tests a computerized accounting system, which of the following is


true of the test data approach?

A. Test date must consist of all possible valid and invalid conditions
B. The program tested is different from the program used throughout the year by
the client
C. Several transactions of each type must be tested
D. Test data are process by the client‘s computer programs under the auditor‘s
control

142.Which of the following is not among the errors that an auditor might include in the
test data when auditing a client‘s EDP system?

A. Numeric characteristics in alphanumeric fields


B. Authorized code
C. Differences in description of units of measure
D. Illogical entries in fields whose logic tested by programmed consistency
checks

143.An auditor who is testing EDP controls in a payroll system would most likely use
test data that contain conditions such as

A. Deductions not authorized by employees


B. Overtime not approved by supervisors
C. Time tickets with valid job numbers
D. Payroll checks with unauthorized signatures
144.Which of the following computer-based assisted auditing techniques allows
fictitious and real transactions to be processed together without client operating
personnel being aware of the testing process?

A. Integrated test facility


B. Input controls matrix
C. Parallel simulations
D. Data entry monitor

145.Which of the following methods of testing application controls utilized a generalized


audit software package by the auditors?

A. Parallel simulation
B. Integrated testing facility approach
C. Test data approach
D. Exception report tests

146.Test data, integrated test data and parallel simulation each require an auditor to
prepare data and computer programs. CPA‘s who lack either the technical expertise
or time to prepare programs should request from the manufacturers or EDP
consultants for

A. The program Code


B. Flowchart checks
C. Generalized audit software
D. Application controls

147.Parallel simulation is an audit technique employed to verify processing logic by


making use of audit test programs. These audit test programs ―simulate‖ the
processing logic of an application program or programs under review. Which
statement indicates the use of parallel simulation audit techniques?

A. Live transactions are processed using live programs


B. Live transactions are processed with test master fil
C. Test transactions are processed using test programs
D. Live transactions are processed using test programs.

148.The output of a parallel simulation should always be

A. Printed on a report
B. Compared with actual results manually
C. Compared with actual results using comparison program
D. Reconciled to actual processing output

149.In auditing through the computer, actual client data is used with:

A. Integrated test facility


B. The test data approach
C. Parallel simulation
D. An expert system

150.Assume that an auditor estimated that 10,000 checks were issued during the
accounting period. If an application control that performs a limit check for each check
request is to be subjected to the auditor‘s test-data approach, the sample should
include

A. Approximately 1,000 test items


B. A number of test items determined by the auditor to be sufficient under the
circumstances
C. A number of test items determine by the auditor‘s reference to the appropriate
sampling tables
D. One transaction
Module 8

Documentation, financial assertions, and audit evidence

PSA- based questions

1. When receiving audit working papers, the primary responsibility of an audit


supervisor is to determine that

A. Each worksheet is properly identified with a decretive heading.


B. Working papers are properly reference and kept in logical groupings.
C. Standard departmental procedures are adhered to with regard to working
paper preparation and technique.
D. Working papers adequately support the audit findings, conclusions, and
report.

2. A difference of opinion concerning accounting and auditing matters relative to a


particular phase of the audit arises between an assistant auditor who is an the senior
auditor responsible for the engagement. After appropriate consultation, the assistant
auditor asks to be disassociated from the resolution of the matter. The working
papers would probably be

A. Silent on the matter because it is an internal matter for the auditing firm.
B. Expanded to document that the assistant auditor is completely disassociated
from responsibility for the auditor‘s opinion.
C. Expanded to document the additional work required because all
disagreements of this type will require further substantive testing.
D. Expanded to document the assistant auditor‘s position and the manner in
which the difference of opinion was resolved

3. During the course of an audit engagement, an auditor prepares and accumulates


audit working papers. The primary purpose of audit working papers is to

A. Aid the auditor in adequately planning his work.


B. Serve as a reference for future audit engagements.
C. Support the underlying concepts included in the preparation of the basi
financial statements.
D. Support the auditor‘s opinion
4. Which of the following characteristics is most important in assuring the achievement
of the primary purpose of worming papers?

A. Working papers must be of standard format and standard content


B. Working papers must be properly indexed and corss-referenced to the draft of
audit report.
C. Working papers must provide sufficient, competent, and useful information to
support the audit report.
D. Working papers must be arranged in logical order following the audit program
sequence.

5. The primary purpose of the auditor‘s working papers is to

A. Provide evidence of planning and execution of audit procedures performed.


B. Serve as a means with which to prepare the financial statements.
C. Document the deficiencies in internal control with recommendations to
management for improvement.
D. Comply with the auditing standards of the profession.

6. What is (are) the purpose(s) of audit documentation?

A. Provide reasonable assurance that the audit is conducted in accordance with


PSAs
B. Provide a basis for determining the appropriate audit report
C. Provide the supervisory personnel an opportunity to assess the sufficiency of
evidence obtained during and audit.
D. Audit documentation serves all the given choices.

7. Which of the following is not a primary purpose of audit working papers?

A. Coordinate the examination


B. Assist in the preparation of the audit report
C. Support the financial statements
D. Provide evidence of the audit work performed

8. Which of the following conditions constitutes inappropriate working paper


preparation?

A. All forms and memoranda used/issued by the auditee department are


included in the working papers.
B. Flowcharts are included in the working papers.
C. The findings are cross-referenced to the supporting documentation.
D. Tick marks are explained in the working papers.

9. Working papers that record the procedures used by the auditor to gather evidence
should be

A. Considered the primary support for the financial statements being audited.
B. Viewed as the connecting link between the books of accounts and the
financial statements.
C. Designed to meet the circumstances of the particular engagement.
D. Destroyed when the particular audit engagement is terminated.

10. Which of the following factors will least affect the auditor‘s judgement as to the
quantity, type, and content of the working papers desirable for a particular
engagement?

A. Nature of the auditor‘s report


B. Nature of the financial statements, schedules, or other information upon
which the auditor is reporting
C. Need for supervision and review
D. Number of personnel assigned to the audit

11. During the audit engagement, data are compiled and included in the audit working
papers. The working papers are

A. Client-owned record of conclusions reached by the auditors who performed


the engagement.
B. Evidence supporting financial statements.
C. Support for the auditor‘s compliance with generally accepted auditing
standards.
D. A record to be used as a basis for the following year‘s engagement.

12. Which of the following is not a factor affecting the independent auditor‘s judgement
about the quantity, type, and content of audit working papers?

A. The need for supervision and review of the work performed by assistants.
B. The nature and condition of the client‘s records and internal controls.
C. The expertise of the client personnel and their participation in preparing the
schedules.
D. The type of financial statements, schedules, or other information on which the
auditor is reporting.

13. Which of the following factors most likely affects the auditor‘s judgment about the
quantity, type and content of the working papers?

A. The assessed level of control risk


B. The content of the client‘s representation letter
C. The timing of substantive tests completed prior to the balance sheet date
D. The usefulness of the working papers as a reference source for the client

14. Audit working papers are used to record the results of the auditor‘s evidence
gathering procedures. When preparing working papers, the auditors should
remember that working papers should be

A. Kept on the client‘s premises so that the client can have access to them for
reference purposes.
B. The primary support for the financial statements being examined,
C. Considered as part of the client‘s accounting records that are retained by the
auditor.
D. Designed to meet the circumstances and the auditor‘s needs on each
engagement.

15. Audit working papers are indexed by means of reference numbers. The primary
purpose of indexing is to

A. Permit cross-referencing and simplify supervisory review.


B. Support the audit report.
C. Eliminate the need for follow-up reviews
D. Determine that working papers adequately support the findings, conclusions
and reports.

16. The principal purpose for cross-indexing the audit working papers is to

A. Give the working papers a professional appearance.


B. Explain the use of tick marks.
C. Provide an explanation on the audit steps performed.
D. Provide a trail for the auditor and the reviewer.

17. Documentation may not be deleted form the working papers after the
A. Audit report delivery date.
B. Date of the audit report.
C. Completion of the assembly of final audit file.
D. Final day of field work

18. The reason why the auditors accumulate evidence is to

A. Defend themselves in the event of lawsuit.


B. Justify the conclusions they have otherwise reached.
C. Satisfy the requirements of the Bureau of Internal Revenue.
D. Enable them to reach conclusions about the fairness of the financial
statements and issue an appropriate audit report.

19. To be competent, evidence must be all of the following except:

A. Sufficient
B. Reliable
C. Relevant
D. Unbiased

20. Which of the following is not one of the determinants of the persuasiveness of
evidence?

A. Competence
B. Physical examination
C. Relevance
D. Sufficiency

21. In determining the sufficiency of evidential matter, which of the following would not
normally be a factor?

A. Cost/benefit considerations
B. The sampling technique used
C. Audit risk
D. Materiality of the account

22. Which of the following statements is not true regarding the competence of audit
evidence?
A. Relevance is enhanced by an effective information system.
B. To be competent, evidence must be both valid and relevant.
C. Validity is related to the quality of the client‘s information system.
D. Relevance must always relate to audit objectives.

23. Which of the following statements concerning evidence is correct?

A. Competent evidential matter supporting management‘s assertions should be


convincing rather than merely persuasive
B. Effective internal control unlikely contributes to the reliability of the evidence
created within the entity.
C. The cost of obtaining evidence is not an important consideration to anauditor
in deciding what evidence should be obtained.
D. A client‘s accounting data cannot be considered a sufficient audit evidence to
support the financial statements.

24. Which of the following statements is incorrect about audit evidence?

A. Evidence obtained from an independent source outside the client organization


is more reliable than that obtained from within.
B. Documentary evidence is more reliable when it is received by the auditor
directly from an independent third party.
C. Documents that originate outside the company are considered more reliable
than those that originate within the client‘s organization.
D. External evidence, such as communications from banks, is generally
regarded as more reliable than the information obtained from the client.

25. Which of the following factors is most important in determining the competence of
audit evidence?

A. The reliability of the evidence in meeting the audit objective


B. The objectivity of the auditor in gathering the evidence
C. The quantity of the evidence obtained
D. The independence of the source of evidence

26. Which of the following pertains to the reliability of audit evidence?

A. The independence of the source of evidence


B. The experience level of the auditor who obtains the evidence
C. Whether the audit client uses a manual or computerized accounting system
D. The quantity of the evidence obtained

27. Which of the following is not one of the characteristics of competent evidence?

A. Independence of the source of the evidence


B. Effectiveness of internal control structure under which the internal evidence
has been developed
C. Size of the sample
D. Degree of objectivity of the auditor

28. Which of the following presumptions does not relate to the competence of audit
evidence?

A. The more effective the internal control is, the more assurance it provides
about the accounting data and financial statements
B. An auditor‘s opinion, to be economically useful, is formed within a reasonable
time and based on evidence obtained at a reasonable cost.
C. Evidence obtained from independent sources outside the entity is more
reliable than evidence secured solely within the entity.
D. The independent auditor‘s direct personal knowledge, obtained through
observation and inspection, is more persuasive than information obtained
indirectly.

29. Which of the following statement relating to the competence of evidential matter is
always true?

A. Evidential matter gathered by an auditor from outside an enterprise is reliable.


B. Accounting data developed under satisfactory conditions of internal control
are more relevant than data developed under unsatisfactory conditions.
C. Oral representations made by managements are not valid
D. Evidence gathered by auditors must be both valid and relevant to be
considered competent.

30. Although the validity of evidential matter is dependent on the circumstances under
which it is obtained, there are three general presumptions that have some
usefulness. The situations given below indicate the relative reliability that a CPA has
placed on two types of evidence obtained in different situations. Which of these is an
exception to one of the general presumptions?
A. The CPA places more reliance on the balance in the scrap sales account at
Plant A, where the CPA has made limited tests of transactions because of
effective controls, than at Plant B where the CPA has made extensive tests of
transactions because of ineffective controls.
B. The CPA places more reliance on the CPAs computation of interest payable
on outstanding bonds than on the amount confirmed by the trustee.
C. The CPA places more reliance on the report of an expert on an inventory of
precious gems than on the CPAs physical observation of the gems.
D. The CPA places more reliance on a schedule of insurance coverage obtained
from the company‘s insurance agent than on one prepared by the internal
audit staff.

31. Which of the following would not be a factor in determining the competence of
evidential matter?

A. The source of the evidence


B. The relevance of the evidence
C. The cost of gathering the evidence
D. Timeliness of the evidence

32. Which of the following statements is incorrect?

A. There are many ways ab auditor can accumulate evidence to meet the overall
audit objectives.
B. Sufficient competent evidence must be accumulated to meet the auditor‘s
professional responsibility.
C. The cost of accumulating the evidence should be minimized.
D. Gathering evidence and minimizing costs are equally important.

33. Each of the following might, by itself, form a valid basis for an auditor of deciding to
omit a test except for the:

A. Difficulty and expense involved in testing a particular item


B. Assessment of control risk at a low level
C. Inherent risk involved
D. Relationship between the cost obtaining evidence and its usefulness

34. The following statements were made in a discussion of audit evidence by two
independent auditors. Which of these statements is not valid?
A. ―I am seldom convinced beyond all doubt about all aspects of the financial
statements being audited.‖
B. ―I would not undertake that procedure because at best the results would only
be persuasive and I‘m looking for convincing evidence.‖
C. ―I evaluate the degree of risk involved in deciding the kind of evidence I will
gather.‖
D. ―I evaluate the usefulness of the evidence I can obtain against the cost to
obtain it.‖

35. Management assertions that are embodied in the financial statements are

A. Directly related to standards on auditing


B. Directly related to financial reporting framework
C. Indirectly related to standards on auditing
D. Indirectly related to financial reporting framework

36. Management assertions are


A. Stated in the footnotes to the financial statements.
B. Implied or expressed representations about the financial statements.
C. Explicit representations about the financial statements.
D. Provided to the auditor in the assertions letter, but are not disclosed in the
financial statements.

37. As used in auditing, which of the following statements best describes ―assertions‖?
A. Assertions are the representations of managements as to the reliability of the
information system.
B. Assertions are the auditor‘s findings to be communicated in his audit report.
C. Assertions are the representations of managements as to the fairness of
presentation of the financial statements.
D. Assertions are found only in the notes to the financial statements.

38. Financial statement assertions include all of the following except:


A. Occurrence
B. Presentation and disclosure
C. Consistency and comparability
D. Completeness

39. The audit objective ―that all transactions and accounts that should be presented in
the financial statements are included‖ is related to which assertion?
A. Occurrence
B. Rights and obligations
C. Completeness
D. Presentation and disclosure

40. The audit objective ―that all footnotes have been included in the financial statements‖
is related most closely to which assertion?

A. Existence or occurrence
B. Rights and obligations
C. Completeness
D. Presentation and disclosure

41. Which of the following is a management assertion that relates to the valuation or
allocation of fixed assets?

A. Fixed assets are properly classified as noncurrent assets


B. Fixed asset depreciation has been correctly calculated
C. The client has title to the machinery and equipment
D. Lien or encumbrance on fixed assets is appropriately disclosed in the notes to
the financial statements

42. Which of the following statements about the existence and completeness objectives
is incorrect?
A. The existence and completeness objectives emphasize opposite audit
concerns
B. Existence deals with overstatements and completeness deals with
understatements.
C. Existence deals with understatements and completeness deals with
overstatements.
D. The completeness objective deals with unrecorded transactions.

43. If reported sales for 2009 erroneously included sales that occurred in 2010, the
assertion violated on the 2009 statements would be

A. Occurrence
B. Completeness
C. Presentation and disclosure
D. Rights and obligation
44. The completeness assertion would be violated if:
A. Fictitious sale transactions were included in accounts receivable.
B. Unbilled shipments had occurred during the period.
C. The balance of accounts payable was overstated.
D. Disclosure in the statements of pledged receivable was inadequate.

45. The rights and obligations assertion applies to


A. Current liability items only
B. Balance sheet items only
C. Both income statement and balance sheet items
D. Assets that are not owned by the company

46. Which of the following is incorrect?


A. It would be a violation of the completeness assertion if managements would
record a sale that did not take place.
B. The completeness assertion deals with matters opposite from those of the
existence/occurrence assertion.
C. The completeness assertion is concerned with the possibility of omitting items
from the financial statements that should have been included.
D. The existence/occurrence assertion is concerned with inclusion of amounts
that should not have been included.

47. Which of the following best describes the primary purpose of audit procedures?
A. To detect errors or irregularities
B. To comply with financial reporting standards
C. To gather corroborative evidence
D. To verify the accuracy of account balances

48. Physical examination of assets is not sufficient form of evidence when the auditor
wants to determine the
A. Existence of the asset
B. Quantity and description of the asset
C. Condition or quality of the asset
D. Ownership of the asset

49. Physical examination is not an objective means of ascertaining an asset‘s


A. Quantity
B. Description
C. Condition or quality
D. Ownership
50. Which of the following statements is not true? ―The evidence gathering technique of
observation
A. Is useful in most parts of the audit.‖
B. Is rarely sufficient by itself.‖
C. Is limited to what the auditor sees.‖
D. Requires the gathering of corroborative evidence.‖

51. A letter to the auditor in response to an inquiry is an example of


A. Physical evidence
B. Confirmation evidence
C. Documentary evidence
D. Analytical evidence

52. A CPA, who is performing an independent audit, would most likely use recalculation
as a substantive test for which of the following expense-related accounts?
A. Purchases of supplies
B. Interest expense
C. Advertising expense
D. Repairs and maintenance expense

53. ―Evaluations of financial information made by a study of plausible relationships


among financial and nonfinancial date involving comparisons of recorded amounts to
expectations developed by the auditor.‖ Refers to:
A. Auditing
B. Tests of balances
C. Tests of transactions
D. Analytical procedures

54. External auditors often confirm assertions contained in the organization‘s financial
statements and accounting records with third parties. Which of the following best
explains why confirmation produces an evidence of high quality?
A. Written assertions from knowledgeable third parties provide sufficient
evidence to achieve most audit objectives.
B. Confirmation by knowledgeable third parties is usually the most relevant
evidence available.
C. Confirmation by knowledgeable third parties is usually the least costly
evidence to obtain.
D. Confirmation by knowledgeable third parties is highly competent because of
its independent source.
55. When comparing the reliability of external versus internal documents, the external
documents are generally considered
A. More reliable
B. Less reliable
C. Equally reliable
D. Unreliable

56. Traditionally, confirmations are used to verify


A. Individual transactions between organizations, such as sales transactions.
B. Fixed asset addition
C. Bank balances and accounts receivable
D. Any of the given responses

57. Analytical procedures are so important that they are required during
A. Planning and completion phases
B. Planning and testing phases
C. Testing and completion phases
D. Planning, testing, and completion phases

58. Analytical procedures used in planning an audit should focus


A. Evaluating the adequacy of evidence gathered concerning unusual balances
B. Testing individual account balances that depend on accounting estimates.
C. Enhancing the auditor‘s understanding of the client‘s business.
D. Identifying material weaknesses in internal control.

59. Analytical procedures are


A. Substantive tests designed to evaluate system of internal control
B. Tests of controls designed to evaluate the validity of management‘s
representation letter.
C. Substantive tests designed to evaluate the reasonableness of financial
information.
D. Tests of controls designed to evaluate the reasonableness of financial
information.
QUIZZERS

DOCUMENTATION

1. In determining the quantity and quality of evidence to gather, the auditor will be
satisfied when the evidence is

A. Irrefutable.
B. Highly persuasive.
C. Conclusive.
D. Completely convincing.

2. The current file of the auditor‘s working papers generally should include

A. A flowchart of the internal controls.


B. A copy of the financial statements.
C. Organizational charts.
D. Copies of bond and note indentures.

3. It refers to the detailed instructions for the collection of a particular type of audit
evidence that is to be obtained at some time during the audit

A. Sampling plan.
B. Audit procedure
C. Audit program
D. Audit plan

4. The auditor‘s working papers will least likely include documentation showing how the

A. Schedules are prepared by the client personnel.


B. Engagement is planned
C. Understanding of the client‘s internal control is obtained and control rise was
assessed.
D. Unusual matters are resolved

5. Although the quantity and content of audit working papers vary with each particular
engagement, an auditor‘s permanent files most likely include

A. Schedules that support the current year‘s adjusting entries.


B. Prior years‘ accounts receivable confirmations that are classified as
exceptions.
C. Documentation indicating that the audit work is adequately planned and
supervised.
D. Analysis of capital stock and other owners‘ equity accounts.
6. The audit working paper that reflects the major components of an amount reported
in the financial statements is the

A. Interbank transfer schedule


B. Supporting schedule
C. Carry forward schedule
D. Lead schedule

7. An auditor ordinarily uses a working trial balance resembling the financial statements
without footnotes, but containing columns for

A. Cash flow increases and decreases


B. Audit objectives and assertions
C. Reclassifications and adjustments
D. Reconciliation and tick marks

8. In the course of an audit of financial statements for the purpose of expressing an


opinion thereon, the auditor will normally prepare a schedule of unadjusted
differences for which he did not propose adjustments when they were uncovered.
The primary purpose of this schedule is to

A. Point out to the responsible client officials the errors made by various
company personnel
B. Summarize the adjustments that must be made before the company can
prepare and submit its income tax returns
C. Identify the potential effects on the financial statement of errors or disputed
items that were considered immaterial when discovered.
D. Summarize the errors made by the company so that corrections can be made
after the audited financial statements are released.

9. Which of the following analyses appearing in a predecessor‘s working papers would


the successor auditor least likely review?

A. Analysis of noncurrent balance sheet accounts


B. Analysis of current balance sheet accounts
C. Analysis of contingencies
D. Analysis of income statement accounts

10. Using personal computers in auditing may affect the methods used to review the
work of staff assistants because

A. The audit fieldwork standards for supervision may differ


B. Documenting the supervisory review may require assistance of consulting
services personnel
C. Supervisory personnel may not have an understanding of the capabilities and
limitations of personal computers
D. Working paper documentation may not contain readily observable details of
calculations

11. In an internal audit, the audit supervisor determines that the working papers are
complete

A. When satisfied that the audit objectives have been met and the working
papers support the conclusions
B. When working papers refer to the steps outlined in the audit program
C. Only after the auditor who prepared the working papers has signed and dated
them
D. When proper cross-references to other working papers are noted.

12. Standardized working papers are often used, chiefly because they allow working
papers to be prepared more

A. Efficiently
B. Comprehensively
C. Neatly
D. Accurately

13. After the fieldwork audit procedures are completed, a partner of the CPA firm who
has not been involved in the audit performs a second or wrap-up working paper
review. This second review usually focuses on

A. The fair presentation of the financial statements in conformity with GAAP


B. Fraud involving the client‘s management and its employees
C. The materiality of the adjusting entries proposed by the audit staff
D. The communication of internal control weaknesses to the client‘s audit
committee

14. An auditor‘s working papers should

A. Not be permitted to serve as a reference source for the client


B. Not contain comments critical of management
C. Show that the accounting records agree or reconcile with the financial
statements
D. Be considered the primary support for the financial statements being audited

15. ―The detailed description of the results of the various evidence decisions for a
specific audit‖ is called an

A. Audit procedure.
B. Audit plan
C. Audit program
D. Audit guide

16. In using the work of a specialist, an understanding should exist among the auditor,
the client, and the specialist as to the nature of the work to be performed by the
specialist. Preferably, the understanding should be documented and would include
all of the following except

A. The objectives and scope of the specialist‘s work


B. The specialist‘s representations as to his relationship, if any, to the client
C. The specialist‘s understanding of the auditor‘s corroborative use of the
specialist‘s findings in relation to the representations in the financial
statements
D. A statement that the methods or assumptions to be used are not inconsistent
with those used by the client

17. Which of the following is an invalid description of why working papers are
developed?

A. Facilitates third-party reviews


B. Aids in the planning, performance, and review of audits
C. Provides the principal evidential support for the auditor‘s report
D. Aids in the professional development of the operating staff

18. During the working paper review, an audit supervisor finds that the auditor‘s reported
findings are not adequately cross-referenced to the supporting documentation. The
supervisor will most likely instruct the auditor to

A. Prepare a working paper to indicate that the full scope of the audit was
carried out
B. Familiarize himself with the sequence of working papers so that he will be
able to answer questions about the conclusions stated in the report
C. Eliminate any cross-references to other working papers since the system is
unclear
D. Provide a working paper indexing system that shows the relationship between
findings, conclusions, and the related facts.

19. The main advantage of properly indexed working papers is to

A. Reduce the size of the file


B. Better organize the working papers
C. Allow division of labor within the audit team
D. Facilitate the efficient use of audit staff

20. Which of the following statements about working papers is correct?


A. Working papers are not permitted to be used as a reference source by the
client
B. The auditor should document his understanding of the client‘s internal control
which is to be used to plan the audit
C. Working papers may be regarded as a substitute for the client‘s accounting
records
D. When reporting on comparative financial statements, the independent auditor
may discard working papers after two years

21. Which of the following is a basic tool used by the auditor to control the audit work
and review the progress of the audit?

A. Time and expense summary


B. Engagement letter
C. Progress flowchart
D. Audit program

22. Which of the following working papers would one normally expect to find in the
permanent file?

A. A copy of a long-term bond indenture


B. The working trial balance
C. An analysis of additions and disposals relating to marketable securities
D. A workpaper analyzing customer replies to confirmation requests

23. The permanent file section of the working papers that is kept for each audit client
most likely contains

A. Review notes pertaining to questions and comments regarding the audit work
performed
B. A schedule of time spent on the engagement by each individual auditor
C. Correspondence with the client‘s legal counsel concerning pending litigation
D. Narrative descriptions of the client‘s internal control policies and procedures

24. Which of the following statements is correct with respect to ownership of audit
documentation?

A. The audit firm owns the audit documentation


B. The audit client owns the audit documentation
C. The audit client and audit firm jointly own the audit documentation
D. The law is not explicit with respect to the ownership of audit documentation

FINANCIAL ASSERTIONS AND AUDIT OBJECTIVES

25. Which of the following statements is true?


A. The auditor‘s objectives follow and are closely related to management
assertions
B. Management‘s assertions follow and are closely related to the auditor‘s
objectives
C. The auditor‘s primary responsibility is to find and disclose fraudulent
management assertions
D. Assertions about presentation and disclosure deal with whether the accounts
have been included in the financial statements at appropriate amounts

26. Which of the following is an incorrect statement about audit objectives?

A. There should be a one-to-one relationship between audit objectives and


procedures
B. Audit objectives should be developed on the basis of management assertions
about the financial statement components
C. Selection of tests to meet audit objectives should depend upon the
understanding of internal control
D. The auditor should resolve any substantial doubt about any of the
management‘s material financial statement assertions

27. Which of the following assertions is least likely to be tested exclusively at an interim
date?

A. Existence for inventory


B. Completeness for accounts receivable
C. Existence for equipment
D. Valuation for marketable securities

28. Assuming a low assessed level of control risk, which of the following audit
procedures is least likely to be performed?

A. Physical inspection of a sample of inventory


B. Search for unrecorded cash receipts
C. Obtaining a client representation letter
D. Confirmation of accounts receivable

29. Which of the following auditing procedures is ordinarily performed last?

A. Reading the minutes of the directors‘ meetings held during the audit year
B. Confirming accounts payable balances
C. Obtaining a management representation letter
D. Testing of control procedures on purchasing function
30. Which of the following is not one of the broad categories of assertions?

A. General or specific transaction objectives


B. Existence or occurrence
C. Valuation or allocation
D. Presentation and disclosure

31. Determining whether amounts are in conformity with GAAP addresses the proper
measurement of assets, liabilities, revenues, and expenses which includes all of the
following except:

A. The reasonableness of management‘s accounting estimates


B. Proper application of valuation principles
C. Proper application of matching principle
D. The reasonableness of management‘s accounting policies

32. For a particular assertion, control risk is the risk that

A. Control will now detect a material misstatement that occurs


B. Audit procedures will fail to detect a weak control system
C. The prescribed control procedures will not be applied uniformly
D. A material misstatement will occur in the accounting process

33. Which of the following is an incorrect statement?

A. An example of a completeness assertion would be that notes payable in the


balance sheet includes all such obligations of the entity
B. An example of an occurrence assertion would be that sales in the income
statement represent exchanges of goods or services that actually take place
C. An example of a rights/obligations assertion would be that amounts
capitalized for leases in the balance sheet represent the cost of the entity‘s
rights to leased property
D. An example of a valuation/allocation assertion would be that property, plant,
and equipment are recorded at market value

34. A distinction must be made between general audit objectives and specific audit
objectives for each account balance. Which of the following is an incorrect
statement?

A. The general audit objectives are applicable to every account balance on the
financial statements
B. The specific audit objectives are applicable to every account balance on the
financial statements
C. The general audit objectives are tailored to the engagement
D. The specific audit objectives are tailored to the engagement
35. Which of the following ―general transaction-related audit objectives‖ is not part of the
valuation or allocation assertion?

A. Completeness
B. Accuracy
C. Classification
D. Timing

36. Only three of the following management assertions are associated with transaction-
related audit objectives. Which one of the following is not?

A. Existence or occurrence
B. Completeness
C. Valuation or allocation
D. Presentation and disclosure
37. Which of the following statements is incorrectly stated?

A. Balance-related audit objectives are applied to account balance


B. Transaction-related audit objectives are applied to classes of transactions
C. Balance-related audit objectives are applied to the ending balance in balance
sheet accounts
D. Balance-related audit objectives are applied to both beginning and ending
balances in the balance sheet accounts

38. The detail tie-in objective is not concerned that the details in the account balance

A. Agree with related subsidiary ledger accounts


B. Are properly disclosed, in accordance with PFRS
C. Foot to the total in the account balance
D. Agree with the total in the general ledger

39. The disclosure objective is concerned that

A. The account balance is properly presented in the financial statements


B. Disclosure requirements are properly presented in the financial statements
and in the footnotes
C. Both responses are correct
D. Both responses are incorrect

40. If a long-term note receivable is included in the account receivable listing, there is a
violation of the

A. Existence objective
B. Completeness objective
C. Classification objective
D. Timing objective
41. After the general objectives are understood, specific objectives for each account
balance on the financial statements can be developed. Which of the following
statements is true?

A. There should be at least one specific objective for each relevant general
objective
B. There will be only one specific objective for each relevant general objective
C. There will be many specific objectives developed for each relevant objective
D. There must be one specific objective for each general objective

42. Which of the following is not a proper matching of auditor‘s objective with
management‘s assertion?

A. Validity matches with existence or occurrence


B. Completeness matches with completeness
C. Ownership matches with rights and obligations
D. Classification matches with presentation/disclosure

43. An audit process is a well-defined methodology for organizing an audit to ensure that

A. The evidence gathered is both sufficient and competent


B. All appropriate audit objectives are specified
C. All appropriate audit objectives are met
D. All of the responses are correct

AUDIT EVIDENCE

44. Which of the following is correct?

A. The evidence that the auditor accumulates remains the same from audit to
audit, but the general objectives vary, depending on the circumstances
B. The general audit objectives remain the same from audit to audit, but the
evidence varies, depending on the circumstances
C. The circumstances may vary from audit to audit, but the evidence
accumulated remains the same
D. The general audit objectives may vary from audit to audit, but the
circumstances remain the same

45. Auditing standards require the auditor to accumulate sufficient competent evidence
to support the opinion issued. Because of the nature of audit evidence, it is

A. Unlikely that the auditor will be completely convinced that the opinion is
correct
B. Likely that the auditor will be completely convinced that the opinion is correct
C. Unlikely that the auditor will arrive at a conclusion
D. Likely that the auditor would change his/her mind about the opinion if he/she
takes the time to gather additional evidence

46. Which of the following ultimately determines the specific audit procedures necessary
to provide an independent auditor with a reasonable basis for the expression of an
opinioin?

A. The audit program


B. The auditor‘s judgment
C. Philippines Standards on Auditing
D. The auditor‘s working papers

47. In the final analysis, the amount and kinds of evidential matter that are required to
support the auditor‘s opinion should be determined by

A. The audit committee


B. Auditor‘s judgment
C. Professional standards
D. Standards of auditing

48. To adequately plan the extent of the audit evidence to gather, the generally
accepted auditing standards require the auditor to gain an understanding of

A. The internal control structure


B. Client‘s organization charts
C. Client‘s procedural manuals
D. All of these

49. When unable to obtain sufficient competent evidential matter to determine whether
certain client management‘s acts are non-compliance to laws and regulations, the
auditor would most likely issue

A. An unqualified opinion with a separate explanatory paragraph


B. Either a qualified opinion or an adverse opinion
C. Either a disclaimer of opinion or a qualified opinion
D. Either an adverse opinion or a disclaimer of opinion

50. An audit evidence is generally considered relevant when it is

A. Derived through valid statistical sampling


B. Objective and unbiased
C. Factual, adequate, and convincing
D. Consistent with the audit objectives

51. Two overriding considerations that affect an auditor‘s judgment in accumulating


evidence are:
1. Sufficient competent evidence must be accumulated to meet the auditor‘s
professional responsibility
2. Cost of accumulating evidence should be minimized

In evaluating these conditions,

A. The first is more important than the second


B. The second is more important than the first
C. They are equally important
D. It is impossible to prioritize one

52. Most of the independent auditor‘s work in formulating an opinion on the financial
statements consists of

A. Studying and evaluating internal control


B. Obtaining and examining evidential matter
C. Examining cash transactions
D. Comparing recorded accountability with assets

53. There are four subcategories of decisions that the auditors must make in
accumulating audit evidence. Which of the following is not one of those
subcategories?

A. Audit procedures to be used


B. Reasons for deciding not to test controls
C. Sample size
D. Timing of the audit procedures

54. Evidential matter supporting the financial statements consists of the underlying
accounting data and all corroborating information available to the auditor. Which of
the following is an example of corroborating information?

A. Minutes of meetings of the board of directors


B. General and subsidiary ledgers
C. Accounting manuals
D. Worksheets supporting cost allocations

55. Which of the following is not one of the major phases in an audit process?

A. Plan and design an audit approach


B. Test controls and transactions
C. Inform client of any adjustments or corrections to be made in the financial
statements
D. Complete the audit and issue the report
56. Evidential matter is generally considered sufficient when

A. It is competent
B. There is enough of it to afford a reasonable basis for an opinion on the
financial statements
C. It has the qualities of being relevant, objective, and free from known bias
D. It has been obtained through random selection

57. In making decisions about evidence for a given audit, the auditor‘s goal is to obtain a
sufficient amount of timely, reliable evidence that is relevant to the information being
verified, and to do so

A. No matter what the cost involved in obtaining such evidence


B. Only if the cost is reasonable
C. At the lowest possible total cost
D. At any cost because the costs are billed to the client

58. Which of the following is not a distinguishing feature of risk-based auditing?

A. Identifying areas posing the highest risk of financial statement errors


B. Analysis of internal control
C. Collecting and evaluating evidence
D. Concentrating audit resources in those areas presenting the highest risk of
financial statement errors

59. The competence of evidence available to an auditor is least likely affected by

A. The relevance of such evidence to the financial statement assertion being


investigated
B. The relationship of the source of such an evidence to the entity being audited
C. The timeliness of the audit evidence obtained
D. The sampling method employed by the auditor to obtain a number of samples
as evidence

60. Which of the following procedures would provide the auditor the most reliable audit
evidence?

A. Inquiries of the client‘s internal audit staff held in private


B. Inspection of prenumbered client purchase orders filed in the vouchers
payable department
C. Analytical procedures performed by the auditor on the entity‘s trial balance
D. Inspection of bank statements obtained directly from the client‘s financial
institution

61. The most reliable forms of documentary evidence are those documents that are
A. Prenumbered
B. Easily documented
C. Internally generated
D. Authorized by a responsible official

62. You have been assigned to audit the maintenance department of an organization.
Which of the following is likely to produce the least reliable audit evidence?

A. Notes on discussions with mechanics in the maintenance operation


B. A schedule comparing actual maintenance expenses with budgeted expenses
and thoe of the prior period and disclosing important differences
C. A narrative covering review of user reports on maintenance service
D. An analysis of changes in certain maintenance department ratios

63. Before applying substantive tests to the details of asset accounts at an interim date,
an auditor should assess

A. Control risk at below the maximum level


B. Inherent risk at the maximum level
C. The difficulty in controlling the incremental audit risk
D. Materiality for the accounts tested as insignificant

64. Before applying principal substantive tests to the details of accounts at an interim
date, an auditor should

A. Assess control risk as below the maximum for the assertions embodied in the
accounts selected for interim testing
B. Determine that the accounts selected for interim testing are not material to the
financial statements taken as a whole
C. Consider whether the amounts of the year-end balances selected for interim
testing are reasonably predictable
D. Obtain written representations from management that all financial records and
related data will be made available

65. If an auditor conducts an audit of financial statements in accordance with generally


accepted auditing standards, which of the following will the auditor most likely
detect?

A. Misposting of recorded transactions


B. Forgery
C. Unrecorded transactions
D. Collusive fund

66. Which of the following best explains the difference between audit objectives and
audit procedures?
A. Audit procedures establish broad general goals; audit objectives specify the
detailed work to be performed
B. Audit objectives are tailor-made for each assignment; audit procedures are
generic in application
C. Audit objectives define specific desired accomplishments; audit procedures
provide the means of achieving audit objectives
D. Audit procedures and audit objectives are essentially the same

67. In gathering audit evidence in the performance of substantive tests, the auditor

A. Should use the test month approach


B. Relies on persuasive rather than convincing evidence in the majority of cases
C. Would consider the client‘s documentary evidence more competent than
evidence gathered from observation and physical inspection
D. Would express an adverse opinion if he has substantial doubt as to any
significant assertion

68. The auditor will not ordinarily initiate discussion with the audit committee concerning
the

A. Extent to which the work of internal auditors will affect the scope of the
examination
B. Extent to which a change in the company‘s organization will influence the
scope of the examination
C. Details of potential problems that the auditor believes might cause a qualified
opinion
D. Details of the procedures that the auditor intends to apply

69. With respect to the auditor‘s planning of a year-end examination, which of the
following statements is always true?

A. An engagement should not be accepted after the fiscal year ends


B. An inventory count must be observed at the balance sheet date
C. The client‘s audit committee should not be told of the specific audit
procedures that will be performed
D. It is an acceptable practice to carry out substantial parts of the examination at
interim dates

70. An auditor test counts a batch of inventory. This is an example of what kind of
evidence?

A. Analytical
B. Documentary
C. Physical
D. Testimony
71. The audit program is basically a list of

A. Detailed audit procedures


B. Account balances and their related assertions
C. Audit procedures to be performed
D. Audit controls

72. Each audit program should have a column for all of the following except:

A. Audit procedures to be performed


B. The initials of the auditor who performs each procedure
C. The date that the performance of the procedure is performed and completed
D. The test of controls related to each procedure

73. Which of the following is not an example of confirmation as an evidence?

A. Requesting the client‘s outside legal counsel to evaluate the possible


outcome of pending litigation
B. Questioning the client‘s employees about existing internal control policies and
procedures
C. Requesting the client‘s customers to verify year-end accounts receivable
balances
D. Requesting payees to respond in writing to the terms contained in notes
payable appearing in the client‘s ledger

74. Accounting for the numeric sequence in the issuance of the sales invoices meets
primarily the

A. Completeness assertion
B. Valuation or allocation assertion
C. Occurrence
D. Presentation or disclosure assertion

75. Which of the following factors affects the competence of evidence obtained by an
auditor?

A. The independence of the information source


B. The competence of the information source
C. The timeliness of the information
D. All of these factors affect the competence of evidence

76. Which one of the following is the least persuasive type of audit evidence?

A. Documents mailed by outsiders to the auditor


B. Correspondence between the auditor and vendors
C. Copies of sales invoices inspected by the auditor
D. Computations made by the auditor

77. Audit evidence takes different forms and varies in persuasiveness. Which of the
following is the least persuasive type of evidence?

A. Vendor‘s invoice
B. Computations made by the auditor
C. Bank statement obtained from the client
D. Canceled checks

78. Which one of the following statements is true?

A. Evidence must pertain to the objective that the auditor is testing before it can
be persuasive
B. Relevance can be considered only in terms of specific audit objectives
C. Evidence may be relevant to one objective but not to other objective
D. All the responses are true

79. A term which is synonymous with competence is

A. Relevance
B. Reliability of evidence
C. Sufficient
D. Any of the given choices

80. Which of the following statements about the competence of evidence is not correct?

A. To be competent, an evidence must be both valid and relevant


B. Competence can be improved by selecting a larger sample size
C. Competence can be improved by selecting audit procedures that contain a
higher quality of the characteristics sought
D. Competence cannot be improved by selecting different population items to
include in the sample size

81. Which one of the following forms of evidence would be least reliable?

A. Monthly bank statement


B. Positive confirmation of customer‘s balance
C. A letter from the client‘s attorney stating that there are no known lawsuits
pending against the client
D. Client‘s file copy of a purchase requisition

82. Which of the following forms of evidence would be most reliable?

A. An insurance policy in the client‘s file


B. The file copy of a purchase requisition
C. The file copy of a receiving room report
D. The file copy of sales invoices

83. Evidence obtained directly by the auditor is more competent than information
obtained indirectly. Which of the following is not an example of the auditor‘s direct
knowledge about an evidence?

A. Physical examination
B. Observation
C. Computation
D. Inquiry

84. When the auditor is gathering evidence, he will conclude that if the source of
information is independent, the evidence will

A. Reliable
B. Not be reliable
C. Be reliable if the provider has no reason to be biased
D. Not be reliable unless the provider is qualified to do so

85. Evidence obtained directly by the auditor will not be reliable if

A. It is provided by the client‘s attorney


B. The auditor lacks the qualifications to evaluate the evidence
C. It is impossible for the auditor to obtain additional corroborating evidence
D. The client denies its veracity

86. Evidence is usually more persuasive for balance sheet accounts when it is obtained

A. From various times throughout the client‘s year


B. Only from transactions occurring on the balance sheet date
C. As close to the balance sheet date as possible
D. From the time period when transactions in that account were most numerous
furing the fiscal period

87. For income statement accounts, evidence is more persuasive if there is a sample
from

A. The entire period under audit


B. The period closest to the end of the fiscal period
C. At least three months of the fiscal year
D. December, since this would include large holiday sales

88. Which of the following statements is not true?


A. A large sample of highly competent evidence is not persuasive unless it is
relevant to the objective being tested
B. A large sample of evidence that is neither competent nor timely is not
persuasive
C. A small sample of only one or two pieces of relevant competent, and timely
evidence lacks persuasiveness
D. The persuasiveness of evidence can be evaluated after considering its
competence and its sufficiency

89. Generally, what source of evidence would most persuasively support audit
comclusions?

A. External
B. Inquiry
C. Oral
D. Informal

90. Observation, though considered a reliable audit procedure, has limited usefulness.
However, it is used in a number of different audit situations. Which of the following
statements is true regarding observation as an audit technique

A. It is the most effective audit methodology to use in filling out internal control
questionnaires
B. It is the most persuasive technique to learn how transactions are really
processed during the period under audit
C. It is rarely sufficient to satisfy any audit assertion other than existence
D. It is the most persuasive audit technique for determining if fraud has really
occurred

91. Which of the following would be the most relevant form of evidence to evaluate the
reasonableness of account balances?

A. Analytical
B. Documentary
C. Physical
D. Representation

92. When an auditor calculates the gross margin as a percent of sales and compares it
with previous periods, this type of evidence is called

A. Physical examination
B. Computation
C. Observation
D. Inquiry
93. Objective evidence is more reliable than evidence that requires considerable
judgment to determine whether it is correct. Which of the following is not an example
of an objective evidence?

A. Confirmation of accounts receivable


B. Confirmation of bank balances
C. Confirmation by client‘s attorney of the likely outcome of outstanding lawsuits
against the client
D. Adding a list of accounts payable to determine the total reconciles with the
general ledger balance

94. Which of the following is an example of subjective evidence?

A. A positive confirmation of an account receivable


B. A bank confirmation
C. Inquiries of the credit manager about the collectability of noncurrent accounts
receivable
D. The physical count of securities and cash

95. Physical examination refers to the inspection or count by the auditor of assets such
as
A. Cash or inventory only
B. Cash, inventory, canceled checks, and sales documents
C. Cash, inventory, securities, notes receivable, and tangible fixed assets
D. Cash, inventory, canceled checks, and tangible fixed assets

96. The distinction between physical examination of assets and examination of


documents is dependent on the item being examined. If the object being examined
has no inherent value, the evidence is called

A. Physical examination
B. Documentation
C. Confirmation
D. Garbage

97. Confirmations are a highly regarded and often used type of evidence because they

A. Are inexpensive
B. Cause no inconvenience for auditor or third party
C. Come from independent sources
D. All of the given choices

98. Three common types of confirmations used by auditors are:

1. Negative confirmations
2. Positive confirmations with a request for information
3. Positive confirmations with the information included

If they were placed in the order of their reliability, from highest to lowest, the
sequence would be

A. 1, 2, 3
B. 3, 2, 1
C. 2, 3, 1
D. 3, 1, 2

99. Whenever practicable and reasonable, the CPA must confirm a sample of

A. Accounts receivable
B. Accounts payable
C. Both accounts receivable and accounts payable
D. Client‘s bank accounts

100.Confirmations lose their value if the client

A. Controls the preparation of the confirmation


B. Does the mailing of the confirmation
C. Receives the responses and turns them over to the auditor
D. Does any of the given choices

101.When the auditor examines the client‘s documents and records to substantiate the
information on the financial statements, it is commonly referred to as

A. Inquiry
B. Confirmation
C. Vouching
D. Physical examination

102.Documents is a form of evidence used

A. In every financial statement audit.


B. In most financial statement audit.
C. When it is both readily available and less costly than other procedures
D. Used when nothing is available that is more competent.

103.A document which the auditor receives from the client, but which is prepared by
someone outside the client‘s organization, is a(n)

A. Confirmation
B. Internal document
C. External document
D. Inquiry
104.An example of vouching would be

A. Trace from receiving reports to the acquisition journal


B. Trace from the acquisitions journal to the supporting vendor‘s invoices
C. Trace from duplicate bank deposit slips to the cash receipts journal
D. Trace from canceled checks to cash disbursements journal

105.Which of the following statements is not true? ―The evidence gathering technique of
inquiry

A. Cannot be regarded as conclusive.‖


B. Requires the gathering of corroborative evidence.‖
C. Is the auditor‘s principal method of evaluating the client‘s internal control
structure.‖
D. Does not provide evidence from an independent source.‖

106.An auditor would be least likely to use confirmations in connection with the
examination of

A. Inventories
B. Long-term debt
C. Property, plant, and equipment
D. Stockholders‘ equity

107.Which of the following is an example of internal evidence that the auditor would
obtain in an audit of accounts receivable?

A. The carrier‘s bill of lading


B. Sales invoice copies
C. A customer‘s purchase order
D. A vendor‘s month-end statement.

108.Ordinarily, what source of evidence should least affect audit conclusions?

A. External
B. Inquiry of management
C. Auditor-prepared analysis
D. Inquiry of company legal counsel

AUDIT PROCEDURES

109.A list of audit procedures that the auditors need to perform to produce evidence is
called an

A. Audit plan
B. Audit program
C. Audit standard
D. Audit budget

110.The procedures specifically outlined in an audit program are primarily designed to

A. Protect the auditor in the event of litigation


B. Detect errors or irregularities
C. Test internal control structure
D. Gather evidence

111.In the context of an audit of financial statement, substantive tests are audit
procedures that

A. May be eliminated under certain conditions


B. Are designed to discover significant subsequent events
C. May be either tests of transactions, direct tests of financial balances, or
analytical tests
D. Will increase proportionately with the auditor‘s assessment of control risk

112.When evaluating the planned level of substantive tests for each significant
assertion, the auditor will consider the evidence obtained from all of the following
except:

A. Procedures to understand the business and industry and related analytical


procedures that have been completed.
B. Evidence about the effectiveness of internal controls gained while obtaining
an understanding of internal control structure.
C. The assessment of detection risk.
D. Evidence of effectiveness of computer control procedures and related follow-
up.

113.A revision of the planned level of detection risk will be necessary whenever

A. Accounts are affected by more than one transaction class.


B. The multiple control risk assessments for the same account balance assertion
differ.
C. The final assessed control risk is not the same as the actual level.
D. The final assessed control risk does not support the planned level.

114.Tests of details of transactions primarily involve

A. Tracing and vouching


B. Confirmation with outsiders
C. Observation
D. Scanning
115.The objective of dual-purpose tests is to

A. Evaluate whether internal controls are operating effectively.


B. Detect material misstatements in the financial statements.
C. Identify unusual trends or patterns in comparative financial statements.
D. Test internal controls as well as transactions and balances using the same
test procedures.

116.To test for unsupported entries in the ledger, the direction of audit testing should be
from he

A. Ledger entries
B. Journal entries
C. Externally generated documents
D. Original source documents

117.The least costly form of testing is usually

A. Test of controls
B. Tests of details of balances
C. Tests of details of transactions
D. Analytical procedures

118.Tracing from source documents to journals most directly addresses which financial
statement assertion?

A. Valuation
B. Completeness
C. Existence
D. Rights

119.An auditor is examining the detailed debut and credit entries in an account. The
auditor is most likely performing

A. Analytical procedures
B. Test of details of balances
C. Test of details of transactions
D. Test of controls

120.Choices about audit evidence are influenced by all of the following except:

A. The auditor‘s understanding of the business and industry


B. Assessment of inherent and control risk
C. Comparisons of the auditor‘s expectation of the financial statements with the
client‘s books and records
D. Decisions about immaterial risk factors

121.The auditor is performing substantive tests several months before the end of the
year. This most likely means that

A. Inherent risk is set at moderate to high


B. Detection risk is set at moderate to high
C. Control risk is set at maximum
D. Detection risk is set at low to very low

122.In testing the existence assertion for an asset, an auditor ordinarily works from the

A. Financial statements to the potentially unrecorded items


B. Potentially recorded items to the financial statements
C. Accounting records to the supporting evidence
D. Supporting evidence to the accounting records

123.WB Industries has significant information that is transmitted, processed,


maintained, and accessed electronically. The auditor has concluded that it is not
possible to reduce detection risk to an acceptable level by performing only
substantive tests for a number of financial statements assertions. The auditor‘
alternative strategy is to

A. Increase the acceptable audit risk


B. Focus audit tests on other assertions for which substantive tests prove to be
effective
C. Require management to change its information system to provide appropriate
evidence
D. Perform tests of controls to gather evidential matter to be used as basis of
assessing control risk related to those assertions

124.The decision on the part of the auditor to perform substantive tests during the
interim period will be based upon

A. Audit risk control and cost effectiveness


B. The approach followed in the past
C. The auditor‘s time convenience
D. The cooperation extended by the client

125.Choose the best illustration of objective audit evidence from the following:

A. The paid invoice file containing invoices matched with the receiving reports
and purchase orders
B. Management‘s assertion that payment procedure requires matching of invoice
with receiving report and purchase order
C. Clerical staff assurances that management policy regarding payment of
invoices—matching of invoice with receiving report and purchase order—is
always followed
D. The treasurer‘s statement of not remembering any exceptions in which an
invoice was submitted for payment that is not accomplished by a covering
receiving report and purchase order

126.Which of the following audit procedures best supports the valuation objective?

A. Performing a lower of cost or market test of the client‘s inventories


B. Reviewing a contingent liability disclosure for proper wording
C. Searching for unrecorded liabilities
D. Observing the client‘s year-end physical inventory taking

127.Which of the following is not an appropriate auditing procedure supporting the


fairness of financial statement presentation?

A. Inspecting plant asset additions for existence


B. Recalculating accrued interest on notes payable
C. Examining invoices in support of legal fees recorded during the fiscal year
D. Reviewing the client‘s production quality control program

128.Audit procedures are normally performed

A. Early in the accounting period being examined


B. Throughout the accounting period being examined, but with emphasis on the
transactions near the end
C. Within one to three months after the close of the accounting period
D. During all three of the above periods

129.The auditor would unlikely perform early substantive testing of account balances
when:

A. A number of significant deviations from control policies and procedures were


detected during tests of controls
B. Due to economic factors, the fourth quarter activity this year is expected to be
somewhat sluggish
C. The client uses a natural business year
D. The taking of the client‘s inventory is performed at an early date

130.As the acceptable level of detection risk decreases, an auditor may change the

A. Timing of substantive tests by performing them at an interim date rather than


at year-end
B. Nature of substantive tests from a less effective to a more effective procedure
C. Timing of tests of controls by performing them at several dates rather than
one at a time
D. Assessed level of inherent risk to a higher amount

131.The auditor is concerned that a client usually fails to bill customers for shipments.
An audit procedure that would gather relevant evidence would be

A. Select a sample of duplicate sales invoices and trace each to related shipping
documents
B. Trace a sample of shipping documents to related duplicate sales invoices
C. Trace a sample of Sales Journal entries to Accounts Receivable subsidiary
ledger
D. Compare the total of the Schedule of Accounts Receivable with the balance
of the Accounts Receivable account in the general ledger

132.The extent of testing normally applies

A. Exclusively to the number of items to be tested


B. To both the number of items tested and the number of tests performed.
C. Exclusively to the number of substantive tests performed
D. To both the nature of items tested and the number of tests performed

133.Which of the following, when performed by the auditor, is not a test of mechanical
accuracy?

A. Extending sales invoices


B. Adding journal and ledgers
C. Tracing amounts from journals to ledgers
D. Calculating the current ratio

134.Which of the following audit procedures would provide the least reliable evidence
about legal title to inventories?

A. Confirmation of inventories at locations outside the client‘s facilities


B. Analytical procedures comparing inventory balances to purchasing and sales
activities
C. Observation of physical inventory counts
D. Examination of paid vendors‘ invoices

135.Which of the following is not a substantive procedure?

A. Tests of details of transactions


B. Tests of purchasing functions
C. Tests of details of balances
D. Analytical reviews
136.Which of the following types of audit tests are not used to satisfy planned detection
risk?

A. Analytical procedures
B. Tests of controls
C. Substantive tests of transactions
D. Tests of details of balances

137.Substantive tests aid the auditor in all, but which of the following ways?

A. Identify monetary misstatements in an account


B. Obtain an understanding of internal control structure
C. Satisfy planned detection risk
D. All of the given choices

138.Auditors usually try to plan the audit to minimize the use of tests of details of
balances because

A. Other types of audit tests are more reliable


B. Other types of audit tests are less costly
C. Other types of audit tests require less experienced audit personnel
D. All of the given choices are correct

139.The independent auditor selects several transactions in each functional are and
traces them through the entire accounting system, paying special attention to
evidence about whether or not the control features are in operation. This audit
procedure is an example of a

A. Sequence test
B. Test of controls
C. Substantive test
D. Functional test

140.Ending account balances may be audited through the use of which of the following
types of audit procedures?

A. Tests of details of balances


B. Analytical procedures
C. Tests of controls
D. Analytical procedures and tests of details of balances

141.Which of the following represents an incorrect pairing of a type of audit test and
evidence?

A. Procedures to obtain an understanding of internal controls - Documentation


B. Analytical procedures – Ratio analysis
C. Substantive tests of transactions – Confirmation
D. Tests of details of balances – Physical examination

142.After finishing the procedures to obtain an understanding of internal control, the


auditor should perform tests of controls on

A. Key controls that have a material effect on the financial statements


B. A random sample key of controls that were reviewed
C. Key controls upon which the auditor intends to rely and plans to assess
control risk below maximum
D. Key controls which represent material weaknesses.

143.Where the auditor has assessed control risk of a particular area at a reduced level,
he will then

A. Eliminate the need to gather evidence in that area


B. Test the effectiveness of the controls in that area
C. Proceed to expand the sample sizes in that area
D. Negotiate with management to determine which controls will be tested in that
area

144.Many tests of controls involve inspecting documents. These tests are commonly
referred to as

A. Tests of transactions
B. Tests of documentations
C. Tests of balances
D. Tests of analytical procedures

145.Upon completion of all the necessary audit procedures, the auditor should combine
the information obtained to reach an overall conclusion as to whether the financial
statements are fairly presented. This is a highly subjective process that relies heavily
on

A. Philippine standards on auditing


B. Philippine financial reporting standards
C. The auditor‘s professional judgement
D. The management representation letter

146.Which of the following is not an information source for developing analytical


procedures used in the audit?

A. Relationships among financial elements


B. Relationships between financial and nonfinancial data
C. Comparison of financial data with anticipated results (e.g., budgets and
forecasts)
D. Comparison of current year financial data with projections for next year‘s
financial results

147.The objective of performing analytical procedures in planning an audit is to identify


the existence of

A. Unusual transactions and events


B. Noncompliance to laws that went undetected because of internal control
weaknesses
C. Related party transactions
D. Recorded transactions that were not properly authorized

148.A major benefit provided by computerized analytical procedures is

A. The ease of doing the calculations


B. The ease of updating the calculations
C. The ease of correcting math calculations
D. The ability to push the work down to lower levels of the audit staff to do the
analysis

149.When performing analytical procedures, an auditor observes that operating income


has declined significantly between the preceding year and the current year the
auditor should

A. Require that the decline be disclosed in the financial statements


B. Consider the possibility that the financial statements may be materially
misstated
C. Inform the management that a qualified opinion on the financial statements
will be necessary
D. Determine the management‘s responsibility for the decline and discuss the
issue with the audit committee

150.Which of the following is ordinarily designed to detect possible material


misstatements in the financial statements?

A. Test of controls
B. Computer controls
C. Analytical procedures
D. Post audit working paper review

151.Which of the following statements is not correct?

A. Analytical procedures use comparisons and relationships to determine which


account balances are in error
B. For certain immaterial accounts, analytical procedures may be the only
evidence needed.
C. In some instances, other types of evidence may be reduced when analytical
procedures indicate that an account balance appears reasonable
D. Analytical procedures are used to isolate accounts or transactions that should
be investigated more extensively

152.Which of the following statements regarding analytical procedures is not correct?

A. The definition of analytical tests places the emphasis on whether the client‘s
recorded date comply with PFRS
B. Analytical procedures are required on all audits
C. Analytical procedures are required on all review service engagements
D. For small accounts with small balances, analytical procedures alone may be
sufficient evidence

153.An aspect of analytical procedures is referred to as ―attention-directing‖ when it


highlights

A. Errors
B. Irregularities
C. Areas of improvements
D. Areas that need more detailed procedures

154.When analytical procedure reveals no unusual fluctuations, the implication is that

A. There are no material errors or irregularities


B. There are no material errors
C. There are no material irregularities
D. The possibility of a material error or irregularity is minimized

155.Which of the following is not one of the major types of analytical procedure?

A. Compare client‘s financial information with industry averages


B. Compare client‘s financial information with prior year
C. Compare client‘s actual data with budget
D. Compare client‘s data with SEC averages

156. Analytical procedures are usually

A. Less expensive to perform than tests of details


B. More expensive to perform than tests of details
C. Just as expensive as tests of details
D. None of them is necessarily correct

157.Most auditors prefer to replace test of details with analytical procedures whenever
possible because
A. The analytical procedures are more reliable
B. The test of details are more expensive
C. The analytical procedures are more persuasive
D. The tests o details are more difficult to interpret

158.An example of an analytical procedure is the comparison of

A. Financial information with similar information regarding the industry in which


the entity operates
B. Recorded amounts of major disbursements with appropriate invoices
C. Results of a statistical sample with the expected characteristic of the actual
population
D. EDP-generated data with similar data generated by a manual accounting
system

159.A schedule set up to combine similar general ledger accounts, the total of which
appears on the working trial balance as a single amount referred to as a:

A. Supporting schedule
B. Lead schedule
C. Audit note
D. Reconciling schedule

160.The auditors use analytical procedures during the course of an audit. The most
important phase of performing these procedures is the:

A. Vouching of all data supporting various ratios


B. Investigation of significant variations and unusual relationships
C. Comparison of client-computed statistics with the industry data on a quarterly
basis
D. Recalculation of industry data
MODULE 9

SAMPLING

PSA-BASED QUESTIONS

1. Select the description which illustrates sampling risk.

A. Applying audit procedures which are inappropriate for the audit objectives.
B. Failing to recognize errors or deviations in the documents examined.
C. Arriving at incorrect statistical conclusions due to computational errors.
D. Choosing a sample which has proportionately more errors than the
population.

2. Sampling risk is an inherent part of sampling that results from

A. the use of inappropriate


B. a failure to recognize exceptions
C. testing a number of items less than the entire population
D. weaknesses in client‘s internal control system

3. Sampling risk refers to the possibility that:

A. The auditor may use a less than optimal statistical method for the
circumstances, e.g. difference estimation instead of ratio estimation.
B. The auditor may fail to recognize an error that exists in the sample.
C. Even though a sample is properly chosen, it may not be representative of the
population.
D. The confidence level and/or precision established by the auditor are not
appropriate.

4. One of the causes of nonsampling error is the:

A. use of inappropriate or ineffective audit procedures


B. failure to draw a random sample
C. failure to draw a representative sample
D. use of attributes sampling instead of variables sampling

5. Statistical sampling:

A. Measures quantitatively the risk from testing only a part of the audit
population.
B. Allows the same degree of confidence as nonstatistical sampling but with
substantially less work.
C. Allows the auditor to replace some judgments with quantitative measures.
D. Measures the reliability of misstatements.
6. Statistical sampling provides a technique for

A. exactly defining materiality


B. greatly reducing the amount of substantive testing
C. eliminating judgments in testing
D. measuring the sufficiency of evidential matter

7. Statistical samples as compared to non-statistical samples permit the auditor to

A. quantify and control sampling risk


B. eliminate any type of non-sampling errors
C. obtain smaller sample sizes in all cases
D. use less complex formulas than those required to evaluate non-statistical
samples

8. Of the following statements, which one of the best differentiates statistical


sampling from nonstatistical sampling?

A. Statistical sampling is a mathematical approach to inference, whereas


nonstatistical sampling is a more subjective approach.
B. Nonstatistical sampling has greater applicability to large populations than
does statistical sampling.
C. Nonstatistical sampling is more subjective, but produces greater consistency
in the application of audit judgment.
D. Nonstatistical sampling has greater applicability to populations that lend
themselves to random selection.

9. One way to reduce sampling risk is to

A. use an appropriate method of selecting sample items from the population


B. carefully design the audit procedures to be used
C. provide proper supervision and instructions to the audit team
D. use variables sampling rather than attributes sampling

10. When the auditor goes through a population and selects items for the sample
without regard to their size, source, or other distinguishing characteristics, it is
called

A. block selection
B. haphazard selection
C. systematic selection
D. statistical selection

11. The tolerable deviation rate has a significant effect on sample size. The
relationship of tolerable deviation rate to the sample size is
A. parallel
B. inverse
C. direct
D. variable

12. The acceptable risk of assessing control risk too low in relation to the sample size
is

A. direct
B. inverse
C. parallel
D. not defined

13. The deviation rate that the auditor will permit in the population and still be willing
to reduce the assessed level of control risk is:

A. tolerable deviation rate


B. estimated population deviation rate
C. acceptable risk of assessing control risk too low
D. sample deviation rate

14. Which of the following statements is correct?

A. The expected population rate has little or no effect on the sample size.
B. As the population size doubles, the sample size should also double.
C. For a given tolerable rate, a larger sample size should be selected as the
expected population deviation rate decreases
D. The population size has little or no effect on sample size except for very small
populations.

15. Which of the following factors is generally not considered in determining the
sample size for a test of controls?

A. Population size
B. Risk of assessing control risk too low
C. Tolerable rate
D. Expected population deviation rate

16. When an auditor does a sampling for attributes, which of the following would
decrease sample size?

Risk of Assessing Tolerable rate of Expected population


control risk too low deviation deviation rate
A. Increase Decrease Increase
B. Decrease Increase Decrease
C. Increase Increase Decrease
D. Increase Increase Increase

17. If all other factors that are specified in a sampling plan remain constant, changing
the expected population deviation rate from 1 percent to 2 percent would cause
the required sample size to

A. increase
B. decrease
C. remain the same
D. become indeterminate

18. Which of the following statements concerning the sample size is true?

A. An increase in the tolerable occurrence rate, other factors remaining


unchanged, increases the sample size.
B. The higher the expected occurrence rate, other factors remaining unchanged,
the larger will be the sample size.
C. The more critical the attribute being tested, the higher will be the tolerable
occurrence rate set by the auditor, and the larger will be the sample size.
D. The lower the acceptable risk of underassessment of control risk, the smaller
will be the sample size be.

19. In attribute estimation, a 10 percent change in which of the following factors


normally will have the least effect on the size of the statistical sample?

A. Population size
B. Reliability
C. Precision interval
D. Standard deviation

20. An important distinction between a statistical sample and a non-statistical


(judgmental) sample is that with a statistical sample:

A. No judgment is required, everything is by formula.


B. A smaller sample size can be used.
C. More accurate results are obtained.
D. Population estimated with measurable reliability can be made.
QUIZZERS

CONCEPTS

1. The application of statistical sampling technique is least related to which of the


following generally accepted auditing standards?

A. The work is to be adequately planned, and assistants, if any are to be


properly supervised.
B. In all matters relating to the assignment, an independence in mental attitude
is to be maintained by the auditor or auditors.
C. A sufficient understanding of internal control is to be obtained to plan the audit
and to determine the nature, timing and extent of the tests to be performed.
D. Sufficient competent evidential matter is to be obtained through inspection,
observation, inquires, and confirmations to afford a reasonable basis for an
opinion about the financial statements under audit.

2. A sample in which the characteristics of the sample are the same as those of the
population is a (an)

A. random sample
B. variable sample
C. acceptance sample
D. representative sample

3. A bank auditor is interested in estimating the average account balance of it


depositors based on a sample. This substantive test is an example of

A. attribute sampling
B. discovery sampling
C. acceptance sampling
D. variables sampling

4. Nonsampling errors occur when audit tests do not uncover existing exceptions in
the

A. population
B. sample
C. planning stage
D. financial statements

5. Which of the following best illustrates the concept of sampling risk?

A. A randomly chosen sample may not be representative of the population as a


whole on the characteristic of interest.
B. An auditor may select audit procedures that are not appropriate to achieve
the specific objective.
C. An auditor may fail to recognize errors in the documents examined for the
chosen sample.
D. The documents related to the chosen sample may not be available for
inspection.

6. The auditors who prefer statistical to non-statistical sampling believe that the
principal advantage of statistical sampling flows from its unique ability to

A. Define the precision required to provide adequate satisfaction.


B. Provide a mathematical measurement of risk.
C. Establish conclusive audit evidence with decreased audit effort.
D. Promote a more legally defensible procedural approach.

7. The primary reason for an auditor to use statistical sampling is to

A. Obtain a smaller sample than would be required by non-statistical sampling


technique.
B. Obtain a sample more representative of the population than would be
obtained by non-statistical sampling technique.
C. Allow the auditor to quantify, and therefore control, the risk of making an
incorrect decision based on sample evidence.
D. Meet requirements of Philippine Standards on Auditing

8. One of the ways to reduce nonsampling risk is through

A. proper supervision and instructions of the client‘s employees


B. proper supervision and instructions of the audit team members
C. the use of attributes sampling rather than variable sampling
D. controls which ensure that the sample drawn is random and representative

9. Detection risk may be subdivided into the risk that analytical procedures and other
substantive procedures will fail to detect a material misstatement and the
allowable:

A. Risk of incorrect acceptance


B. Risk of incorrect rejection
C. Control risk
D. Audit risk

10. Which of the following statements is incorrect?

A. It is acceptable for auditors to use statistical sampling method.


B. It is acceptable for auditors to use non-statistical sampling method.
C. The primary benefit of statistical sampling method is the quantification of
sampling risk.
D. An advantage of using statistical sis that the cost/benefit ratio is always
positive.

11. In applying variables sampling, an auditor attempts to

A. Estimate a qualitative characteristic of interest.


B. Determine various rate of occurrence for specified attributes.
C. Discover at least one instance of a critical error.
D. Predict a monetary population value within a range of precision.

12. Which of the following statements is a valid criticism against the use of
nonstatistical sampling methods?

A. Many audit tests, such as footing of journals, must be performed outside a


statistical sampling context.
B. The cost of performing random selection or testing often exceeds the
benefits.
C. Nonstatistical sampling does not differ substantially from statistical sampling
method.
D. Conclusions may be drawn in more precise ways when using statistical
sampling method.

13. Which of the following statements is not correct regarding probabilistic and
nonprobabilistic sample selection?

A. In probabilistic selection, every population item has a known chance of being


selected.
B. Probabilistic selection is required for all statistical sampling methods.
C. It is not acceptable to make nonstatistical evaluation using probabilistic
selection.
D. Both methods are acceptable and commonly used.

14. Which one is not a sample selection method commonly associated with
nonstatistical audit sampling?

A. Directed sample selection.


B. Block sample selection.
C. Probability proportional to size sample selection.
D. Haphazard sample selection.

15. Which one is not a sample selection method commonly associated with statistical
audit sampling?

A. Simple random sample selection.


B. Systematic sample selection.
C. Block sample selection.
D. Stratified sample selection

16. The major weakness of nonstatistical sampling is that it

A. usually requires larger sample size that statistical sampling


B. does not allow sampling risk to be objectively measured
C. frequently results in samples that are not representative of the population
D. gives less accurate point estimates of parameters than statistical sampling
does

17. Which of the following statements regarding statistical sampling in auditing is


true?

A. In as much as audits are test-based, generally accepted auditing standards


require the use of statistical sampling methods whenever the auditor decided to
examine only a part of the population.
B. Although statistical sampling may be applied to test controls, it is required for
substantive testing purposes.
C. Sampling methods are used by auditors in both testing of controls and
substantive testing.
D. Statistical sampling methods are more appropriate for testing of controls
when the auditor elects to reprocess transactions, than when controls are tested
by means of document examination.

18. Which of the following is not an aspect of sampling risk?

A. Risk of assessing control risk too high.


B. Risk of not identifying a misstatement included in a sample.
C. Risk of incorrect acceptance.
D. Risk of sampling results indicating that a population is materially misstated
when it is not.

19. Which of the following statement is true?

A. The audit procedures will vary as a result of using either statistical or


nonstatistical sampling.
B. The audit procedures will be the same for either statistical or non-statistical
sampling, but they must be performed differently for each.
C. Statistical sampling requires quantitative audit procedures whereas
nonstatistical sampling requires judgmental audit procedures.
D. The same audit procedures are performed in the same manner for either
statistical or nonstatistical sampling.

20. An underlying feature of random sampling is that each


A. stratum of the accounting population be given equal representation in the
sample
B. item in the accounting population be randomly ordered
C. item in the accounting population should have an opportunity to be selected
D. item must be systematically selected using replacement

21. In which of the following cases would the auditor be most likely

The measurement of Error frequency is


Tolerable error is expected to be
A. Large Low
B. Small High
C. Large High
D. Small Low

22. Auditors who prefer statistical sampling to non-statistical sampling may do so


because statistical sampling helps the auditor

A. Measure the sufficiency of the evidential matter obtained.


B. Eliminate subjectivity in the evaluation of sampling results.
C. Reduce the level of tolerable error to a relatively low amount.
D. Minimize the failure to detect a material misstatement due to non-sampling
risk.

23. In order to quantify the risk that the sample evidence leads to erroneous
conclusions about the sampled population

A. Each item in the sampled population must have an equal chance of being
selected.
B. Each item in the sampled population must have a chance of being selected
proportional to its book value.
C. Each item in the sampled population must have an equal or known probability
of being selected.
D. The precise number of items in the population must be known.

24. In examining cash disbursements, an auditor plans to choose a sample using


systematic selection with a random start. The primary advantage of such a
systematic selection approach is that population items

A. which include errors will not be overlooked when the auditor exercises
compatible reciprocal options.
B. may occur in a systematic pattern, thus making the sample more
representative.
C. may occur more than once in a sample.
D. do not have to be prenumbered in order for the auditor to use the technique.
25. If certain forms are not consecutively numbered

A. Selection of a random sample probably is not possible.


B. Systematic sampling may be appropriate.
C. Stratified sampling should be used.
D. Random number tables cannot be used.

26. If all other factors in a sampling, plan are held constant, changing the measure of
tolerable error to a smaller value would cause the sample size to be:

A. Smaller.
B. Larger.
C. Unchanged.
D. Indeterminate.

27. Which of the following is an element of sampling risk?

A. Choosing an audit procedure that is inconsistent with the audit objective.


B. Choosing a sample size that is too small to achieve the sampling objective.
C. Failing to detect an error on a document that has been inspected by the
auditor.
D. Failing to perform audit procedures that are required by the sampling plan.

28. Other factors- remaining constant, the audit risk is increased by an increase in:

A. Materiality.
B. The effectiveness of analytical procedures.
C. The risk of incorrect rejection.
D. Detection risk.

29. In assessing sampling risk, the risk of incorrect rejection and the risk of assessing
control risk too high relate to the

A. efficiency of the audit


B. selection of the sample
C. effectiveness of the audit
D. audit quality controls

SAMPLING FOR ATTRIBUTES

30. Which type of sampling plan is most frequently used in testing control activities?

A. Attributes sampling.
B. Discovery sampling.
C. Probability-proportional-to-size sampling.
D. Classical variables sampling.
31. Tests of controls provide reasonable assurance that controls are applied as
prescribed. A sampling method that is useful when testing controls is:

A. Nonstatistical sampling
B. Discovery sampling
C. Attribute estimation sampling
D. Stratified random sampling

32. Attributes sampling would be an appropriate method to use on which one of the
following procedures in an audit program?

A. Review sales transactions for large and unreasonable amount.


B. Observe whether the duties of the accounts receivable clerk are separate
from handling cash.
C. Examine a sample of duplicate sales invoices for credit approval by the credit
manager.
D. Review the aged schedule of accounts receivable to determine if the
receivables from officers are included.

33. Which of the following is a valid statement about the assessment of control risk?

A. There is a positive relationship between detection risk and the combined level
of inherent and control risk.
B. Misstatements discovered by conducting substantive procedures may cause
the auditor to modify the previous assessment of control risk.
C. The auditor should consider the assessed levels of inherent and control risks
in determining the nature, timing, and extent of substantive procedures required to
eliminate audit risk.
D. The assessed level of inherent and control risks can be sufficiently low in
order to eliminate the auditor's need to perform substantive tests on some
assertions.

34. Discovery sampling should be used to estimate whether a population contains

A. errors of any kind


B. critical deviations
C. noncritical errors
D. no errors

35. Establishing the tolerable deviation rate requires

A. statistical frequency probability tables


B. random number tables
C. a computer program
D. auditor judgment
36. Acceptable risk of assessing control risk too low or too high is directly related to

A. nonsampling risk
B. sampling risk
C. inherent risk
D. control risk

37. Which of the following statements is true about nonstatistical sampling, in tests of
controls?

A. Nonstatistical sampling plans provide a quantitative measure of sampling risk.


B. The auditor's judgment in nonstatistical sampling is guided by classical
statistical sampling concepts.
C. The calculated nonstatistical sample should never be altered by the auditor.
D. The auditor considers the same parameters when determining a nonstatistical
sample size as when determining a statistical sample size.

38. The risk which the auditor is willing to take of accepting a control as being
effective when, in fact, it is not is the

A. tolerable deviation rate


B. acceptable risk of assessing control risk too low
C. estimated population deviation rate
D. finite correction factor

39. If the auditor is concerned that a population may contain exceptions, the
determination of a sample size sufficient to include at least one such exception is
a characteristic of

A. Discovery sampling
B. Variables sampling
C. Random sampling
D. Monetary-unit sampling

40. Statistical sampling may be applied to test controls when a client's control
procedures:

A. Depend primarily on segregation of duties.


B. Are carefully reduced to writing and are included in client accounting
manuals.
C. Leave an audit trail as evidence of compliance.
D. Enable the detection of fraud.

41. Which of the following conditions suggest an increase in sample size?


A. Internal controls in the area of payroll processing are found to be much
stronger than the auditor's initial assessment.
B. Tests of internal controls in nearly all transaction cycle subsets have
produced numerous and widespread errors.
C. The application of analytical procedures reveals a favorable sales budget
variance that is material and that remains unexplained.
D. Study of business and industry reveals a material decline in both the industry
and the client's revenue during the current year.

42. If the size of the sample to be used in a particular test of attributes has not been
determined by utilizing statistical concepts but the sample has been chosen in
accordance with random selection procedures,

A. no inferences can be drawn from the sample


B. the auditor has committed a non-sampling error
C. the auditor may or may not achieve desired allowance for sampling risk at the
desired level of confider
D. the auditor will have to evaluate the results by reference to the principles of
discover/ sampling

43. Which of the following factors does an auditor generally need to consider in
planning a particular audit sample for a control test?

A. Number of items in the population.


B. Total peso amount of the items to be sampled.
C. Estimated standard deviation of the population.
D. Tolerable error.

44. An auditor plans to test a sample of 20 checks for counter signatures as


prescribed by the client's control procedures. One of the checks in the chosen
sample of 20 cannot be found. The auditor should consider the reasons for this
limitation and

A. evaluate the results as if the sample size had been 19.


B. treat the missing check as a deviation for the purpose of evaluating the
sample.
C. treat the missing check in the same manner as the majority of the other 19
checks, i.e., countersigned or not.
D. choose another check to replace the missing check in the sample.

45. The tolerable rate of deviation for tests of controls necessary to justify a control
risk assessment depends primarily on which of the following?

A. The cause of errors


B. The extent of reliance to be placed on the procedures
C. The amount of any substantive errors
D. The limit used in audits of similar clients

46. An auditor who is examining inventory may appropriately apply sampling for
attributes in order to estimate the

A. average price of inventory items


B. percentage of slow-moving inventory items
C. peso value of inventory
D. physical quantity of inventory items

47. The tolerable rate of deviations for a test of controls is generally

A. lower than the expected rate of errors in the related accounting population
B. higher than the expected rate of errors in the related accounting records
C. identical to the expected rate of errors in the related accounting records
D. unrelated to the expected rate of errors in the related accounting records

48. To determine whether the client's internal control operated effectively in


minimizing the likelihood of failing to bill customers for inventory shipped to them,
the auditor would select a sample of transactions from the population represented
by the

A. customer order file


B. bill of lading file
C. open invoice file
D. sales invoice file

49. The precision limit for control testing necessary to justify lowering the assessed
control risk level depends primarily on which of the following?

A. The cause of the errors


B. The materiality of the attribute(s) to be tested
C. The amount of any substantive errors
D. The limit used in audits of similar clients

50. An auditor performs a test to determine whether all merchandise for which the
client was billed was received. The population for this test consists of all

A. Merchandise received.
B. Vendors' invoices.
C. Canceled checks.
D. Receiving reports.

51. Although mathematically based, statistical sampling does not replace audit
judgment. In utilizing statistical sampling techniques, the auditor must apply
judgment in all but which of the following tasks?
A. Selecting a tolerable rate of error:
B. Determining an acceptable risk of underassessing control risk.
C. Calculating the actual error rate.
D. Assessing the materiality of control weaknesses.

52. If an auditor, in planning to use statistical sampling, is concerned with the number
of a client's sales invoices that contain mathematical errors, he would most likely
utilize

A. random sampling with replacement


B. sampling for attributes
C. sampling for variables
D. stratified random sampling

53. An auditor wishes to determine if the error rate on travel reimbursement claims is
within the five-percent tolerance level set by management. What sampling plan
should the auditor use?

A. Variables sampling.
B. Attribute sampling.
C. Judgment sampling.
D. PPS sampling.

54. An auditor samples cash disbursement records for significant errors of P5, 000 or
more. Upon finding 'one such error, these records are scheduled for a complete
review. This conclusion is most likely based on a

A. Cluster sample.
B. Discovery sample.
C. Systematic sample.
D. Stratified sample.

55. In attribute estimation, which of the following must be known in order to appraise
the results of the auditor's sample?

A. estimated peso value of the population


B. standard deviation of the values in the population
C. actual occurrence rate of the attribute in the population
D. sample size

56. Given random sampling, the same sample size, and the same tolerable error for
the testing of two unequal populations, the risk of assessing control risk too low on
the smaller population is

A. the same as the risk of assessing control risk too low on the larger population
B. higher than the risk of assessing control risk too low on the larger population
C. lower than the risk of assessing control- risk too low on the larger population
D. not determinable relative to the risk of assessing control risk too low on the
larger population

57. When using statistical sampling for tests of controls, an auditor's evaluation would
include a statistical conclusion about whether:

A. Deviations in the population are within an acceptable range.


B. Monetary precision exceeds a predetermined amount.
C. The population's total monetary value is not in error by more than a
predetermined amount.
D. Population characteristics occur at least once in the population.

58. An auditor is testing credit authorization procedures by examining sales invoices


for credit approval by the credit department. The procedures will be considered to
be working adequately if 96% of all sales invoices either indicate approval or are
cash sales. The auditor selects a random sample of 100 invoices. In this situation,
which of the following outcomes illustrates underassessment?

A. The auditor finds five deviations and concludes that procedures work
inadequately. The actual population deviation rate is 2%.
B. The auditor finds no deviations and concludes that, procedures work
adequately. The true population deviation rate is 5%.
C. The auditor finds no deviations and concludes that the procedures work
adequately. The true population deviation rate is 2%.
D. The auditor finds five deviations and concludes that procedures work
inadequately. The true population deviation rate is 6%.

59. An auditor who uses statistical sampling for attributes in testing internal controls
should increase the assessed level of control risk when the

A. Sample occurrence rate is less than the expected occurrence rate used in
planning the sample.
B. Tolerable rate less the allowance for sampling risk exceeds the sample
occurrence rate.
C. Sample occurrence rate plus the allowance for sampling risk exceeds the
tolerable rate.
D. Sample occurrence rate plus the allowance for sampling risk equals the
tolerable rate.

60. If a selected random number matches the number, of a voided voucher, the
voucher ordinarily should be replaced by another one if it

A. constitutes a deviation
B. cannot be located
C. has been properly voided
D. represents an immaterial peso amount

61. Assuming the tolerable deviation rate is 5 percent; the expected population rate is
3 percent, and the allowance for sampling risk is 2 percent, what should an
auditor conclude if the test of 100 randomly selected documents reveals 4
deviations?

A. accept the sample results as a support for assessing control risk below the
maximum because the tolerable rate less the allowance, for sampling risk equals
the expected population deviation rate
B. assess control risk at the maximum because the sample deviation rate plus
the allowance for sampling risk exceeds the tolerable rate
C. assess control risk at the maximum because the tolerable rate plus the
allowance for sampling risk exceeds the expected population deviation rate
D. accept the sample results as support for assessing control risk below the
maximum because the sample deviation rate plus the allowance for sampling risk
exceeds the tolerable rate

62. What is ordinarily the preferable course of action when an auditor finds a higher
than expected deviation rate when he is sampling controls?

A. Triple the size of the sample to further analyze potential problems.


B. Project the level of deviation to the entire sample, and if material qualify the
audit opinion.
C. Continue to selection items from the population until the error rate diminishes
to a tolerable level.
D. Increase the assessed level of control risk and expand substantive testing
procedures.

63. Which one is most likely to have more serious consequences, assessing control
risk as too high or too low, and why?

A. Too high, because too much reliance will be put on weak controls, increasing
overall audit risk.
B. Too high, because of audit inefficiency and consequently audit reliability will
be inhibited.
C. Too low, because too much reliance will be put on weak controls, increasing
overall audit risk.
D. Too low, because audit efficiency and consequently audit reliability will be
inhibited.

64. Assessing control risk too high is the risk that the sample result

A. Does not support tolerable error for some or all of management's assertions.
B. Contains proportionately more deviations from prescribed control procedures
than what actually exist in the population as a whole.
C. Contains monetary misstatements that could be material to the financial
statements when aggregated with misstatements in other account balances or
classes of transactions.
D. Contains proportionately fewer deviations from prescribed control procedures
than what actually exist in the population as a whole.

65. Assessing control risk too low relates to:

A. The efficiency of the audit.


B. The effectiveness of the audit.
C. The preliminary estimate of materiality.
D. Tolerable error.

66. At times a sample may indicate that the auditor's assessed level of control risk for
a given control is reasonable when, in fact, the true compliance rate does not
justify the assessed level. This situation illustrates the risk of

A. assessing control risk too low


B. incorrect precision
C. assessing control risk too high
D. incorrect rejection

67. In the examination of the financial statements of Delta Company, the auditor
determines that in performing a test of internal control effectiveness, the rate of
error in the sample does not support the auditors preconceived notion of a
tolerable occurrence rate when, in fact, the actual error rate in the population does
meet the auditors notion of effectiveness. This situation illustrates the risk of

A. underassessment of control risk


B. overassessment of control risk
C. incorrect rejection
D. incorrect acceptance

68. Several risks are inherent in the 'evaluation of audit evidence which has been
obtained through the use of statistical sampling. Which of the following risks is an
example of the risk of underassessment of control risk?

A. Failure to properly define the population to be sampled.


B. Failure to draw a random sample from the population.
C. Failure to accept the statistical hypothesis that internal control is unreliable
when, in fact, it is.
D. Failure to accept the statistical hypothesis that a book value is not materially
misstated when the true book value is not materially misstated.
69. As a result of tests of controls, an auditor underassessed control risk and
decreased substantive testing. This underassessment occurred because the true
occurrence rate in the population was

A. Less than the risk of underassessment in the auditor's sample.


B. Less than the occurrence rate in the auditor's sample.
C. More than the risk of underassessment in the auditor's sample.
D. More than the occurrence rate in the auditor‘s sample.

SAMPLING FOR VARIABLES

70. Which of the following sampling plans would be designed to estimate a numerical
measurement of a population, such as a peso value?

A. Numerical sampling.
B: Discovery sampling.
C. Sampling for attributes.
D. Sampling for variables.

71. Which of the following statements is an advantage of classical variables


sampling?

A. If no errors are expected, classical variables sampling will result in a smaller


sample size than probability-proportional-to-size sampling.
B. A classical variables sampling plan can begin before the completed
population is available.
C. Classical variables sampling may .result in a smaller sample size than
probability-proportional-to-size sampling if there are many differences between
recorded and audited amounts.
D. Classical variables sampling does not require recorded values for individual
sampling units.

72. What is the primary objective of using stratification as a sampling • method in


auditing?

A. To increase the confidence level at which a decision will be reached from the
results of the sample selected.
B. To determine the occurrence rate for a given characteristic in the .population
being studied.
C. To decrease the effect of variance in the total population.
D. To determine the precision range of the sample selected.

73. An auditor is applying PPS sampling. In determining the sample size, which of the
following is not necessary?

A. a reliability factor for overstatement errors


B. a reliability factor for understatement errors
C. tolerable error
D. anticipated error

74. In a variable sampling plan, an auditor must generally consider each of the
following except

A. variation within the population


B. acceptable risk of incorrect acceptance
C. tolerable error
D. population

75. When sampling methods are used in a substantive test, all of the following factors
must be considered in determining an optimum sample size, except the

A. variation in the population


B. risk levels that the auditor is willing to accept
C. deviation occurrence rate that the auditor expects to exist in the sample
D. tolerable misstatement

76. Which of the following factors would influence the sample size for a substantive
test of details for a specific account?

Expected amount of Measure of tolerable


misstatements misstatement
A. No No
B. Yes Yes
C. No Yes
D. Yes No

77. An auditor initially planned to use unrestricted random sampling with replacement
when testing accounts receivable. Later, the auditor decided to use unrestricted
random sampling without replacement. As a result only of this decision, the
sample size should

A. increase
B. remain the same
C. decrease
D. be recalculated using a binomial distribution

78. The relationship between the sampling risk of incorrect acceptance and the
sample size of substantive tests is

A. inverse
B. indeterminate
C. positive
D. linear

79. The use of the difference estimation sampling technique to estimate peso
amounts is inappropriate when

A. The total book value is known and corresponds to the sum of all the individual
book values.
B. A book value for each sample item is unknown.
C. There are some observed differences between audited values and book
values.
D. The audited values are nearly proportional to the book value.

80. Which of the following situations would increase the sample size? A decrease in:

A. risk of incorrect rejection


B. estimated population standard deviation
C. expected frequency of errors
D. tolerable error

81. A number of factors influence the sample size for a substantive test of details of
an account balance. All other factors being equal, which of the following would
lead to a larger sample size?

A. Lowering the assessed level of control risk.


B. Lowering the assessed inherent risk through the use of analytical review
procedures.
C. Smaller expected frequency of errors.
D. Smaller measure of tolerable error.

82. For variables sampling purposes, changes in certain parameters affect sample
size positively while changes in others have a negative effect. In this regard,
which of the following statements is true?

A. An increase in beta risk reduces sample size.


B. Population size affects sample size inversely, i.e., as population size
increases, sample size decreases.
C. An increase in alpha risk, increases sample size.
D. As materiality (M) increases, sample size increases.

83. An auditor is primarily concerned with substantial overstatements of accounts


receivable balances and expects few, if any, errors. In an effort to concentrate on
the large peso values, the auditor would logically employ

A. ratio, estimation
B. probability proportional to size sampling
C. discovery sampling
D. mean per unit sampling

84. Ratio estimation is inappropriate when:

A. The total population book value is known and corresponds to the sum of all
population items.
B. There are some observed differences between audited end recorded book
values
C. Differences between recorded and audited values are nearly proportional to
the recorded values.
D. There are no recorded values for some items in the population.

85. A population that is physically separated into two or more groups based on
sample variation being less than that for the entire population is called a

A. systematic sample
B. judgment sample
C. simple random sample
D. stratified sample

86. In probability-proportional-to-size sampling, each invoice:

A. Has an equal probability of being selected.


B. Can be represented by no more than one peso unit.
C. Has an unknown probability of being selected.
D. Has a probability proportional to its peso value or being selected.

87. Which of the following is improper when using probability-proportional-to-size


sampling?

A. Combining negative and positive peso error items


B. Using a sample selection technique in which the same account balance could
be selected more than once
C. Selecting a random starting point and then sampling every nth peso unit.
D. Defining the sampling unit as an individual peso and not as an individual
account balance.

88. In comparison with classical variables sampling, which of the following Is an


advantage of probability-proportional-to-size (PPS) sampling?

A. PPS sampling automatically results in a stratified sample.


B. PPS sampling results in a smaller sample size if many differences are
expected between audited and recorded amounts.
C. PPS sampling is particularly appropriate when understatement errors are
expected
D. PPS sampling is less likely to overstate the allowance for sampling risk when
errors are found in the sample

89. PPS sampling is less efficient if

A. Computerized account balances are being audited.


B. Statistical inferences are to be made.
C. The audit objective is oriented to understatements.
D. The account contains a large number of transactions.

90. Which of the following courses of action would an auditor most likely follow in
planning a sample of cash disbursements if the auditor is aware of several
unusually large cash disbursements?

A. Increase the sample size to reduce the effect of the unusually large
disbursements.
B. Continue to draw new samples until all the unusually large disbursements
appear in the sample.
C. Set the tolerable rate of deviation at a lower level than originally planned.
D. Stratify the cash disbursements population so that .the unusually large
disbursements are selected.

91. PPS sampling is most appropriate when the auditor

A. anticipates understatement errors


B. anticipates overstatement errors
C. expects no errors
D. has assessed control risk at the maximum

92. The mean-per-unit estimation method calculates the estimated total audited value
of a population of accounts receivable as:

A. A summation of the total individual accounts values in the population.


B. The sample mean audited value multiplied by the number of items in the
population.
C. The estimated total audited value of the population multiplied by the number
of items in the sample.
D. The summation of the sample multiplied by the number of discrete samples in
the population.

93. What is the best description of "tolerable misstatement" for mean-per-unit


estimation?

A. The maximum misstatement that may exist without causing an account to be


materially misstated.
B. The "bounds" around the sample mean that we would expect the value to fall
within to be correct.
C. The "projected" misstatement in the population based upon the sample
chosen.
D. The upper limit (or lower limit for liabilities) of asset values for which the book
value may exceed that sample mean without being materially misstated.

94. When are the ratio estimation and difference estimation techniques most likely to
be preferable to the mean-per-unit estimation method?

A. The choice between any of the methods is irrelevant, since they all provide
similar results.
B. When differences between book and audited values are infrequent.
C. When differences between book and. audited values are frequent.
D. When differences between book and projected misstatement is estimated to
be small.

95. What is one of the main advantages of the probability-proportional-to-size


sampling technique over the classical variables approach?

A. It provides a more accurate estimation of the sample mean.


B. It provides a wider range for acceptance so that less substantive testing
needs to be done.
C. It provides a smaller range for acceptance so that more errors are discovered.
D. If often requires a smaller sample size to be selected.

96. Probability-proportional-to-size sampling will result in what type of sample items


being selected?

A. Highly representative of the population because it is wholly randomized.


B. A higher proportion of small value items then large value items because of the
sampling interval used.
C. A higher proportion of large value items then small value items because of the
sampling interval used.
D. A biased sample means that may not be representative of the population.

97. While performing a substantive test of details during an audit, the auditor
determined that the sample results supported the conclusion that the recorded
account balance was materially misstated. It was, in fact, not materially misstated.
This situation illustrates the risk of

A. alpha risk
B. assessing control risk too low
C. beta risk
D. assessing control risk too high
98. The risk of incorrect acceptance relates to the:

A. Effectiveness of the audit.


B. Efficiency of the audit.
C. Preliminary estimate of materiality.
D. Allowable risk of tolerable error.

99. Sample results support the conclusion that a recorded account balance is
materially misstated but, unknown to the auditor, the account is not misstated,
suggesting the risk of

A. incorrect rejection
B. assessing control risk too high
C. incorrect acceptance
D. assessing control risk too low

100. In conducting a substantive test of an account balance, an auditor hypothesizes


that no material error exists. The risk that sample results will support the
hypothesis when a material error actually does exist is the risk of

A. Incorrect rejection
B. Alpha error
C. Incorrect acceptance
D. Type I error
MODULE 10

TESTS OF CONTROLS AND SUBSTANTIVE PROCEDURES

REVENUE AND RECEIPTS CYCLE

Tests of Controls

1. Which of the following business functions is associated with the revenue/receipt


cycle?

A. Obligations are paid to vendors and employees.


B. Resources are distributed to outsiders in exchange for promises of future
payments.
C. Resources are used, held, or transformed.
D. Capital funds are received from investors and creditors.

2. Which of the following is not a common activity in the revenue/receipt cycle?

A. Order entry
B, Receiving
C. Inventory control
D. Cash collection

3. The cash account is involved in which cycle?

A. Revenue and collection.


B. Acquisition and. expenditure.
C. Production and conversion.
D. All of the given choices.

4. Which of the following is an appropriate audit procedure to test cancelled checks


for authorized signatures?

A. Compare the check date with the first cancellation date.


B. Determine that all checks are to be signed by individual officers who are
authorized by the board.
C. Examine a representative sample of signed checks and trace their signatures
to the specimen signature book of authorized signatories.
D. Confirm the signatures from a sample of checks directly with the bank.

5. Which of the following is not likely a source of information about the accounting
system in the revenue area?

A. Direct inquiry of customers.


B. Prior experience with the client.
C. Systems flowcharts prepared by the EDP department.
D. Financial reporting manuals.

6. Which of the following gives an indication of a potential fraudulent activity?

A. Numerous credit memoranda have been issued to the company's biggest


customer.
B. Internal auditor cannot locate several credit memoranda to support reductions
of customers' balances.
C. The year-end bank reconciliation has no outstanding checks or deposits older
than 15 days.
D. No one was absent the day the auditors handed out the paychecks.

7. Which of the following control procedures could prevent or detect errors or frauds
arising from shipments made to unauthorized parties?

A. Document policies and procedures for scheduling the shipments of goods.


B. Establish procedures for reviewing and approving the prices and sales terms
before sale.
C. Prenumber the bills of lading and assure that the related billings are made on
a periodic basis.
D. Prepare and periodically update the lists of authorized customers.

8. Which of the following control procedures would most likely assure that access to
shipping, billing, inventory control, and accounting records is restricted to
personnel authorized by management?

A. Segregate the responsibilities 'for authorization, execution, and recording, and


prenumber and control the custody of documents.
B. Establish the cash receipts function in a centralized location and require a
daily reconciliation of cash, receipts records with deposit slips.
C. Establish policy and procedures manuals, organization charts, and supporting
documentation.
D. Periodically substantiate and evaluate the recorded account balances.

9. An entity has implemented a control procedure which requires that authorized


personnel reconcile the total of individual customer accounts receivable with
control totals. This control relates to which of the following control objectives?

A. Sales, cash receipts, and related transactions should be recorded at the


correct amounts, in the proper period, and should be properly classified.
B. Recorded accounts receivable balances should reflect underlying
transactions and events.
C. Billings, collections, and related adjustments transactions should be posted
accurately to individual customer accounts.
D. Access to cash and cash-related records should be restricted to personnel
authorized by management.

10. Which of the following internal control_ procedures most likely would deter lapping
of collections from customers?

A. Independent internal verification of dates of entry in the cash receipts journal


with dates of daily cash summaries.
B. Authorization of writeoffs of uncollectible accounts by a supervisor who is
independent of credit approval.
C. Segregation of duties between receiving cash and posting collections to the
accounts receivable ledger.
D. Supervisors comparison of the daily cash summary with the sum of the cash
receipts journal entries.

11. What sequence of steps does an auditor undertake when identifying control
procedures that are potentially reliable in assessing control risk below the
maximum?

A. Consider the errors or frauds that might occur, determine control procedures,
identify control objectives, and design tests of controls.
B. Determine control procedures, design tests of controls, consider the errors or
frauds that might occur, and identify control objectives.
C. Identify control objectives, consider the errors or frauds that might occur,
determine control procedures, and design tests of controls.
D. Design tests of controls, determine control procedures, consider the errors or
frauds that might occur, and identify control objectives.

12. Assuming cash receipts from credit sales have been misappropriated, which of
the following is likely to conceal the misappropriation and unlikely to be detected?

A. Understating the sales journal.


B. Overstating the accounts receivable control account.
C. Overstating the accounts receivable subsidiary ledger.
D. Overstating the cash receipts journal.

13. Which of the following is most likely to provide management with incentives to
overstate earnings?

A. Projected quarterly dividends.


B. Issuance of preferred stock.
C. Unbudgeted increase in materials prices.
D. A projected stock split.

14. Under which of the following circumstances does management have some
discretion in timing the recognition of revenue?
A. The timing of revenue is not reasonably determinable and the earnings
process is not complete.
B. The amount and timing of revenue is reasonably determinable.
C. The earning process is complete or reasonably complete.
D. The transaction is at arm's length.

15. After preparing a flowchart of internal control for sales and cash receipts
transactions and evaluating the design of the system, the auditor would perform
tests of controls on all control procedures

A. That are documented in the flowchart.


B. That are considered to be deficiencies that might allow errors to enter the
accounting system.
C. That are considered to be strengths that the auditor plans to rely on in
assessing control risk.
D. That would help in preventing irregularities.

16. Which of the following would the auditor consider to be an incompatible operation
if the cashier receives remittances from the mail room?

A. The cashier posts the receipts to the accounts receivable subsidiary ledger.
B. The cashier makes the daily deposit at a local bank.
C. The cashier makes the daily deposit of cash collections.
D. The cashier endorses the checks.

17. Which of the following is not a universal rule for achieving control over cash?

A. Separate the ash-handling and record-keeping functions.


B. Decentralize the receiving of cash as much as possible.
C. Deposit each day's cash receipts by the end of the day.
D. Have bank reconciliation prepared by employees who do not handle cash.

18. On conducting an audit in which point in an ordinary sales transaction of a


wholesaling business is a lack of specific authorization of least concern to the
auditor?

A. Granting of credit.
B. Shipment of goods.
C. Determination of discounts.
D. Selling of goods for cash.

19. A company has computerized sales and cash receipts journals. The computer
programs for these journals have been properly debugged. The auditor
discovered that the total of the accounts receivable subsidiary accounts differs
materially from the accounts receivable control account. This discrepancy could
indicate

A. Lapping of receivables.
B. Credit memoranda being improperly recorded.
C. Receivables not being properly aged.
D. Statements being intercepted prior to mailing.

20. To achieve control when there is no billing department, the billing function should
be performed by the

A. accounting department
B. sales department
C. shipping department
D. credit and collection department

21. The person who opens the mail commonly prepares a remittance advice when a
customer fails to return one with the payment. Consequently, mail should be
opened by the

A. credit manager
B. receptionist
C. sales manager
D. accounts receivable clerk

22. Which of the following control procedures will likely prevent the concealment of a
cash shortage that was, perpetrated by improperly writing off a trade account
receivable?

A. Write off must be approved by a responsible officer after reviewing the credit
department's recommendations and supporting evidence.
B. Write Off must' be supported by an aging schedule showing that only
receivables that are several months overdue have been written off.
C. Write off must be approved by the cashier.
D. Write off must be authorized by field sales representatives.

23. Which of the following would unlikely improve control over an entity's cash?

A. Separating cash record keeping from the custody of cash.


B. Preparing the monthly bank reconciliation.
C. Processing of checks in batches, rather than intermittently.
D. Separating cash receipts from cash disbursements.

24. Which of the following would best protect a company that wishes to prevent
lapping?
A. Segregating duties so that accounting staff has no access to an incoming
mail.
B. Segregating duties so that no employee has access both to checks from
customers and money from daily cash receipts.
C. Arrange that customers send payments directly to the company's bank.
D. Requesting that customers checks be made payable to the company and be
addressed to the treasurer.

25. Which of the following is the greatest drawback of using subsequent collections
that are evidenced only by a deposit slip as an alternative procedure when
responses to positive accounts receivable confirmations are not received?

A. Checking of subsequent collections can never be used as an alternative


auditing procedure.
B. A deposit slip is not received directly by the auditor.
C. A customer may not have made a payment on a timely basis.
D. By examining a deposit slip only, the auditor does not know whether the
payment is for the receivable at the balance sheet date or a subsequent
transaction.

26. In considering internal control within the revenue/receipt cycle, what is the
purpose of a transaction walk through?

A. To gain an assurance that employees are performing assigned functions


accurately.
B. To confirm the results of the auditor's understanding of the internal control
structure.
C. To select documents, for detailed tests of controls.
D. To verify the results of the auditor's sampling plan.

27. Which of these assignments of duties would least likely lead to an embezzlement
or theft?

A. Inventory warehouse manager has responsibility for making the physical


inventory observation and reconciling the discrepancies to the perpetual inventory
records.
B. The cashier prepares the bank deposit, endorses the checks with a company
stamp, takes the cash and checks to the bank for deposit, and reconciles the bank
statement.
C. Accounts receivable clerk opens customer payments so he could make
entries in the customers' accounts receivable subsidiary accounts.
D. Financial vice president receives the checks payable to suppliers and the
supporting invoices, signs the checks, and mails them to the payees.

28. Standard control procedures over customer remittances received through the mail
include the policy that requires the mailroom personnel to
A. Forward the remittances, unopened, directly to the cashier.
B. Open the mails, restrictively endorses the checks, and then prelist each
remittance in triplicate copies.
C. Forward the remittances, unopened, directly to the accounts receivable clerk.
D. Open the mails, restrictively endorses the checks, then, forward the
remittances directly to the accounts receivable clerk.

29. After making the deposit, the daily cash summaries and the validated deposit slips
should be forwarded by the cashier directly to the:

A. Treasurer.
B. Accounts receivable clerk.
C. General accounting.
D. Internal auditor.

30. The accounting and the cash receipts functions should be handled by which
department(s)?

A. Both functions should be under the control of the company Treasurer


B. Both functions should be under the control of the company Controller
C. The Controller should have control of accounting functions and the Treasurer
should have control of cash receipt functions.
D. The Treasurer should have control of accounting, functions and the Controller
should have control of cash receipt functions.

31. When auditing cash, the auditor should mostly be concerned with:

A. Detective risk.
B. Inherent risk.
C. Adjunct risk.
D. Nonsampling risk.

32. Which of the following is most likely to indicate a fraud?

A. Several overpayments are made for goods received from a supplier.


B. The yearend cash balance does not include cash in transit to the company at
year-end.
C. A check received after year end is inadvertently recorded us if received
before year-end.
D. A documented loan to an officer of the company.

33. Jolas embezzled P50,000 from the company's account in Bank A. At year-end he
concealed the shortage by drawing a check in Buy and deposited it in Bank A. He
has not recorded the transaction the books. This is an example of:
A. Lapping.
B. Kiting.
C. An effective cash management.
D. Related-party transactions.

34. A client maintains two bank accounts. One of the accounts, Bank A, has an
overdraft of P10, 000. The other account, Bank B, has a positive balance of P5,
000. To conceal the overdraft from the auditor, the client may decide to

A. Draw a check for at least P 10,000 on Bank A for deposit in Bank B. Record
the receipt but not the disbursement and list the receipt as a deposit in transit.
Record the disbursement at the beginning of the following year.
B. Draw a check for P 10,000 on Bank B for deposit in Bank A. Record the
disbursement but not the receipt. List the disbursement as an outstanding check,
but do not list the receipt as a deposit in transit. Record the receipt at the
beginning of the following period.
C. Draw a check for at least P10, 000 on Bank B for deposit in Bank A. Record
the receipt but not the disbursement and list the receipt as a deposit in transit.
Record the disbursement at the beginning of the following year.
D. Draw a check for at least P10, 000 on Bank A for deposit in Bank B. Record
the disbursement but not the receipt and list the disbursement as an outstanding
check. Record the receipt at the beginning of the following year.

35. A company has a policy of rotating employees' assigned duties. This policy is
most important for employees who:

A. are not bonded


B. maintain the detailed accounting records
C. handle cash receipts
D. have access to the general ledger

36. Alpha Company uses its sales invoices for posting to perpetual inventory records.
Inadequate internal control procedures over the invoicing function allow goods to
be shipped that are not yet invoiced. The inadequate controls could cause an

A. understatement of revenues, receivables, and inventory


B. overstatement of revenues and receivables, and an understatement of
inventory
C. understatement of revenues and receivables, and an overstatement of
inventory
D. overstatement of revenues, receivables, and inventory

37. Which of the following control procedures may prevent the failure to bill customers
for some shipments?
A. Each shipment should be supported by a pre-numbered sales invoice that is
accounted for.
B. Each sales order should be approved by authorized personnel.
C. Sales journal entries should be reconciled to daily sales summaries.
D. Each sales invoice should be supported by a shipping document.

38. The most effective control for ensuring that customers are billed only for goods
shipped is to

A. require that carriers sign properly completed bills of lading


B. implement a policy that prevents the mailing of sales invoices to customers in
the absence of a properly approved shipping order and a bill of lading signed by
the carrier.
C. require that all shipments be approved by an accounting personnel
D. prevent goods from leaving the warehouse without being accompanied by a
signed bill of lading and a properly approved shipping order

39. A company policy should clearly indicate that defective merchandise returned by
customers is to be delivered to the

A. Sales clerk.
B. Receiving clerk.
C. Inventory control clerk.
D. Accounts receivable clerk.

40. During the review of a small business client's internal control system, the auditor
discovered that the accounts receivable clerk approves credit memos and has
access to cash. Which of the following controls would be most effective in
offsetting this weakness?

A. The owner reviews errors in billings to customers and postings to the


subsidiary ledgers.
B. The controller receives the monthly bank statement directly and reconciles
the checking accounts.
C. The owner reviews credit memos after they are recorded.
D. The controller reconciles the total 'of the detailed accounts receivable to the
amount shown in the ledger.

41. The most effective control to prevent unbilled and unrecorded shipments of
finished goods is to

A. Require all outgoing shipments to be accompanied by a prenumbered


shipping' order and bill of lading (signed by the carrier). Forward a copy of these
documents to accounting, to be placed in an open file awaiting receipt of the
customer invoice copy.
B. Forward a copy of the shipping order and bill of lading to billing.
C. Implement a policy that prevents sales invoices from being mailed to
customers in the absence of a properly approved shipping order and bill of lading
signed by the carrier.
D. Forward a copy of the signed bill of lading to the store‘s manager.

42. Controls over approving credit relate to the:

A. completeness assertion
B. rights and obligation
C. valuation or allocation
D. occurrence

43. To determine whether internal control operates effectively to minimize errors of


failure to bill a customer for a shipment, the auditor would select a sample of
transactions from the population represented by the

A. customer order file


B. shipping records file
C. subsidiary customer accounts ledger
D. sales invoice

44. To verify that all sales transactions' have been recorded, a test of transactions
should be completed on a representative sample drawn from

A. entries in the sales journal


B. the billing clerk's file of sales orders
C. a file of duplicate copies of sales invoices for which all prenumbered forms in
the series have been accounted for
D. the shipping clerk's file of duplicate copies of shipping documents

45. To gather audit evidence about the .proper credit approval of sales, auditor would
select sample of documents from the population represented by the

A. customer order file


B. bill of lading file
C. subsidiary customers' account ledger
D. sale invoice file

46. The purpose of tests of controls over shipping is to determine whether

A. billed goods have been shipped


B. shipments are billed
C. shipping department personnel are competent.
D. credit is approved before goods are shipped

47. The purpose of tests of controls over billing is to determine whether


A. billed goods have been shipped
B. shipments are billed
C. billing department personnel are competent
D. credit is approved before goods are billed

48. An effective procedure to test for unbilled shipments is to trace from the

A. sales journal to the shipping documents


B. shipping documents to the sales journal
C. sales journal to the accounts receivable ledger
D. sales journal to the general ledger sales account

49. To determine whether refunds granted to customers were properly approved, the
auditor should trace accounts receivable entries to:

A. Sales invoices.
B. Remittance advices.
C. Shipping documents.
D. Credit memos.

50. The following are four steps that an auditor undertakes in assessing control risk:

A) Determine what control procedures are used by the entity


B) Identify the system's control objectives
C) Design tests of controls
D) Consider the potential errors or irregularities that could result

In what order would an auditor perform these steps?

A. DBAC
B. BCDA
C. BDAC
D. DCAB

51. In order for the auditors to be able to recognize potential fraud, they must be
aware of the basic characteristics of fraud. Which of the following is a
characteristic of fraud?

A. Unintentional deception.
B. Taking unfair or dishonest advantage of uninformed individuals.
C. Lack of training.
D. Negligence on the part of executive management.

52. Which of the following statements about "window dressing‖ is correct?


A. Window dressing involves an intentional overstatement of receivables and
sales through decreases in the percentage of completion of long-term
construction projects.
B. An audit is not intended to provide assurance of detecting any forms of
window dressing.
C. A number of window dressing practices represent proper and appropriate
business practices.
D. Window dressing ordinarily .involves the intentional overstatement of liability
and equity accounts.

SUBSTANTIVE TESTS

53. As one of the year-end audit procedures, the auditor instructed the client's
personnel to prepare a standard bank confirmation request for a bank account
that had been closed during the year. After the client's treasurer had signed the
request, it was mailed by the assistant treasurer. What is the major flaw in this
audit procedure?

A. The confirmation request was signed by the treasurer.


B. Sending the request was meaningless because the account was closed.
C. The request was mailed by the assistant treasurer.
D. The CPA did not sign the confirmation request before it was mailed.

54. A proof of cash is normally used

A. for all engagements


B. to test the transactions process when controls over cash are weak
C. when control risk for cash is low
D. when lapping is suspected.

55. To gather evidence regarding the balance per bank in a bank reconciliation, an
auditor would examine all of the following except:

A. cutoff bank statement


B. year-end bank statement
C. bank confirmation
D. general ledger

56. An auditor requests a cutoff bank statement primarily to:

A. verify the cash balance reported on the bank confirmation


B. verify reconciling items on the client's bank reconciliation
C. detect lapping
D. detect kiting
57. An auditor gathers evidence regarding the validity of deposits in transit by
examining the

A. bank confirmation
B. cutoff bank statement
C. year-end bank statement
D. bank reconciliation

58. Which of the following audit procedures is most likely to detect a cash balance
that is restricted as to withdrawal?

A. Review the cutoff bank statement.


B. Prepare an interbank transfer schedule.
C. Make inquiries of management.
D. Compare cash balance with cash budget.

59. An auditor should trace bank transfers for the last part of the audit period and the
first part of the subsequent period to detect whether

A. the cash receipts journal was held open for a few' days after the year-end
B. cash balances were overstated because of kiting
C. the last checks recorded before the year-end were actually mailed by the
year-end
D. any unusual payments to or receipts from related parties occurred.

60. An unrecorded check issued during the last week of the year would Most likely be
discovered by the auditor when the

A. check register for the last month is reviewed


B. cutoff bank statement is reconciled
C. bank confirmation is reviewed
D. search for unrecorded liabilities is performed

61. An auditor compares information on canceled checks with ii formation contained


in the cash disbursement journal. The objective this test is to determine that

A. recorded cash disbursement transactions are properly authorized


B. proper cash purchase discounts have been recorded
C. cash disbursements are for goods and services actually received
D. no discrepancies exist between the date on the checks and the data in the
journal

62. Working papers ordinarily would not include

A. Initials of the in-charge auditor indicating a review of the staff assistants' work.
B. Cut-off bank statements received directly from the banks.
C. A memo describing the preliminary review of the internal control structure.
D. Copies of the client's inventory count sheets.

63. Which of the following auditing procedures would: the auditor not apply to a cutoff
bank statement?

A. Trace year-end outstanding checks and deposits in transit to the cutoff bank
statement.
B. Compare dates, payees and endorsements on returned checks with the cash
disbursements record.
C. Determine that the year-end deposit in transit was credited by the bank on the
first working day of the following accounting period.
D. Reconcile the bank account as of the end of the cutoff period.

64. Which of the following would be the most appropriate audit procedure to test the
processing of interbank transfers?

A. Analyze a sample of interbank transfers throughout the period including


period-end reconciliations.
B. Obtain cutoff bank statements for each bank account and reconcile them to
accounting records.
C. Send bank confirmation requests to each bank in which accounts are
maintained and reconcile the completed forms to accounting records.
D. Trace all bank deposits recorded in the accounting records near the end of
the fiscal period to their supporting documentation and to the bank statements.

65. While performing an audit of cash, an auditor begins to suspect kiting. Which of
the following is the best evidence that the auditor could obtain concerning whether
kiting is taking place?

A. Documentary evidence obtained by vouching entries in the cash account to


supporting documents.
B. Documentary evidence obtained by .vouching credits on the latest bank
statement to supporting documents.
C. Evidence obtained by preparing a schedule of interbank transfers.
D. Oral evidence obtained through discussions with controller personnel.

66. The auditor uses a cutoff bank statement to compare:

A. deposits in transit on the year-end bank reconciliation to deposits in the cash


receipts journal
B. checks dated prior to year-end to the outstanding checks listed on the year-
end bank reconciliation
C. deposits listed on the cutoff statement to disbursements in the cash
disbursements journal
D. checks dated subsequent to year-end to the outstanding checks listed on the
year-end bank reconciliation

67. An auditor who examines check disbursements discovers a missing check


number. Upon inquiry to the person responsible for disbursements and
reconciliation of the cash account, he is told that the check number is missing
because the check was voided. What is the auditor's next step?

A. Prepare a bank transfer schedule to identify the check.


B. Examine the bank confirmation to determine whether the check cleared.
C. Since the person responsible for disbursements also reconciles the account,
no additional procedures are necessary.
D. Examine the voided checks file to determine whether the check is in the file.

68. Of the following, which procedure or document is most effective for defecting
kiting?

A. A bank cut-off statement.


B. A bank statement.
C. A bank kiting statement.
D. Confirmation of bank balance.

69. Which of the following is confirmed on the standard form used for cash balances
at financial institution?

A. Factored accounts receivable.


B. Loss contingencies.
C. Loans payable.
D. Safe deposit boxes controlled by the entity.

70. When counting cash on hand, the auditor must exercise control over all cash and
other negotiable assets to prevent

A. theft
B. irregular endorsement
C. substitution
D. deposits in transit

71. Which of the following is not a primary objective of the auditor in the tests of
accounts receivable?

A. Determining the approximate realizable value.


B. Determining the adequacy of internal control.
C. Establishing, the validity of the receivables.
D. Determining the approximate time of collectability of the receivables.

72. The negative form of accounts receivable confirmation request is particularly


useful except when
A. control procedures surrounding accounts receivable are considered to be
effective
B. a large number of small balances are involved
C. the auditor has reason to believe the persons receiving the requests are likely
to give them consideration
D. individual account balances are relatively large

73. A sales cutoff test complements tests of

A. sales returns
B. cash
C. accounts receivable
D. sales allowances

74. Most part of the audit of sales and collection cycle

A. cannot be performed until the audit of cash is completed


B. can be performed independently of the audit of other cycles
C. must be performed simultaneously with the audit of the purchases and
disbursements cycle
D. must be performed first so that the audit of the other cycles can rely on the
data

75. The audit objective: "The accounts receivable balance represents gross claims on
customers and agrees with the sum of the accounts receivable subsidiary ledger"
is derived from the assertion of

A. presentation and disclosure


B. completeness
C. valuation or allocation
D. existence

76. A shipping document used in vouching will primarily meet the:

A. completeness assertion
B. valuation or allocation assertion
C. rights and obligations assertion
D. occurrence assertion

77. A shipping document used in tracing will primarily meet the:

A. completeness assertion
B. valuation or allocation assertion
C. rights and obligations assertion
D. occurrence assertion

78. An auditor is examining accounts receivable. Which one is the most competent
type of evidence in this situation?

A. Interviewing the personnel who records accounts receivable.


B. Verifying that postings to the receivable account from journals have been
made.
C. Receipt by the auditor of a positive confirmation.
D. No response received for a request for a negative confirmation.

79. Negative confirmation of accounts receivable is less effective than positive


confirmation of accounts receivable because

A. a majority of recipients usually lack the willingness to respond objectively


B. some recipients may report incorrect balances that require extensive follow-
up
C. the auditor cannot infer that all nonrespondents have verified their account
information
D. negative confirmations do riot produce evidential matter that is statistically
quantifiable

80. Although most substantive testing is performed during the final audit, M IMO
substantive tests may be done during the interim period. Which of the following
statements concerning the timing of substantive tests is true?

A. When internal control is weak, extensive substantive testing should be


performed during the interim audit.
B. Substantive testing should be performed during the interim audit only under
conditions of excellent internal control.
C. As a general rule, the auditor performs substantive tests of balances as of the
balance sheet date and tests of transactions during the interim as well as the
year-end audit.
D. If internal control is weak, the auditor should confirm accounts receivable as
of a point in time at least one month prior to the client's fiscal year-end.

81. Before applying principal substantive tests to the details of asset and liability
accounts at an interim date, the auditor should

A. assess the difficulty in controlling incremental audit risk


B. investigate significant fluctuations that have occurred in the asset and liability
accounts since the previous balance sheet date
C. select only those accounts which can effectively be sampled during year-end
audit work
D. consider the control tests that must be applied at balance sheet date to
extend the audit conclusions reached at the interim date

82. Confirming accounts receivable is required whenever:

A. they are material and it is practicable and reasonable to do so


B. they are material in amount
C. it is practicable to do so
D. it is reasonable to do so

83. In the processing of accounts receivable confirmations, the auditor would not
normally be expected to:

A. reconcile the information to the corresponding customer's account


B. personally deposit the requests in the mail
C. include his own return address envelope
D personally prepare the confirmation letter

84. The auditor should ordinarily mail confirmation requests to all banks with which •
the client has conducted any business during the year, regardless of the year-end
balance, since

A. the confirmation form, also seeks information about indebtedness to the bank
B. this procedure will detect kiting activities which would otherwise not be
detected
C. the mailing of confirmation, forms to all the client's depository banks is
required by Philippine standards on auditing
D. this procedure relieves the auditor of any responsibility with respect to non-
detection of forged checks.

85. An analysis of the aged accounts receivables is most directly related in which
substantive test objective?

A. Existence and occurrence.


B. Presentation and disclosure.
C. Rights and obligations.
D. Valuation.

86. The tests of balances to evaluate the adequacy of the allowance for uncollectible
accounts do not involve which of the following?

A. Considering the evidence concerning the collectability of past due amounts.


B. Testing the aging of the amounts shown in the aging categories on the aged
trial balance.
C. Considering the evidence concerning the collectability of current amounts.
D. Assessing the reasonableness of the percentages used to compute the
allowance component required for each aging category and the adequacy of the
overall allowance.

87. When scheduling audit work, the auditors are most likely to confirm accounts
receivable balances at an interim date if:

A. negative confirmations are being used


B. internal control is weak
C. internal control is strong
D. there is a simultaneous examination of cash and accounts payable

88. Which of the following is the best argument against the use of negative accounts
receivable confirmations?

A. The cost-per response is excessively high.


B. There is no way of knowing if the intended recipients actually receive them,
C. The recipients are likely to feel that in reality the confirmation is a subtle
request for payment.
D. The inference drawn from receiving no reply may not be correct.

89. Which of the following procedures least likely helps the auditors to assess the
adequacy of management's accounting estimate of the allowance for doubtful
accounts?

A. Investigate confirmation exceptions for any indication of amounts in dispute.


B. Review the accounts which have been written off as uncollectible prior to
year-end.
C. Investigate credit ratings for large accounts receivable.
D. Discuss with the credit manager the current status of doubtful accounts.

90. Which of the following is a proper alternative audit procedure for no responses to
positive accounts receivable confirmation requests?

A. Examination of subsequent cash receipts in payment of the receivable.


B. Mailing of negative confirmation requests to nonrespondents.
C. Expansion of the sample by the number of nonrespondents.
D. Reduction of accounts receivable by the amount of the no responses.

91. Which of the following might be detected by an auditor's review of the client's
sales cut-off?

A. Excessive goods returned for credit.


B. Unrecorded sales discounts.
C. Lapping of year end accounts receivable.
D. Inflated sales for the year.
92. During the process of confirming receivables as of December 31, 2009, a positive
confirmation was returned indicating that the "balance owed as of December 31
was paid by a customer on January 9, 2010." The auditor would most likely

A. determine whether there were any changes in- the account between January
1 and January 9, 2010
B. determine whether a customary trade discount was taken by the customer
C. reconfirm the zero balance as of January 10, 2010
D. verify that the amount was received

93. Which of the following analytical audit findings would most likely indicate a
possible problem?

A. A material decrease in the receivables turnover.


B. A material increase in inventory turnover.
C. A material decrease in days' sales outstanding.
D. A material increase in the acid test ratio.

94. When the objective of the auditor is to evaluate the appropriateness adjustments
to sales, the best available evidence would normally be

A. oral evidence obtained by discussing adjustment-related procedures with


controller personnel
B. analytical evidence obtained by comparing sales adjustments to gross sales
for a period of time
C. physical evidence obtained by inspection of goods returned for credit
D. documentary evidence obtained by inspecting documents supporting entries
to adjustment accounts

95 Two types of accounts receivable confirmation requests are used in practice -


positive and negative. Negative confirmations may be

A. when internal control over sales and accounts receivable is weak


B. only when the auditor has assessed inherent risk and control risk as low, the
auditor believes that the recipients will review the request, and a large number of
small balances are involved
C. only when internal control over sales and accounts receivable is strong
D. only when the auditor has assessed inherent risk and control risk as low, the
auditor believes that the recipients will review the request, and a small number of
large balances are involved

96. In which type of evidential matter would an auditor primarily rely upon when
evaluating the collectability of accounts receivable?

A. Positive confirmation.
B. Negative confirmation.
C. Aged accounts receivable listing.
D. Management's representations.

97. A client who wishes to inflate earnings decides to hold the sales record open
beyond year-end and record 2011 sales in 2010. Although the invoices are dated
as of year-end, the shipments were made in the following period. Moreover, the
goods were included in the ending inventory of the period under audit. Which of
the following auditing procedures would not assist in detecting this form of
fraudulent financial reporting?

A. The auditor confirms accounts receivable on a positive basis as of year-end.


B. The auditor examines shipping documents relating to sales recorded during
the last few days of the year.
C. The auditor examines shipping documents relating to sales recorded during
the first few days of the year following the period under audit.
D. The auditor applies analytical procedures that compare gross profit rates and
sales volume by month for the current and preceding years.

98. Which of the following substantive field work procedures provides the best
evidence about the completeness of recorded revenues?

A. Reconciling the sales journal to the general ledger control account.


B. Vouching charges made to the accounts receivable subsidiary ledger to
supporting shipping records.
C. Vouching shipping records to the customer order files.
D. Reconciling shipping records to recorded sales.

99. Which source document should an auditor use to verify the correct sales date for
an item sold FOB shipping point?

A. Carrier's bill of lading.


B. Customer's payment document.
C. Customer's purchase order.
D. Sales invoice.

100. Which of the following procedures would an auditor most likely rely on to verify
management's assertion of completeness?

A. Review standard bank confirmations for indications of kiting.


B. Compare a sample of shipping documents to related sales invoices.
C. Observe the client‘s distribution of payroll checks.
D. Confirm a sample of recorded receivables by direct communication with the
debtors.
101. Which account balance is most likely to be misstated if an aging of accounts
receivable is not performed?

A. Sales revenue.
B. Sales returns and allowances.
C. Accounts receivable.
D. Allowance for bad debts.

102. Confirmation is most likely to be a relevant form of evidence with regard to


assertions about accounts receivable when the auditor has concerns about the
receivables

A. valuation
B. classification
C. existence
D. completeness

103. An auditor confirms a representative number of open accounts receivable as of


December 31, 2010, and investigates respondents' exceptions and comments. By
this procedure which of the following would the auditor most likely learn of?

A. One of the cashiers has been covering a personal embezzlement by lapping.


B. One of the sales clerks has not been preparing charge slips for credit sales to
family and friends.
C. One of the computer department's staff has been removing all sales Invoices
applicable to his account from the data file.
D. The credit manager has misappropriated remittances from customers whose
accounts have been written off.

104. Which of the following audit objectives is not served by confirming customers'
accounts receivable?

A. Valuation of accounts receivable as of the balance sheet date.


B. Existence of customers represented in the accounts receivable trial balance.
C. Completeness of customers represented in the accounts receivable trial
balance.
D. Sales and accounts receivable cutoff.

105. For customers who are not responding to a first request for positive confirmation
requests, the auditor should next

A. contact the customer by telephone and attempt to confirm the balance orally
B. analyze subsequent remittances from the customer to see if the year-end
balance has been paid
C. send a second request for confirmation
D. examine underlying documentation supporting the, year-end balance
106. An aged trial balance of accounts receivable is usually used by the auditor to

A. Verify the validity of recorded receivables.


B. Ensure that all accounts are promptly credited.
C. Evaluate the results of compliance tests.
D. Evaluate the provision for bad debt expense.

107. An auditor reconciles the total of the accounts receivable subsidiary ledgers to the
general ledger control account balance, as of December 31, 2008. Which of the
following would the auditor most likely learn?

A. An October invoice was improperly computed.


B. An October check from a customer was posted in error to the account of
another customer with a similar name.
C. An opening balance in a subsidiary ledger account was improperly carried
forward from the previous accounting period.
D. An account balance is past due and should be written off.

108. An auditor's preliminary analysis of accounts receivable revealed the following


turnover rates:
2010 2009 2008
4.3 6.2 7.3

Which of the following is the most likely cause of the decrease in accounts
receivable turnover?

A. Increase in the cash discount offered.


B. Liberalization of credit policy.
C. Shortening of due-date terms.
D. Increased cash sales.

109. The auditor should use positive confirmation of accounts receivable:

A. When variables estimation sampling technique is not used.


B. For individual account balances that are immaterial in amount.
C. When internal controls over receivables are believed to be strong.
D. When the possibility of disputes in various accounts is greater than usual.

110. When there are a large number of relatively small account balances, negative
confirmation of accounts receivable is acceptable if internal control is

A. strong and the individuals receiving the confirmation requests are unlikely to
give them adequate consideration
B. weak and the individuals receiving the confirmation requests are likely to give
them adequate consideration
C. weak and the individuals receiving the confirmation requests are Unlikely to
give them adequate consideration
D. strong and the individuals receiving the confirmation requests are likely to
give them adequate consideration

111. In the confirmation of accounts receivable, the auditor would most likely

A. request confirmation of a sample of the inactive accounts


B. seek to obtain positive confirmations for at least 50% of the total amount of
the receivables
C. require confirmation of all receivables from government agencies
D. require that confirmation requests be sent a Month before year-end

112. An auditor would perform alternative procedures to substantiate the existence of


accounts receivable when

A. no reply to a positive confirmation request is received


B. no reply to a negative confirmation request is received
C. collectability of the receivables is doubtful
D. pledging of the receivables is probable

113. The auditors may use positive' and/or negative forms of confirmation requests for
accounts receivable. An auditor most likely will use

A. the positive form to confirm all balances, regardless of size


B. a combination of the two forms, with the positive form used for large balances
and the negative form for small balances
C. a combination of the two forms, with the positive form used for trade
receivables and the negative form for other receivables
D. the positive form when the control structure related to receivables are
satisfactory, and the negative form when controls are unsatisfactory

114. Which of the following statements is correct concerning the use of negative
confirmation requests?

A. Unreturned negative confirmation requests rarely provide significant explicit


evidence.
B. Negative confirmation requests are effective when defection risk is low.
C. Unreturned negative confirmation requests indicate that alternative
procedures are necessary.
D. Negative confirmation requests are effective when understatements of
account balances are suspected.

115. Which of the following might be detected by sales cutoff tests?

A. Understated receivables
B. Overstated sales
C. Kiting
D. Misappropriated inventory

116. An auditor's purpose in reviewing credit ratings of customers with delinquent


accounts receivable most likely is to obtain evidence concerning management's
assertions about

A. valuation or allocation
B. presentation and disclosure
C. existence or occurrence
D. rights and obligations

117. In auditing accounts receivable, which of the following questions would add value
to an audit?

A. Are accounts receivable pledged?


B. Are customers satisfied with your billing procedures?
C. Are any accounts receivable due from related parties?
D. Is there a separation of duties between the recording of cash receipts and the
handling of cash?

118. Which is the most persuasive evidence to support accounts receivable (not
including the allowance for doubtful accounts)?

A. Sales invoices held by the client.


B. Written confirmation of the balances, sent by the customers directly to the
auditor.
C. Shipping documents held by the client, showing the peso amount of
merchandise sent to the customers.
D. Deposit slips held by the client, showing the amount of cash received from
customers during the month after year-end.

119. Which of the following procedures could reveal unrecorded sales as of balance
sheet date?

A. Comparing shipping documents with sales records.


B. Applying gross profit percentages to inventory shipped during the period.
C. Tracing payments received after the balance sheet date to accounts
receivable records.
D. Sending accounts receivable confirmations.

120. Once an auditor has determined that accounts receivable have increased due to
slow collections in a "tight money" environment, the auditor would likely

A. propose an increase in the balance in the allowance for bad debts account
B. review the going concern ramifications
C. review the credit and collection policy
D. expand tests of collectability

121. Customers with substantial due balances have failed to reply after a second
requests had been mailed to them directly. Which of the following audit
procedures is most appropriate?

A. Examining shipping documents.


B. Reviewing cash collections during the year being audited.
C. Intensifying the study of internal controls for receivables.
D. Increasing the balance in the accounts receivable allowance account.

122. An auditor's primary concern when performing tests of controls over purchasing is
to determine whether:

A. purchases are properly authorized


B. purchases are properly recorded
C. purchase orders agree to purchase requisitions
D. purchasing personnel are performing their assigned functions properly

123. A client erroneously recorded a large purchase twice. Which of the following
control procedures would most likely detect this error in a timely and efficient
manner?

A. Footing the purchase journal.


B. Reconciling vendors' monthly statements with subsidiary payable ledger
accounts.
C. Tracing totals from the purchases journal to the ledger accounts.
D. Sending written quarterly confirmations to all vendors.

124. The accounts payable department receives a purchase order form to accomplish
all of the following except

A. comparing invoice price to purchase order price


B. ensuring that the purchase had been properly authorized
C. ensuring that the goods had been received by the party requesting the goods
D. comparing quantity ordered to quantity purchased

125. For effective internal control purposes, which of the following individuals should be
responsible for mailing signed checks?

A. Receptionist
B. Accounts payable clerk
C. Treasurer
D. Payroll clerk

126. Which of the following is a primary function of the purchasing department?

A. Authorizing the acquisition of goods.


B. Ensuring the acquisition of goods of a specified quality.
C. Verifying the propriety of goods of a specified quality.
D. Reducing expenditures for goods acquired.

127. How con an auditor determine whether the Receiving Department procedures are
applied properly?

A. Test a sample of receiving documents.


B. Observe receiving procedures on a surprise basis.
C. Review procedures manuals.
D. Interview Receiving personnel.

128. Whit of the following control procedures could prevent or detect payment for
goods that have not been received?

A. Counting goods when received.


B. Matching the purchase order, receiving report, and vendor's invoice.
C. Comparing goods received with goods requisitioned.
D. Verifying vouchers for accuracy and approval.

129. An Internal control questionnaire indicates that an approved receiving report


accompanies every check request. To test this control, an auditor should select
and examine:

A. receiving reports, to determine that the related canceled checks are dated no
earlier than the receiving reports
B. receiving reports, to determine that the related canceled checks are dated no
later than the receiving reports
C. canceled checks, to determine that the related receiving reports are dated no
earlier than the checks
D. canceled checks, to determine that the related receiving reports are dated no
later than the checks

130. Omitting quantities from copies of purchase orders sent to the receiving
department is a control procedure intended mainly to

A. ensure that goods received are physically counted by receiving department


personnel
B. identify and return damaged goods as soon as they are received
C. provide a cross-check for verifying the accuracy of perpetual inventory
records
D. prevent theft of goods by receiving department personnel

131. Which of the following is not an appropriate activity for the treasurer's department?

A. Prepares checks.
B. Cancels vouchers.
C. Forwards checks to vendors.
D. Prepares vouchers.

132. As a senior auditor, you are reviewing a write-up of internal control in cash
receipts and disbursement procedures. Which of the following deficiencies alone
should cause you the least concern?

A. Checks are signed by only one person.


B. Signed checks are distributed by the controller 6 approved payees.
C. The treasurer fails to establish bona fide names and addresses of check
payees.
D. Cash disbursements are made directly out of cash receipts.

133. Matching the suppliers' invoice, the purchase order, and the receiving report
normally should be the responsibility of the

A. receiving department
B. accounts payable department
C. purchasing department
D. treasury function

134. To avoid potential errors and irregularities, well-designed controls in the accounts
payable area should include a separation of which of the following functions?

A. Cash disbursements and vendor invoice verification.


B. Vendor invoice verification and merchandise ordering.
C. Physical handling of merchandise received and preparation of receiving
reports.
D. Check signing and cancellation of payment documentation.

135. Which of the following is a necessary control procedure for cash disbursements?

A. Checks should be signed by the controller and at least one other employee of
the company.
B. Checks should be sequentially numbered, and the numerical sequence
should be accounted for by the person preparing the bank reconciliation.
C. Checks and supporting documents should be marked "paid" immediately after
the check is returned with the bank statement.
D. Checks should be sent directly to the payee by the employee who prepares
documents that authorizes check preparation.
136. Which of the following functions is not appropriate for the accounts payable
department?

A. Comparing purchase requisitions, purchase orders, receiving reports, and


vendors' invoices.
B. Preparing purchase orders.
C. Preparing voucher and daily summary.
D. Filing voucher package by due date.

137. The accounts payable department generally should

A. cancel supporting documentation after a cash payment is mailed


B. approve the price and quantity of each purchase requisition
C. assure that the quantity ordered is omitted from the receiving department's
copy of the purchase order
D. agree the vendor's invoice with the receiving report and purchase order

138. When goods are received, the receiving clerk should match the goods with the

A. purchase order and requisition


B. vendor's invoice and the receiving report
C. vendor's shipping document and the purchase order
D. receiving report and the vendor's shipping documents

139. To improve control over merchandise purchases, a company's receiving


department should

A. accept merchandise only if an approved purchase order is on hand


B. accept and count all merchandise received from known vendors
C. rely on shipping documents to prepare receiving reports
D. be responsible for handling merchandise but not for preparing receiving
reports

140. To assure that disbursements are neither improper nor inaccurate, an entity
should require that all checks be

A. signed by an officer after supporting documentation has been examined


B. reviewed by the treasurer before mailing
C. numbered sequentially and accounted for by internal auditors
D. canceled when they are returned with the bank statement

141. The mailing of disbursement checks and remittance advices should be controlled
by the employee who

A. signs the checks last


B. approves the vouchers for payment
C. matches the receiving reports, purchase orders, and vendor invoices
D. verifies the mathematical accuracy of the vouchers and remittance advices

142. Expenditure/disbursement cycle begins with requisitions from us departments and


ends with the receipt of materials and the recognition of a liability. An auditor's
primary objective in reviewing the cycle is to:

A. obtain an understanding of the client's prescribed policies and procedures


sufficient to plan the audit
B. investigate the handling and recording of unusual acquisitions
C. consider the need to increase substantive tests of purchases and accounts
payable
D. assure that materials ordered, received, and paid for are actually on hand

143. Which of the following control questions relates to the existence and Occurrence
objective in purchasing and accounts payable?

A. Are the purchase order forms prenumbered and the numerical sequence
checked for missing documents?
B. Does the accounting department check invoices for mathematical accuracy?
C. Does the chart of accounts and accounting manual give instructions for
classifying debit entries?
D. Are receiving reports prepared for each item received?

144. The purchasing department:

A. Should obtain competitive bids from vendors.


B. Should inspect incoming goods and forward them to the receiving
department.
C. Should inspect vendor invoices and forward them to the accounting
department.
D. All of the given choices are correct.

145. To adequately provide for the segregation of duties, the purchase requisitions for
regular inventory stock should be initiated by which of the following departments?

A. Purchasing department.
B. Sales department.
C. Warehouse.
D. Shipping.

146. In a properly designed accounts payable system, a voucher is prepared after the
invoice, purchase order, requisition, and receiving report are verified. The next
step in the system is to
A. cancel the supporting documents
B. enter the check amount in the check register
C. approve the voucher for payment
D. post the voucher amount to the expense ledger

147. Which of the following is an internal control procedure that would prevent a paid
disbursement voucher from being presented for payment a second time?

A. Vouchers should be prepared by individuals who are responsible for signing


disbursement checks.
B. Disbursement vouchers should be approved by at least two responsible
management officials.
C. The date on a disbursement voucher should be within a few days of the date
the voucher is presented for payment.
D. The official signing the check should cancel the paid voucher after examining
the documentation supporting the disbursement.

148. Which of the following may be considered an appropriate means for further testing
controls over vendor payments?

A. Confirm year-end balances with vendors.


B. Search for unrecorded invoices at year-end.
C. Develop a set of hypothetical transactions designed to test existing controls
over vendor payments (e.g., introduce into the system, a voucher containing an
invoice for raw materials but lacking a purchase order and/or receiving report.
D. Constructing an internal control covering the payment processing function.

149. Which of the following would be the best procedure to determine whether
purchases were properly authorized?

A. Discuss authorization procedures with personnel in the controllers and


purchasing functions.
B. Review and evaluate a flowchart of purchasing procedures.
C. Determine whether a supported by properly sample of entries in the purchase
journal is executed purchase orders.
D. Vouch payments for selected purchases to supporting receiving reports.

150. In examining liabilities of a company, what is the auditors' primary concern?

A. Completeness.
B. Presentation.
C. Rights.
D. Valuation.

151. Purchase cutoff procedures should be designed to test that merchandise is


included in the inventory of the client when the client:
A. has paid for the merchandise
B. has physical possession of the merchandise
C. holds legal title to the merchandise
D. holds the shipping documents for the merchandise issued in the company's
name

152. When the auditors discover an overstatement of accounts payable, they would
most likely expect to find an overstatement of:

A. accrued liabilities
B. inventory
C. retained earnings
D. revenues

153. An auditor usually examines receiving reports that support entries in the:

A. voucher register and sales returns journal


B. sales journal and sales returns journal
C. voucher register and sales journal
D. check register and sales journal

154. Which of the following transactions would an auditor most likely repose an
adjustment to the financial statements?

A. Inventory is included on the balance sheet at year-end, but the check for
payment has not been paid until January 12.
B. An order for office supplies that has not been recorded because the goods
have neither been received nor paid for by year-end.
C. Purchase of P5,000 of office furniture that was ordered on December 22 with
a P1,000 deposit being made with an entry debiting "deposit on furniture" for
P1,000 and a credit to cash for P1,000. The office furniture was received on
January 5.
D. Shop supplies are included on the balance sheet at year-end, but the payable
and subsequent cash disbursements are not recorded until after year-end.

155. Only one of the following four statements which compare confirmation of accounts
payable with suppliers and confirmation of accounts receivable with customers is
true. The true statement is that

A. confirmation of accounts payable with suppliers is a more widely accepted


auditing procedure than is confirmation of accounts receivable with customers
B. it is less likely that the confirmation request sent to the supplier will show the
amount owed him or her than that the request sent to the customer will show the
amount due from him or her
C. statistical sampling techniques are more widely 'accepted in the confirmation
of accounts payable than in the confirmation of accounts receivable
D. compared to the confirmation of accounts payable, the confirmation of
accounts receivable will tend to emphasize accounts with zero balances at the
balance sheet date

156. Confirmation of accounts payable balances is

A. usually performed at interim dates rather than at year end


B. not effective in testing for unrecorded liabilities
C. particularly useful when the auditor suspects liabilities may be materially
understated
D. required by generally accepted auditing standards

157. Which of the following is true about the audit procedure of confirming accounts
payable?

A. Confirmation of payables is most appropriate when the auditor expects


understatement errors.
B. It is not productive to mail second requests.
C. The auditor is not required by current professional pronouncements to justify
his or her opinion on financial statements when payables are not confirmed.
D. Payables are usually confirmed as of an interim date.

158. Which of the following best explains why accounts payable confirmation
procedures are not always used?

A. Inclusion of representations on accounts payable in the client representation


letter eliminates the need in most situations.
B. Accounts payable generally are immaterial and may be audited through using
analytical procedures.
C. Creditors will press for payment when they receive the confirmation.
D. Confirmations are better at identifying overstatements than understatements,
and overstatements are not typically the major concern with accounts payable.

159. Which of the following audit procedures is not designed primarily to test for the
correctness of purchases and sales cutoff?

A. Observe shipping and receiving areas during physical inventory observation


and relate goods to the last receipt and shipment for the year. Determine that
these are the final entries in the purchases and sales records for the year.
B. Examine sales and purchases invoices for a few days before and after year
end. Compare with dates of receipt and shipment and with freight terms to
determine that the transactions were recorded in the proper accounting period.
C. Record last document numbers (sales invoice, voucher, check, receiving
report) for the year and relate to goods in shipping and receiving areas at year
end.
D. Trace client's unit costs to the auditor's copies of audited price lists.

160. Which of the following procedures relating to the audit of accounts payable could
the auditor delegate entirely to the client's employees?

A. testing footings in the accounts payable ledger


B. reconciling unpaid invoices to vendors' statements
C. preparing a schedule of accounts payable
D. mailing confirmations for selected account balances

161. Assume an auditor's interim consideration of internal control in the


expenditure/disbursement cycle reveals that control risk can be assessed below
the maximum and detection risk above the minimum for some assertions. Based
on the foregoing, which of the following is true about the substantive tests applied
to accounts payable?

A. The auditor is more apt to confirm payable balances.


B. The auditor is less apt to perform substantive tests at the balance sheet date.
C. The auditor is more apt to increase the extent of substantive tests.
D. The auditor is more apt to ignore the risk of incorrect acceptance when
sampling accounts payable.

162. Which of the following procedures would help an auditor test for overstatements of
accounts payable at the balance sheet date?

A. Trace entries in the cash disbursements records to items in the accounts


payable trial balance.
B. Agree items in the file of unmatched receiving reports to the accounts payable
balance.
C. Trace items in the accounts payable trial balance to documentation contained
in canceled voucher packages.
D. Coordinate cutoff tests performed for receiving and for shipping.

163. In testing cutoff for purchases and payables at December 31, an auditor is
confronted with the following four scenarios. Which of the four most likely
represents a cutoff error, requiring that the auditor propose an adjusting journal
entry?

A. Shipping terms are FOB shipping point. Goods were shipped on December
31. The purchase was recorded on December 31.
B. Shipping terms are FOB destination. Goods were shipped on December 31.
The purchase was recorded on December 31.
C. Shipping terms are FOB shipping point. Goods were shipped on January 2.
The purchase was recorded on January 4.
D. Shipping terms are FOB destination. Goods were shipped on December 31.
The purchase was recorded on January 2.

164. When an auditor selects a sample of items from the vouchers payable register for
the last month of the period being audited and traces the items to underlying
documents, the auditor, is gathering evidence primarily in support of the assertion
that

A. recorded obligations were paid


B. incurred obligations were recorded in the correct period
C. recorded obligations were valid
D. cash disbursements were recorded as incurred obligations

165. Which of the following audit procedures is the most efficient for defecting
unrecorded liabilities at the balance sheet date?

A. confirming large accounts payable balances at the balance sheet date


B. comparing cash disbursements in the subsequent period with the accounts
payable trial balance at year end
C. examining purchase orders issued for several days prior to the close of the
year
D. obtaining a letter from the client's attorney

166. Unrecorded liabilities are most likely to be found during the review of which of the
following documents?

A. Unpaid bills
B. Bills of lading
C. Shipping records
D. Unmatched sales invoices

167. Which of the following is not a step in the search for unrecorded liabilities?

A. Examine the open purchase order file.


B. Examine disbursements for the period immediately before the end of the
period.
C. Examine the unmatched receiving reports.
D. All of the given choices are steps searching unrecorded liabilities.

168. To determine whether accounts payable are complete, an auditor performs a test
to verify that all merchandise received is recorded. The population of documents
for this test consists of all:

A. Vendors' invoices
B. Receiving reports
C. Purchase orders
D. Canceled checks

169. Which of the following audit procedures is best for identifying unrecorded trade
accounts payable?

A. Reviewing cash disbursements recorded subsequent to the balance sheet


date to determine whether the related payables apply to the prior period.
B. Investigating payables recorded just prior to and just subsequent to the
balance sheet date to determine whether they are supported by receiving reports.
C. Examining unusual relationships between monthly accounts payable
balances and recorded cash payments.
D. Reconciling vendors' statements to the file of receiving reports to identify
items received just prior to the balance sheet date.

170. Which of the following procedures is least likely to alert the auditors to unrecorded
accounts payable?

A. Confirmation of accrued liabilities.


B. Reconcile recorded liabilities with monthly statements from creditors.
C. Examine disbursement transactions recorded following year-end.
D. Analytical procedures involving year-end accounts payable.

171. An audit procedure applicable to testing the year-end cutoff of liabilities is

A. Tracing recorded liabilities to supporting documents.


B. Preparing an aging schedule for accounts payable.
C. Reviewing the general journal for unusual entries recorded immediately after
year-end.
D. Examining vendor invoices received subsequent to year-end for shipment
date and terms of shipment.

172. The audit procedures applied to accrued liabilities differ from those applied to
accounts payable because

A. accrued liability balances are less material than accounts payable balances.
B. evidence supporting accrued liabilities is nonexistent, whereas evidence
supporting accounts payable is readily available.
C. accrued liabilities usually pertain to services of a continuing nature, whereas
accounts payable are the result of completed transactions.
D. accrued liabilities at year-end will become accounts payable during the
following year.

173. Accrued liabilities generally differ from accounts payable in that occurred
liabilities:
A. Accumulate over time.
B. Are usually confirmed at year-end.
C. Can be found by a review of unpaid invoices.
D. Are never included in cost of goods sold.

174. Which of the following best describes the auditors' approach to the audit of
accrued liabilities?

A. Confirmation.
B. Observation.
C. Plan a low assessed level of control risk.
D. Test computations.

175. Inventory should be recorded when

A. The company has title to it.


B. When it is received.
C. When the related revenue is recognized.
D. When it is from the vendor.

176. The overall production authorization starts with a

A. Purchase order.
B. Production plan.
C. Sales forecast.
D. Bill of materials.

177. The source of authorization for preparation of materials requisitions is

A. Bill of materials.
B. Purchase order.
C. Production plan.
D. Sales forecast.

178. Comparing material usage reports to raw material stores issue slips is a control to
help insure which assertion?

A. Existence and occurrence.


B. Completeness.
C. Rights and obligations.
D. Valuation or allocation.

179. Client's inventory instructions should include all the following except

A. names of client personnel responsible for the count.


B. instructions for recording accurate description.
C. instructions for auditors' test counts.
D. plans for controlling movement of goods.

180. Tracing a test count to the inventory compilations provides evidence for which
assertion?

A. Existence and occurrence.


B. Completeness.
C. Valuation or allocation.
D. Presentation and disclosure.

181. Which of the following least likely serves as a substitute for performing other audit
procedures?

A. The physical observation of the counting of company inventory.


B. Sending letters to debtors or creditors of the company .to confirm amount the
company owes or is owed, respectively.
C. Tracing an amount on the financial statements back through the accounting
system to the underlying supporting documentation.
D. Obtaining a representation letter signed by top management.

182. Which of the following is an internal control weakness for a company whose
inventory of supplies consists of .a large number of individual items?

A. Supplies of relatively little value are expensed when purchased.


B. The cycle basis is used for physical counts.
C. The storekeeper is responsible for maintenance of perpetual inventory
records.
D. Perpetual inventory records are maintained only for items of significant Value.

183. An auditor would most efficiently test for the misclassification of capital
acquisitions as expenses by

A. Taking a physical tour of plant facilities before starting an audit.


B. Reviewing company capital acquisition policies with purchasing personnel.
C. Tracing capital additions back to source documents.
D. Scanning repair and maintenance records and investigating large peso-value
entries.

184. Ordinarily, are auditors more concerned with an under- or overstatement of


ending inventory, and why?

A. Overstatement, because net income would be overstated also.


B. Understatement, because net income would be overstated.
C. Overstatement, because this would indicate that liabilities are likely to be
understated.
D. Understatement, because this would indicate that liabilities are likely to be
understated.

185. In conjunction with the observation of a client's physical inventory, the auditors
should:
A. plan the physical inventory
B. segregate damaged and obsolete goods
C. supervise the client's personnel
D. evaluate the adequacy of the client's counting procedures

186. An auditor has accounted for a sequence of inventory tags and is now tracing
information on a representative number of tags to the inventory summary sheets.
Which assertion does this procedure relate in most directly?

A. Completeness.
B. Existence.
C. Presentation.
D. Valuation.

187. An inventory turnover analysis most likely helps the auditors to detect:

A. inadequacies in inventory pricing


B. methods of avoiding cyclical holding cost
C. optimum automatic reorders points
D. obsolete merchandise

188. During an audit of a non-public entity, which of the following is primarily an overall
audit approach to audit plant assets and equipment by a continuing auditor?

A. Analysis and inquiry.


B. Direct tests of year-end ending balances.
C. Tests of controls.
D. Tests of transactions during the year.

189. Which of the following is the legitimate reason why a responsible party would
have difficulty estimating environmental cleanup costs?

A. Possible as yet unknown contamination.


B. Possible future legislation.
C. Possible insurance recoveries.
D. Possible preexisting contamination.

190. Which of the following statements concerning plant assets is not correct?
A. Few transactions ordinarily occur.
B. Typically there is little change in accounts from year to year.
C. Plant assets need to be tightly controlled to prevent defalcation.
D. Year-end cutoff of plant assets has no effect net income.

191. An auditor has found many new assets on the plant floor, which coincides with an
increase in the equipment subsidiary ledger. However, the auditor has noticed
that lease payments are being made to an equipment leasing company. The
auditor should primarily be concerned with which financial statement assertion?

A. Rights and obligations.


B. Relevance.
C. Clerical accuracy.
D. Completeness.

192. The accuracy of perpetual inventory records may be established in part by


comparing perpetual inventory records with:

A. purchase requisitions
B. receiving reports
C. purchase orders
D. vendor payments

193. When auditing merchandise inventory at year end, the auditor performs a
purchase cutoff test to obtain evidence that

A. goods purchased before year end are received- before the physical inventory
count
B. no goods held on consignment for customers are included in the inventory
balance
C. no goods observed during the physical count are pledged or sold
D. all goods owned at year end are included in the inventory balance

194. A client's physical count of inventories was higher than the inventory quantities
per the perpetual records. This situation could be the result of the failure to
record:

A. sales
B. sales discounts
C. purchases
D. purchase returns

195. Which of the following audit procedures is not appropriate for addressing the
assertion of valuation?

A. verifying accounts payable trial balance


B. confirming with creditors
C. testing for unrecorded liabilities
D. performing analytical procedures

196. When there are few property and, equipment transactions during the year, the
continuing auditor usually makes a

A. complete review of the related internal controls and assesses control risk
relative to them
B. complete review of the related internal controls and performs analytical review
tests to verify current year additions to property and equipment
C. preliminary review of the related internal controls and performs a thorough
examination of the balances at the beginning of the year
D. preliminary review of the related internal controls and performs extensive
tests of current year property and equipment transactions

197. In analyzing the plant assets account, why is the examination of repairs and
maintenance records important?

A. Rights.
B. Existence.
C. Valuation.
D. Presentation and disclosure.

198. In examining the miscellaneous revenue account, an auditor discovers income


from plant assets. What should be a primary audit concern?

A. That such assets have been removed from the ledger of property owned.
B. That such assets are not available for physical examination.
C. That the assets sold were fully depreciated prior to the decision to sell them.
D. That such assets have been replaced by comparable equipment.

199. Which of the following statements is not correct concerning intangible assets?

A. Auditors review the reasonableness of the client's amortization program.


B. A lack of physical substance.
C. Valuation is a primary audit concern.
D. Proper presentation as current, assets.

200. When performing an, audit of the property, plant, and equipment accounts, an
auditor should expect which of the following to be most likely to indicate a
departure from generally accepted accounting principles?

A. A gain was recognized when .a new asset was acquired at a price lower than
its listed retail price.
B. Interest has been capitalized for self-constructed equipment.
C. Assets have been acquired from affiliated corporations with the related
transactions recorded and described in the financial statements.
D. The cost of freight-in on an acquisition has been capitalized.

201. The auditors are least likely to learn of retirements of equipment through which of
the following?

A. Review of the purchase returns and allowances account.


B. Review of depreciation.
C. Analysis of the debits to the accumulated depreciation account.
D. Review of insurance policy riders.

202. A weakness in internal accounting control over the recording of retirements of


equipment may cause the auditor to

A. inspect certain items of equipment in the plant and trace those Items to the
accounting records
B. review the subsidiary ledger to ascertain whether depreciation was taken on
each item of equipment during the year
C. trace additions to the "other assets" account to 'search for equipment that is
still on hand but no longer being used.
D. select certain items of equipment from the accounting records and locate
them in the plant

203. When auditing inventories of raw materials, purchased parts, and/or Merchandise
inventory, the auditor's most effective means for evaluating the valuation assertion
is to

A. examine recent invoices from vendors, along with freight bills and compare
with client's unit costs, as adjusted for freight and discount.
B. compare purchases with prior year and with industry averages and account
for significant fluctuations.
C. trace quantifies from tags or count sheets to final inventory listings.
D. scan inventory listings for large extended amounts, and trace related
quantities to auditor's copy of the inventory tag or listing.

204. The auditor tests the quantity of materials charged to work in process by tracing
these quantities to

A. cost ledgers.
B. perpetual inventory records
C. receiving reports
D. material requisitions
205. Which of the following accounts would most likely be reviewed by the auditor to
gain reasonable' assurance that additions to the equipment account are not
understated?

A. Repairs and maintenance expense.


B. Depreciation expense.
C. Gain on disposal of equipment.
D. Accounts payable.

206. The most significant audit step in substantiating additions to the office furniture
account balance is

A. examination of vendors' invoices and receiving reports for current year's


acquisitions
B. review of transactions near the balance sheet date for proper period cutoff
C. calculation of ratio of depreciation expense to' gross office equipment cost
D. comparison to prior year's acquisitions

207. Instead of taking a physical inventory count on the balance sheet date, the client
may take physical counts prior to the year-end if internal controls are adequate
and

A. computerized records of perpetual inventory are maintained


B. inventory is slow moving
C. CBIS error reports are generated for missing pre-numbered inventory tickets
D. obsolete inventory items are segregated and excluded

208. Which of the following is not one of the independent auditor's 'objectives regarding
the audit of inventories?

A. Verifying that inventory counted is owned by the client.


B. Verifying that the client has used proper inventory pricing.
C. Ascertaining the physical quantities of inventory on hand.
D. Verifying that all inventory owned by the client is on hand at the time of the
count.

209. An auditor is verifying the existence of newly acquired fixed assets recorded in the
accounting records. Which of the following is the best evidence to help achieve
this objective?

A. Documentary support obtained by vouching entries to subsidiary records and


invoices.
B. Physical examination of a sample of newly recorded fixed assets.
C. Oral evidence obtained by discussions with operating management.
D. Documentary support obtained by reviewing titles and tax returns.
210. Which of the following procedures is most relevant to testing the completeness
assertion for prepaid insurance?

A. resting whether insurance coverage exceeds the replacement value of


insured tangible property.
B. Confirming insurance policies with carriers.
C. Reconciling, premium payments with cash disbursement records.
D. Agreeing total expense and unexpired premiums with the general ledger.

211. In a manufacturing company, which of the following audit procedures would give
the least assurance of the valuation of inventory at the audit date?

A. Testing the computation of standard overhead rates.


B. Examining paid vendors' invoices.
C. Reviewing direct labor rates.
D. Obtaining confirmation of inventories pledged under loan agreements.

212. When perpetual inventory records are maintained in quantities and in pesos, and
internal accounting control over inventory is weak, the auditor would probably

A. want the client to schedule the physical inventory count at the end of the year
B. insist that the client perform physical counts of inventory items several times
during the year
C. increase the extent of tests for unrecorded liabilities at the end of the year
D. have to disclaim an opinion on the income statement for that year

213. In auditing plant assets and accumulated depreciation for proper valuation, the
auditor should do all of the f0llowing, except:

A. recalculate depreciation expense on a test basis


B. physically inspect major plant assets additions
C. vouch major additions by reference to underlying documentation
D. vouch repairs and maintenance expense on a test basis

214. To verify the proper value of costs charged to real property records for
improvements to the property, the best source of evidence would be:

A. inspection by the auditor of real property improvements


B. a letter signed by the real property manager asserting the propriety of costs
incurred
C. original invoices supporting entries into the accounting records
D. a comparison of billed amounts to contract estimates

215. An auditor has accounted for a sequence of inventory tags and is now going to
trace information on a representative number of tags to the physical inventory
sheets. The purpose of this procedure is to obtain assurance that
A. the final inventory is valued at cost
B. all inventory represented by an inventory tag is listed on the inventory sheets
C. all inventory represented by an inventory tag is bona fide
D. inventory sheets do not include untagged inventory items.

216. An auditor analyzes repairs and maintenance accounts primarily to obtain


evidence in support of the audit assertion that all
A. non-capitalizable expenditures for repairs and maintenance have been
properly charged to expense
B. expenditures for property and equipment have not been charged h expense
C. non-capitalizable expenditures for-repairs and maintenance have been
recorded in the proper period
D. expenditures for property and equipment have been recorded in proper period

217. The auditor may conclude that depreciation charges are insufficient by noting

A. large amounts of fully depreciated assets


B. continuous trade-ins of relatively new assets
C. excessive recurring losses on assets retired
D. insured values greatly in excess of book values

218. To test the accuracy of the current year's depreciation charges, an auditor should
rely most heavily on

A. vouching of the current year's fixed asset acquisitions


B. comparison of depreciation schedule detailed with schedules supporting the
income tax return
C. tracing of totals from the depreciation schedule to properly approved journal
entries and ledger postings
D. recomputation of depreciation for a sample of plant assets

219. In forming an audit on the existence of inventory contained in a d louse, an auditor


is primarily concerned with

A. observing and testing the number of units on hand


B. determining if the value of the inventory is reasonable
C. identifying the ownership of the inventory
D. locating slow moving items contained in inventory

220. Which of the following audit procedures would provide the least reliable evidence
that the client has legal title to inventories?

A. confirmation of inventories at locations outside the client's facilities


B. analytical review of inventory balances compared to purchasing and sales
activities
C. observation of physical inventory counts
D. examination of paid vendors' invoices

221. An auditor wants to develop an audit test to evaluate the reasonableness of the
quantity of scrap material resulting from a certain production process compared to
industry standards. Which would be the most competent type of evidence
available to satisfy this objective?

A. Documentary.
B. Indirect testimony.
C. Physical.
D. Analytical.

222. Which of the following controls would be the most appropriate means to ensure
that terminated employees had been removed from the payroll?

A. Mailing checks to employees' residences.


B. Establishing direct-deposit procedures with employees' banks.
C. Reconciling payroll and time-keeping records.
D. Establishing computerized limit checks on payroll rates.

223. Which of the following departments should have the responsibility for authorizing
payroll rate changes?

A. Human Resources.
B. Payroll.
C. Treasurer.
D. Timekeeping.

224. Effective internal control over the payroll function should include which of the
following?

A. Total time recorded on time clock cards should be reconciled to job reports by
employees responsible for those specific jobs.
B. Payroll department employees should be supervised by the management of
the human resources department.
C. Payroll department employees should be responsible in maintaining the
personnel records.
D. Total time spent on jobs should be compared to the total number of hours
indicated on time clock cards.

225. The purpose of segregating the duties, of hiring personnel and distributing payroll
checks is to separate the

A. administrative controls from the internal accounting controls


B. human resources function from the controllership function
C. operational responsibility from the record keeping responsibility
D. authorization of transactions from the custody of related assets

226. An auditor would consider internal control procedures relating to a client‘s payroll
procedures to be ineffective if the payroll department supervisor is responsible for

A. hiring subordinate payroll department employees


B. having custody over unclaimed paychecks
C. updating employee earnings records
D. applying pay rates to time tickets

227. The human resources department receives an edit listing of payroll changes
processed at every payroll cycle. If they do not verify the tinges processed, this
could result in:

A. undetected errors in payroll rates for new employees


B. inaccurate social security deductions
C. labor hours charged to the wrong account in the cost reporting system
D. employees not being asked if they want to contribute to the company pension
plan

228. Estimates in the finance and investment cycle include

A. probability of a correlated hedge


B. classification of equity investments
C. actuarial assumptions for pension costs
D. all of the given choices are correct

229. Inspecting marketable securities provides primary evidence about the assertion of

A. existence
B. rights and obligations
C. valuation
D. all of the above

230. If market prices are not readily available for fair value measurements,
management should use

A. auditors' best estimates


B. historical cost
C. their own assumptions as long as there are no contrary data
D. the previous year's value

231. Goodwill impairment refers to

A. loss of trust from customers


B. the amount of amortization
C. a permanent decline in value of recorded goodwill
D. a need for increased professional skepticism

232. Controls over making estimates include all of the following except

A. management communication of the need for proper accounting estimates


B. comparison of prior estimates with subsequent results
C. consideration of whether estimates are consistent with the company's
operational plans
D. ensuring the effects of the estimate are in line with analysts forecasts

233. Which of the following is least likely to be included in the audit program for
debenture bonds?

A. Examine security pledged relating to, the loan


B. Confirm
C. Review bonds paid during the period
D. Perform analytical procedures

234. An auditor has calculated the interest paid on a company's recorded bonds and
found the interest paid was 10%; in examining the bonds he notes that they are
8% bonds sold without a premium or a discount. Which of the following is most
likely?

A. Understated debt outstanding.


B. Understated interest expense.
C. Overstated common stock.
D. Overstated accrued interest receivable.

235. An audit found that P10, 000,000 of long term debt on the financial statements will
become due in 6 months. The financial statement assertion that must be
addressed in determining that the proper amount of debt is included as current is:

A. existence
B. completeness
C. rights
D. presentation

236. Sole of capital stock and large debt financing transactions are usually authorized
by

A. the transfer agent


B. the shareholders
C. the board of directors
D. management
237. For a large publicly traded client the auditors' examination of capital stock account
will not ordinarily include:

A. analysis of capital stock accounts


B. confirmation of shares issued with the independent registrar
C. accounting for the proceeds of a major stock issuance
D. reconciliation of a stock certificate book with the general ledger

238. An audit program for the examination of the retained earnings count should
include a step that requires the verification of the:

A. market value used to charge retained earnings to account for a two-for-one


stock split
B. approval of the adjustment of the beginning balance as a result of a write-
down of an account receivable
C. authorization for both cash and stock dividends
D. gain or loss resulting from disposition of treasury shares

239. When verifying dividend amounts paid, an auditor will typically do all except which
of the following?

A. Determine dates and amounts of dividends paid


B. Send confirmations to shareholders to verify payments
C. Examine arrearages of preferred stock dividends.
D. Examine treatment of unclaimed dividends.

240. When a corporation has convertible debentures or stock options, which of the
following procedures should the auditor perform?

A. Verify that dividends paid are being held in a secured account.


B. Determine that enough shares are held in reserve to fulfill the obligations.
C. Determine that all stock options and convertible debentures have been
recorded in the stockholder ledger.
D. Confirm options and debentures with stock transfer agent.
MODULE 11

Completing The Audit

PSA-BASED QUESTIONS

1. Which of the following matters do auditors need not communicate to the audit
committee of a public company?

A. All critical accounting policies


B. Compensation arrangements related to the chief executive officer
C. Schedule of unadjusted differences
D. Management letter comments

2. Analytical procedures are required to be performed during the:

A. planning and substantive test stages.


B. substantive test and overall review stages.
C. planning and overall review stages.
D. planning stage only.

3. Which of the following factors would least influence an auditor‘s consideration of


the reliability of data for purposes of analytical procedures?

A. Whether the data are processed in a computer system or in a manual


accounting system
B. Whether sources within the entity are independent of those who are
responsible for the amount being audited
C. Whether the data are subjected to audit testing in the current year or prior
year
D. Whether the data are obtained from independent sources outside the entity or
from sources within the entity

4. Analytical procedures are

A. substantive tests designed to evaluate a system of internal control


B. tests of control procedures designed to evaluate the validity of management‘s
representation letter
C. substantive tests designed to evaluate the reasonableness of financial
information
D. tests of control procedures designed to detect errors in reported financial
information

5. The auditor notices significant fluctuation in key elements of the company‘s


financial statements. If management is unable to provide an acceptable
explanation, the auditor should
A. consider the matter as a scope limitation.
B. Perform additional audit procedures to investigate the matter further.
C. intensify the examination with the expectation of detecting management fraud.
D. withdraw from the engagement.

6. Who is responsible for establishing the process and controls for preparing
accounting estimates?

A. The independent auditor


B. The internal auditor
C. The management
D. The controller

7. The auditor' should adopt one or a combination of the following approaches in


the audit of an accounting estimate:

I. Review and test the process used by management to develop the estimate.
II. Use an independent estimate for comparison with what the management
prepares.
Ill. Review subsequent events which confirm the estimate made.

A. Any of them
B. None of them
C. Either I or II
D. I only

8. Which of the following is not one of the primary approaches that the auditors may
use when evaluating the reasonableness of accounting estimates?

A. Review and test management‘s process of developing estimates.


B. Confirm estimates directly with outsiders.
C. Independently develop an estimate of the amount to be compared to
management‘s estimate.
D. Review subsequent events or transactions that have bearing on the estimate.

9. The auditor should normally concentrate on the key factors and assumptions
used by management including all of the following except that those that are

A. insignificant to the accounting estimates.


B. sensitive to variations.
C. deviations from historical patterns.
D. susceptible to misstatements and biases

10. In evaluating the assumptions on which the estimate is based, the auditor would
need to pay particular attention to assumptions which are
A. reasonable in light of actual results in prior periods
B. consistent with those used for other accounting estimates
C. consistent with management's plans which appear appropriate
D. subjective or susceptible to material misstatements

11. Subsequent events refer to

A. only significant events that occur between the balance sheet date and the
date of the auditor‘s report which have been discovered by the auditor during
the same period
B. only significant events that occur between the balance sheet date and the
date of the auditor‘s report irrespective of the date they have been
discovered by the auditor
C. only significant events that occur between the balance sheet date and the
date the audited financial statements have been released to the client,
irrespective of the date of their discovery by the auditor.
D. all significant events that occur after the balance sheet date.

12. Which of the following is not correct concerning a type I and a type II subsequent
event?

A. A type I may require adjustment to financial statements while a type II would


not.
B. Both a type I and a type II subsequent event may require disclosure.
C. A type I is an event that occurred prior to year end, but was discovered after
while a type II is one that arises subsequent to year end.
D. A type II event may require adjustment to the financial statements and a type
II may require note disclosure.

13. Which of the following statements that relates to subsequent events is


inappropriately described?

A. The auditor is expected to conduct a continuing review of all matters to which


previously applied procedures have provided satisfactory conclusions.
B. The auditor should consider the effect of subsequent events on the financial
statements and on the auditor‘s report.
C. The procedures to identify events that may require adjustment as of, or
disclosure in the financial statements would be performed as near as practicable
to the date of the auditor's report.
D. The procedures that are designed to obtain sufficientty appropriate audit
evidence that all events up to the date of the audit report that may require
adjustment of, or disclosure in, the financial statements are in addition to routine
procedures which may be applied to specific transactions.

14. The auditor‘s formal review of subsequent events normally should be extended
through the date of the
A. auditors report.
B. next formal interim financial statements.
C. delivery of the audit report to the client.
D. mailing of the financial statements to the stockholders.

15. Which of the following appropriately describes the auditor‘s procedures with
respect to subsequent events?

A. The procedures to identity events that may require adjustments of, or


disclosure in, the financial statements would be performed as early as
practicable.
B. Those routine procedures that are applied to specific transactions occurring
after the period ends are designed lo obtain sulticient appropriate audit
evidence that all events up to the date of the audit report have been
identified.
C. When a component is audited by another CPA, the auditor would consider
the other auditor's procedures regarding events after period end and the need
to inform the other auditor of the planned date of the audit report.
D. The auditor is responsible to inquire regarding the financial statements after
the date of the auditor's report.

16. Which of the following is least likely a procedure that would be performed by the
auditor near the auditor's report date?

A. Reading the minutes of the meetings of shareholders, the board of directors


and audit executive committees held throughout the audit year.
B. Reading the entity's latest available interim financial statements
C. Inquiring of the client's legal counsel concerning litigations and claims.
D. Reviewing the procedures that management has established to ensure that
subsequent events are identified.

17. Which of the following procedures would an auditor most likely perform to obtain
evidence about the occurrence of subsequent events?

A. Confirming a sample of material accounts receivable established after year-


end.
B. Comparing the financial statements being reported on with those of the prior
period.
C. Investigating personnel changes in the accounting department occurring after
year-end.
D. Inquiring as to whether any unusual adjustments were made after year-end.

18. Which of the following should the auditor do the least when, after the financial
statement have been issued, the auditor becomes aware of a fact that existed at
the date of the auditor‘s report?
A. Consider whether the financial statements need revisions.
B. Discuss the matter within management.
C. Take the action appropriate in the circumstance.
D. Inform those users who are currently relying on the financial statements about
the fact that has been discovered.

19. If subsequent to the issuance of the audited financial statements, the auditor
becomes aware of material misstatements in the financial statements that exist
prior to the date of the audit report, the auditor should

A. notify the parties who are currently relying on the financial statements.
B. discuss the matter with the management, and should take the action
appropriate in the circumstances.
C. document such information in the audit plan for succeeding audit.
D. submit a revised copies of the financial statements and audit report to the
stockholders.

20. If, after the audited financial statements have been issued, the auditor becomes
aware that some information included in the statements is materially misleading,
he or she has

A. no obligation to disclose it, assuming he or she acted in good faith and without
negligence in arriving at the audit opinion.
B. an obligation to inform the board of directors of the misleading statements.
C. an obligation to inform all users who are relying on the financial statements.
D. an obligation to make certain that users who are relying on the financial
statements are informed.

21. When a new audit report is issued on financial statements because of


subsequent discovery of material misstatements on previously issued financial
statements, the audit report should include

A. no modification.
B. qualified opinion because of scope limitation.
C. qualified opinion because of inadequate disclosure.
D. emphasis of a matter paragraph that refers to a note to the financial
statements that more extensively discusses the reason for the revision of the
previously issued financial statements.

22. When a fact, that existed before the date of the report is discovered and the
management revises the previously issued audited financial statements, the
following are appropriate except the:

A. new auditor's report should include an emphasis of a matter paragraph that


refers to a note to the financial statements that discusses the reason for the
revision of the financial statements and to the earlier report issued by the auditor.
B. new auditor‘s report should contain the original date.
C. performance of the procedures that are designed to obtain sufficient evidence
as to subsequent events would ordinarily be extended to the date the revised
financial statements are approved by the entity's management.
D. auditor is permitted to restrict the audit procedures regarding the financial
statements to the effects of the subsequent event that necessitated the revision.

23. The management should assess those events that may cast significant doubt
about the entity's ability to continue as a going concern for at least

A. two years from the balance sheet date.


B. two years from the date of the audit report.
C. one year from the balance sheet date.
D. one year from the date of the audit report.

24. Which of the following is incorrect about the management's responsibility to make
an assessment of an entity's ability to continue as a going concern?

A. In assessing whether the going concern assumption is appropriate, the


management takes into account all the available information for the foreseeable
future, which should be at least twelve months from the balance sheet date.
B. Though there is a history of profitable operations and a ready access to
financial resources, management must make its assessment with detailed
analysis.
C. Management's assessment of the going concern assumption involves making
a judgment, at a particular point of time, about the future outcomes of events or
conditions which are inherently uncertain.
D. Management should make explicit assessment of its ability to continue as a
going-concern entity.

25. Which of the following least likely indicate a potential going-concern problem of
on the entity?

A. Historical negative operating cash flows


B. Failure to comply with loan covenants
C. Refinancing of large short-term obligation with a medium-term loan
D. Pending regulatory proceedings against the entity

26. Which of the following is correct about the auditor's responsibility with respect to
the entity's ability to continue as a going concern?

A. The auditor is responsible to make an assessment of the entity's ability to


continue as a going concern.
B. The auditor's responsibility is to consider the appropriateness of the
management‘s use of the going concern assumption in the preparation of the
financial statements.
C. The auditor can predict future events or conditions that may cause an entity to
discontinue as a going concern.
D. The auditor may allow the management to make an assessment of its ability
to continue as a going concern if the management is ,believed to be objective in
doing such an assessment.

27. In evaluating the management's assessment of the entity's ability to continue as


a going concern, he should consider the following, except,

A. the independence of the management.


B. the process that the management has followed to make its assessment.
C. the assumptions on which the assessment is based and management‘s plan
for future action.
D. whether the assessment has taken into account all relevant information of
which the auditor is aware of as a result of the audit procedures.

28. Which of the following is an appropriate procedure to test for an indication of


events or conditions that cast significant doubt on the entity's ability to continue
as a going concern beyond the period assessed by management?

A. Inspection
B. Inquiry
C. Observing
D. Analysis

29. When events or conditions have been identified to cast significant doubt on the
entity's ability to continue as a going concern, the auditor should

A. consider reassessing control risk at the maximum.


B. consider the issuance of disclaimer of opinion due to scope limitation.
C. review management plans for future actions based on its going-concern
assessments.
D. report the matter to the board of directors and stockholders.

30. Which of the following audit procedures would most likely assist an auditor in
identifying conditions and events that may indicate that there could be substantial
doubt about an entity's ability to continue as a going concern?

A. Review compliance with the terms of debt agreements


B. Confirm accounts receivable from principal customers
C. Reconcile interest expense with debt outstanding
D. Confirm bank balances
31. The auditor relies on the client representation letter to:

A. confirm written representations given to the auditor.


B. document the continuing materiality of client representations.
C. guarantee the absence of management fraud.
D. reduce the possibility of misunderstanding concerning management's
representations.

32. The auditors are required to obtain a letter of representation from their clients.
Which of the following statements regarding the letter of representation is
correct?

A. A letter of representation should impress upon management its responsibility


for the assertions in the financial statements.
B. A letter of representation should be signed by a company's financial officials
and attorneys.
C. A letter of representation documents the responses from the management to
inquiries about various aspects of the audit.
D. A letter of representation is a written statement from a non-independent party
and as such should not be regarded as a valid evidence.

33. A purpose of of management representation letter is to reduce

A. audit risk to an aggregate level of misstatement that could be considered


material.
B. an auditor's responsibility to detect material misstatements only to the extent
that the letter is relied on.
C. the possibility of a misunderstanding concerning management's responsibility
for the financial statements.
D. the scope of an auditor's procedures concerning related party transactions
and subsequent events.

34. Which of the following statements is true with respect to management


representations?

A. Management representations are dated as of the balance sheet date.


B. Management representations may serve as a substitute tor various types of
substantive procedures.
C. Management representations are signed by the auditor and delivered to the
client's officers.
D. Management representations are used to corroborate information obtained
during the audit.

35. When considering the use of management's written representations as audit


evidence about the completeness assertion. on auditor should understand that
such representations
A. complement, but do not replace, substantive tests designed to support the
assertion.
B. constitute sufficient evidence to support the assertion when considered in
combination with a sufficiently low assessed level of control risk.
C. are not part of the evidence considered to support the assertion.
D. replace a low assessed level of control tisk as evidence to support the
assertion.

36. The auditor should obtain evidence that the management acknowledges its
responsibility for the fair presentation of the financial statements in accordance
with PERS, and has approved the financial statements.
The auditor can obtain evidence of management's acknowledgment of such
responsibility approval

I. From relevant minutes of meetings of the board of directors or similar body.


II. By obtaining a written representation from the management.
III. By obtaining a signed copy of the financial statements.

A. Any of the given procedures


B. Either I or II
C. I only
D. None of the procedures given

37. A management representation letter would ordinarily be dated as of the

A. date the report is delivered to the entity audited.


B. date the financial statements were approved by the client management.
C. balance sheet date of the latest period reported on.
D. date a letter of audit inquiry is received from the entity's attorney of record.

38. A written representation from a client's management that, among other matters.
acknowledges its responsibility for the fair presentation of the financial
statements, should normally be signed by the

A. chief executive officer and the chief financial officer.


B. chief financial officer and the chair of the board of directors.
C. chair of the audit committee of the board of directors.
D. chief executive officer, the chair of the board of directors, and the client's
lawyer.

39. If the management refuses to furnish certain written representations that the
auditor believes are essential, which of the following is appropriate?

A. The auditor can rely an oral evidence relating to the matter as a basis for an
unqualified opinion.
B. The client's refusal does not constitute a scope limitation that may lead to a
modification of the opinion.
C. The client's refusal may have an effect on the auditor's ability to rely on other
representations of the management.
D. The auditor should express an adverse opinion because of management‘s
refusal.

40. For which of the following matters should on auditor obtain written management
representations?

A. Management's cost-benefit justifications for not correcting internal control


weaknesses.
B. Management's knowledge of future plans that may affect the price of the
entity's stock.
C. Management's compliance with contractual agreements that may affect the
financial statements.
D. Management's acknowledgement of its responsibility for employee's violations
of laws.

41. A written management representation letter is most likely to be an auditor‘s best


source of corroborative information of a client's intention to

A. terminate an employee pension plan.


B. make a public offering of its common stock.
C. settle an outstanding lawsuit for an amount less than the accrued loss
contingency.
D. discontinue a line of business.

42. Which of the following matters would an auditor most likely include in a
management representation letter?

A. Communications with the audit committee concerning weaknesses in the


internal control structure.
B. The completeness and availability of minutes of stockholders' and directors'
meetings.
C. Plans to acquire or merge with other entities in the subsequent year.
D. Management's acknowledgment of its responsibility for the detection of
employee fraud.
QUIZZERS

1. Which of the following is not among the characteristics of the procedures being
performed in completing the audit?

A. They are optional since they have only an indirect impact on the opinion to be
expressed.
B. They involve a lot of subjective judgment by the auditor.
C. They do not pertain to specific transaction cycles or accounts.
D. They are usually performed by the audit managers or other senior rnembers of
the audit team who have extensive audit experience with the client.

2. Before reaching a final decision on the opinion to be issued, a conference is


generally is held with the client. At this meeting, all of the following may be
expected, except:

A. an oral report of the auditor's major findings.


B. the auditor's rationale for proposed adjustments or additional disclosures.
C. an agreement between the auditor and the client on the changes to be made
in the financial statements
D. the delivery of the management letter.

3. Which of the following activities is ordinarily performed prior to year end?

A. Audit documentation review


B. Interim testing
C. ―Roll-forward" work
D. Subsequent event review

4. Which of the following statements is true?

A. It is more difficult to discover unrecorded transactions or events than to verify


recorded information
B. It is more difficult to verily recorded information than to discover unrecorded
transactions or events
C. It is equally difficult to verify recorded information and to discover unrecorded
transactions or events.
D. None of the given choices is true.

5. Upon completion of the audit, the auditor needs to consider uncorrected


misstatements because:

A. The aggregate of uncorrected misstatements, when considered, makes the


financial statements materially misstated.
B. There is a need to revise the financial statements after their issuance.
C. They are basis of whether the auditor needs to redocument internal control.
D. The aggregate of uncorrected misstatements is the basis of the auditor to
reassess materiality level.

6. What should a prudent auditor do when the aggregate of uncorrected


misstatements approaches the materiality level?

A B C D
Perform additional procedures YES NO NO YES
Request management to adjust
financial statements for identified YES YES NO YES
misstatements
Request management to adjust
financial misstatements for YES NO YES NO
projected misstatements

7. If based on the aggregate of uncorrected misstatements the auditor believes


there may be material misstatements, the auditor should perform additional
procedures. If the client refuses to adjust the financial statements and the auditor
is not able to conclude that the aggregate of uncorrected misstatements is not
material, the auditor should,

A. issue a standard opinion.


B. consider resigning from the engagement.
C. appropriately modify the audit report.
D. obtain additional representation letter covering uncorrected misstatements.

8. Which of the following communications is ordinarily signed by the auditor?

A. Attorney's letter
B. Management representation letter
C. Internal control deficiency letter
D. All of these are signed by the auditor

9. They involve analysis of significant ratios and trends including the resultant
investigation of fluctuations and relationships that are inconsistent with other
relevant information or expectation:

A. inquiry
B. analytical procedures.
C. account analysis.
D. inspection.

10. Analytical procedures performed in the overall review stage of on audit suggest
that several accounts have unexpected relationships. The results of these
procedures most likely indicate that
A. unaccounted effects of irregularities exist.
B. internal control activities are not operating effectively.
C. additional tests of details are required.
D. the communication with the audit committee should be revised.
11. When substantive tests are performed before the balance sheet date, at a
minimum the auditors should, at or after the balance sheet date:

A. changes occurred in the account balances between the two dates.


B. perform analytical procedures, including comparison of the account balances
of the two dates.
C. reconfirm all balances that were confirmed at interim date.
D. confirm all balances that were not confirmed at interim date.

12. An assumption underlying analytical procedures is that

A. these procedures cannot replace tests of balances and transactions.


B. statistical tests of financial information may lead to the discovery of material
errors in the financial statements.
C. the study of financial ratios is an acceptable alternative to the investigation of
unusual fluctuations.
D. relationships among data may reasonably be expected to exist and continue
in the absence of known conditions to the contrary.

13. An auditor suspects that fictitious sales may have been recorded during the year.
Which of the following analytical review results would most likely indicate that
fictitious sales were recorded?

A. Uncollectible account write-offs increased by 10 percent, sales increased by


10 percent and accounts receivable increased by 10 percent.
B. Gross margin decreased from 40 to 35 percent.
C. The number of days' sales in accounts receivable decreased from 64 to38.
D. Accounts receivable turnover decreased from 7.1 to 4.3.

14. Auditors apply analytical procedures on client's operations in order to identify

A. improper separation of accounting and other financial duties.


B. weaknesses of a material nature in the client's internal control.
C. unusual transactions.
D. noncompliance with prescribed control procedures.

15.Of the following procedures, which one does not produce analytical evidence?

A. Compare revenue, cost of sales, and gross profit with the prior year and
investigate significant variations.
B. Examine monthly performance reports and investigate significant revenue and
expense variances.
C. Confirm customers‘ accounts receivable and clear all material exceptions.
D. Compare sales trends and profit margins with industry averages and
investigate significant differences.

16. The extent to which analytical procedures provide useful substantive evidence
depends on

A. the effectiveness of client‘s internal control system


B. the integrity and training of client's personnel.
C. their reliability in the circumstances.
D. the experience of the auditor using them.

17. Analytical procedures:

A. are required to be performed in the planning phase of the audit.


B. are often done during examination's testing stage.
C. are required to be done during the completion phase of the audit.
D. All of them

18. An important benefit from industry comparisons is:

A. an aid to understanding the client's business


B. an indicator of errors.
C. an indicator of irregularities.
D. a least-cost indicator for audit procedures.

19. A benefit obtained from comparing client's data with industry average is that it
provides

A. an indication of the likelihood of financial failure.


B. an indication where errors exist in the statements.
C. a benchmark to be used in evaluating client's budget.
D. a comparison of "what is" with "what should be."

20. When the current year's unaudited trial balance is compared to the prior year's
audited trial balance.

A. errors are identified.


B. discrepancies are discovered.
C. irregularities become apparent.
D. changes are highlighted.

21. When a higher than normal ratio of long-term debt to net worth is coupled with a
lower than average ratio of profits to total assets, the company

A. is highly successful.
B. is comparable with industry standards.
C. has a high risk of financial failure.
D. has a liquidity problem.

22. Which of the following discoveries through the use of analytical procedures would
indicate a relatively high risk of financial failure?

A. A decline in gross margin percentages.


B. An increase in the balance of fixed assets.
C. A higher than normal ratio of long-term debt to net worth as well as a lower
than average ratio of profits to total assets.
D. An increase in the ratio of allowance for uncollectible accounts to gross
accounts receivable, while at the same time accounts receivable turnover also
decreased.

23.―Unusual fluctuations" occur when

A. Significant differences are not expected but do exist.


B. Significant differences are expected but do not exist.
C. there is a material accounting error or irregularity.
D. Any one of the given three situations may occur

24. Which method of analytical procedure is most useful because many expenses,
such as cost of goods sold, might be expected to bear a predictable relationship
to net sales?

A. Horizontal analysis
B. Trend analysis
C. Vertical analysis
D. Reasonable analysis

25. One type of analytical procedure is trend analysis. Which of the following is the
best example of trend analysis?

A. Comparison of company financial ratios to that of its competitors.


B. Comparison of accounting records to budgeted amounts.
C. Comparison of inventory levels over the past 3 years.
D. Comparison of interest expenses to outstanding loan balances.

26. Analytical procedures are those that

A. evaluate the accuracy of the account balances.


B. assess the overall reasonableness of transactions and balances.
C. review the effectiveness of internal control procedures.
D. analyze the effect of management procedures on the accounting system.
27. Analytical procedures enable the auditor to predict the balance or quantity of an
item under audit. Information to develop this estimate can be obtained from all of
the following except

A. tracing transactions through the system to determine whether procedures are


being applied as prescribed.
B. comparison of financial data with data for comparable prior periods,
anticipated results (e.g., budgets and forecasts), and similar data for the industry
in which the entity operates.
C. study of the relationships of elements of financial data that would be expected
to conform to a predictable pattern based upon the entity's experience.
D. study of the relationships of financial data with relevant nonfinancial data.

28. Which of the following statements is true concerning analytical procedures?

A. Analytical procedures usually involve comparisons of ratios developed from


recorded amounts with assertions developed by the management.
B. Analytical procedures used in planning an audit ordinarily use data aggregated
at a high level.
C. Analytical procedures can replace tests of controls in gathering evidence to
support the assessed level of control risk.
D. Analytical procedures are more efficient, but not more effective, than tests of
details and transactions

29. Which of the following items tend to be the most predictable for purposes of
analytical procedures applied as substantive tests?

A. Relationships involving balance sheet accounts


B. Transactions subject to management discretion
C. Relationships involving income statement accounts
D. Data subject to audit testing in the prior year

30. According to professional standards, analytical procedures are least likely to be


applied to:

A. test disclosures about reportable operating segments.


B. review the financial statements or interim financial information.
C. compile the financial statements.
D. plan and audit and assist in the final review.

31.Which of the following is not a typical analytical procedure?

A. Study of relationships of financial information with relevant nonfinancial


information.
B. Comparison of financial information with similar information regarding the
industry in which the entity operates.
C. Comparison of recorded amounts of major disbursements with appropriate
invoices.
D. Comparison of recorded amounts of major disbursements with budgeted
amounts.
32. Which of the following would be least likely to be comparable between similar
corporations in the same industry line of business?

A. Earnings per share


B. Return on total assets before interest and taxes
C. Accounts receivable turnover
D. Operating cycle

33. Sales commissions as a percentage of sales declined significantly during the


year under audit. Of the following possible causes, the most likely is

A. sales increased during the year.


B. the sales force was reduced at the end of the year.
C. sales commission rates were increased at the beginning of the year
D. fictitious sales were recorded at year-end to inflate earnings. Commissions
were not recorded on these sales.

34. In evaluating the effectiveness of a company‘s credit and collection policies, the
ratio most likely to be used by an auditor is

A. quick ratio.
B. accounts receivable turnover.
C. working capital turnover.
D. return on sales.

35. During an audit of the accounts receivable function, you found that the accounts
receivable turnover rate had fallen from 7.3 to 4.3 over the last three years. What
is the most likely cause of the decrease in the turnover rate?

A. An increase in the discount offered for early payment.


B. A more liberal credit policy.
C. A change from net 30 to net 25.
D. Greater cash sales.

36. Significant unexpected fluctuations identified by analytical procedures will


usually necessitate a(an)

A. consistency qualification.
B. review of internal control.
C. explanation in the representation letter.
D. auditor investigation.
37. An auditor compares 2010 revenues and expenses with those of the prior year
and investigates all changes exceeding 10%. By this procedure the auditor would
be most likely to learn that

A. an increase in property tax rates has not been recognized in the client's
accrual.
B. the 2010 provision for uncollectible accounts is inadequate, because of
worsening economic conditions.
C. fourth quarter payroll taxes were not paid.
D. the client changed its capitalization policy for small tools in 2010.

38. Of the following procedures, which is the most important that on auditor should
use when performing an analytical review of the income statement?

A. Select sales and expense items and trace their amounts to related supporting
documents.
B. Compare actual revenues and expenses with the corresponding figures of the
previous year and investigate significant differences.
C. Obtain from the proper client representatives, inventory certificates for the
beginning and ending inventory amounts that were used to determine cost of
sales.
D. Ascertain that the net income amount in the statement of changes in financial
position (statement of cash flows) agrees with the net income amount in the
income statement.

39.The auditor's analytical procedures will be facilitated if the client

A. uses a standard cost system that produces variance reports.


B. segregates obsolete inventory before the physical inventory count.
C. corrects material weaknesses in internal control before the beginning of the
audit.
D. reduces inventory balances to the lower of cost or market.

40. Which of the following is not a purpose served by the application of analytical
procedures?

A. As part of audit planning to assist in locating significant changes in revenues


and expenses.
B. To provide a basis for lowering materiality thresholds where significant
earnings inflation is indicated.
C. To determine the economic substance of related party transactions.
D. As part of audit review to determine that all significant abnormalities have
been resolved to the auditor's satisfaction.
41. Auditors sometimes use comparison of ratios as audit evidence. For example, an
unexplained decrease in the ratio of gross profit to sales may suggest which of
the following possibilities?

A. Unrecorded purchases
B. Unrecorded sales
C. Merchandise purchases being charged to selling and general expense
D. Fictitious sales

42. In applying analytical procedures, the auditor discovered that gross profit as a
percent of sales declined sharply during the current year. A possible cause might
be

A. the client has significant amounts of obsolete inventory carried at full cost.
B. a significant quantity of finished goods located in a distant warehouse was
inadvertently omitted from the ending inventory.
C. recorded sales included goods that were shipped the following year.
D. depreciation of office equipment was overstated.

43. An abnormal fluctuation in gross profit that might suggest the need for extended
audit procedures for sales and inventories would most likely be identified in the
planning phase of the audit by the use of

A. tests of transactions and balances.


B. a preliminary review of internal control.
C. specialized audit programs.
D. analytical procedures.

44. What form of analytical review might uncover the existence of obsolete
merchandise?

A. Inventory turnover rates


B. Decrease in the ratio of gross profit to sales
C. Ratio of inventory to accounts payable
D. Comparison of inventory values to purchase invoices.

45. What is ordinarily the primary concern when auditing the income
statement?

A. Overstatement of Revenues. Expenses and Net Income


B. Overstatement of Revenues and Expenses, and understatement of Net
Income
C. Overstatement of Net Income and understatement of Revenues and
Expenses
D. Overstatement of Revenues and Net Income. understatement of Expenses
46. Compared to balance sheet accounts, the audit of income statement accounts
generally relies more heavily on:

A. tests of details of transactions.


B. tests of details of balances.
C. analytical procedures.
D. tests of controls.

47. What audit procedure is not ordinarily used to examine selling, general and
administrative expenses?

A. .Analytical procedures
B. Use of budgets to identify unexpected differences
C. Confirmations of amounts paid with advertising agencies
D. Detailed tests of balances

48. Which of the following income statement accounts is least likely to be subject to
extensive detailed tests of balances?

A. legal and professional fees


B. Contributions
C. Cost of sales
D. Officers' salaries

49. Which of the following procedures is normally not considered in the auditor's
substantive procedures for revenue and expense accounts?

A. Evaluate evidence gathered in the audit of balance sheet accounts and


examination of transaction cycles.
B. Individually confirm significant transactions with third parties.
C. Perform analytical procedures to verify the overall reasonableness of revenue
and expense accounts.
D. Examine "miscellaneous", "other and "clearing"' accounts that are classified as
revenues and expenses.

50. The auditors' best course of action with respect to "Other financial information"
included in a client prepared annual report containing the auditor‘s report is to:

A. indicate in the auditors' report that the "other financial information" is


unaudited.
B. consider whether the "other financial information is accurate by performing a
review.
C. obtain written representations from the management as to the material
accuracy of the "other financial information."
D. read and consider the manner of presentation of the "other financial
information.
51. Which of the following events in the subsequent period is on example of a Type
2 subsequent event?

A. Realization of recorded year-end receivables of a different amount than what


is recorded
B. Settlement of recorded year-end estimated product warranty liabilities at an
amount different from what is recorded
C. Purchase of a business
D. Purchase of a machine

52. Which of the following statements best expresses the auditor's responsibility with
respect to events occurring in the subsequent period?

A. The auditor has no responsibility for events occurring in the subsequent period
unless these events affect transactions recorded on or before the balance sheet
date.
B. The auditor's responsibility is to determine that transactions recorded on or
before the balance sheet date actually occurred.
C. The auditor is fully responsible for events occurring in the subsequent period
and should extend all detailed procedures through the last day of the field work.
D. The auditor is responsible for determining that a proper cutoff has been made
and for performing a general review of events occurring in the subsequent
period.

53. An auditor concludes that the omission of a substantive procedures considered


necessary at the time of the audit may impair the auditor's current ability to
support the opinion that had been previously issued. The auditor need not apply
the omitted procedure if the

A. risk of adverse publicity or litigation is low.


B. results of other procedures that were applied tend to compensate for the
omitted procedure.
C. auditor's opinion is qualified because of a departure from generally accepted
accounting principles.
D. results of the subsequent period's tests of controls make the omitted
procedure less important.

54. Which of the following procedures can be performed only in the subsequent
period?

A. Examination of data to determine that a proper cutoff has been made


B. Tests of the details of balances
C. Tests of the details of transactions
D. Reading of the minutes of the board of directors' meetings
55. A major customer of an audit client suffers a fire just prior to completion of year-
end fieldwork. The audit client believes that this event could have a significant
direct effect on the financial statements. The auditor should:

A. Advise the management to disclose the event in notes to the financial


statements.
B. Disclose the event in the auditor‘s report.
C. Withhold submission of the auditor‘s report until the extent of the direct effect
on the financial statements is known.
D. Advise the management to adjust the financial statements.

56. An auditor is concerned with completing various phases of the audit after the
balance sheet date. This subsequent period extends to the date of the

A. auditor's report.
B. final review of the audit working papers.
C. public issuance of the financial statements.
D. delivery of the auditor‘s report to the client.

57. Which of the following procedures should an auditor ordinarily perform regarding
subsequent events?

A. Compare the latest available interim financial statements with the financial
statements being audited.
B. Send second requests to client's customers who failed to respond to the first
accounts receivable confirmation requests.
C. Communicate material weaknesses in internal control to the client's audit
committee.
D. Review the cutoff bank statements for several months after the year-end.

58. Which of the following events occurring after the issuance of an auditor‘s report
most likely would cause the auditor to make further inquiries about the previously
issued financial statements?

A. A technological development that could affect the entity's future - ability to


continue as a going concern.
B. The discovery of information regarding a contingency that existed before the
financial statements were issued.
C. The entity's sale of a subsidiary that accounts for 30% of the entity's
consolidated sales.
D. The final resolution of a lawsuit which is adequately explained in a separate
paragraph of the auditor's report.

59. After issuing a report, an auditor has no obligation to make continuing inquiries or
perform other procedures concerning the audited financial statements, unless
A. an information, which existed at the report dote that affects the report comes
to the auditor's attention.
B. the control environment changes after the issuance of the report.
C. an information about on event that occurred after the end of field work comes
to the auditor's attention.
D. the final determinations or resolutions are made of contingencies that had
been disclosed in the financial statements.

60. Subsequent to the issuance of the auditor's report, the auditor become aware of
facts existing at the report date that would have affected the report had the
auditor then been aware of such facts. After determining that the information is
reliable, the auditor should next

A. notify the board of directors that the auditor‘s report must no longer be
associated with the financial statements.
B. determine whether there are persons relying or likely to rely on the financial
statements who would attach importance to the information.
C. request the management to disclose the effects of the newly discovered
information by adding a footnote to subsequently issued financial statements.
D. issue a revised set of pro-forma financial statements that consider the newly
discovered information.

61. On March 15, 2010, Kiel, CPA, expressed an unqualified opinion on a client's
audited financial statements for the year ended December 31, 2009. On May 4,
2010, Kiel's internal inspection program disclosed that engagement personnel
failed to observe the client's physical inventory. Omission of this procedure
impairs Kiel's current ability to support the unqualified opinion. If the shareholders
are currently relying on the opinion, Kiel should first

A. advise the management to disclose to the shareholders that his unqualified


opinion should not be relied on.
B. undertake to apply alternative procedures that would provide a satisfactory
basis for an unqualified opinion.
C. reissue the auditor's report and add an explanatory paragraph describing the
departure from PFRS.
D. compensate for the omitted procedure by performing tests of controls to
reduce audit risk to a sufficiently low level.

62. Six months after issuing an unqualified opinion on audited financial statements,
an auditor discovered that the engagement personnel failed to confirm several of
the client's material accounts receivable balances. The auditor should first:

A. request permission of the client to undertake the confirmation of accounts


receivable.
B. perform alternative procedures to provide a satisfactory basis for an
unqualified opinion.
C. assess the importance of the omitted procedures to the auditor's ability to
support the previously issued opinion.
D. inquire whether there are persons currently relying, or likely to rely on the
unqualified opinion.
63. When a fact is discovered alter the date of the report but before the financial
statements are issued and the client amends the financial statements, would the
following procedures or actions be necessary?

A B C D
Procedures to obtain evidence with
respect to subsequent events are YES YES NO NO
extended.
An emphasis of a matter is required. YES NO NO YES

64. The auditor's primary means of obtaining corroboration of management's


information concerning litigation is a

A. letter of audit inquiry to the client's lawyer.


B. letter of corroboration from the auditor's lawyer upon review of the legal
documentation.
C. confirmation of claims and assessments from other parties to the litigation.
D. confirmation of claims and assessments from an officer of the court presiding
over the litigation.

65. Which of the following is not a procedure to discover unasserted claims or


contingent liabilities?

A. Review of Board of Director minutes


B. Sending a letter of inquiry to the client's attorney
C. Substantive testing of company accounts receivable
D. Searching newspapers and other periodicals for stories about the client and its
industry

66. Which of the following statements concerning litigation, claims and assessments
which were extracted from a letter from a client's lawyer is most likely to cause
the auditor to request clarification?

A. "I believe that the possible liability to the company is nominal in amount."
B. "I believe that the action can be settled for less than the damages claimed."
C. ―I believe that the plaintiff's case against the company is without merit."
D. "I believe that the company will be able to defend this action successfully."

67. In evaluating whether there is a sufficiently low probability of material


misstatement in the financial statements, the auditors accumulate:
A. likely misstatements in the financial statements.
B. known misstatements in the financial statements.
C. known, projected and other estimated misstatements in the financial
statements.
D. known, projected and potential misstatements in the financial statements.

68. Which of the following is typically the auditor‘s initial procedure to be performed
to identify litigation, claims, and assessments?

A. Perform analytical procedures


B. Confirm litigation, claims, and assessments with third-party litigants
C. Obtain a letter from the client's legal counsel
D. Inquire of the client regarding the existence of litigation, claims, and
assessments

69. The auditors should request that on audit client sends a letter of inquiry to those
attorneys who have been consulted concerning litigation, claims, or
assessments. The primary reason for this request is to provide:

A. an information concerning the progress of cases to date.


B. corroborative evidential matter.
C. an estimate of the peso amount of the probable loss.
D. an expert opinion regarding whether a loss is possible, probable, or remote.

70. In an audit of contingent liabilities, which of the following procedures would be


least effective?

A. Reviewing a bank confirmation letter


B. Examining the customer confirmation replies
C. Examining the invoices for professional services
D. Reading the minutes of the board of directors meetings

71. An attorney, responding to an auditor as a result of the client's letter of audit


inquiry, may appropriately limit the response to

A. items which have high probability of being resolved to the client‘s detriment.
B. asserted claims and pending or threatened litigation.
C. legal matters subject to unsettled points of law, uncorroborated information or
other complex judgments.
D. matters to which the attorney has given substantial attention in the form of
legal consultation or representation.

72. The primary reason why an auditor requests that letters of inquiry be sent to the
client's legal counsel is to provide the auditor with
A. a description and evaluation of litigation, claims, and assessments that existed
at the balance sheet date of.
B. an expert opinion as to whether a loss is possible, probable, or remote.
C. the opportunity to examine the documentation concerning litigation, claims,
and assessments.
D. corroboration of the information furnished by the management concerning
litigation, claims, and assessments.

73. An auditor should obtain evidential matter relevant to each of the following actors
concerning third-party litigation against a client except the

A. period in which the underlying cause for legal action occurred.


B. probability of an unfavorable outcome.
C. jurisdiction in which the matter will be resolved.
D. existence of a situation indicating an uncertainty as to the possible loss.

74. The primary source of information about litigation, claims, and assessments is the:

A. board of directors.
B. client's attorneys.
C. management.
D. reply through direct confirmation with the other party involved.

75. The letter of audit inquiry to the client's lawyer(s) is the auditor's primary means
of obtaining:

A. corroboration of the information on litigation. claims, and assessments


provided by the auditor's attorneys.
B. corroboration of the information on litigation, claims, and assessments,
provided by management.
C. corroboration of the information on litigation, claims, and assessments
provided by the other party to the matter.
D. initial information about litigation, claims. and assessments.

76. A lawyer's refusal to respond to a letter of audit inquiry normally requires the
auditor to issue a(n):

A. qualified opinion or a disclaimer of opinion.


B. unqualified opinion with an explanatory paragraph.
C. qualified or adverse opinion.
D. standard three-paragraph unqualified opinion.

77. Which of the following auditing procedures is ordinarily performed last?

A. Reading of the minutes of the directors' meetings held subsequent to balance


sheet date
B. Confirming accounts payable
C. Obtaining a management representation letter
D. Testing of the purchasing function.

78. When litigation or claims have been identified or when the auditor believes they
may exist, the auditor should

A. seek direct communication with the entity's lawyers.


B. disclose the litigation and claims in the auditor‘s report.
C. issue unqualified opinion with explanatory paragraph.
D. issue qualified or adverse opinion.

79. When an audit is made in accordance with the Philippine Standards on Auditing,
the auditor should always

A. document the understanding of the client's internal control and the basis for all
conclusions about the assessed level of control risk to financial statement
assertions.
B. employ analytical procedures as substantive tests to obtain evidence about
specific assertions related to account balances.
C. obtain appropriate representations from the management.
D. observe the taking of physical inventory on the balance sheet date.

80. Written management representations obtained by the auditor in connection with a


financial statement audit should include

A. a summary of all corrected misstatements.


B. management's belief that the effects of uncorrected misstatements are not
material.
C. a summary of all uncorrected misstatements
D. management's belief that any uncorrected misstatements are in fact not
misstatements.

81. An auditor accepted an engagement to audit the 2009 financial statements of


DRL Corporation and began the fieldwork on September 30, 2009. DRL gave the
auditor the 2009 financial statements on January 7, 2010. The auditor completed
the fieldwork and simultaneously obtains approval of the financial statements by
the management on February 10, 2010. The auditor delivered the report on
February 16, 2010. The management representation letter should normally be
dated:

A. December 31, 2009


B. January 17, 2010
C. February 10, 2010
D. February 16, 2010
82. Which of the following is least likely an action that may mitigate an entity's
difficulty to continue as a going concern?

A. Increased cash dividends


B. Retirement of outstanding capital stock in order to improve earnings per share
C. Retirement of long-term debt in order to improve profitability
D. Disposal of property in a sale-leaseback arrangement

83. Road, CPA, believes there is "substantial doubt about the ability of Kennon
Company to continue as a going concern for a reasonable period of time. In
evaluating Kennon's plan for dealing with the adverse effects of future conditions
and events, Rood most likely would consider, as a mitigating factor, Kennon's
plans to:

A. Make the credit terms for sales on account more lenient.


B. strengthen internal controls over cash disbursements.
C. purchase the production facilities currently being leased from a related party.
D. postpone those expenditures for research and development projects.

84. The auditor is most likely to discover omitted audit procedures during:

A. preparation of the management letter.


B. follow-up procedures performed in compliance with generally accepted
auditing standards.
C. a post engagement review performed as part of the firm's quality control
inspection program.
D. the final review of the working papers.

85. Which of the following types of audit documentation review is focused on


ensuring that the quality of audit work and reporting is consistent with the qualitys
standards of the firm?

A. Review of staff work by audit supervisor


B. Review of staff work by audit manager
C. Review of work staff by audit manager and audit partner
D. Review of work by second (reviewing) partner
MODULE 12

INTERNAL AUDITING

1. An audit committee must comprise of outside directors. Which of the following is


considered an outside director? A member of the board of directors who is

A. the treasurer.
B. the company‘s president.
C. a retired executive from another company.
D. a consultant to the company.

2. The primary difference between operational auditing and financial auditing is that
In operational auditing

A. the auditor is not concerned with whether the audited activity is generating
Information in compliance with financial accounting standards
B. The operational auditor is seeking to help management use resources in the
most effective manner possible.
C. the auditor starts with the financial statements of an activity being audited and
works backward to the basic processes involved on producing them.
D. the auditor can use analytical skills and tools that are not necessary in
financial auditing.

3. Internal auditing often extends beyond examinations leading to the dtermination


of the fairness of financial presentation and includes audits of efficiency,
effectiveness, and

A. internal control
B. evaluation
C. accuracy
D. compliance

4. Which of the following factors are essential to an effective internal auditing


organization?

I. Operating responsibility
II. Organizational status
III. Objectivity
IV. Authority over operations

A. I and II
B. II and III
C. Ill and IV
D. I and IV
5. Internal auditors are most likely to issue a report on which of the following?

A. Annual financial statement reporting


B. Internal control
C. Tax compliance
D. Quarterly financial statement reporting

6. The internal audit staff has been asked to conduct an audit of the purchasing
department. Top management feels that there have been some production
bottlenecks recently because of out-of-stock situations. What is the primary
objective of the auditors in this assignment?

A. To appraise the economy with which resources are employed.


B. To review the reliability and integrity of financial and operating information.
C. To review the means of safeguarding assets and verifying the existence of
such assets.
D. To ascertain whether results are consistent with established objectives and
whether operations are being carried out as planned.

7. The best description of the scope of internal auditing is that it encompasses

A. primarily operational auditing.


B. both financial and operational auditing.
C. primarily the safeguarding of assets and verifying the existence of such
assets.
D. primarily financial auditing.

8. The role of the internal auditor with respect to a purchasing deportment audit is to

A. evaluate the adequacy of purchasing policies and procedures and determine


the extent of compliance.
B. review and appraise the adequacy of controls over the creation of all types of
company obligations.
C. review and appraise the conditions by which notes payable come into
existence and the control exercised over them.
D. establish that the purchasing deportment is independent of receiving.
inspection, stores, and accounts payable activities.

9. Which of the following statements regarding assurance services is/are true with
respect to internal auditors?

I. Assurance services by internal auditors can be provided only to internal


management and the audit committee: that is. they cannot be provided to outside
entities.
II. Assurance services require the specification of assertions to be tested.
III. Assurance services can be provided bray in the areas of controls and financial
reporting.

A. I and II
B. II only
C. I. II, and III
D. None of the above

10. Which of the following statements is correct regarding the performance of


consulting activities by internal auditors?

A. Consulting activities: by definition, impair the independence of the auditor and


therefore should be performed only in areas that the internal audit department
does not plan to audit in the future.
B. Consulting activities are simply an extension of the auditor's current work in
providing recommendations.
C. Consulting is a more proactive approach in which the auditor takes the lead in
analyzing problems, deciding the best course of action, and assisting the
management in implementing solutions.
D. Consulting should be limited to internal controls because that is the auditor‘s
area of primary competence.

11. The primary audience for the written report issued by the internal auditor at the
completion of an audit should be

A. the external auditors when they intend to rely on the internal auditor's work.
B. managers outside the area of audit so as to inform them of what is going on in
other areas of the organization.
C. the audit committee who needs to be kept informed on the risks to which the
organization is exposed.
D. the management inside or outside the audited area who can take corrective
action.

12. Which of the following statements is/are true regarding independence and
objectivity as applied to internal auditing?

I. Independence is a departmental feature that affects the scope of audits.


II. Only the audit committee can determine independence.
III. Objectivity is a personal feature that is to be exhibited by all internal audit
team members on an audit.

A. I and ll
B. I and III
C. I, II. and III
D. Ill only
13. An internal auditor determines that the actual procedures differ from prescribed
control procedures. The auditor should

I. Require operating personnel to conform to prescribed procedures


2. Document the discrepancies and make any appropriate recommendations to
management.
3 Expand all aspects of the audit to determine other differences from prescribed
procedures.
4. Modify the audit plan as warranted by the differences noted.

A. I and 3
B. 2 and 3
C. I and 4
D. 2 and 4

14. The internal audit function should be responsible to:

A. the audit committee.


B. senior management.
C. the external auditor.
D. both the audit committee and senior management.

15. Follow-up activities by the internal auditor may be terminated, even though
corrective action has not been taken, when the

A. recommendations concern activities not included in the scope of the original


audit program.
B. board of directors or management has assumed the risk of not taking
corrective action.
C. auditor has not convinced operating personnel of the soundness of the audit
recommendations.
D. auditor has no authority or responsibility to prescribe or direct the corrective
action.

16. Which of the following would not be an audit activity that contributes to improved
corporate governance?

A. Perform a compliance audit to determine if the marketing department operates


in conformance with company policies.
B. Help train operational managers in Control Self-Assessment.
C. Assist the external auditors in conducting the annual external audit.
D. Perform on operational audit of the company's procurement processes.

17. An internal auditor is preparing a final audit report to management. There is,
however, disagreement between the auditor .and the auditee on one finding
which details the auditee‘s violation of corporate purchasing policy. The auditee
believes the purchasing policy is open to interpretation and that there was no
violation. The auditor believes that the policy is clearly stated and that the
auditee's actions were a violation. In this circumstance, the auditor should

A. delete the finding from the audit report.


B. present only those facts, with support the audit finding and ignore those which
detract from it.
C. present both the auditor's and auditee's positions in the report.
D. not issue the audit report until the auditor and auditee agree on all audit
findings and recommendations.

18. Which of the following statements is not correct regarding the internal and
external audit profession?

A. External auditors cannot perform internal audit work for their public company
financial statement audit clients.
B. Internal audit work is broader in scope.
C All auditors performing internal audits must be Certified Internal Auditors.
D. The audit committee is an important client.

19. According to the Standards, the internal audit director should ensure follow-up of
prior audit findings and recommendations:

A. to determine if corrective action was taken and is achieving the desired


results.
B. unless management rejected the recommendation in their initial response.
C. unless the audit schedule does not allow time for follow-up..
D. unless management has accepted the recommendation.

20.The major objective of operational audit is to:

A. analyze operational areas for control deficiencies, especially those that would
allow a fraud to go undetected.
B. perform trend analysis to identify high risk areas that merit management
attention.
C. analyze operations to identify potential deficiencies as a basis for improving
operational performance.
D. determine mismanagement or ineffective management by department or
divisional managers.

21. Internal auditing has been a dynamic profession. Which of the following best
describes the scope of internal auditing as it has developed to date?

A. Internal auditing involves appraising the economy and efficiency with which
resources are employed.
B. Internal auditing involves evaluating compliance with policies, plans,
procedures, laws and regulations.
C. Internal auditing has evolved to verifying the existence of assets and
reviewing the means of safeguarding assets.
D. Internal auditing has evolved to more of an operational orientation from a
strictly financial orientation.

22. You have been selected to develop an internal auditing department for your
company. Your approach would most likely be to hire:

A. internal auditors each of whom possesses all the skills required to handle all
audit assignments.
B. inexperienced personnel and train them the way the company wants them
trained.
C. degreed accountants since most audit work is accounting related.
D. internal auditors who collectively have the knowledge and skills needed to
complete all internal audit assignments.

23. Operational auditing is primarily oriented toward:

A. future improvements to accomplish management's goals.


B. the accuracy of data reflected in management's financial records.
C. verifying that the company's financial statements are fairly presented.
D. post protection provided by existing internal control.

24. Which of the following is the most appropriate method of reporting disagreement
between the auditor and the auditee concerning audit findings and
recommendations?

A. State the auditor's position because the report is designed to provide the
auditor's independent view.
B. State the auditee's position because management is ultimately responsible for
the activities reported.
C. State both positions and identify the reasons for the disagreement.
D. State neither position. If the disagreement is ultimately resolved, there will be
no reason to report the previous disagreement. If the disagreement is never
resolved, the disagreement should not be reported, because there is no
mechanism to resolve it.

25. Workpapers document the work done by the internal auditor. The internal auditor
should

A. prepare the workpapers and permit no access to them by auditees.


B. encourage qualified auditees to assist in preparing audit workpapers on
nonsensitive subjects.
C. prepare the workpapers but review them with the auditee to be sure that the
evidence is appropriately compiled.
D. defer preparation of workpapers for a business segment until the audit work
on that phase is completed.

26. Which of the following could be an organization factor that might adversely affect
the ethical behavior of the director of internal auditing?

A, The director reports directly to an independent audit committee of the board of


directors.
B. The director of internal auditing is not assigned any operational
responsibilities.
C. A director of internal auditing may not be appointed without concurrence of the
board of directors.
D. The director's annual bonuses are based on peso recoveries of recommended
future savings as a result of audits.

27. Operational audit planning includes

A. establishing the audit objectives and the scope of work


B. determining the efficiency with which the organization uses its resources
C. establishing qualifications for members of the internal audit department
D. reviewing existing operating systems on a preliminary basis

28. The president wants to know whether the purchasing function is properly meeting
its charge to ―purchase the right material at the right time in the right quantities."
Which of the following types of audits addresses the president's request?

A. A financial audit of the purchasing department


B. An operational audit of the purchasing department
C. A compliance audit of the purchasing function
D. A full-scope audit of the manufacturing operation

29. Prior to commencing an operational audit, the auditee management requests that
the auditor include certain tests in the audit program. The auditor should

A. refuse to add them to the audit program as requested by the auditee.


B. comply if there is sufficient time available in the budget.
C. give priority to these requests.
D. perform the tests when they are approved by the audit committee or its
designee.

30. A company's new president meets the director of internal audit for the first time,
and asks the director to briefly describe his department's overall responsibility.
The director should state that the internal audit's overall responsibility is to:
A. act as an independent appraisal function to review operations as a service to
management by measuring and evaluating the effectiveness of controls.
B. review the means of safeguarding assets and, as appropriate, verify the
existence of such assets.
C. ensure compliance with policies, plans, procedures, laws, and regulations,
which could have a significant impact on operations and reports.
D. review the reliability and integrity of financial and operating information and
the means used to identify, measure, classify, and report such information.

31. Which of the following shortcomings that was uncovered during a review of
recent audit should cause the greatest concern to the supervisor of internal
auditing?

A. Incomplete documentation related to certain findings


B. Lack of approval for additions to the audit program
C. Overrun on the time budget and completion schedule
D. Lack of compliance with the prescribed format for workpapers

32. Recommendations in audit reports may or may not actually be implemented.


Which of the following best describes internal auditor's role in follow-up on audit
recommendations? Internal auditor:

A. has no role; follow-up is a management's responsibility.


B. should be charged with responsibility for implementing audit
recommendations.
C. should follow up to ascertain that appropriate action is taken on audit
recommendations.
D. should request that independent auditors follow up on audit recommendations.

33. The most important function of operational audit report is to

A. report findings and recommendations.


B. state the auditor's opinion or conclusion.
C. direct management to take specified actions.
D. report the objective of the audit

34. A major difference between operational auditing and financial auditing is

A. operational auditing focuses on activities rather than financial statement


assertions.
B. operational auditing extends beyond the entity to analysis of industry and
economic data.
C. unlike financial auditing, operational auditing does not generally culminate in a
formal audit report.
D. operational auditing is narrower than financial auditing in its breadth of
coverage.
35. In an operational audit of a computerized inventory control system that generates
purchase orders based on reorder points and economic order quantities, which of
the following approaches would provide the best evidence that purchase orders
are authorized?

A. Trace purchase orders to the computer listing


B. Compare receiving reports to purchase order details
C. Test the system to determine that only authorized individuals can change the
parameters in the program that generates purchase orders
D. Review the system documentation to determine proper functioning of the
program

36. A major part of an operational audit involves appraising the efficiency and
effectiveness of the activity or unit being audited. Efficiency and effectiveness
appraisal include all but the following steps:

A. assessing the goal selling process.


B. determining the extent to which goals are being achieved and the degree of
goal congruence.
C. assessing the financial stability of the unit.
D. appraising the effectiveness of resource utilization within the unit or activity.

37. The operational auditor should evaluate the organization's overall planning
process to determine whether.

A. all plans, policies, and procedures are compatible with the organization's
objectives.
B. expected benefits do not exceed the cost of drawing them up.
C. each plan is developed or approved by senior management.
D. the premises for plans are based on historical data.

38. A governmental audit may extend beyond an examination leading to the


expression of an opinion on the fairness of financial presentation to include

Economy &
Program Results Compliance Efficiency
A. Yes Yes No
B. Yes Yes Yes
C. No Yes Yes
D. Yes No Yes

39. The purpose of governmental effectiveness or program results auditing is to


determine if the desired results of a program are being achieved. The first step in
conducting such an audit should be to:

A. evaluate the system used to measure results.


B. determine the time frame to be audited.
C. collect quantifiable data on the program's success or failure.
D. identify the legislative intent of the program being audited.

40. The major objective of the internal auditor‘s review of computer program testing
procedures in system development is to determine

A. the tested controls are cost effective.


B. significant errors could occur in processing date.
C. programmed tests will allow continuity of the operation tested.
D. a cost-benefit analysis of processing alternatives has been performed.

41. Which one of the following is not reason to conduct an operational audit?

A. Efficiency
B. Economy
C. Correctness of accounting procedures
D. Effectiveness

42. An operational audit resulted in a finding regarding the efficiency of operations.


Select the best approach to gain the auditee's cooperation in this situation.

A. Allow the auditee to participate in the development of recommendations for


improvement.
B. Emphasize the personal responsibility of the auditee management.
C. Document the adverse finding with a complete list of all operational
deficiencies.
D. Submit a draft copy of the final report to higher-level management.

43. To maximize independence, the director of internal auditing should report to the

A. audit committee
B. controller
C. chief financial officer
D. director of information systems.

44. Independence of the internal auditors would be least likely achieved if internal
auditors report to the?

A. President
B. Controller
C. Audit Committee
D. The Board of Directors

45. Lydia previously worked in accounts payable before transferring to internal audit.
She has been assigned to do some testing in the accounts payable because of
familiarity with that department. Which of the following is the best response to
this situation?

A. The assignment makes good sense because of her expertise.


B. She should not audit this function, since she may be reviewing some of her
own previous work.
C. The assignment should only be made if she works under a supervisor.
D. Lydia should have not transferred to internal audit since she previously
worked in operations.

46. In performing an operational audit, internal auditors are most likely to focus upon
which of the following attributes?

A. Controls relating to the adequacy of the financial statements.


B. All controls and not just those relating to financial statements.
C. Efficiency of operations.
D. Operations of the business including efficiency, effectiveness, internal control
and others.

47. The primary difference between on operational audit and a compliance audit is
which of the following?

A. An operational audit focuses on business efficiencies and effectiveness, while


a compliance audit focuses on whether laws and other requirements are being
followed.
B. An operational audit focuses on reliability of the financial statements, while a
compliance audit focuses on whether laws and other requirements are being
followed.
C. An operational audit focuses on business efficiencies and effectiveness, while
a compliance audit focuses on reliability of the financial statements.
D. A compliance audit focuses on business efficiencies and effectiveness, while
an operational audit focuses on whether laws and other requirements are being
followed.

48. What is the final step that should be undertaken in an operational audit?

A. Reporting of the findings


B. Follow-up procedures to determine if recommendations have been
implemented
C. Report on the organization's internal control and compliance with legal
requirements
D. An operational meeting with Board of Directors

49. Operational audit generally have been performed by internal auditors but may be
performed by independent accountants. A primary purpose of an operational
audit is to provide:
A. a means of assuring that internal controls are functioning as planned.
B. aid to the independent auditor, who is auditing the financial statements.
C. the results of internal examinations of financial and accounting matters to an
entity's top-level management.
D. a measure of management performance in meeting organizational goals.

50. Which of the following is generally considered one of the main ieasons why
internal auditors evaluate the effectiveness of their company's internal control
structure?

A. To determine whether all risks and exposures of the company have been
either reduced or eliminated.
B. To determine whether the established internal controls are functioning as
intended by management.
C. To determine the extent of reliance the internal auditors can place on the
controls in the process of evaluating the financial statements.
D. To reduce the amount of detailed testing for their external auditors.

51. Which of the following is not one of the three phases in an operational audit?

A. Planning
B. Review of the internal control structure
C. Evidence accumulation and evaluation
D. Reporting and follow-up

52. There are several sources that the operational auditor can utilize in developing
specific evaluation criteria. An area that is not considered a source is

A. generally accepted accounting principles.


B. historical performance, such as results from prior periods.
C. comparable performance. such as the data of comparable entities.
D. discussion and agreement between the management of the entity to be
audited, the operational auditor, and the entity or persons to whom the findings
will be reported.

53. An operational auditor is most likely to be concerned with whether o transaction


was

A. necessary.
B. reasonable.
C. properly approved.
D. properly supported with documentation.

54. When planning an operational audit, the auditor should consider the risk of

A. misleading financial statements.


B. inadequate internal controls.
C. inefficient operations.
D. lawsuits for inadequate auditing.

55. When performing an operational audit, the auditor would normally be concerned
with all of the following except

A. calculation of earnings per share.


B. investigation of budget variance.
C. follow up on inventory shortages.
D. reasons for idle equipment.

56. The benefits of an operational audit generally include all of following except

A. increased revenue.
B. increased reliability of the financial statements.
C. increases productivity.
D. decreased costs.

57. An operational audit report is not likely to be addressed to the

A. department supervisors.
B. audit committee.
C. stockholders.
D. top management.

58. In a compliance audit, an auditor is concerned with whether an entity's


transactions are in conformance with

A. management objectives.
B. board of director directives.
C. laws and regulations.
D. accepted business practices.

59. A major responsibility of internal auditing is to:

A. install sound accounting, financial, and operating controls at reasonable cost.


B. determine the extent of compliance with established policies, plans, and
procedures.
C. account for the company's assets and safeguard them from losses.
D. develop reliable management data.

60. In comparison to the independent auditor, an internal auditor is more likely to be


concerned with:

A. legal and regulatory compliance.


B. cost accounting procedures.
C. operational auditing.
D. internal control.

61. When an independent auditor decides that the work performed by internal
auditors may have a bearing on the nature, timing, and extent of planned audit
procedures, the independent auditor should evaluate the objectivity of the
internal auditors. Relative to objectivity, the independent auditor should

A. consider the organizational level to which internal auditors report.


B. review the quality control program in effect for the internal audit staff.
C. examine the quality of the internal audit reports.
D. consider the qualifications of the internal audit staff.
MODULE 13

OTHER PROFESSIONAL SERVICES

PSA preface, PSRE, PSRS, PSAE –BASED QUESTIONS

1. The primary standards for assurance engagement other than audits or reviews of
historical financial statements are the:

A. Accounting and Review Services.


B. Philippine Standard on Assurance Engagement.
C. Generally Accepted Auditing Standards.
D. Philippine Standards on Review Engagement.

2. The objective of a review of financial statements is

A. to enable the auditor to express an opinion whether the financial statements


are prepared, in all material respects, in accordance with Philippine financial
reporting standards.
B. for the auditor to carry out procedures of an audit nature to which the auditor
and the entity and any appropriate third parties have agreed and to report on
factual findings.
C. for the accountant to use accounting expertise, as opposed to auditing
expertise, to collect, classify: and summarize financial information.
D. to enable an auditor to state whether, on the basis of procedures which do not
provide all the evidence that would be required in an audit, anything has come to
the auditor's attention that causes the auditor to believe that the financial
statements are not prepared, in all material respects, in accordance with
Philippine financial reporting standards (negative assurance).

3. Performing inquiry and analytical procedures that provide the accountant with a
reasonable basis for expressing limited assurance that there are no material
modifications that should be made to the financial statements in order for them to
be in conformity with PFRS or with other comprehensive basis of accounting is
the definition of

A. compilation
B. audit
C. review
D. agreed-upon procedure

4. Engagement letter for review of financial statements least likely includes

A. the objective of the service being performed


B. the fact that the engagement cannot be relied upon to disclose errors, illegal
acts or other irregularities, for example, fraud 0r defalcations that may exist
C. a statement that an audit is not being performed and that an audit opinion will
not be expressed
D. the fact that because of the test nature and other inherent limitations of an
audit, together with the inherent limitations of any accounting and internal control
system, there is an unavoidable risk that even same material misstatements may
remain undiscovered

5. Before performing a review of a non-public entity's financial statements, an


accountant should

A. make inquiries of the management


B. apply analytical review procedures to identify unusual fluctuations
C obtain a sufficient level of knowledge about accounting principles and
practices in the industry wherein the entity operates
D. inquire as to whether the management has significantly omit disclosures in
the financial statements

6. In planning a review of financial statements, the auditor should obtain or update


his knowledge of the business. Which of the following is not one of this knowledge
of the business?

A. Entity's organization
B. Nature of entity's assets, liabilities, revenues and expenses
C. Accounting system
D. Internal control

7. Which of the following is not used as a basis by the auditor in determining the
specific nature, timing and extent of review procedures?

A. Assessed level of control risk


B. The extent to which a particular item is affected by management judgment
C. The materiality of transactions and account balances
D. Any knowledge acquired by carrying out a review of the financial statements
of prior periods.

8. Which statement is incorrect regarding procedures and evidence obtained in a


review engagement?

A. The auditor should apply his judgment in determining the specific nature,
timing and extent of review procedures.
B. The auditor should apply the same materiality considerations as would have
been applied had an audit opinion on the financial statements been expressed.
C. There is a greater risk that misstatements will not be detected in an audit than
in a review.
D. The judgment as to what is material is made by reference to the information
on which the auditor is reporting and the needs of those relying on that
information, not to the level of assurance provided.

9. Which of the following is least likely done by the auditor in conducting a review of
financial statements?

A. Study of the relationships of the elements of the financial statements


B. Comparison of the financial statements with those statements of prior periods
C. Comparison of the financial statements with anticipated results and financial
position
D. Comparison of inventory listing with physical inventory count

10. In a review of interim financial information of a publicly-held company, the CPA is


expected to have an understanding of all of the following except the:

A. industry in which the client operates


B. client's internal control structure
C. nature of the entity's organization
D. entity's accounting practices

11. Which of the following procedures is not included in a review of financial


statements of a nonpublic entity?

A. Inquiries of management
B. Inquiries regarding events subsequent to the balance sheet date
C. Any procedures designed to identify relationships among data that appear to
be unusual
D. Communicating any material weaknesses discovered during the study and
evaluation of internal accounting control

12. Which of the following is generally more important in a review than in a


compilation?

A. Determining the accounting basis on which the financial statements are to be


presented
B. Gaining familiarity with the industry's accounting principles and practices
C. Obtaining a signed engagement letter
D. Obtaining a signed representation letter

13. Which of the following procedures is ordinarily performed by a CPA in a review


engagement of a non-public entity?

A. Analytical procedures designed to test the accounting records by obtaining


corroborating evidential matter
B. Inquiries concerning entity's procedures summarizing transactions
C. Analytical review designed to internal control
D. Inquiries of the entity's legal counsel concerning contingent liabilities

14. Which of the following should the auditor perform in a review engagement?

A. Understand matters that are relevant to the financial statements


B. Understand the entity's internal control system
C. Observe the physical count of inventory
D. Inquire of legal counsel of pending litigations

15. The review of unaudited financial statements consists of:

A. infernal control evaluation and management representation


B. inquiry of management and documentation of internal controls
C. analytical procedures and compliance with laws and regulations
D. inquiry of management and analytical procedures

16. When performing a review of the financial statements of a non public entity the
CPA should:

A. obtain an understanding of internal control


B. inquire about actions taken at the meetings of stockholders and board of
directors
C. send letters of audit inquiry to attorneys
D. read the minutes of meetings of stockholder and board of directors

17. Which of the following is a major difference between a review and an audit of the
financial statements?

A. The scope of the procedures performed and the assurance provided.


B. The level of knowledge of professional standards needed to perform the
procedures.
C. The type of accounting used - reviews are typically on non-PFRS accounting,
while audits are based upon PFRS accounting.
D. The type of company involved in reviews may only be publicly-held.

18. Which of the following is not included in the scope, paragraph of a review report?

A. A statement that a review is limited primarily to inquiries and analytical


procedures.
B. A reference to Philippine Standard on Auditing applicable to review
engagement.
C. A statement that the review included an evaluation of reasonableness of
accounting estimates made by management.
D. A statement that an audit has not been performed.
19. Where do you find the f0llowing paragraph?

… Nothing has come to our attention that causes us to believe that the
accompanying financial statements are not presented fairly, in all material
respects in accordance with generally accepted accounting principles in the
Philippines.

A. Opinion paragraph of an auditor's report


B. Opinion paragraph of a review report
C. Negative assurance paragraph
D. Scope paragraph of a review report

20. In a review engagement, if there has been a material scope limitation, the auditor
should describe the limitation in the review report and either

A. express a qualification of the negative assurance or not issue any assurance


B. express a qualification of the negative assurance provided or give an adverse
statement that the financial statements are not presented fairly
C. express an adverse statement that the financial statements are not presented
fairly or the auditor does not issue any assurance
D. not modify the negative assurance or not issue an assurance

21. An accountant's standard report on a review of the financial statements of a


nonpublic entity should state that the accountant

A. does not express an opinion or any form of limited assurance on the financial
statements
B. is not aware of any material modifications that should be made to the financial
statements for them to conform with PFRS
C. obtained reasonable assurance about whether the financial statements are
free of material misstatement
D. examined evidence, on a test basis, supporting the amounts and disclosures
in the financial statements

22. A review report states that

A. a review includes assessing the accounting principles used and significant


estimates made by the management
B. a review includes examining on a test-basis.
C. the accountant is not aware of any material modifications that should be
made to the financial statements
D. the accountant does not express an opinion or any other form of assurance

23. The statement that the reviewer "is not aware of any material modification that
should be made to the financial statements in order for them to be in conformity
with PERS" is known as:
A. reasonable assurance
B. negligent performance
C. negative assurance
D. necessary ignorance

24. The professional accountants issued the following statement in their report: ...,
nothing came to our attention that caused us to believe that the accompanying
financial statements are not presented fairly,…

What is the nature of the report?

A. Special report on financial statements prepared under comprehensive basis


of accounting
B. Qualified audit report
C. Review report
D. Audit report-with limited reporting objective

25. The objective of an agreed upon-procedures engagement is for the auditor to:

A. carry out procedures of an audit nature to which the auditor and the entity and
any appropriate third parties have agreed and to report on factual findings
B. carry out procedures of a review nature to which the auditor and the entity
and any appropriate third parties have agreed and to report on factual findings
C. carry out procedures of a review nature and to express limited assurance
based on those agreed procedures
D. carry out procedures of an audit nature and to express limited assurance

26. According to PSRS, engagement to perform agreed-upon procedures, the


procedures employed in doing agreed-upon procedures are:

A. designed to enable the accountant to express a limited assurance


B. designed to enable the accountant to express a negative assurance
C. designed to enable the accountant to provide the identified user(s) factual
findings
D. less extensive than compilation procedures but more extensive than review
procedures

27. Rivera, CPA, has significant indirect financial interest on Mother Corporation.
Mother Corporation engaged Rivera to apply agreed upon procedures on
accounts .receivable and thereafter submits a Report of Factual Findings to
Discount Finance. According to Philippine Standards on Auditing that applies to
this engagement, Rivera:

A. should decline the engagement because of his lack of independence


B. should convince Mother Corporation to change the engagement to
compilation due to his lack of independence
C. can accept the engagement, issue the Report of Factual Finding and state in
the report his lack of independence
D. perform agreed-upon procedures and withhold the findings due to his lack of
independence

28. Which of the following ethical principles governing the auditor's professional
responsibilities for agreed-upon procedures engagement is not required of
auditors?

A. Technical standards
B. Confidentiality
C. Integrity
D. Independence

29. Which of the following is incorrect about agreed-upon procedures engagement?

A. An engagement to perform agreed-upon procedures may involve the auditor


in performing certain procedures concerning individual items of financial data.
B. Users of the agreed-upon procedures report assess for themselves the
procedures and findings reported by the auditor and draw their conclusion from
the auditor's work.
C. The auditor should be independent of the financial data or financial
statements where agreed procedures have to be applied.
D. The report is restricted to those parties that have agreed to the procedures to
be preformed.

30. Which statement is incorrect regarding agreed-upon procedures?

A. Users of the report assess for themselves the procedures and findings
reported by the auditor and draw their own- conclusions from the auditor's work.
B. The report is restricted to those parties that have agreed to the procedures to
be performed since others, unaware of the reasons for the procedures, may
misinterpret the results.
C. The auditor should conduct an agreed-upon procedures engagement in
accordance with PSA and the terms of the engagement.
D. Where the auditor is not independent, a statement to that effect need not be
made in the report of factual findings.

31. Matters to be agreed in an agreed-upon procedures engagement include the


following, except:

A. stated purpose of the engagement


B. limitations on distribution of the report of factual findings
C. anticipated form of the report and the level of assurance to be provided
D. nature, timing and extent of the specific procedures to be applied

32. The following procedures may be performed by CPAs engagement.

I. Consideration of internal control


II. Observation
III. Inquiry and analysis
IV. Inspection
V. Confirmation
VI. Obtaining management representation letter

Which of the foregoing may be performed by the auditor in an agreed-upon


procedures engagement?
A. II and VI only
B. I, II and VI only
C. II, III, IV and V only
D. II, Ill, IV and VI only

33. The report on an agreed-upon procedures engagement needs to describe the


purpose and the agreed-upon procedures on the engagement in sufficient detail
to enable the reader to understand the nature and the extent of the work
performed. The report of actual findings should not contain:

A. addressee (ordinarily the client who engaged the auditor to perform the
agreed-upon procedures)
B. identification of the purpose for which the agreed-upon procedures were
performed
C. a description of the auditor's .factual findings including sufficient details of
errors and exceptions found
D. a statement that the procedures performed constitute an audit and, as such,
an opinion is expressed

34. A report for an agreed-upon procedure ordinarily includes:

A. Findings--Yes; Negative Assurance-- Yes


B. Findings--Yes; Negative Assurance—No
C. Findings--No; Negative Assurance—Yes
D. Findings--No; Negative Assurance—No

35. Which of the following report is for limited distribution?

A. Review Report
B. Compilation Report
C. Report of Factual Findings
D. Audit Report
36. An accountant may accept an engagement to apply agreed-upon procedures to
prospective financial statements provided that:

A. distribution of the report is limited to the specified parties involved


B. the prospective financial statements are also examined
C. the responsibility for the adequacy of the procedures performed is taken by
the accountant
D. negative assurance is expressed on the prospective financial statements
taken as a whole

37. Distribution of a report is always restricted when:

A. negative assurance is given


B. there is a positive expression of opinion
C. agreed-upon procedures have been performed
D. a review has been performed

38. Which of the following would not be appropriate to a report on an engagement to


apply agreed-upon procedures to specified financial statement items?

A. indicate the intended distribution of the report


B. Provide an opinion on the specified elements, accounts, or items
C. Enumerate the procedures performed
D. State that the report relates only to the elements, accounts, or items specified

39. Which of the following is not appropriate for the accountant's report on the results
of applying agreed-upon procedures to prospective financial statements?

A. Express an opinion on the results of applying the agreed-upon procedures.


B. Indicate the prospective financial statements reported on.
C. Specify that the use of the report is limited to certain user(s).
D. Indicate that the prospective results may not be achieved.

40. The report on agreed-upon procedures -engagement needs to describe the


purpose and the procedures that have been agreed upon in sufficient details. The
report should appropriately include Iii "title"

A. Report of Agreed-Upon Procedures.


B. Report of Factual Findings.
C. Report of agreed Procedures and Finding.
D. CPA's Report of Agreed-Upon Procedures.

41. Compilation is an example of which one of the following types of services?

A. Auditing
B. Review
C. Consulting
D. Accounting

42. A CPA has been, engaged to compile financial statements for a nonpublic client.
Which the following statements best describes this engagement?

A. The CPA must perform the basic accepted auditing procedures necessary to
determine that the statements are in conformity with PFRS
B. The CPA is performing an accounting services rather than an examination of
financial statements
C. The financial statements are representation of both management and the
CPA.
D. The CPA may prepare the statements from the books but may not assist in
adjusting and closing the books

43. A CPA who is not independent may issue a

A. compilation report
B. compilation report and review report
C. comfort letter
D. report of any type

44. Which statement is incorrect regarding compilation engagement?

A. This ordinarily entails reducing detailed data to a manageable and


understandable form without a requirement to test the assertions underlying that
information.
B. The procedures .employed are designed to enable the accountant to express
limited assurance on the financial information.
C. Users of the compiled financial information derive some benefit as a result of
the accountant's involvement because the service has been performed with
professional competence and due care.
D. In all circumstances when an accountant's name is associated with financial
information compiled by the accountant, the accountant should issue a report.

45. When compiling financial information, the accountant is ordinarily required to

A. obtain a general knowledge of the business and operations of the entity


B. make any inquiries of management to assess the reliability and completeness
of the information provided
C. verify any matters
D. verify any explanations

46. Indicate whether the following procedures performed in an audit engagement are
also required when performing related services.
A B C D
• Agreeing on the terms of
engagement Yes Yes Yes No
• Engagement planning Yes Yes Yes No
• Documentation Yes Yes No No
• Issuance of report Yes No No No

47. Which statement is incorrect regarding the procedures -performed in a


compilation engagement?

A. If the accountant becomes aware that information supplied by the


management is incorrect, incomplete, or otherwise unsatisfactory, the accountant
should consider performing appropriate procedures and request the management
to provide additional information.
B. The accountant should read the compiled in and consider whether it appears
to be appropriate in form and fro from obvious material misstatements.
C. The Philippine financial reporting standards and any known departures
thereof should be disclosed within the financial information, and their effects
should be quantified.
D. The accountant should obtain an acknowledgment from the management of
its responsibility for the appropriate presentation of the financial information and of
its approval of the financial information.

48. If the accountant becomes aware of material misstatements, the accountant


should try to agree appropriate amendments with the entity. If such amendments
are not made and the financial information is considered to be misleading, the
accountant should

A. do nothing
B. withdraw from the engagement
C. issue a qualified or adverse opinion
D. issue a negative assurance

49. In performing a compilation of financial-statements, the accountant feels that a


modification of the standard report is not adequate to indicate deficiencies in the
financial statements taken as a whole, and the client is not willing to correct the
deficiencies. The accountant should therefore

A. perform a review of the financial statements


B. issue a special report
C. withdraw from the engagement
D. express an adverse opinion

50. Reports on compilation engagements should contain the following, except:


A. a statement that the engagement was performed in accordance with the
PSAs applicable to compilation engagements
B. identification of the financial information indicating that it is based on
information provided by the management
C. a statement that the management is responsible for the financial information
compiled by the accountant
D. a statement that the accountant does not express an opinion but expresses
only a limited assurance on the financial statements

51. A compilation report should include all of the following except:

A. a statement that the compilation has been performed in accordance with the
Philippine Standards on Related Services applicable to compilation
B. a statement that the financial statements are the representation of the
management
C. a statement that adequate disclosure has been made concerning accounting
policy and practice
D. a statement that the financial statement have not been audited or reviewed

52. Which of the following statements should not be included in an accountant's


standard report based on the compilation of client's financial statements?

A. The compilation was performed in accordance with the applicable Philippine


Standards on Related Services.
B. The accountant has not audited or reviewed the financial statements.
C. The accountant expresses only a limited assurance on the financial
statements.
D. The compilation is limited to representing in the form of financial statements,
information that is the representation of management.

53. The level of assurance that is provided by the CPA on a compilation report is

A. low
B. high
C. moderate
D. none

54. An accountant's compilation report should state that

A. a compilation includes assessing the accounting principles used and


significant estimates made
B. a compilation is substantially less in scope than an audit
C. the accountant is not aware of any material modifications that should be
made to the financial statements
D. the accountant does not express an opinion or any form of assurance on the
financial statements
55. An accountant's compilation report on a financial forecast should include a
statement that the

A. compilation does not include an evaluation of the support of the assumptions


underlying, the forecast
B. hypothetical assumptions used in the forecast are reasonable
C. range of assumptions selected is one in which one end of the range is less
likely to occur than the other
D. prospective statements are limited to presenting, in the form of a forecast,
information that is the accountant's representation

56. On each page of the financial information or on the front of the complete set of
financial statements, the financial information compiled by the accountant should
contain a reference such as

A. "Unaudited"
B. "Compiled without Audit or Review"
C. "Refer to Compilation Report"
D. Any of the given choices

57. Each page of the financial statements compiled by an accountant should include a
reference such as

A. see accompanying accountant's footnotes


B. unaudited, see accountant's disclaimer
C. see accountant‘s compilation report
D. subject to compilation restriction

58. In an engagement to examine prospective financial information, the auditor should


obtain sufficient appropriate evidence as to whether:

1. The management's best-estimate assumptions on which the prospective


financial information is based are not unreasonable and, in the case of
hypothetical assumptions, they are consistent with the purpose of, the information.
2. The prospective financial information is properly prepared on the basis of the
assumptions.
3. The prospective financial information is properly presented and all material
assumptions are adequately disclosed, including a clear indication as to whether
they are best-estimate or hypothetical assumptions.
4. The financial information is prepared on a consistent basis with historical
financial statements, using appropriate accounting principles.

A. I, II, Ill and IV


B. I, II and III
C. I and ll
D. I, II and IV
59. An examination of a financial forecast is a professional service that involves

A. compiling or assembling a financial forecast that is based on management


assumptions
B. limiting the distribution of the accountant's report to the management and the
board of directors
C. assuming a responsibility to update the management on key events for one
year after the report's date
D. evaluating the preparation of a financial forecast and the support underlying
management's assumptions

60. Forecast means a

A. financial information based on assumptions about events that may occur in


the future and possible actions by an entity
B. prospective financial information prepared on the basis of assumptions as to
future events which the management expects to take place and the actions that
the management is expected to take as of the date the information is prepared
(best-estimate assumptions).
C. prospective financial information prepared on the basis of hypothetical
assumptions about future events and management actions which are not
necessarily expected to take place.
D. prospective financial information prepared on the basis of a mixture of best-
estimate and hypothetical assumptions.

61. What is meant by a financial forecast under Philippine Standard on Assurance


Engagements?

A. A prospective, financial statement that predicts an expected financial position,


results of operations, and cash flows.
B. A prospective financial statement that is prepared on the basis of
assumptions as to future events which the management expects to take place and
the actions that the management is expected to take as of the date the
information is prepared.
C. A prospective financial statement that presents •an entity's expected financial
position, results of operations, and cash flows based on one or more hypothetical
assumptions.
D. A prospective financial statement that predicts an entity's expected financial
position, results of operations, and cash flows based on one or more hypothetical
assumptions.

62. Given one of more hypothetical assumptions, a responsible party may prepare, to
the best of its knowledge and belief, an entity's expected financial position, results
of operations, and changes in financial position. Such prospective financial
statements are most commonly known as
A. special purpose financial statements
B. financial projections
C. partial presentations
D. financial forecasts

63. Prospective financial information can include financial statements or one or more
elements of financial statements and may be prepared for distribution to third
parties in

A. a prospectus to provide potential investors with infc3rmation about future


expectations
B. an annual report to provide information to shareholders, regulatory bodies
and other interested parties
C. a document for the information of lenders which may include, for example,
cash flow forecasts
D. Any of the given choices

64. When prospective financial statements are prepared, the "responsible party" is
usually the

A. management
B. CPA who examines them
C. government entity that requires them
D. audit committee

65. Which statement is incorrect regarding the examination of prospective financial


information?

A. The auditor should not accept or should withdraw from an engagement when
the assumptions are clearly unrealistic or when the auditor believes that the
prospective financial information will be inappropriate for its intended use.
B. The auditor and the client should agree on the terms of the engagement.
C. The auditor should obtain a sufficient level of knowledge of the business to be
able to evaluate whether all significant assumptions required for the preparation of
the prospective financial information have been identified.
D. The auditors need not obtain written representations from the management
regarding the intended use of the prospective financial information, the
completeness of significant management assumptions and the management's
acceptance of its responsibility for the prospective financial information.

66. Prospective financial statements are for

A. general use
B. limited use only
C. either general or limited use
D. use by internal management only

67. When an accountant issues on examination report on a financial forecast, the


report ordinarily should:

A. state that the forecast is presented in conformity with the generally accepted
accounting principles in the Philippines
B. provide an explanation of the differences between an examination and an
audit
C. state that the accountant is responsible for events and circumstances not to
exceed one year after the report's date
D. disclaim an opinion on whether the assumptions provide a reasonable basis
for the projection

68. An auditor should not issue a report on

A. the achievability of forecasts


B. internal control
C. management performance
D. quarterly financial information

69. When the auditor believes that, the presentation and disclosure of the prospective
financial information is not adequate, the auditor should

A. express a qualified or adverse opinion in the report on the prospective


financial information
B. withdraw from the engagement
C. disclaim an opinion in the report on the prospective financial information
D. either modify the opinion or withdraw from the engagement

70. When an accountant examines financial forecast that fails to disclose several
significant assumptions used to prepare the forecast, the accountant should
describe the assumptions in the accountant's report and issue a (n)

A. qualified in a negative-assurance form of opinion


B. unqualified opinion with a separate explanatory paragraph
C. disclaimer of opinion
D. qualified or adverse opinion

71. Assurance services are independent professional services that improve the
quality of information or its

A. context
B. profitability
C. reliability
D. sufficiency
72. When an accountant performs more than one level of service, generally should
issue a report that is appropriate for:

A. the lowest level of service rendered


B. a compilation engagement
C. the highest level of service rendered
D. a review engagement
QUIZZERS

1. An auditor who was engaged to perform an examination of the financial


statements of a non-public entity has been asked by the client to refrain from
performing various audit procedures and changed the nature of the engagement
to a review of financial statements. The client made the request because of the
significant cost of completing the examination. The auditor would most likely

A. qualify the auditor's report and refer to the scope limitation


B. view the request as an indication of possible irregularity
C. complete the examination in progress
D. grant the client request

2. The degree of certainty that the practitioner has attained and wishes to convey is
a (n)

A. assertion
B. conveyance
C. assurance
D. declaration

3. A practitioner who is performing an assurance engagement may ordinarily report


upon the subject matter or a (n):

A. assertion about the subject matter


B. control criterion
C. reliability statement
D. suitable criteria

4. Which of the following types of assurance engagements is similar to an assurance


provided by an audit of historical financial statements?

A. Assessment
B. Detailed review
C. Evaluation
D. Examination

5. Which of the following statements is true concerning interim financial information?


A. An audit of interim financial information is required for SEC-registered
companies.
B. The accountant needs to obtain sufficient knowledge of the entity's business
and industry before undertaking an engagement on interim financial information.
C. An accountant may not report on financial information presented separately
from the audited financial statements.
D. Interim financial information may not be included as part of a note to audited
financial statements.
6. What type of assurance is provided by an auditor when he issues a review report?

A. Limited
B. Neutral
C. None
D. Positive

7. The circumstance most likely to make it impossible for a practitioner to issue a


review report is when the

A. criteria are only available to specified users


B. subject matter contains a departure from the criteria
C. company faces a going concern uncertainty
D. scope of the engag6ment has been significantly limited

8. The objective of a review of interim financial information is to provide the CPA with
a basis for

A. expressing a limited opinion that the financial information is presented in


conformity with generally accepted accounting principles
B. expressing a compilation opinion on the financial statements
C. reporting whether material modifications should be made to such information
to make it conform with generally accepted accounting principles
D. reporting limited assurance to the board of directors only

9. Which of the following procedures is not appropriate to a review of interim


financial information?

A. Confirm cash balances with all banks and depositories.


B. Make inquiries concerning the accounting system and any significant changes
in the internal control structure.
C. Perform analytical procedures to identify and provide a basis for inquiry about
relationships and individual items that appear unusual.
D. Inquire about the actions taken on meetings of stockholders, the board of
directors, and committees of the board.

10. In a review engagement, the independent accountant's procedures include:

A. examining bank reconciliation


B. confirming accounts receivable with debtors
C. reading the financial statements to consider whether they appear to conform
to PFRS
D. obtaining a letter of audit inquiry from all legal counsels
11. Which of the following procedures is not normally performed by the accountant in
a review engagement of a non-public entity?

A. Communicating any material weaknesses discovered during the study and


evaluation of internal accounting control.
B. Reading the financial statements to determine whether they are in conformity
with PFRS.
C. Writing an engagement letter to establish an understanding regarding the
services to be performed.
D. Issuing report stating that the review was performed.

12. Performing inquiry and analytical review procedures is the primary basis for an
accountant to issue

A. compilation reports
B. management advisory services report
C. review report
D. audit report

13. The use of negative assurance in an audit report on financial statements

A. is a violation of the standards of reporting


B. is encouraged by PSAs
C. helps in clarifying the degree of responsibility being assumed by the auditor
D. is properly located in the opinion paragraph of the unqualified report

14. Claire, CPA, was engaged to review the financial statements of Emir Company, a
nonpublic entity. Evidence came to Claire's attention that indicated a substantial
doubt as to Emir's ability to continue as a going concern. The principal conditions
and events that caused the substantial doubt have been fully disclosed in the
notes to Emir's financial statements. Which of the following statements best
describes Claire's reporting responsibility concerning this matter?

A. Claire is not required to modify the accountant's review report


B. Claire is not permitted to modify the accountant's review report
C. Claire should issue an 'accountant's compilation report instead of a review
report
D. Claire should express a qualified opinion in her review report

15. The objective of a review of the interim financial information: of a public company
is to

A. provide the accountant with a basis for expressing an opinion


B. estimate the accuracy of the financial statements based on limited tests of
accounting records
C. provide the accountant with a basis for reporting to the board of directors or
shareholders
D. obtain corroborating evidence through inspection, observation, and
confirmation.

16. If requested to perform a review engagement for a non-public entity for which the
accountant has immaterial direct financial interest, the accountant is:

A. not independent and, therefore, may issue a review report but not an auditor's
opinion
B. not independent and, therefore, may not issue a review report
C. not independent and, therefore, may not be associated with the financial
statements
D. independent because the financial interest is immaterial

17. An accountant is requested to issue a review report on the balance sheet of a


non-public entity but not on the other basic financial statements. The accountant
may not do so

A. because compliance to this request will result to an incomplete review


B. because compliance to this, request is a violation of ethical standards
C. if the scope has been restricted
D. if the review discloses material departure from PFRS

18. When providing a limited assurance that the financial statements of a non-public
entity requires no material modifications in order to present them in accordance
with PFRS, the accountant should

A. understand the system of accounting controls


B. test the accounting records that identify inconsistencies with the prior year's
financial statements
C. understand the accounting principles in the industry wherein the business
entity operates
D. develop an audit program

19. The statement that "nothing came to our attention which would indicate that these
statements are not fairly presented" expresses which of the following?

A. Disclaimer of opinion
B. Negative assurance
C. Negative confirmation
D. Piecemeal opinion

20. Which of the following would not be included in a CPA's report based upon a
review of the financial statements?
A. A statement that the review is in accordance with PSAs.
B. A statement that all information included in the financial statements is the
representation of management.
C. A statement describing the principal procedures performed.
D. A statement describing the auditor's conclusions based on the results of the
review.

21. In a review service where the client failed to follow PFRS, the accountant is

A. not required to determine the effect of a departure if the management has not
done so, but that fact must be disclosed in the report
B. required to determine the effect of a departure if the management has not
done so and that fact must be disclosed in the report
C. not required to determine the effect of a departure if management has not
done so and that fact need not be disclosed in the report
D. required to determine the effect of a departure if management has not done
so but, that fact heed not be disclosed in the report

22. The objective of a review of interim financial information of a public entity is to


provide an accountant with a basis for reporting whether

A. material modification in the financial statements should be made to conform


with PFRS
B. a reasonable basis exists for, expressing an updated opinion regarding the
financial statements that were previously audited
C. condensed financial statements should be included in a registration
statement.
D. the financial statements are presented fairly in conformity with PFRS

23. An accountant who reviews the financial statements should issue a report stating
that a review

A. is substantially less in scope than an audit


B. provides negative assurance that the internal control is functioning as
designed
C. provides only a limited assurance that the financial statements are fairly
presented
D. is substantially more in scope than a compilation

24. In a review engagement, if the CPA believes that the financial statements lack a
material disclosure that the management refuses to Include, the CPA should

A. issue a qualified opinion


B. issue an adverse opinion
C. express only limited assurance
D. disclose this departure in a separate paragraph
25. Which of the following procedures would normally be included in the review
engagements?

A. Preparing a bank transfer schedule


B. Inquiring about related party transactions
C. Assessing the internal control structure
D. Perform cut-off tests

26. Which of the following procedures is ordinarily performed by CPA in a review


engagement of a non-public entity?

A. Verify changes in key account balances


B. Read the minutes of board of directors' meeting
C. Inspect the open purchase order file
D. Search for unrecorded liabilities

27. Before an independent CPA agrees to a change from an audit engagement to a


review engagement, he should consider the

I. Additional audit effort necessary to complete the engagement


II. Reason for the change in the terms of the engagement

A. Yes, Yes
B. Yes, No
C. No; Yes
D. No, No

28. If the CPA is not familiar with the specialized industry accounting principles but
plans to obtain certain level of knowledge, which of the following engagements
may be accepted?

I. Compilation
II. Review
III. Audit

A. I only
B. I and II only
C. All of the them
D. None of them

29. An accountant should perform analytical procedures in engagement to do

I. Audit
II. Review
III. Compilation

A. Yes, Yes, Yes


B. No, Yes, No
C. Yes, Yes, No
D. Yes, No, No

30. Which of the following would not be included in a review engagement?

A. Obtaining a representation letter.


B. Considering whether financial statements conform with PFRS
C. Assessing control risk
D. Inquiring about subsequent events

31. Which statement is incorrect regarding the general principles on a review


engagement?

A. The auditor is not required to comply with the "Code of Professional Ethics for
Certified Public Accountants" promulgated by the Board of Accountancy.
B. The auditor should conduct a review in accordance with PSRE 2400.
C. The auditor should plan and perform the review with an attitude of
professional skepticism, recognizing that circumstances may exist cause the
financial statements to be materially misstated.
D. For the purpose of expressing a negative assurance in the review report, the
auditor should obtain sufficient appropriate evidence primarily through inquiry and
analytical procedures to be able to draw conclusions.

32. Which of the following is required to be performed in an audit but not in a review
engagement?

A. Complying with the Code of Professional Ethics for Certified Public


Accountants
B. Planning the engagement
C. Agreeing on the terms of engagement
D. Studying and evaluating internal control structure

33. An auditor's report would be designated as a special report when it is issued in


connection with which of the following set of financial statements?

A. Financial statements for an interim period that are subjected to a limited


review
B. Financial statements that are prepared in accordance with a comprehensive
basis of accounting other than PFRS
C. Financial statements that purport to be in accordance with PFRS but do not
include a statement, of cash flows
D. Financial statements that are unaudited and are prepared from a client's
accounting records

34. A CPA's report on applying agreed-upon procedures to prospective financial


statements always includes a (n):

A. disclaimer of opinion
B. adverse opinion
C. restrictions on its distribution
D. unqualified opinion

35. Clients sometimes engage auditors to perform a specified set of procedures


concerning an element of a financial statement. This type of, engagement is
called

A. individual account engagement


B. agreed-upon procedures engagement
C. assurance service
D. compliance audit

36. A CPA is not required to comply with the "Code of Professional Ethics for Certified
Public Accountants" when performing a(n):

A. review
B. agreed-upon procedures
C. compilation
D. None of the choices given

37. An accountant's compilation report should be dated as of the date of

A. completion of fieldwork
B. completion of the engagement
C. transmittal of the compilation report
D. the latest subsequent event referred to in the notes to the financial
statements.

38. Which of the following would not be considered an attestation engagement?

A. A compilation of financial statements


B. A letter to an underwriter
C. A report on the application of an accounting principle
D. A report on financial statements that are prepared on a comprehensive basis
of accounting other than PFRS
39. An accountant has been asked to compile the financial statements of a nonpublic
company that omit substantially all the disclosures required by generally accepted
accounting principles. The accountant may issue a compilation report if

A. the report indicates the lack of disclosures


B. the absence of disclosure is not, to the CPA‘s knowledge intended to mislead
the users of the financial statements
C. the financial statements are intended primarily for management purposes only
D. all of the given conditions are met

40. You own Garter, Inc., which manufactures wooden tables. You need to hire some
accountants to prepare your monthly financial statements. The preparation of your
financial statements is referred to as a(n):

A. audit
B. compilation
C. review
D. special report

41. A compilation report is not required when compiled financial statements are
expected to be used by:

A. management only
B. management and third parties
C. third parties only
D. a compilation report is required whenever financial statements are compiled.

42. Which of the following is not an assurance form of report?

A. Compliance
B. Compilation
C. Examination
D. Review

43. Which of the following procedures is normally performed in connection with a


compilation engagement?

A. Making a risk assessment


B. Making inquiries of management concerning actions taken at the board
meetings
C. Applying analytical review procedures
D. Reading the financial statements for obvious mistakes in the application of
accounting principles

44. An accountant who is not independent may issue a


A. compilation report
B. review report
C. comfort letter
D. qualified opinion

45. When a CPA compiles a non-public entity's financial statements that omit
substantially all disclosures required by PFRS; the CPA should indicate in the
compilation report that the financial statements are

A. restricted for internal use only


B. not to be given to financial institutions for the purpose of obtaining credit
C. compiled in conformity with other comprehensive basis of accounting
D. not designed for those who are not informed of the omitted disclosures

46. When an independent CPA assists in preparing the financial statements of a


publicly-held entity but has not audited or reviewed them, the CPA should issue a
disclaimer of opinion. In such situations, the CPA has no responsibility to apply
any procedures beyond

A. documenting that the internal control is not being relied on


B. reading the financial statements for obvious material misstatements
C. ascertaining whether the financial statements are in conformity with PFRS
D. determining whether management has elected to omit substantially all the
required disclosures

47. When reporting on financial statements prepared on a comprehensive basis of


accounting other than generally accepted accounting principles, the independent
auditor should include in the report a paragraph that

A. states that the financial statements are not intended to be in conformity with
generally accepted accounting principles
B. states that the financial statements are not examined in accordance with
generally accepted auditing standards
C. refers to the authoritative pronouncements that explain, the, comprehensive
basis of accounting being used
D. justifies the comprehensive basis of accounting being used.

48. Which of the following is not a type of special report?

A. A report for a company that uses the cash basis of accounting


B. A report on a financial presentation prepared in compliance with the forms of
a debt agreement
C. A report for the board of directors on the company's internal control.
D. A report with an opinion on whether a company's accounts receivable and
cash follow PFRS.
49. A comfort letter is typically sent to whom and for what reason?

A. Banks when a company is applying for a loan


B. The Board of Directors of a company on a report on internal control
C. The underwriters of a company's securities to assist them in their reasonable
investigation of a registration statement
D. Company management indicating the types of procedures performed during
the audit and general findings

50. Which statement is correct regarding report on a component of financial


statements?

A. This type of engagement may be undertaken as a separate engagement or in


conjunction with an audit of the entity's financial statements.
B. In determining the scope of the engagement, the auditor need not consider
those financial statement items that are interrelated and which could materially
affect the information on which the audit opinion is to be expressed.
C. The auditor's examination will ordinarily be less extensive than if the same
component were to be audited in connection with a report on the entire financial
statements.
D. When an adverse opinion or disclaimer of opinion on the entire financial
statements has been expressed/the auditor Mc& report on components of the
financial statements even if those components are so extensive as to constitute, a
major portion of the financial statements.

51. Which statement is incorrect regarding report on compliance with contractual


agreements?

A. The auditor must not be requested to report on an entity's compliance with


certain aspects of contractual agreements, such as bond indentures or loan
agreements.
B. Engagements to express an opinion as to an entity's compliance with
contractual agreements should be undertaken only when the overall aspects of
compliance relate to accounting and financial matters within the scope of the
auditor's professional competence.
C. When there are particular matters forming part of the engagement that are
outside the auditor's expertise, the auditor would consider using the work of an
expert.
D. The report should state whether, in the auditor's opinion, the entity has
complied with the particular provisions of the agreement.

52. Which of the following statements is incorrect regarding report on summarized


financial statements?
A. Unless the auditor has expressed an audit opinion on the financial statements
from which the summarized financial statements were derived, the auditor should
not report on summarized financial statements.
B. Summarized financial statements are presented in considerably less detail
than annual audited financial statements.
C. Summarized financial statements need to be appropriately titled in order to
identify the audited financial statements from which they have been derived.
D. Summarized financial statements contain all the information required by the
financial reporting framework used for the annual audited financial statements.

53. The auditor's report on summarized financial statements least likely include

A. an identification of the audited financial statements from which the


summarized financial statements were derived
B. a reference to the date of the audit report on the general-purpose financial
statements and the type of opinion given in that report
C. an opinion as to whether the information in the summarized financial
statements is presented fairly, in all material respects
D. a statement which indicates that for a better understanding of an entity's
financial performance and position and of the scope of the audit performed, the
summarized financial statements should be read in conjunction with the general-
purpose financial statements and the audit report thereon

54. Which of the following best describes the auditor's responsibility for "other
information" that, is included in the annual report to stockholders which contains
financial statements and the auditor's report?

A. The auditor has no obligation to read the "other information"


B. The auditor has no obligation to corroborate the "other information" but should
read "other information" to determine whether it is materially inconsistent with the
financial statements
C. The auditor should extend the examination to the extent necessary to verify
the "other information"
D. The auditor must modify the auditor's report to state that the "other
information" is not audited or "not covered by the auditor's report"

55. Comfort letters are ordinarily signed by the

A. independent auditor
B. client
C. client's lawyer
D. internal auditor

56. An auditor has been engaged to audit a set of financial statements that were
prepared on a cash basis. The auditor
A. must ascertain that there is a proper disclosure of the fact that the cash basis
of accounting has been used, the general nature of material items omitted, and
the net effect of the omissions
B. may not be associated with statements that are not in accordance with PFRS
C. must render a qualified report explaining the departure from PFRS in the
opinion paragraph
D. must restate the financial statements on an accrual basis and then issue the
standard report.

57. Which of the following statements concerning prospective financial statements is


correct?

A. Only a financial forecast would normally be appropriate for limited use


B. Only a financial projection would normally be appropriate for general use
C. Any type of prospective financial statements would normally be appropriate
for limited use
D. Any type of prospective financial statements would normally be appropriate
for general use

58. Accepting an engagement to examine an entity's financial projection most likely


would be appropriate if the projection were to be distributed to

A. all employees who work for the entity


B. potential stockholders who request a prospectus or a registration statement
C. a bank with which the entity is negotiating for a loan
D. all stockholders of record as of the report date

59. Which of the following services is not normally performed in connection with
prospective financial statements?

A. Examination
B. Review
C. Agreed-upon procedures
D. Compilation

60. If a CPA has both compiled and reviewed the financial statements of a nonpublic
entity the CPA should issue:

A. a compilation report only


B. both a review and compilation report
C. a combination review and compilation report
D. a review report only

61. Which of the following represents the highest to lowest level of auditors in the
assurance or even no assurance provided by auditors in the performance of the
engagement?
A. An audit; a compilation; a review.
B. A compilation; a review; an audit.
C. A review; an audit; a compilation.
D. An audit; a review; a compilation.

62. Which statement is incorrect regarding special purpose audit engagements?

A. Before undertaking a special purpose audit engagement, the auditor should


ensure that there is agreement with the client as to the exact nature of the
engagement and the form and content of the report to be issued.
B. To avoid the possibility of the auditor's report being used for purposes for
which it was not intended, the auditor may wish to indicate in the report the
purpose for which the report is prepared and any restrictions on its distribution
and use.
C. When requested to report in a prescribed format, the auditor should consider
the substance and wording of the prescribed report.
D. The auditor need not consider whether any significant interpretations on an
agreement on which the financial information is based are clearly disclosed in the
financial information.

63. A comprehensive basis of accounting comprises a set of criteria used in preparing


financial statements which applies to all material items and which has substantial
support. Other comprehensive financial reporting frameworks may include the
following, except

A. a conglomeration of accounting conventions devised to suit individual


preference
B. that one used by an entity to prepare its income tax return
C. the cash receipts and disbursements basis of accounting
D. the financial reporting provisions of a government regulatory agency

64. The CPA is asked to audit financial statements prepared on a modified cash
basis. This is acceptable provided the CPA

A. converts the financial statement to an accrual basis before rendering an audit


report
B. qualifies the audit opinion due to a departure from PFRS
C. issues an adverse opinion
D. states clearly in the audit report that fairness was evaluated within the
framework of the other basis rather than PF
SIMULATED EXAMINATION I

1. The subject matter of auditing consists of:

A. Assertions.
B. Established criteria.
C. Evidence.
D. Written reports.

2. A typical objective of an operational audit is for the auditors to

A. determine whether the financial statements fairly present the entity's operations.
B. evaluate the feasibility of attaining the entity's operational objectives.
C. make recommendations for improving performance.
D. report on the entity's relative success in attaining its profit maximization.

3. Which one of the following best describes the assurance process?

A. Proving the accuracy of the books and records


B. Gathering evidence about specific and known assertions
C: Assisting management in the successful operations of the company
D. Assembling and filing tax returns and related supplemental information

4. Which one of the following is an example of management expectations for


independent auditors?

A. An expert providing a written communication as the product of the engagement


B. Individuals who perform day-to-day accounting functions on behalf of the company
C. An active participant in management decision making
D. An internal source of expertise on financial and other matters

5. Public accountants are not prohibited from providing which of the following services
for public audit clients
A. audits of the effectiveness of internal controls
B. the function of the internal auditors
C. selection and implementation of an accounting information system
D. quarterly financial statement bookkeeping

6 Assurance services involve allot the following except:

A. Improving the quality of information for decision purposes.


B. Improving the quality of the decision model used
C. Improving the relevance of information.
D. Implementing a system that improves the processing of information.

7. In forming an opinion on the financial statements.


A. the auditor should evaluate the conclusions drawn from the audit evidence obtained
during the course of the audit
B. the auditor evaluates whether there is a reasonable assurance about whether the
financial statements are free from any misstatements
C. the auditor evaluates whether sufficiently appropriate audit evidence has been
obtained to eliminate the risk of material misstatements
D. the auditor verifies that all errors that misstate the financial statements have been
corrected by the client

8. If the auditor encounters circumstances that lead him to conclude that compliance
with a specific requirement results to financial statements that are misleading the
auditor:
A. considers the need to appropriately modify the auditor's report
B. does not need to modify the report.
C. needs to issue qualified opinion
D. needs to disclaim his opinion

9. An expectation of the public is that the auditor will recognize that the primary users of
audit services are:

A. the employees
B. the Securities and Exchange Commission
C. the investors and creditors
D. the board of directors

10. Which of the following represents a situation in which an auditor is reasonably


independent of the client?

A. The auditor is paid by the client organization rather than the users of the financial
statements.
B. The auditor takes a personal loan from the president of the company.
C. The auditor's dependent son holds 25 shares of the client's common stock.
D. The auditor has not received payment for the previous audit services.

11. A CPA firm is considered independent when it performs which of the following
services for a publicly traded audit client?

A. Serving as a member of the client's board of directors.


B. Determining which accounting policies will be adopted by the client.
C. Accounting information system design and implementation.
D. Tax return preparation as approved by the board of directors.

12. The following are factors that a professional accountant should use the basis of his
acceptance of an assurance engagement, except:
A. The auditor believes that a conclusion based on suitable criteria can be expressed.
B. The subject matter is identifiable.
C. The conclusion can be meaningful to the intended user of the report of the
practitioner.
D. The likelihood that the conclusion to be expressed always support', the assertion of
the responsible party.

13. Which of the following represents a procedure the auditor may use because
plausible relationships .among financial" statement balances ore expected to exist?

A. Attributes testing
B. Enterprise risk assessment
C. Inherent tests of control
D. Analytical review

14. An auditor compares expenses as a percent of sales to expectations. This is an


example of:

A. ratio analysis
B. trend analysis
C. internal control analysis
D. vertical analysis

15. Management's responsibility for the financial statements is

A. implicitly represented in the auditors standard report.


B. Explicitly represented in the opening paragraph of the auditor‘s standard report.
C. Explicitly represented in the responsibility of the management paragraph of the
auditors standard report.
D. Explicitly represented in the opinion paragraph of he auditor: standard report.

16. What is the primary purpose of effective internal control in or organization?

A. Achievement of certain organizational goals.


B. Completion at a successful audit for the entity.
C. Shareholder involvement in the company's success
D. Obtaining profitability and financial strength.

17. Which of the following is not a major emphasis in the design a effective internal
control?

A. Assets are property protected.


B. Duties are segregated.
C. Transactions are authorized.
D. Processes are efficient.
18. Which one of the following is the most relevant factor in assessing the control risk of
a computerized environment?

A. Computerized environments provide management with effective replacement


controls
B. Computerized accounting systems enhance efficiency for users
C. An auditor's method of testing the effectiveness of the system controls is the same in
a computerized system as in a mantic II system
D. The control risk over computerized accounting systems must be assessed during
planning

19. What is the overriding objective of the international Auditing Standards that are
issued by the International Auditing Practice Committee of the International Federation
of Accountants?

A. To improve the uniformity of auditing practices and related services throughout the
world.
B. To override a country's regulations governing the audit of financial statements.
C. To replace the generally accepted auditing standards.
D. To provide a uniform application of specific audit procedures that are acceptable
worldwide.

20. Individual CPAs, Firms or Partnerships of CPAs, including partners mid staff
members thereof shall register with the BOA and the PRC. IIII application for
registration of Ocampo and Co., CPAs was approved on July 31, 2006, the registrant
should apply for the renewal on or before:

A. Sept. 30, 2008


B. Dec. 31, 2009
C. Dec. 31, 2008
D. July 31, 2008

21. Management's assertions in the financial statements are of relevant to the audit
process because:

A. they embody the procedures that will be performed by the audit team
B. they include representations of financial statement in accordance with the applicable
reporting criteria
C. they provide evidence that -auditors have prepared financial statements in
accordance with GAAP
D. they relate to regulator's expectations about audit results

22. To establish the validity of account balances and transactions relating to recorded
amounts, auditors may resort to:

A. vouching
B. tracing
C. representing
D. footing

23. The assertion of existence can be audited directionally by considering balances and
transactions from:

A. recorded amounts to evidence regarding the source


B. evidence regarding the source to recorded amounts
C. general ledgers to trial balances
D. all of these choices

24. Which of the following types of audits are most similar?

A. Operational audits and compliance audits


B. Independent financial statement audits and operational audits
C. Compliance audits and Independent financial statement audits
D. Internal audits and independent financial statement audits

25. Which of the following best represents an auditor's responsibility for fraud?
A. Auditors are only required to find securities fraud
B. Auditors defer to management to discover the extent of fraud
C. Auditors are required to discover misstatements resulting from material fraud D.
Auditors are required to seek out and find all fraud, regardless of its magnitude

26. An auditor will most likely estimate the tolerable failure rate in order to:

A. determine which type of sampling approach to use


B. calculate the probable control risk
C. determine the population to be tested
D. determine the appropriate sample size

27. How many CPE credit units must be accumulated by a registered accounting
professional within the three-year period coinciding the renewal of a CPA's registration?

A. 90 credit units
B. 60 credit units
C. 40 credit units
D. 20 credit units

28. For a company with strong internal control over receivables, the distinguishing
feature of the audit use of positive confirmations in contrast to negative confirmations is:

A. the population sampled


B. the involvement of the client in the confirmation process
C. the type of information included
D. the volume of confirmations returned

29. Auditors test management's estimates of an asset's impaired value through


reference to all of the following except:

A. inquiry of fixed asset personnel


B. evidence of fair market value
C. estimated cash flow
D. financial plans

30. The auditor may determine that fixed assets that should have been capitalized as
assets have been recognized as expenses during the period under audit by testing:

A. assets for impairment


B. repairs and maintenance expenses
C. depreciation expense
D. useful lives of assets

31. The product of inherent risk and control risk is assessed as low. How would an
auditor with this assessment most likely test depreciation expense?

A. As a ratio of total assets


B. As a percent of sales
C. By recomputing all depreciation figures
D. By fagging and tracing transactions through the system

32. Misstatements that are found during an audit and aggregated at the conclusion of
the audit for further consideration by the auditor for their impact on the financial
statements typically include:

A. those material items that have been proposed by the auditor for adjustment and
accepted by the client
B. those of an immaterial magnitude that have been passed by the auditor until the
completion of the audit
C. Those of a material nature that have been ignored by the auditor due to the risk of
sampling error
D. those of immaterial amounts that were not documented by the auditor because they
ore of an inconsequential matter to the audit

33. The application of due professional care means that the auditor work conforms with
all of the following except:

A. Current auditing standards as promulgated by Auditing and Assurance Standards


Council.
B. The work that a reasonably prudent auditor would have performed in the same
situation.
C. The work that would have been performed by a reasonable person who was not
necessarily trained in auditing.
D. The work is at least equal to that which had been performed on the audit
engagement during the preceding year.

34. The second standard of fieldwork requires the auditor to do all of the following
except

A. Understand the business and its environment.


B. Understand the risks related to financial reporting
C. Perform analytical procedures to identify potential misstatements in the financial
statements
D. Obtain an understanding of internal control and potential 'weaknesses in controls.

35. Which of the following is incorrect regarding professional competence?

A. Professional accountants may portray themselves as having the required expertise


or experience they do not possess
B. Professional competence may be divided into two separate phases.
C. The attainment of professional competence requires initially a high standard of
general education.
D. The maintenance of professional competence requires a continuing awareness of
development in the accountancy profession

36. Balboa, a senior auditor, is the - team leader of the audit team assigned in the audit
of HCB Company. His first assignment as audit assistant was the audit of inventory of
NCB Company. Since then, he has been a member, and far the last 5 years, the in-
charge of the team for the audit of HCB. What kind of threat to independence, if any,
has been created by the foregoing fact?

A. Advocacy threat.
B. Self-interest threat.
C. Self-review threat.
D. Familiarity threat.

37. What kind of threat to independence most likely occurs when any product or
judgment of a previous assurance engagement or non assurance. engagement needs
to be re-evaluated in reaching conclusions on the assurance engagement or when a
member of that, assurance team was previously a director or officer of the assurance
client, or was an employee in a position to exert direct and significant influence over the
subject matter of the assurance engagement?

A. Self-interest threat
B. Advocacy threat
C. Self-review threat
D. Familiarity threat.
38. Which of the following is prohibited by the Code of Professional Ethic for CPAs?

A. Use of a firm name which includes the name of a retired partner.


B. Announcement in a newspaper of the opening of a public accounting firm.
C Engaging, in civic activities during business hours.
D. Accepting an engagement or employment which one cannot reasonably expect to
complete or discharge with professional competence.

39. The example(s) of circumstances that may create self-interest threat include

A. Charging contingent fees relating to an assurance engagement.


B. A direct financial interest or material indirect financial interest in an assurance client.
C. A loan or guarantee to or from an assurance client or any of it directors or officers.
D. All of these create self-interest threat.

40. Jennifer Gomez, CPA, forgot to test a client's assessment of goodwill impairment
during an audit. Such an act is probably an example of:

A. ordinary negligence
B. due diligence
C. reckless professional behavior
D. fraud

41. Similar to auditors in the CPA realm, internal auditors also strive to possess:

A. independence
B. Objectivity.
C. Competence.
D. All of these choices.

42 In the auditor's responsibility paragraph of the audit report, the use of the term
"misstatements" conveys that the auditors are responsible to each for

A. minor misstatements.
B. significant Misstatements.
C. fraudulent misstatements.
D. all misstatements.

43. The appropriate date for the audit report is one on which the

A. client's fiscal year ends.


B. auditor and client entered into a contract.
C. client management approves the audited financial statements.
D. auditor types and delivers the report to client.
44. A responsibility of the auditor and opinion qualification can be issued only when the
auditor

A. is not independent.
B. is not able to accumulate all the evidence required by generally accepted auditing
standards.
C. has accumulated all the evidence required by generally accepted auditing standards.
D. has been restricted by the client from gathering inc needed information and the
possible effect of such a limitation requires a disclaimer of opinion .

45. A qualified opinion is appropriate when the auditor is satisfied that the overall
financial statements are

A. fairly stated
B. materially misstated
C. misleading
D. fairly stated, even though there is an immaterial exception

46. Analytical procedures are used

A. to set materiality limits.


B: to assess the reasonableness of financial statement amounts.
C. to provide direct evidence about the numbers in the financial statements.
D. to test internal controls.

47. The risk that material misstatements have occurred in transactions entering the
accounting system is

A. audit risk.
B. inherent risk.
C. control risk.
D. detection risk.

48. In the audit risk model, if an auditor wanted to keep audit risk at a low level, but
there was a great inherent risk of material Misstatement and the internal control was
ineffective, then procedures would need to be designed so that
A. detection risk was at a low level.
B. detection risk was at a high level.
C. control risk was at a low level.
D. inherent risk was at a high level.

49. Physical observation by an auditor would include

A. examination of a soles invoice.


B. recalculation of depreciation.
C. examination of securities certificates.
D. scanning the accounts for unusual transactions.

50. When qualified or adverse opinion is issued, the qualifying paragraph is inserted

A. between the introductory and scope paragraphs.


B. before the opinion paragraph.
C. after opinion paragraph, as last paragraph.
D. immediately otter the address, as the first paragraph.

51. If the auditor believes there is minimal likelihood that the resolution of an uncertainty
will have a material effect on the financial statements. the auditor would issue a(n)

A. qualified opinion
B. unqualified opinion
C. adverse opinion
D disclaimer of opinion

52. How is the responsibility of the practitioner affected by an involvement of an expert


in an assurance engagement?

A. The practitioner should always refer to the work of the expert in his report.
B. The practitioner should divide the responsibility between him and the expert.
C. The auditor should have an understanding of the aspects of the subject matter for
which the expert has used, sufficient to enable the practitioner to accept responsibility
for the conclusion on subject matter being expressed.
D. The practitioner should first obtain a level at knowledge of the business, sufficient to
at least equal the expertise of the expert, so that he can review the results of the work at
the expert.

53. The concept of materiality is not used by auditors as d guide to

A. planning the audit program.


B. evaluation of the evidence.
C. application of general standards.
D. making decisions about the audit report.

54. Inherent risk is not a characteristic of the

A. client's business.
B. substantive procedures.
C. major types of transactions.
D. effectiveness of the client's accountants.

55. Which of the following risks is entirely a quality criterion based on professional
judgment?
A. Audit risk
B. Inherent risk
C. Control risk
D. Detection risk

56. Misstatements must be compared to some measurement base before a decision


can be made about the materiality of the failure to follow GAAP. A commonly accepted
measurement base would be

A. net income.
B. total assets.
C. working capital.
D. all of the given choices.

57. The only unqualified reports which use modified wordings are those involving

A. the use of other auditors.


B. material uncertainties.
C. substantial doubt about going concern.
D. lack of consistent application of GAAP.
58. Financial statement assertions include ail of the following except:

A. occurrence.
B. presentation and discios6re.
C. consistency and comparability.
D. completeness.

59. Which of the following types of evidence will be gathered in order to test internal
controls?

A. Confirmation: of accounts receivable with customers.


B. Observation of client personnel receiving inventory shipments.
C. Observation of inventory counts.
D. Inquiry of management regarding significant litigation.

60. If, when performing analytical procedures, an, auditor observes that operating
income has declined significantly between the preceding year and the current year, the
auditor should next:

A. require that the decline be disclosed in the financial statements.


B. consider the possibility that the financial statements may be
materially misstated.
C. inform management that a qualified opinion on the financial statements will be
necessary.
D. determine management's responsibility for the decline and discuss the issue with the
audit committee.
61. The purpose of analytical procedures in these audit planning stage is to

A. aid in planning the observation of physical inventory.


B. identify unusual circumstances that the auditor may need investigate further. C. flag
individual transactions for further review.
D. determine whether sales transactions were approved.

62. Which of the following is not a change that affects consistency and -therefore does
not require an explanatory paragraph?

A. Change in accounting principle, such as a. change from weighted-average to FIFO.


B. Change in reporting entity, such as the inclusion of an additional company in the
combined financial statements.
C. Change in an accounting estimate, such as a decrease in the life of an asset for
depredation purposes.
D. Correction of errors by changing from non-GAAP to GAAP.

63. When there is a subsequent discovery of omitted procedures, although the financial
statements are fairly presented, the auditor may not have met due diligence
requirements. The auditor:

A. is under no obligation to perform addition audit procedures.


B. must contact the client and perform the omitted procedures
C. must notify the SEC of the omitted procedures.
D. immediately resign from the engagement.

64. A statement in a report such as "Nothing came to our attention that is would lead us
to question the fairness of the presentations" is referred to as a(n):

A. unqualified opinion.
B. disclaimer of opinion.
C. negative assurance.
D. positive assurance.

65. In a financial statement audit, management is responsible for the following except:

A. the financial statements.


B. for establishing and maintaining internal control.
C. for meeting budget projections.
D. for assuring compliance with laws and regulations.

66. Contingent fee based pricing of accounting services is:

A. Always strictly prohibited in public accounting practice.


B. Allowed in an engagement to corner financial statements..
C. Not prohibited if associated with report based agreed-upon procedures.
D. Always considered an act discreditable to the profession.

67. Adequate planning and design of an audit is necessary for an audit to restrict which
type of component of audit risk?

A. Control risk.
B. Detection risk.
C. Sufficiency risk.
D. Inherent risk.

68. Prior to beginning the field work on a new audit engagement in which a CPA does
not possess expertise in the industry in which the client operates, the CPA should

A. reduce audit risk by lowering the preliminary levels of materiality


B. Design special substantive tests to compensate for the lack industry expertise.
C. Engage financial experts familiar with the nature of the industry.
D. Obtain a knowledge of matters that relates to the nature of the entity's business.

69. An auditor anticipates of assessing control risk at a low level in computerized


environment. Under these circumstances, on which the following procedures would the
auditor initially focus?

A. Output control procedures.


B. General control procedures.
C. Application control procedures.
D. Program control procedures.

70. Engagement risk is, influenced by the risks associated with the following, except:
A. The sufficiency and appropriateness of the evidence likely to be available.
B. The nature and extent of the process used to collect and evaluate evidence. C. The
identified user of the assurance engagement report.
D. The nature and form of the subject matter.

71. Discussions with the owner-manager of an audit client reveal to the auditor that the
company is more concerned with minimizing their income tax payments than
maximizing income. Based on this information, which management assertion will the
auditor be concerned about verifying with regard to sales revenue?

A. Existence and occurrence.


B. Completeness
C. Rights and obligations.
D. Valuation.

72. Which of the following primary assertions is satisfied when an au observes the
client's physical count of inventory
A. Valuation
B. Completeness
C. Existence
D. Rights and obligations

73. Which of the following acts are considered fraudulent?


I. Alteration of records or documents.
II. Misinterpretation of facts.
III. Misappropriation of assets.
IV. Recording of transactions without substance
V. Clerical mistakes.

A.III only
B. I and Ill only
C. I, Ill, and IV only
D. All the given acts are fraudulent

74. Which of the following statements best represents the reason auditors prepare
engagement letters to be signed by their clients?

A. They provide documentation of management's responsibility the financial statements.


B. They document the audit fees and deadlines that have agreed upon with their clients.
C. They communicate and clarify the expectations responsibilities of both the client and
the auditor.
D. They help to limit auditor liability in the event of misunderstandings.

75. Which of the following series of steps represent the correct sequence of evidence as
represented in the audit testing hierarchy?

A. risk assessment, tests of controls, tests of details, substantive analytical procedures.


B. risk assessment, tests of controls, substantive analytical procedures, tests of details.
C. tests of controls, risk assessment, tests of details, substantive analytical procedures.
D. tests of controls, risk assessment, substantive • analytical procedures, tests of
details.

76. Professional skepticism dictates that when management makes a statement to the
auditors, the auditors should

A. Disregard the statement because such a statement will be overruled by external


evidence.
B. Corroborate the evidence with other supporting documentation whenever possible.
C. Ask the client management to put it in a form of board resolution.
D. Accept the statement at its face value because the management has no reason to
put itself in shameful misrepresentation.
77. Philippine Standards on Auditing require the auditors to assess the risk of material
misstatements due to fraud

A. for every audit


B. for first-time audits
C. sufficient to find any frauds which may exist
D. whenever it would be appropriate

78. Which of the following conditions supports an increase in detection risk?

A. internal control over cash receipts is excellent


B. Application of analytical procedures reveals a significant increase in sales revenue in
December, the last month of the fiscal year.
C. Internal control over shipping, billing, and recording of sales revenue is weak. D.
Study of the business reveals that the client recently acquired a new company in an
unrelated industry.

79. An auditor obtains knowledge about a prospective client's business and industry to:

A. identify areas of specific risk to the engagement.


B. determine whether the client's management is sufficiently trustworthy to justify
accepting the engagement.
C. make preliminary judgments about material misstatements in the client's financial
statements.
D. document weaknesses in the client's internal control.

80. Inspection of tangible assets provides evidence for which assertion?

A. Existence and occurrence.


B. Completeness.
C. Rights and obligations.
D. Presentation and disclosure.

81. The auditor has assessed the level of control risk (together with the assessed level
of inherent risk) to determine the acceptable level of detection risk for financial
statement assertions for receivable account balances. As the acceptable, level of
detection risk decreases, which of the following adjustments to the accounts receivable
audit program would the audit team normally make?

A. Change the, nature of substantive tests to less efficient procedures, such as using
negative rather than positive confirmations.
B. Change the timing of the confirmation process to an interim date.
C. Increase the sample size of the accounts for confirmations.
D. Change the sampling method from random to convenience sampling.
82. An auditor determines that the management integrity is high, the risk of account
misstatements is low, and the client's information system is fellable. Which of the
following conclusions can be reached regarding the need to perform direct tests of
account balances?

A. Direct tests should be limited to material account balances, and the extent of testing
should be sufficient to corroborate the auditor's assessment of low risk.
B. Direct tests of account balances are not needed..
C. Direct tests of account balances are necessary if audit risk was set at a low level, but
are not necessary if audit risk was set at a high level.
D. Direct tests should be performed on all account balances to independently verify the
correctness of the financial statements.

83. A lest of an asset for overstatement provides corresponding evidence on:

Expense Revenue Liability


A. Overstatement Overstatement Understatement
B. Understatement Overstatement Overstatement
C. Understatement Understatement Understatement
D. Overstatement Overstatement Overstatement

84. Which of the following best describes the auditor's responsibility' for, detecting
financial reporting fraud versus detecting a defalcation?
A. There is more responsibility for detecting financial reporting fraud because audits are
designed to look for financial misstatements.
B. The auditor is responsible for detecting financial reporting fraud only if it is material
but he is responsible for detecting all defalcations caused by a known deficiency in the
client's internal control.
C. The auditor is responsible for detecting material misstatements to the financial
statements; thus there is no difference in the responsibility of detecting financial
reporting fraud or - a defalcation as long as they are material.
D. The auditor is responsible for detecting financial reporting fraud of any amount if
collusion and red flags are present.

85. Fraudulent companies will prepare financial statements that materially misleading by
doing all of the following except

A. understate revenues and assets:


B. understate expenses and liabilities.
C. show financial performance better than industry average.
D. have performance exactly meet announced targets.

86. A program for understanding the client's inherent risk and contra would not include
the procedure to:

A. understand economy and industry with which the client opera


B. study previous year audit documentation.
C. evaluate the competence and independence of the internal auditors.
D. obtain written representation from the client concerning collectability of receivables.

87. A management representation letter is prepared on each engagement for which of


the following primary reasons?

A. It clearly documents the audit procedures that were performed by the auditors.
B. It further acknowledges that management is responsible for fraud Contained in the
financial statements.
C. It provides the auditor with comfort that the client has integrity and is not misleading
the engagement team.
D. It clarifies certain matters included in the letter and documents them for the auditor
as further evidence from the client.

88. A company issues preferred stock. Which of the following will the auditor evaluate
for disclosure purposes in the financial statements relating to the outstanding shares of
preferred stock?

A. Liquidation preference
B. Fair market value
C. Number of shareholders of record
D. Dividends per share ratio

89. Which of' the following statements is (are) correct regarding the auditor's use of
materiality as if applies to a financial statement audit?

A. The auditor is required to report all incidences of material fraud to the audit
committee.
B. There is no difference between the peso amount of planning materiality when
searching for a defalcation versus .searching for financial reporting fraud.
C. The auditor must consider qualitative factors such as whether senior management is
involved in determining the materiality of fraud.
D. All of the statements are correct.

90. Which of the following would not be considered a motivation to commit fraud?

A. Personal financial problems.


B. Stock compensation program.
C. Ineffective internal controls.
D. Tight debt covenants.

91. The likelihood of assessing control risk too high is the risk that sample selected to
test controls
A. does not support the auditor's planned assessed level of control risk when the true
operating effectiveness of the control justifies such an assessment.
B. contains misstatements that could be material to the financial statements when
aggregated with misstatements in other account balances or transaction classes.
C. contains proportionately fewer deviations from prescribed controls than what exist in
the balance or class or a whole.
D. does not support the tolerable misstatement for some or all of management's
assertions.

92. When an incoming auditor becomes aware of certain material misstatement in the
prior period's comparative financial statements on which the predecessor auditor
previously issued unmodified report, the incoming auditor should

A. Modify the opening paragraph by referring to the predecessor auditor, the type of
opinion issued and the date of the report
B. Restate the financial statements of the prior period
C. Discuss the matter with the management and, after having obtained management's
authorization, contact the Predecessor auditor and propose that the prior period financial
statements be restated.
D. Obtain management's authorization for the revision of the prior year's financial
statements and include an emphasis of matter paragraph to describe si tch o revision
mode

93. What is the reason for ensuring that every copy of a vendor's invoice has a receiving
report?

A. To ascertain that merchandise billed by the vendor Was received by the company.
B. To ascertain that the merchandise received by the company was billed by the
vendor.
C. To ascertain that the invoice was correctly prepared:
D. To ascertain that a check was prepared for every invoice.

94 Which of the following is not done -by an auditor when obtaining an understanding of
an entity's internal control?

A. Identification Of the types of potential misstatements that can occur.


B. Consideration d the operating-effectiveness of the internal controls.
C. Designation of substantive tests
D. Consideration of factors that affect the risk of material misstatements.

95. The auditor attempts to determine the operation of the accounting system by tracing
one or few transactions through the accounting system. This procedure is referred to as

A. tracing.
B. tests of controls.
C. vouching.
D. a walk-through.

96. A difference of opinion regarding the results of a sample cannot be resolved


between the, assistant who performed the auditing procedures and the in-charge
auditor. The assistant should

A. Refuse to perform any further work on the engagement.


B. Accept the judgment of the more experienced in-charge auditor.
C. Document the disagreement and ask to be disassociated from the resolution of the
matter.
D. Notify the client that a serious audit problem exists.

97. When providing consulting services, the CPA acts primarily as a(n):

A. independent accountant.
B. expert on compliance with industry standards.
C. technology specialist
D. objective advisor on the use of information.

98. The Primary deliverable of an engagement to perform based on procedures


prescribed by the intended user of the report is (are):

A. the financial statements


B. The Review Report
C. Report of Factual Findings
D. Management Letter

99. A review engagement differs in scope as compared to an audit due to:

A. the subject matter of the service


B. the quantity and type of evidence obtained
C. ethical requirements with respect to independence
D. the users of the financial statements

100. An internal auditor's report to management will typically include:

A. An opinion paragraph.
B. Issues and findings.
C. A commitment to implement solutions.
D. All of these choices.
SIMULATED EXAMINATION 2

1. Which of the following best describes the reason why on independent auditor reports
on financial statements?

A. A management fraud may exist and it is more likely to be detected by independent


auditors
B. Different interests may exist between the company preparing the statements and the
persons using the statements
C. A misstatement of account balances may exist and is generally corrected as a result
of the independent auditor's work
D. A poorly designed internal control system may be in existence

2. Operational auditing refers to the study of business operations for the purpose of
making recommendations for which of the following?

A. Economic and efficient resources


B. Compliance with rules and regulations.
C. Attesting to the fairness of the financial statements.
D. All of the above

3. The accuracy of information that is included in the footnotes that accompany the
audited financial statements of a company whose shares are traded on a stock
exchange is the primary responsibility of the:

A. stock exchange officials.


B. management.
C. lead engagement partner.
D. Securities and Exchange Commission.

4. When the professional' accountant has obtained sufficient appropriate evidence to be


satisfied that the subject matter is plausible in the circumstances, he or she can provide
what level of assurance?

A. Positive
B. High
C. Moderate
D. Absolute

5. The auditor's responsibility in an audit engagement is limited to:

A. Expression of an opinion on the financial statements.


B. Expression of an opinion on the financial statements and adequacy of summary or
accounting policies ad other notes.
C. Opinion issued and faire of presentation of financial statements
D. Expression of opinion and inclusion of supplementary information, if necessary.
6. The existence of audit risk is recognized by the statement in the scope paragraph
auditor‘s responsibility of the auditor's standard report that the

A. auditor is responsible for expressing an opinion on the financial statements.


B. financial statements are presented fairly, in all material respects, in conformity with
PFRS.
C. audit includes examining, on a test basis, evidence supporting the amounts and
disclosures in the financial statements.
D. auditor obtains reasonable assurance about whether the financial Statements are
free of material misstatements.

7. Which of the following is, explicitly cited in the Revised Accountancy Law IRA 9298)?

A. The Professional Regulation Commission has the authority to replace any member of
the Board of Accountancy for negligence, incompetence, or any other just cause
B. Insolvency is a ground for proceedings against a CPA
C. A person shall be considered to be in the professional practice of accounting if, as an
officer in a private enterprise, he makes decisions requiring professional accounting
knowledge
D. After .three years, subject to certain conditions, the Board of Accountancy may order
the reinstatement of a CPA whose certificate of registration has been revoked

8. Which of the following is not a requisite in applying for the CPA licensure
examinations?

A. Natural-born citizen-of the Philippines


B. Good moral character
C. Holder of the degree of Bachelor of Science in Accountancy
D. Has not been convicted of any criminal offense involving moral turpitude

9. Individual CFAs, Firms Cr Partnerships of CPAs, including partners and staff,


members thereof shall register with the BOA and the PRC. If the application for
registration of Arnaldo & Associates, CPAs was approved on May 10, 2006; the
registration will aspire on

A. September 30, 2008


B. December 31, 2008
C. December 31, 2009
D. May 10, 2009

10. Which of the following was a modification made to the iFAC Code to consider
Philippine regulatory requirements and circumstances?

A. The period for rotation of the lead engagement partner was changed front seven to
five years
B. Advertising and solicitation, by individual professional accountants in public practice
is allowed in the Philippines
C. When a professional accountant performs services in a country other than the home
country, the professional accountant should always adhere to the ethical requirements
of the country in which services are being performed
D. When a professional accountant .performs services in a country other than the
home country, the professional accountant should always adhere to the ethical
requirements of the home country

11. To which principle does the following relate?

"The intended user needs to have confidence that the profession accountant hat no.
interest that creates an unacceptable risk of bias with respect to the subject matter‖

A. Confidentiality.
B. Integrity
C. Professional competence and due care
D. Objectivity

12. Which of the following is most likely a violation of the Code of Ethics?

A. The professional accountant in public practice bills a client a fee, lower than what
previously has been charged for similar services
B. The professional accountant makes a representation that specific professional
services in the current or future periods will be performed for a stated fee if it is likely at
the time of representation that such fee will be substantially increased
C. The professional accountant in public practice is the one who determines the
appropriate billing rates of each professional staff engaged in performing services
D. The professional accountant agrees to the client's proposal for a professional fee
that is dependent to the number of service hours rendered

13. Which of the following acts may potential create a self-review threat?

A. Providing advices on accounting principles for audit clients


B. Determining journal entries without obtaining the approval of audit client
C. Assisting an audit client in resolving account reconciliation
D. Drafting the notes .to the financial statements for audit client

14. Which of the following types of loan granted to a member of the assurance Team by
an assurance client that is a -bank is mostly a potential threat to independence?
A: Car loan
B. Credit card balance
C. Clean loan
D. Home-mortgage loan
15. The criteria are the standards or benchmarks used to evaluate the subject matter of
an assurance engagement. Among the following criteria, which one is the least
objective?

A. Philippine financial reporting standards governing the preparation of financial


statements
B. Specific agreements in a contract
C. Control policies-and procedures
D. Effectiveness and efficiency in carrying out operating procedures

16. How is the auditor's report on the financial statements that require final approval by
stockholders before 'such financial statements are issued publicly dated?

A. The auditor's report should be dated coinciding the date of approval of the financial
statements by the stockholders
B. The auditor's report should be dated after the approval of the financial statements by
the stockholders
C. The date of the auditor's report coincides the date of approval of the financial
statements by the board of directors
D. The audit report should be dual dated, the first date coinciding the approval by the
board of directors and the second date to coincide with the approval by The
stockholders

17. In pursuing its quality control objectives with respect to independence, a CPA -firm
may use policies and procedures such as

A. Emphasizing independence in mental attitude in firm training 'programs and in


supervision and review of work
B. Prohibiting employees from owning shares of stock of publicly traded companies
C. Suggesting that employees conduct their banking transactions with banks that do not
maintain accounts with client firms
D. Assigning employees 'who may lack independence to research positions that do not
require participation in' field audit work

18. Which of the following is an element of a CPA firm's quality control system that
should be considered in establishing its qualify control policies and procedures?

A. Complying with laws and regulations


B. Using statistical sampling techniques
C. Assigning personnel to engagements
D. Considering audit risk and materiality

19. Before the practitioners rely on the work of the expert, he should obtain sufficient
appropriate evidence that the work of the expert is adequate by considering the
following, except:
A. the reasonableness and significance of the expert's findings in relation to the
objective of the engagement and the conclusion on the subject matter
B. the professional competence, experience and objectivity of the expert
C. the findings of the expert support the assertion issued by the party responsible to the
subject matter
D. the reasonableness of the assumptions, methods and source data used by the
expert

20. Which of the following is an invalid reason why an auditor cannot issue an absolute
assurance?

A. Most audit evidences ore conclusive rather than persuasive


B. The inherent limitations of any accounting and internal control system.
C. Audit is based on testing
D. Audit procedures that are effective in detecting ordinary misstatements are
ineffective in detecting intentional misstatements.

21. The audit procedures deemed necessary in the circumstances to achieve the
objective of the audit refer to:

A. audit program
B. audit objective
C. substantive procedures
D. scope of an audit

22. In which of the following would the judgment of the auditor be most critical?

A. Verification of the authenticity of the transfer certificate of title


B. Classifying whether a liability is current or long-term
C. Verification of the cut-off of transactions
D. Assessing the reasonableness of the estimates made by management

23. Which of the following does not require the auditor to send a new engagement
letter?

A. An indication that the client misunderstands the objective and scope of the audit
B. A change of engagement from higher to lower level of assurance
C. A recent change in the engagement
D. Legal requirements and other government agencies‘ pronouncements

24. An auditor who agreed for a change in the type of engagement from higher to lower
level of assurance, should issue the report based on the revised engagement

A. but should discuss in a separate paragraph the reason for the change
B. and should always refer to any procedures that should have been performed in the
original engagement
C. and should -qualify the opinion due to scope limitation
D. and should omit reference to the original engagement

25. Which of the following is an appropriate reason why the auditor considers the
professional competence of assistants whom the work will be delegated?

A. To assure that the assistants will be objective.


B. To have reasonable assurance that such work will be performed with due care by the
audit assistant.
C. To effectively reduce the working paper documentation
D. To eliminate detection risk

27. Detection of noncompliance, regardless of materiality, requires considerations of the


following:

I. Integrity of management.
II. Possible effect on other aspects of the audit.
III. Legal determination of the act of non-compliance.
A. A, B, C
B. A, B
C. A, C
D. C

27. The auditor should perform the following risk assessment procedures to obtain an
understanding of the entity and its environment, including its internal control, except:
A. inquiries of management and others within the entity
B. inquiries of the entity‘s external legal counsel or of valuation experts that the entity
has used
C. analytical procedures
D. observation arid inspection

28. Which the following distinctions between general audit objectives and specific audit
objectives for each account balance is correct?

A. The general audit objectives are applicable to every account balance on the financial
statements
B. The specific audit objectives are applicable to every account balance on the financial
statements
C. The general audit objectives are stated in terms tailored to the engagement
D. The specific audit objectives are stated in. terms tailored to the engagement

29. The risk that the audit will fail to uncover a material misstatement is eliminated

A. it‘s a client has strong internal controls.


B. if a client follows Philippine financial reporting standards (PFRS).
C. when the auditor has complied with generally accepted auditing standards (GAAS).
D. under no circumstances

30. What is the relationship between materiality and the level of control risk?

A. Direct
B. Parallel
C. Inverse
D. Positive

31. Which of the following statements about internal control is correct?

A. Internal control system refers to all the Policies and procedures adopted by, the
management of an entity to assist in eliminating material errors but not fraud
B. A strong environment, by itself, ensures the effectiveness of the internal control
system
C. The internal control system is confined to those matters which relate directly to the
functions of the accounting system
D. In the audit of financial statements, the auditor is only concerned with those policies
and procedures within the accounting and internal control system that are relevant to
the financial statements

32. When obtaining an understanding of the accounting and internal control system the
auditor may trace a few transactions through the accounting system. This technique is:

A. internal control review


B. test of transactions
C. walk-through test
D. validity test

33. Which of the following least likely affects the nature, timing, and extent of the
procedures performed by the auditor to obtain an understanding of the accounting and
internal control system of an audit client

A. Materiality considerations
B. The expected level of detection risk
C. The auditor‘s assessment of inherent risk
D. The complexity of the accounting process

34. Which of the following is incorrect about internal control?


A. Accounting and internal control system provides the management with reasonable
assurance that organizational objectives are to be achieved
B. One of the inherent limitations of accounting and internal control systems is that the
possibility that the procedures may become inadequate due to changes in conditions,
and compliance with procedures may deteriorate.
C. Most internal controls tend to be directed at routine transactions.
D. Management should not consider the costs of the accounting and internal control
systems because such a consideration makes the system ineffective.

35. Which statement is incorrect regarding the nature of tests of controls?

A. As the planned level of assurance increases, the auditor seeks more reliable audit
evidence
B. Those controls subject to testing by performing Inquiry combined with inspection or
re-performance ordinarily provide more assurance than those controls for which the
audit evidence consists solely of inquiry and observation
C. The absence of misstatements detected by a substantive procedure provides audit
evidence that controls related to the assertion being tested are effective
D. A material misstatement detected by the auditor's procedures that was not identified
by the entity ordinarily is indicative of the existence of a material weakness in internal
control.

36. Which statement is incorrect regarding the extent of tests of controls?

A. The auditor designs tests of controls to obtain sufficient appropriate audit evidence
that the controls operated effectively throughout the period of reliance
B. The more the auditor relies on the operating effectiveness of controls in the
assessment of risk, the lesser is the extent of the auditor's tests of controls
C. If the rate of expected deviation is too high, the auditor may determine that tests of
controls for a particular assertion may not be effective
D. Because of the inherent consistency of IT processing, the auditor may not need to
increase the extent of testing, of an automated control

37. When would an auditor typically not perform additional tests of a computer systems
controls?

A. When the assessed level of control risk is at a minimum


B. When computer controls appear to be strong and risk is at a minimum
C. When controls appear to be weak
D. When inherent risk is at a maximum

38. The evaluation of deviations that were observed upon completing the tests of
controls:
A. May require the need for doing more extensive understanding of control.
B. may require more extensive tests of controls.
C. may not require modification of the nature, timing, and extent of the planned
substantive procedures.
D. requires a documentation of the basis of assessment of control risk if the assessed
level of control risk is assessed at the maximum.

39. Which of the following statement is true?


A. If the auditor assesses the level of control risk at the maximum, no documentation of
the reason is necessary
B. If the auditor assesses the level control risk at less than maximum, no documentation
of the reason is necessary
C. If the auditor assesses the level of control risk at the maximum, documentation of the
reason is necessary
D. All of the given choices

40. Which of the following is appropriate about risk assessment?

A. The assessed level of inherent and control risk can be sufficiently low, thus resulting
to eliminating the need for substantive tests
B. There is an inverse relationship between detection risk and the combined level of
inherent and control risks
C. Audit risk may be more appropriately determined by assessing inherent and control
risk 'separately
D. Detection risk is eliminated if an auditor were to examine 100 percent of the account
balance or class of transactions

41. The audit risk model consists of: AR = IR x CR x DR


The detection risk is the dependent variable. What is the acceptable level of detection
risk if the assessed level of Inherent risk is Medium and the Control risk is Low?

A. Highest
B. Lower
C. Medium
D. Higher

42. Which of the following should the auditor not consider of having specialized CIS
skills in an audit?

A. The auditor needs to obtain a sufficient understanding of the accounting and internal
control system affected by the OS environment
B. The auditor needs to determine the effect of the CIS environment on the assessment
of overall risk and of risk at the account balance and class of transactions level
C. The need of the auditor to make analytical procedures during the completion stage of
the audit
D. Design and perform appropriate tests of controls and substantive procedures

43. Which of the following characteristics of CIS environment should the auditors be
least concerned?

A. Lack of segregation of functions


B. Cost-benefit relationships
C. Lack of transaction trails
D. Access control.
44. Which of the following is least likely a risk characteristic associated with CIS
environment?

A. Error embedded in an application's program logic maybe difficult to manually detect


on a timely basis.
B. The separation of functional responsibilities diminishes ir a computerized
environment
C. Initiation of changes in the master file is exclusively handled by respective users
D. The potential unauthorized access to data or to alter them without visible evidence
maybe greater

45. Which of the following is not one of the auditor's motor concerns when he has to
make a documentation of the internal control in a computerized environment?

A. The organizational structure of the client's CIS activities


B. The access controls
C. The significance and complexity of computer processing in each significant
accounting application
D. The use of software packages instead of customized software

46. Which of the following is not an advantage of a computerized accounting system?

A. Computers process transactions uniformly


B. Computers help alleviate human errors
C. Computers can process many transactions quickly
D. Computers leave a thorough audit trail which can be easily followed

47. A common difficulty in auditing a computerized accounting system is:

A. Data can be erased from the computer with no visible evidence


B. Because of the lack of an audit trail, computer systems have weaker controls and
more substantive testing is required
C. Because of the uniform, nature of transaction processing, computer systems have
strong controls and less substantive testing is required
D. The large dissemination of entry points into the computer system leads to weak
overall reliance on information generated by a computer

48. Which of the following should the auditor least consider in determining the
significance of service organization activities to the client and its relevance to the audit?

A. The control policies and procedures of the client in requiring that all payments for
goods and services be supported by receiving reports
B. The client's internal controls that are applied to the transactions processed by the
service organization
C. The material financial statement assertions that are affected by the use of the service
organization
D. Terms of contract and relationship between the client and the service organization

49. When the auditor considers that the service organization activities are significantly
relevant to the audit and he concludes that it would be efficient to obtain evidence from
tests of controls, such evidence may be obtained by, except:

A. visiting the service organization


B. performing tests of the client‘s control over activities of the service organization
C. reviewing the service contract between the client and the service organization
D. obtaining a service organization auditor‘s report that expresses an opinion as to the
operating effectiveness of the service organization‘s accounting and internal control
systems for the processing applications and internal control systems for the processing
applications relevant to the audit

50. Which of the following statements about the existence and completeness objectives
are not true?

A. The existence and completeness objectives emphasize opposite audit concerns


B. Existence deals with overstatements and completeness deals with understatements
C. Existence deals with understatements and completeness' deals with overstatements
D. The completeness objective deals with unrecorded transactions

51. In testing for cutoff, the objective is to determine

A. whether all of the current period's transactions are recorded


B. that no transactions from the prior period are included in I current period's balances
C. that no transactions of the current period have been delayed a recorded in a future
period.
D. whether the transactions are recorded in the proper period

52. The determination of the appropriate sample size based on t tolerable deviation rate
in a test of control procedure relates to

A. sufficiency.
B. relevance.
C. validity.
D. appropriateness.

53. Which of the following least likely affect the sufficiency of the appropriate audit
evidence?

A. Nature of the accounting and internal control systems


B. Materiality of the item being examined
C. Source and reliability of information available
D. The type of sampling approach - statistical or judgmental

54. Which of the following statements that relate to the persuasiveness of audit
evidence is invalid?

A. The audit evidence obtained directly by the auditor is more reliable than that one
provided by the client management
B. The oral representation by the dent management is invalid evidence
C. The effectiveness of accounting and internal control adds to the reliability of internal
audit evidence
D. While internal audit evidence is considered to be acceptable, the auditor usually
prefers audit evidence from external sources

55. The following statements are discussions about financial' assertions and audit
evidence. Which of them is (are) correct?

I. When substantial doubt as - to a material financial statement assertion exists, the


auditor likely attempts to obtain sufficient appropriate audit evidence to remove such
doubt
II. The difficulty and cost involved are valid basis for omitting a necessary procedure
Ill. In obtaining audit evidence, the auditor needs to consider the relationship between
the cost of obtaining it and its usefulness
IV. When there are inconsistent evidences provided by two different sources, the
auditor may need to modify his audit opinion

A. II,III, IV
B. I, III
C. II, IV
D. I, III, IV

56. The three major categories of documentary audit evidence are:

1. Documentary audit evidence created by third-parties and held by the client entity.
2. Documentary audit evidence created and held by third parties.
3. Documentary audit evidence created and held by the client entity.

A. 1, 3, 2
B. 3, 1, 2
C. 2, 1, 3
D. 2, 3, 1

57. The inspection of property and equipment primarily relates to which financial
assertion?

A. ownership
B. Valuation
C. Completeness
D. existence

58. If the reported sales for 2007 erroneously include sales that occurred in 2008, the
assertion that caused misstatements on the 2007 financial statements would be:

A. occurrence
B. Valuation or allocation
C. completeness
D. presentation and disclosure

59. The completeness assertion would be violated if:

A. fictitious sales transactions were included in accounts receivable


B. the allowance for doubtful accounts was understated
C. unbilled shipments had occurred during the period
D. disclosures in the statements of pledged receivables were inadequate

60. According to Philippine Standards on Auditing, the auditor's responsibility for failure
to detect fraud arises

A. when such failure clearly results from failure to comply with generally accepted
auditing standards.
B. whenever the amounts involved are material.
C. only when the examination was specifically designed to detect fraud.
D. only when such failure clearly results-from negligence so gross as to sustain an
inference of fraud on the part of the auditor.

61. If an auditor believes a client may have committed illegal acts, which of the following
actions should the auditor take?

A. Consult with that the client's counsel and the auditor's counsel to determine how the
suspected illegal acts will be communicated to the stockholders
B. Extend auditing procedures to determine whether the suspected illegal acts have a
material effect on the financial statements
C. Make inquiries of the clients management and obtain an understanding of the
circumstances underlying the acts and of other evidence to 'determine the effects of
such acts on the financial statements
D. Notify each member of the audit committee of the board at directors about the nature
of the acts and request that they advise an approach to be taken by the auditor

62. An audit program should be sufficiently detailed to provide all of the following
except:

A. Evidential support for the audit opinion.


B. An outline of the work to be done.
C. A record of the work performed.
D. A basis for controlling the audit

63. The primary purpose of the audit working papers is to:

A. Provide an evidence of compliance with auditing standards.


B. Provide management with an independent copy of financial records.
C. Provide a protection against litigation.
D. Document the deficiencies in client's policies and procedures.

64. Early substantive testing of account balances is done only when


A. the client has a natural business year.
B. evidence indicates effective control policies and procedures.
C. Internal control structure is weak.
D. The primarily substantive approach is taken.

65. Test of details of balances generally tend to be the

A. most costly, and least effective audit procedures.


B. most costly, and most effective audit procedures.
C. least costly, yet most effective audit procedures.
D. least costly, and least effective audit procedures.

66. Which of the following analytical procedures, might highlight a possible


overstatement or understatement of a balance in an expense account? Compare:

A. gross margin percentage with previous year


B. individual expenses with previous year
C. inventory turnover ratio with previous year
D. individual asset and liability balances with previous year

67. The test of details of balances, when .applied by the auditor to examine rent and
lease expense for- capitalized leases, would satisfy the audit objective of

A. Existence
B. detail tie-in
C. presentation and disclosure
D. classification

68. Negative confirmation of accounts receivable is less effective than positive


confirmation of accounts receivable because

A. A majority of recipients usually lack the willingness to respond objectively.


B. Some recipients may report incorrect balances that require extensive follow-up.
C. The auditor cannot infer that all non-respondents have verified their account
information.
D. Negative confirmations do not produce evidential matter that is statistically
quantifiable.

69. Which of the following circumstances would most likely cause an auditor to suspect
that material fraud exists in a client's financial statements?

A. Property and equipment items are usually sold at a loss before being fully
depreciated.
B. Significantly fewer responses to a positive confirmation requests are received than
what is expected.
C. Monthly bank reconciliations usually include several in-transit items.
D. Clerical errors are listed on an CBIS-generated exception report.

70. Auditor's examine invoices for accounts such as repairs and maintenance primarily
to determine whether

A. expenditures for fixed assets have been 'recorded in the proper period
B. expenditures have been improperly authorized
C. noncapitalized expenditures have been properly expensed
D. expenditures that were expensed should have been capitalized

71. When auditing-contingent liabilities, which of the following procedures would be


least effective?

A. Reading the minutes of the board of directors' meetings.


B. Reviewing the bank confirmation letter.
C. Examining customer confirmation replies.
D. Examining invoices for professional services.

72. When obtaining an evidence regarding litigation against a client, the CPA would be
least interested in determining

A. An estimate of when the matter will be resolved.


B. The period in which the underlying cause of the litigation occurred.
C. The probability-of an unfavorable outcome
D. An estimate of the potential loss.

73. Banaue Company is an affiliate of the audit client and is audited by another firm of
auditors. Which of the following is roost likely to be used by the auditor to obtain
assurance that all guarantees of the affiliate's indebtedness have been detected?

A. Send the standard bank confirmation request to all the client's lender banks. B.
Review client minutes and obtain representation letter.
C. Examine supporting documents for all entries in intercompany accounts.
D. Obtain written confirmation of indebtedness from the auditor of the affiliate.
74. Which of the following subsequent events will be least likely to result an adjustment
to the financial statements?

A. Culmination of events affecting the realization of accounts receivable owned as of


the balance sheet date.
B. Culmination of events affecting the realization of inventories owned as ot the balance
sheet date.
C. Material changes in the settlement of liabilities which were estimated as of the
balance sheet date.
D. Material changes in the quoted market price of listed investment securities since the
balance sheet date.

75. An auditor is concerned with completing various phases of the examination after the
balance sheet date. This "subsequent period" extends to the date of the:

A. auditor's report.
B. final review of the audit working papers.
C. issuance of the financial statements to the users.
D. delivery of the auditor's report to the client.

76. Which of the following material events occurring subsequent to balance sheet date
would require an adjustment to the financial statements?

A. Sale of long-term debt or capital stock.


B. Loss of a plant as a result of a flood.
C. Major purchase of a business which is expected to double sales volume.
D. Settlement of litigation, in excess at the recorded liability.

77. Selena, CPA, is preparing an audit program for the purpose of ascertaining the
occurrence of subsequent events that may require adjustment or disclosure essential to
a fair presentation of the financial statements in conformity with the Philippine financial
reporting standards. Which one of the following procedures would be least appropriate
for this purpose?

A. Confirm, as of the completion of the fieldwork, those accounts receivable that have
increased significantly from the year-end date.
B. Read the minutes of the board of directors' meetings.
C. Inquire of management concerning events which may have occurred.
D. Obtain a lawyer's letter as of the completion of fieldwork.

78. An auditor's decision concerning whether or not to "dual date" the audit report is
based upon the auditor's willingness to

A. extend auditing procedures.


B. accept responsibility for subsequent events.
C. permit inclusion of a footnote captioned event (unaudited) subsequent to the
date of the auditor's report.
D. assume responsibility for events subsequent to the issuance of the auditor's
report.

79. As part of an audit, a CPA often requests a representation letter tram the client.
Which one of the following is not a valid purpose of such a letter?

A. To provide audit evidence.


B. To emphasize to the client the auditor's responsibility for the fairness of the financial
statements.
C. To satisfy himself by means of other auditing procedures when certain customary
auditing procedures are not performed.
D. To provide possible protection to the CPA against a charge of knowledge in where
fraud is subsequently discovered have existed in the accounts.

80. The risk that the auditor gives an unqualified audit opinion when the financial
statements are materially misstated refers to

A. Audit risk
B. Inherent risk
C. Detection risk
D. Control risk

81. Which of the following disagreements with the management may potentially cause
the auditor to disclaim his opinion?

A. Acceptability of accounting policies


B. Method of application of accounting principles
C. Omission of certain required disclosures in the financial statements.
D. Refusal of the client management for the auditor‘s observation of physical inventory
count

82. The auditor's report may be appropriately addressed to the dients:

Board of Directors Stockholders Audit Committee


A. Yes Yes Yes
B. Yes No Yes
C. Yes Yes No
D. Yes No No

83. Which of the following is not explicitly included in the paragraph of the auditor's
report?

A. Financial reporting framework


B. Reasonable assurance
C. Generally accepted auditing standards
D. Assessment of accounting estimates

84. The term present fairly, in all material respect," means:

A. the financial statements conform with PFRS.


B. The auditor considers only those matters that are material to the financial
statements.
C. The financial statements may still be materially misstated because the auditors may
not have discovered the errors.
D. The financial statements are accurately Prepared.

85. When the auditor concludes that the financial statements are presented fairly in
material respects, in accordance with the identified financial reporting framework, he
should issue:

A. Standard unqualified opinion


B. Qualified opinion
C. Disclaimer of opinion
D. Unqualified opinion with explanatory paragraph

86. In which of the following situations does an auditor need to issue an unqualified
opinion?

A. A significant doubt about the ability of the company to continue as a going concern;
such concern is adequately disclosed by the entity in the notes to financial statements.
B. A limitation of the scope of the audit the possible effect of which is material to the
financial statements.
C. The auditor has disagreement .with, management regarding the acceptability of the
accounting policies, the effect of which is material.
D. The omission of significant information in the financial statement.

87. Which of the following disagreements with the management may potentially cause
the auditor to disclaim his opinion?

A. Acceptability of accounting policies


B. Method of application of accounting principles
C. Omission of certain required disclosures in the financial statements
D. Refusal of the client management for the auditor‘s observation of physical inventory
count

88. When the client does not disclose restrictions of future cash dividends and the CPA
discloses the omitted information in the separate paragraph of the audit report, the
opinion should be:

A. Qualified due to scope limitation


B. Qualified due to inadequate disclosure
C: Adverse
D. Unqualified opinion with an explanatory paragraph

89. When the financial -statements of the prior period were audited by another CPA, the
incoming auditor's report indicates:

A B C D
1. The dates and periods covered by prior years' financial YES YES NO NO
statements
2. That the financial statements of the prior period* were YES YES YES NO
audited by another CPA
3. The type of opinion issued by another CPA and if YES NO YES YES
modified, the reason thereof

90. When the auditor's report on the prior period, as previously issued, included a
modified opinion, the auditor's report should refer to the corresponding figures:

A. though the matter which gave rise to the modification of the audit report had been
subsequently resolved
B. if the matter that gave rise to the modification of the previous auditor's report is
unresolved and results in a modification of the auditor's report regarding the current
figures.
C. only when the matter that gave rise to the modification is unresolved and results to a
modification of the auditor's report regarding the current figures.
D. in all cases where the matter which gave rise to the modification of the previous
audit report was satisfactorily resolved and properly dealt with in the financial
statements, the current report would not refer to the previous modification.

91. In which of the following circumstances would a modification of the tor's report on
the current period not necessary?

A. If the prior period financial statements had been revised and reissued with a new
auditor's report.
B. If the prior period financial statements had not been revised and reissued but the
matter that gave rise to the modification, though material to the current period is
resolved.
C. If the matter that gave rise to the modification of the previous audit report was related
to an unresolved beginning balance of property accounts.
D. The report of the prior period issued by a continuing auditor.

92. They are not presented as complete financial statements capable of landing alone,
but are an integral part of the current period.

A. Comparative financial statements


B. Corresponding figures
C. Notes of financial statements
D. Supplementary report

93. Which of the following statements applies to consultancy service engagement?

A. A practitioner should obtain an understanding of the internal control structure to


assess control risk.
B. A practitioner is not permitted to compile a financial forecast.
C. A practitioner should obtain sufficient relevant data to complete the engagement.
D. A practitioner is to maintain an appearance of independence.

94. Prospective financial statements are for

A. general use
B. limited use only
C. either general or limited use
D. use by internal management only

95. Given one of more hypothetical assumptions, a responsible party may prepare, to
the best of its knowledge and belief, an entity's expected financial position, results of
operations, and changes in financial position. Such prospective financial statements are
most commonly known as

A. special purpose financial statements


B. financial projections
C. partial presentations
D. financial forecasts

96. When reporting on financial statements prepared on a comprehensive basis of


accounting other than Philippine financial reporting standards, the independent auditor
should include in the report a paragraph that

A. states that the financial statements are not intended to be in conformity with
Philippine financial reporting standards.
B. states that the financial statements were not examined in accordance with generally
accepted auditing standards.
C. refers to the authoritative pronouncements that explain the comprehensive basis of
accounting being used.
D. justifies the comprehensive basis of accounting being used.

97. The objective of a review of interim financial information is to provide the CPA with a
basis for
A. expressing a limited opinion that the financial information presented in conformity
with Philippine financial reporting standards
B. expressing compilation opinion on the financial statements.
C. reporting whether material modifications should be made to such standards.
information to make it conform with Philippine financial reporting
D. reporting limited assurance to the board of directors only standards.

98. Audit reports issued in connection with which of the following are generally not
considered to be special reports or special purpose reports?

A. specified elements, accounts, or items of a financial statement.


B. Compliance with aspects of contractual agreements related to audited financial
statements.
C. financial statements prepared in conformity with the price-level
D. Compiled financial statements prepared in accordance with basis of accounting.
appraised liquidation values.

99. The party that is responsible for the assumptions identified in the preparation of
prospective financial statements is usually

A. A third-party lending institution.


B. The client's management.
C. The reporting accountant.
D. The client's independent auditor. ion

100. In extreme cases such as situations involving multiple uncertainties that are
significant to the financial statements, the auditor

A. may consider to express a disclaimer of opinion


B. may quality his opinion instead of issuing-on unqualified opinion ‗nth emphasis of
matter paragraph
C. may issue an adverse opinion because of their significance
D. may issue a "subject to" opinion because the situations related to uncertainties
SIMULATED EXAMINATION 3
1. An audit that involves obtaining and evaluating evidence about the efficiency and
effectiveness of an entity's operating activities in relation to specified objectives is a(n)
A. external audit
B. Financial statement audit.
C. Operational audit.
D. Compliance audit.

2. Which of the following best describes due care?

A. tact in avoiding legal liability


B. requisite skill and diligence
C. reasonable infallibility
D. freedom from undue influence

3. In which of the following situations would qualified opinion be inappropriate?

A. Financial statements are materially misstated.


B. A doubt that is more than substantial about the ability of the company to continue as
a going concern.
C. A significant scope limitation.
D. The management insisted of not attaching the statement of cash flows.

4. The auditor's standard report states that the financial statements are presented tairly

A. with reasonable assurance.


B. in all material respects.
C. without significant errors.
D. on a consistent basis.

5. Which of the following statements about the report that conveys a high level of
assurance on the subject matter is correct?

A. The practitioner's report should contain a subjective expression of his opinion about
a subject matter based on the identified suitable criteria on the evidence obtained.
B. The practitioner must not issue an oral conclusion about a subject matter based on
the identified suitable criteria and the evidence obtained.
C. The report can take various forms, such in writing, electronic form, oral or by
symbolic representation.
D. The report should be in a standard format for reporting on all assurance
engagement.

6. How did the Code of Ethics define public interest?


A. A distinguishing mark of a profession is the acceptance of its responsibility to the
public.
B. The accountancy profession's public consists of clients, credit grantors,
governments, employers, employees: investors, the business and financial community,
and others who rely on the objectivity and integrity of professiOnc.il accountants.
C. The collective well being of the community of people and institution-3 that the
professional accountant serves.
D. The standards of the accountancy profession are heavily determined by the public
interest.

7. Which of the following is incorrect regarding professional competence?

A. Professional accountants may portray themselves as having the expertise or


experience they do not possess.
B. Professional competence may be divided into two separate phases.
C. The attainment of professional competence initially requires a high
standard of general education.
D. The maintenance of professional competence requires a Continuing awareness of
development in the accountancy profession.

8. Which of the following statements about the failure of the auditor of detecting material
misstatements always true?

A. The auditor's failure to discover material misstatements is an evidence of inadequate


planning: performance or judgment.
B. The auditor did not comply with Philippine Standards on Auditing.
C. The auditor has the burden of defending the quality of his audit.
D. Such a failure is an indication of the absence of the auditor‘s, professional
competence and due care.

9. A professional accountant may be associated with a fax return that:

A. contains a false or misleading statement.


B. contains statements or information furnished recklessly or without any real
knowledge of whether they are true or false.
C. omits or obscures information required to be submitted and such omission or
obscurity would mislead the revenue authorities.
D. uses of estimates if such use is generally acceptable or if it impractical under the
circumstances to obtain exact data.

10. The Code of Professional Conduct would be violated if a member accepted a fee for
services and the fee was:

A. fixed by a public authority.


B. based on a price quotation submitted in competitive bidding.
C. based on the result of judicial proceedings.
D. payable after a specified finding was attained.

11. No person shall serve the Professional Regulatory Board at Accountancy for more
than

A. 3 years
B. 6 years
C. 9 years
D. 12 years

12. A common difficulty in auditing a computerized accounting system is:

A. data can be erased from the computer with no visible evidence.


B. because of the lack of an audit trail, computer systems have weaker controls and
more substantive testing is required.
C. because of the uniform nature of transaction processing, computer systems have
strong controls and less substantive testing is required.
D. the large dissemination of entry points into the computer system leads to weak
overall reliance on information generated by a computer.

13. A partner of the firm who is serving as a company secretary for an audit client
creates which of the-following threats?

Advocacy Familiarity Self-review


A. Yes No Yes
B. yes Yes No
C. No Yes Yes
D. Yes Yes Yes

14. Which of the following represents a situation in which an auditor is independent of


the client?

A. The auditor is paid by the client organization rather than the SEC.
B. The auditor takes a personal 'rim from the president of the company.
C. The auditor‘s dependent son holds 25 shores (one-half percent) of the client's
outstanding common stock.
D. The auditor has not received .payment for the previous audit services.

15. Which of the following represents a situation in which the auditors may disclose
client information to outside parties?

A. Bringing working papers to a professional CPA workshop as an example of quality


work.
B. Complying with a validly issued and enforceable subpoena or summons.
C. Showing the client's bank statement to a neighbor who is a shareholder to
emphasize its cash position.
D. Explaining to the local television news station why the client is likely to miss payroll in
the 'forthcoming periods.

16. Which of the following constitutes a situation involving a contingent fee?


A. An hourly rate and estimated total fee disclosed in a tax engagement letter
B. An audit fee based upon a budget and respective professional billing rates in a
proposal
C. A promise to deliver the audit opinion prior to a certain deadline for an estimated fee
D. An additional audit fee based on the positive outcome of securities offering

17. The rules on independence require that a member in public practice shall be
independent in the performance of professional services as required by the Code of
Professional Ethics. For which of the following services does the rule apply?

A. Audits of historical financial statements.


B. Review services.
C. Examinations of prospective financial statements.
D. All the three services given.

18. When the users of financial statements have confidence in a independence, it is


referred to as independence in:

A. fact.
B. appearance.
C. conduct.
D. total.

19. A significant aspect of conducting an audit with due professional care is the auditor's
attitude of professional:

A. Pessimism
B. Skepticism
C. Optimism
D. Courtesy

20. Which one of the following is not considered a valid source information about the
client's processes?

A. confirmation of third-parties
B. review of the client's budget
C. a tour of the client's plant
D. management inquiry

21. The risk that financial statements are likely to be misstated materially without regard
to the effectiveness of internal control is which type of risk?
A. inherent risk
B. Audit risk
C. client risk
D. control risk

22. When must an auditor perform analytical review procedures in a financial statement
audit?

A. Testing controls over financial cycles


B. Performing tests to substantiate balances
C. Planning the nature, timing and extent of procedures
D. Performing tests to substantiate transactions

23. Which of the following would an auditor least likely discuss with the former auditors
of a potential client prior to acceptance?

A. Integrity of management
B. Fees charged for services
C. Predecessor's disagreements with management regarding the use of accounting
principles
D. Reasons for changing audit firms

24. Which of the following -represents a procedure that the auditor may use because
plausible relationships among financial statement balances are expected to exist?

A. Attributes testing
B. Review engagement
C. Inherent test of control
D. Analytical review

25. What is the primary purpose of an effective internal control in an organization?

A. Achievement of certain organizational goals.


B. Completion of a successful audit for the entity.
C. A shareholder's involvement in the company's success.
D. Obtaining profitability and financial strength.

26. Which of the following is not a major emphasis in the effective internal control?

A. Assets are properly protected.


B. Duties are segregated.
C. Transactions are authorized.
D. Processes are efficient.

27. One of the major components of an organization's internal structure includes:


A. audit control risk.
B. the financial environment.
C. risk assessment.
D. telecommunication equipment.

28. The payroll department should be responsible for:

A. authorization of new employees.


B. processing payroll transactions.
C. timekeeping.
D. signing payroll checks.

29. Which of the following is an example of a type of control that tested?

A. Contingent liabilities are disclosed adequately.


B. Property and equipment is classified as a long-term asset
C. Purchase orders are signed by an authorized purchasing officer
D. An undocumented review of the expense budgets

30. Which of the following is a valid statement about the assess control risk?

A. Misstatements discovered by conducting substantive pro may cause the auditor to


modify the previous assess control risk.
B. There is a positive relationship between detection risk combined level of inherent and
control risk.
C. The auditor should consider the assessed levels of inherent control risks in
determining the nature, timing, and e substantive procedures required to eliminate audit
risk.
D. The assessed level of inherent and control risks can be sufficiently low in order to
eliminate the auditor's need to-perform substantive tests on some assertions.

31. Which of the following statements about tests of controls is not valid?

A. The auditor may perform inquiry and observation and gathering audit evidence about
the operating effectiveness of the control.
B. Audit evidence obtained by doing observation pertains only to the point in time at
which the procedure was applied.
C. Ordinarily, making inquiries provides more reliable audit evidence than doing
observation when testing segregation of functional responsibilities.
D. Observation of who applies a control procedure is useful as a test of control
procedures when evaluating control effectiveness of both computerized and manual
system

32. The auditor should consider whether the assessment of control risk confirmed:
A. Upon completion of the understanding of infernal control.
B. Upon the conclusion of the audit, based on the results substantive procedures and
other audit evidence obtained.
C. Upon completion of tests of controls ft
D. Before the final audit program is completed.

33. In general, a material weakness in internal control may be defined as a condition in


which material errors or irregularities may occur and not be detected within a timely
period by

A. an independent auditor during tests of controls.


B. employees in the normal course of performing their assigned functions.
C. management when reviewing interim financial statements aria reconciling account
balances.
D. outside consultants who issue a special purpose report on internal control structure

34. A weakness in internal control procedures over recording retirement of equipment


may cause the auditor to

A. inspect certain items of equipment in the plant and trace those items to- tile
accounting records
B. review the property master file to ascertain whether depreciation was taken on each
item of equipment during the year
C. trace additions to the "other assets" account to search for equipment that is still on
hand but no anger being used
D. select certain items of equipment fran the accounting records and locate them in the
plant

35. When the auditor performs tests of control regarding the policy counting materials
received from the supplies, the auditor most likely

A. inspect the receiving reports.


B. verify that cash-vouchers are accompanied by receiving reports
C. inquire with the receiving clerk of whether count of incoming deliveries is being
made.
D. observe several times as the receiving section is receiving the incoming deliveries.

36. Which of the following statements reflects on auditor's responsibility lot detecting
errors and fraud?

A. An-auditor is responsible for detecting employee errors and fraud, but not for
discovering fraud involving employee collusion it management override.
B. An auditor should plan the .audit to detect errors and fraud that are caused- by
departures from GAAP
C. An auditor is not responsible for detecting errors and fraud unless the application of
GAAS would result in such detection.
D. An auditor should design the audit to provide reasonably assurance of detecting
errors and .fraud that are material to the financial statements

37. The audit risk is the, product of inherent, control and detection risks. Which of these
risks is the dependent variable?

A. Detection risk
B. Control risk
C. Inherent risk
D. All of these are dependent variables.

38. A type of fraud in which an employee takes assets from an organization for
personal gain:
A. Fraudulent financial reporting.
B. Defalcation.
C. Window dressing.
D. Inside trading.

39. The risk of financial fraud may significantly increase if there is

A. A system of profit-sharing incentive plan.


B. A substantial growth in sales.
C. A change from manual processing to computerized system.
D. Frequent changes in suppliers.

40. In planning an audit engagement, which of the following best likely affects the
independent auditors judgment as to the quantity, type, and content of working papers?
A. The estimated occurrence rate of attributes.
B. The preliminary evaluations -based on substantive testing.
C. The content of the client's representation letter.
D. The anticipated nature of the auditor report.

41. Which statement is incorrect regarding the discussion among the engagement team
about the susceptibility of the entity's financial statements to material misstatements?

A. The members of the engagement team should discuss the susceptibility of the
entity's financial statements to material misstatements.
B. The objective of this discussion is for members of the engagement team to gain a
better understanding of the potential for material misstatements of the financial
statements resulting from fraud or error in the specific areas assigned to them, and to
understand how the results of the audit procedures that they perform may affect other
aspects of the audit.
C. The discussion provides an opportunity for more experienced engagement team
members, including the engagement partner, to shore their insights based on their
knowledge of the entity, and for the team members to exchange information about the
business risks.
D. All the team members should have a comprehensive knowledge of all aspects of the
audit.

42. In which of the following situations would materiality be least likely considered
critical?

A. A decision of whether the auditor has to obtain a representation letter from the
management.
B. In determining the nature, timing and extent of audit procedures.
C. A decision of whether to modify the audit opinion.
D. Evaluating the effect of misstatements.

43. Which of the following statements about materiality is incorrect?


A. The assessment of what is material is a matter of professional judgment.
B. When planning the audit. the auditor should consider what would make the financial
statements materially misstated.
C. The assessed level of materiality should not normally be revised by the auditor.
D. Materiality is addressed in the auditor's report.

44. They are EDP control procedures that provide reasonable assurance that all
transactions are authorized and recorded, and are processed completely, accurately
and on a timely basis.

A. General controls
B. Application controls
C. Hardware controls
D. Software controls

45. A collection of files that is shared and used by a number of different users:

A. Database
B. Flat file
C. Master file
D. Transaction file

46. The applications of auditing procedures using the computer as an audit tool refer to

A. Integrated test facility.


B. Data-based management system
C. Auditing through the computer
D. Computer assisted audit techniques

47. It refers to an act of omission or commission by the audit client entity. either
intentional or unintentional, which are contrary to the prevailing laws or regulations
A. Fraud
B. Negligence
C. Noncompliance
D. Defalcation

48. Which of the following elements ultimately determines the specific auditing
procedures that are necessary in the circumstances to afford a reasonable basis for an
opinion?

A. auditor judgment
B. relative risk
C. materiality
D. reasonable assurance

49. "Unusual fluctuations" occur when

A. significant differences are not expected but do exist.


B. significant differences are not expected and do not exist.
C. significant differences are expected but do exist.
D. none of the given choices.

50. Which of the following statements is generally correct about the competence of
evidential matter?

A. The auditor's direct personal knowledge obtained through observation and


inspection, is more persuasive than information obtained indirectly from independent
outside sources.
B. To be competent, evidential matter 'must be either valid or relevant, but need not be
both.
C. Accounting data alone may be considered sufficient competent evidential matter to
issue an unqualified opinion on financial statements.
D. Competence of evidential matter refers to the amount of corroborative evidence to be
obtained.

51. To adequately plan the appropriate audit evidence, generally accepted auditing—
standards require the auditor to gain an understanding of the internal control structure.
This understanding is obtained by:

A. Reviewing organizational charts and procedural manual.


B. Discussions with client personnel.
C. Observing client activities.
D. All of these

52. Which of the following statements concerning analytical procedures is true?


A. Analytical procedures may be omitted entirely for some financial statement audits
B. Analytical procedures used in planning the audit should not use nonfinancial
information
C. Analytical procedures usually are effective and efficient for tests of controls
D. Analytical procedures alone may provide the appropriate level of assurance for some
assertions

53. Each of the following might, by itself, form a valid basis for an auditor to decide to
omit a test except for the

A. Difficulty and expense involved in testing a particular item


B. Assessed level of control risk.
C. Relative risk involved.
D. Relationship between the cost of obtaining evidence and its usefulness.

54. Another entity that executes or records transaction on behalf of a client is called:

A. Third Party provider.


B. Service organization.
C. Outsourcer.
D. Profit center.

55. Testing in the direction from the source documents to the general ledger involves
testing transactions or balances primarily for which type of error?

A. Overstatement
B. Understatement
C. Neither overstatement nor understatement
D. Either overstatement or understatement

56. Management's assertions in the financial statements 'are relevant to the audit
process because:

A. they embody the procedures that will be performed by the audit team
B. they include representations that the financial statements are in accordance with
GAAP
C. they provide evidence that auditors have prepared financial statements in
accordance with GAAP
D. they relate to regulators expectations about audit results

57. When may audit procedures be performed?

A. in the interim period


B. At period end
C. Subsequent to period end
D. At of the choices given
58. Existence, as an assertion, can be audited directionally by considering balances and
transactions from

A. recorded amounts to evidence regarding the source


B. evidence regarding the source to recorded amounts
C. general ledgers to trial balances
D. all of these choices

59. Which of the following audit tests is usually the most costly to perform?

A. Analytical procedures.
B. Tests of balances.
C. Tests of controls.
D. Substantive tests of transactions.

60. Auditors usually try to plan the audit to minimize the use of tests of
A. the other tests are more reliable.
B. the other tests are less costly.
C. the other tests require less experienced audit person
D. any of the choices is true.

61. An auditor concludes that the omission of a substantive procedure considered


necessary at the time of the examination may impair the auditor's present ability to
support the previously expressed opinion. The auditor need not apply the omitted
procedure if
A. the risk of adverse publicity or litigation is low.
B. the results of other procedures that were applied tend to compensate for the
procedure omitted.
C. the auditor's opinion was qualified because of a departure from generally accepted
accounting principles.
D. the results of the subsequent period's tests of controls make the omitted procedure
less important.

62. Which of the following accounts would most likely be reviewed by the auditor to gain
reasonable assurance that additions to the equipment account are not understated?
A. Repairs and maintenance expense
B. Depreciation expense
C. Gain on disposal of equipment
D. Accounts payable

63. Which of the following is a qualitative misstatement?

A. Inadequate allowance for uncollectible accounts.


B. Padded sales.
C. Unrecorded short-term obligations.
D. Failure to disclose loan restrictions in payment of dividends.

64. In relation to opening balances, which of the following may cause the auditor to
disclaim his opinion?

A. The opening balances contain misstatements that could materially affect the current
period‘s financial statements and such misstatements have not been corrected.
B. The current period's accounting policies have not been consistently applied in
relation to opening balances and the effect of such change is not properly accounted for
or disclosed.
C. The inability of the auditor to obtain sufficient appropriate audit evidence concerning
opening balances.
D. The assessed substantial doubt about the entity's ability to continue as a going
concern as-indicated by consistent negative cash flows.

65. Which of the following does not affect the sufficiency and appropriateness of the
audit evidence that the incoming auditor will need to obtain regarding opening
balances?

A. Materiality of the opening balances in relation to the current period's financial


statements.
B. The nature of the accounts and the risk of misstatements in the current period's
financial statements.
C. The number of years the client is in business.
D. Whether the prior period's financial statements were audited.

66. Which of the following is the least concern of the, client auditor in reviewing the
report of service organization auditor on the effectiveness of the internal control design
of the service organization?

A. The system's controls have been placed in Operation.


B. The comprehensiveness of a description of the service organization's accounting
and internal control systems which is ordinarily prepared by the management of the
service organization.
C. The manner of the documentation of the understanding of internal control made by
the service organization's auditor.
D. The accounting and internal control systems are suitably designed to achieve the
stated objectives.

67. Which of the following is least likely entitled to the report of the service organization
auditor on the suitability of internal control design and operating effectiveness of the
service organization?

A. Service organization management


B. Service organization stockholders
C. Service organization‘s customer
D. Client auditors.

68. Upon completion of the audit, the auditor needs to consider uncorrected
misstatements because:

A. The aggregate of uncorrected misstatements, when considered, makes the financial


statements materially misstated,
B. There is a need to revise the financial statements after their issuance.
C. They are basis of whether the auditor needs to redocument internal control. D. The
aggregate of uncorrected misstatements is the basis of the auditor to reassess
materiality level.

69. What should a prudent auditor do when the aggregate of uncorrected misstatements
approaches the materiality level?

A B C D
Perform additional procedures YES NO NO YES
Request management to adjust financial YES YES NO YES
statements for identified misstatements
Request management to adjust financial YES YES YES NO
statements for projected misstatements.

70. They involve analysis of significant ratios and trends including the resultant
investigation of fluctuations and relationships that are inconsistent with other relevant
information or expectation:
A. Inquiry.
B. Analytical procedures.
C. Account analysis.
D. Inspection

71. Which of the following procedures may provide the auditor with information not
previously possessed by him?

A. Inspection.
B. Inquiry.
C. Analytical procedures.
D. Computation.

72. It consists of looking at a process or procedures being performed other persons:

A. Inquiry.
B. Observation.
C. Tracing.
D. Inspection.
73. If the current period's accounting policies have not been consistently applied in
relation to opening balances and if the chance has not been property accounted for or
disclosed, the auditor should issue either a (an)

A. Qualified or disclaimer of opinion.


B. Qualified or adverse opinion.
C. Adverse or disclaimer of opinion.
D. Standard unqualified opinion or unqualified opinion with explanatory paragraph.

74. An audit of the General Lizard Company, a home appliance manufacturer company,
detects material misapplication of the measurement of the lower of cost or market
principle. General Lizard's executive and financial management will not change the
recorded amounts or disclosures to the auditor's satisfaction. Such a situation will most
likely result in which type of report?

A. Unqualified
B. Disclaimer
C. Qualified or adverse
D. Negative assurance

75. A client company has issues that cause substantial doubt regarding the entity's
ability to continue as a going concern. If this is the only major audit issue, which type of
opinion will the auditor usually refrain from issuing?

A. Adverse
B. Unqualified with explanatory language
C. Clean opinion
D. Disclaimer of opinion

76. The auditors of White Stained Sheets, Inc. are unable to obtain evidence regarding
accounts receivable which is a material balance. Instead, the auditors are able .to
satisfy themselves with other alternative procedures relating to the White Stained
Sheets audit. Which report will the auditors most likely issue in this situation?

A. Qualified
B. Unqualified
C. Adverse
D. Disclaimer
77. Ajax Wilson audits Doornail, Inc. without having independence. Such a situation
would lead to the issuance of a(n):

A. unqualified opinion with explanatory language


B. qualified opinion
C. disclaimer of opinion
D. unqualified opinion
78. When management does not amend the financial statements in circumstances
where the auditor believes they need to be amended and the auditor's report has not
been released to the entity, the auditor should issue either a (n):

A. adverse opinion or disclaimer of opinion


B. standard unqualified opinion or qualified opinion
C. Unqualified opinion with explanatory paragraph or a qualified opinion.
D. qualified opinion or an adverse opinion.

79. When financial statements are audited by an accounting firm, the partner-in-charge
of engagement ordinarily signs in the name of the firm because:

A. The practice is impliedly required by the Accountancy Law of 2004.


B. This assures the users of financial statements that the firm assumes the entire
responsibility for the financial statements.
C. The firm assumes responsibility for the audit.
D. The opinion becomes more credible if signed in name of the firm

80. Which of the following is least considered a scope limitation in an audit


engagement?

A. The auditor is unable to carry out an audit procedure believed to be desirable. B. The
timing of auditor's appointment is too late which results to inability of the auditor to
perform prescribed procedures.
C. The audit engagement requires a limited reporting objective.
D. The entity's accounting records are inadequate.

81. Which of the following documentation is required for an audit in accordance with
Philippine standards on auditing?
A. An internal control questionnaire.
B. A client engagement letter.
C. A planning memorandum or Checklist.
D. A client representation letter.

82. Which of the following scenarios regarding a lawsuit filed against a client by a third
party would qualify as a "contingent liability"?

A. A lawsuit has been filed, but not yet resolved.


B. A lawsuit has been filed and has concluded with the client winning.
C. A lawsuit has been filed and has concluded with a third parity winning an award of
P100,000 but client has not paid yet.
D. A lawsuit has been filed and concluded, With the third party winning an award of
P100,000 which the client paid after the balance sheet date but before the statements
are issued.
83. Which of the following factors will least affect the independent auditor's judgment as
to the quantity, type, and content of the working papers desirable for a particular
engagement?
A. Nature of the auditor's report.
B. Nature of the financial statements schedules or other information upon which the
auditor is reporting.
C. Need for supervision and review.
D. Number of personnel assigned to the audit.

84. Before releasing the audit report, the auditor should perform which of the following?
A. Instruct the client to release the financial statements to the bank
B. Perform planning for the subsequent period audit
C. Review the financial statements analytically for any inconsistencies
D. Secure a deposit from the client to safeguard against lawsuits

85. What is the primary purpose of the auditors' request for an attorney's letter relating
to a client?

A. Receive guidance from experts regarding the interpretation of Generally Accepted


Accounting Principles
B. Gather independent information pertaining to Client's legal matters in order to assess
estimates and disclosures
C. Determine if fraud has occurred on the client premises during the period under audit
D. Confirm the existence of inventory and accounts receivable that are held by the
attorney in proxy and on behalf of the client

86. Which one of the following subsequent events will most likely result to an
adjustment to the financial statements?

A. Material change in the amount of settlement, of a lawsuit which had been estimated
at year end
B. Entry into a significant-new line of products and business
C. Proceeds received-from a 'related party note payable
D. Signing of a letter-of-intent by the' client to acquire 55% of another entity for stock
87. Which one of the following is a key condition indicating doubt regarding an entity's
ability to continue as a going-concern?

A. Improvement in key financial ratios


B. Litigation in the normal course of business
C. The company‘s auditors were voted out in the most previous shareholder's meeting
D. Inability to make principal and interest payments as they become due
88. Misstatements that are found during-an audit and aggregated at the conclusion of
the audit for further consideration by the auditor for their impact on the financial
statements typically include:
A. those material items that have been proposed by the auditor for adjustment and
accepted by the client.
B. those of an immaterial magnitude that have been passed by the auditor until the
completion of the audit.
C. those of a material nature that have been ignored by the auditor due to the risk of
sampling error.
D. those of immaterial amounts that were not documented the auditor because they are
of an inconsequential matter to the audit.

89. If, after the audited financial statements have been issued, the auditor becomes
aware that some information included in the statements is materially misleading, he has

A. no obligation to disclose it, assuming he acted in good faith and without negligence
in arriving at the audit opinion.
B. an obligation to inform the board of directors of the misleading statements.
C. an obligation to inform all users who are relying on the financial statements.
D. an obligation to make certain that users who are relying on the financial statements
are informed.

90. According to Philippine Standards on 'Auditing, which of the following combinations


of procedures is appropriately required in performing review engagement?
A. Inquiry and analytical procedures
B. Analytical procedures and documentation.
C. Observation and analytical procedures
D. Inquiry and inspection.

91. Which of the following is an inappropriate specific procedure that the auditor may
perform in completing-a review engagement?

A. Inquiring with management about the extent of related party transactions.


B. Inquiring with client's legal counsel regarding litigations.
C. Comparing the client's gross profit to industry data.
D. Inquiring with management about its plans of restructuring its long-term

92. Which of the following is true of the report based on-an agreed-upon procedures
engagement?

A. The report is restricted to those parties who have agreed to those specific
procedures to be performed.
B. The CPA provides the recipients of the report a tinged assurance reasonableness of
the assertion(s) presented in the financial information.
C. The report states that the auditor has not recognized any basis that requires revision
of financial statements.
D. The report should state that the procedures performed are limited to analytical
procedures and inquiry.

93. Which of the following services is not considered related to audit?


A. Agreed-upon procedure
B. Compilation.
C. Consultancy
D. Review.

94. Which one of the following the most relevant factor ki assessing the control risk of a
computerized environment?

A. Computerized environment provides management with effective replacement


controls
B. Computerized accounting systems enhance efficiency jar users
C. An auditor‘s method of testing the effectiveness of the system controls is the same in
a computerized system as in a manual system
D. The control risk over computerized accounting systems must be assessed during
planning

95. Which application most effectively allows users to state data in on organized
manner and gather information in a usable format?

A. Database management system


B. Operating system
C. Spreadsheet
D. Access control software

96. Which of the following statements is correct with respect to obtaining an


understanding with a client?

A. Auditors are not required to obtain an understanding with their clients.


B. Auditors must obtain an understanding only if an audit is to be conducted.
C. Auditors must document their understanding of the engagement.
D. Auditors must obtain an engagement letter.
97. Which of the following does not strengthen internal control over the electronic
environment?

A. Users are not allowed to make changes to the applications


B. Programmers test new developments prior to release in production
C. Backup of programs and data is performed only by accounting and infrequently
D. Internal auditors test changes to software prior to release in production
98. Which one of the following is often utilized to authenticable the user a computer
system?

A. Username and Password


B. Name and address
C. Data and reports
D. Default and null

99. Auditors review the adequacy of client's documentation accounting information


systems in order to do which of the following?

A. Rewrite the coding used to generate programs to satisfy financial audit assertions
B.Perform analytical procedures associated with the information technology
environment
C. Determine which computer assisted audit techniques are used by the client
D. Understand the use of systems to determine the audit procedures to be performed

100. Which of the following represents a hash total?


A. Encryption
B. Summary of invoice numbers
C. Financial total of payroll
D. Validation of product ID number
SIMULATED EXAMINATION 4

1. An operational audit performed by an internal auditor: is best described as an:

a. audit of a company's compliance with management's policies and procedures


b. evaluation of a company's operations to determine if the company is
susceptible to fraud or other material irregularities.
c. Audit of the operations of a company's computer systems
d. audit of a company's operations to determine The economy and-efficiency
with which resources are employed.

2. In the absence of pronouncements issued by the AASC and the PICPA,


published statements and guidelines by other authoritative bodies, like AICPA,
IAPC, and AFA are the basis of determining generally accepted auditing
standards. What effect do these pronouncements provide in determining the
generally accepted auditing standards?

a. Authoritative
b. Alternative
c. Persuasive
d. Parallel

3. Which of the following statements best describes why the profession of CPAs
has deemed it essential to promulgate a code of ethics and to establish a
mechanism for enforcing observance of the code?

a. Distinguishing mark of a profession is its acceptance of responsibility to the


public.
b. A prerequisite for success is the establishment of an ethical code that
primarily stresses the professional's responsibility to clients and colleagues.
c. A requirement of most laws calls for the profession to establish acode of
ethics.
d. An essential means of self-protection for the profession is the establishment
of flexible ethical standards by the profession

4. A person whose CPA certificate has been revoked

a. can no longer be reinstated as a CPA.


b. Is automatically reinstated as a CPA after two years if he has acted in on
exemplary manner.
c. May be reinstated as a CPA by the Board of Accountancy after two years if
he has acted in an exemplary manner
d. May be reinstated by the PRC after two years if he has acted in an exemplary
manner.

5. The CPA should not undertake an engagement if his fee is to be based upon

a. Rates set by a city ordinance


b. Per diem rates plus expenses
c. A percentage of audited net income
d. The findings of a fax authority

6. The objectives of the Philippine Accountancy Act of 2004 are the following,
except:

a. Standardization and regulation of accounting education


b. Integration of accountancy profession
c. Examination for registration of certified public accountants.
d. Supervision, control and regulation of the practice of accountancy.

7. The level of assurance provided by an audit of detecting a material misstatement


is referred to as:

a. Absolute assurance.
b. High assurance
c. Negative assurance
d. Reasonable assurance.

8. Which of the following most completely describes how independence has been
defined by the CPA profession?

a. Performing an audit from the viewpoint of the public.


b. Avoiding the appearance of significant interests in the affairs of an audit
client.
c. Possessing the ability to act with integrity and objectivity.
d. Accepting responsibility to act professionally and in accordance with a
professional code of ethics.
9. Which of the following is a significant: difference between the Accountancy Law
of 1975 and the Accountancy Law of 2004 with respect to the appointment of
members of the Board of Accountancy?

a. The number of years comprising one appointment term.


b. The number of consecutive terms of appointment of a member.
c. The maximum number of years of membership in the Board of Accountancy
d. Functions of a member of the Board of Accountancy.

10. Which of the following best describes what is meant by the term generally
accepted auditing standards?

a. Pronouncements issued by the Auditing Standards and Practices Council


b. Rules acknowledged by the accounting profession because of their universal
application
c. Procedures to be used to gather evidence to support financial statements.
d. Measures of the quality of the auditor's performance.

11. If requested to perform an audit engagement for a non-public entity in which an


auditor has an immaterial direct financial interest, the he is:

a. Independent because the financial interest is immaterial; therefore, a review


report may be issued.
b. Not independent and, therefore, may not be associated with the financial
statement
c. Not independent and, therefore, may not issue an unqualified audit report
d. Not independent and therefore, may issue a review report but may not issue
an auditor's opinion.

12. The concept of materiality would be least important to an auditor when


considering the

a. adequacy of disclosure of a client's illegal act.


b. discovery of weaknesses in a client's internal control structure.
c. effects of a direct financial interest in the client on the independence.
d. decision whether to use positive or negative con accounts receivable.

13. The Code of Professional Ethics states, in part, that a CPA should maintain
integrity and objectivity. Objectivity refers to the CPA's ability to
a. Determine accounting practices that were consistently applied.
b. Maintain an impartial attitude on all matters which come under review.
c. Determine the materiality of items.
d. Insist on all matters regarding audit procedures.

14. Which of the following engagements allows the CPA practitioner charging
contingent fees?

a. Agreed upon procedures.


b. Compilation.
c. Review.
d. Examination of a forecast.

15. Are the following CPAs required to comply with the Code Professional Conduct?
A BCD
CPAs in Commerce and-Industry. Yes Yes Yes Yes
CPAs in Public Accounting No Yes Yes No CPAs in Education/Acaderne Yes No
Yes No CPAs in Government Yes No Yes No

16. If the firm performs a simultaneous services of auditing the client' financial
statements and bookkeeping services, the CPA ma potentially face a

a. Self-interest threat
b. Intimidation threat
c. Self-review threat
d. Familiarity threat

17. Financial interest is either direct or indirect. Which one of the following relatives
of the auditor who holds financial interest is construed as direct financial interest
of the auditor?

a. Sibling
b. Parent
c. Spouse
d. Non-dependent child

18. The exercise of due core requires that an auditor

a. Use error-free judgement.


b. Study and review internal accounting control, including compliance tests
c. Critically review the work done at every level of supervision.
d. Examine all corroborating evidence available.

19. Karen, CPA, has been retained to audit the financial statements of Redeemer
Company. Redeemer Company's predecessor auditor, Gino, CPA, who has been
notified by Redeemer that Gina's services have been terminated. Under these
circumstances, which party should initiate the communications between Karen
and Gino?

a. Karen, the successor auditor


b. Redeemer's controller or CFO
c. Gina, the predecessor auditor
d. The chairman of Redeemer's board of directors

20. A successor auditor would most likely make specific inquiries of the predecessor
auditor regarding

a. Specializedaccounting principles of the client's industry.


b. The competency of the client's internal audit staff
c. The uncertainty inherent in applying sampling procedures.
d. Disagreements With management as to auditing procedures.

21. Which of the following factorsmost likely would cause an auditor not to accept a
new audit engagement?

a. An inadequate understanding of the entity's internal control structure.


b. The close proximity to the end of the entity's fiscal year.
c. Concluding that the entity's managementprobably lacks integrity.
d. An inability to perform preliminary, analytical assessing control risk.

22. Which of the following is least likely included in an audit engagement letter?

a. The responsibility of the auditor to third party users statements


b. Management responsibility for the financial statements
c. The form of any reports or other communication of the results of the
engagement
d. Arrangement concerning the involvement of other experts in some aspects of
the audit

23. Which of the following least likely influence the auditor's decision to send a
separate engagement letter to a component of parent entity client?
a. Legal requirements
b. Degree of ownership by parent
c. Reporting requirements of the component entity
d. Who appoints the auditor of the component

24. Which of the following is not an acceptable reason for a change of the
engagement from a higher to a lower level of assurance?

a. Cost considerations.
b. Restriction on the scope of the engagement.
c. Misunderstanding as to the nature of the engagement originally requested
d. Audited financial statements are no longer needed because the client was
able to obtain alternative financing.

25. The following are quality control procedures that are observed firm:

I. identifies on a timely basis the staffing requirements of specific


audits
II. Periodically counsels personnel as to their progress and career
opportunities
III.Prepares time budget for an audit engagement to determine
manpower requirements and to schedule audit work.
IV.Evaluates partners periodically by means of senior partner or fellow
partner evaluation and counseling as to whether they continue to
have the qualifications to fulfil their responsibilities.

Which of the foregoing .procedures is (are} necessary to achieve the objectives


of assignment of personnel?

a. I
b. I, III
c. II, IV
d. I, II, III, IV

26. It involves informing assistants at their responsibilities and the objectives of the
procedures they have to perform:

a. Supervision
b. Delegation
c. Direction
d. Review

27. The main purpose of risk assessment procedures is to

a. Obtain an understanding of the entity and its environment, including its


internal control, to assess the risks of material misstatement at the financial
statement and assertion levels.
b. Test the operating effectiveness of controls in preventing, or detecting and
correcting, material misstatements at the assertion level
c. Detect material misstatements at the assertion level.
d. All of the given choices are main purposes of risk assessment

28. Which of the following statements is incorrect regarding obtaining an


understanding of the entity and its environment?

a. Obtaining an understanding of the entity and its environment is an essential


aspect of performing an audit in accordance with PSAs
b. Understanding of the entity and its environment establishes a frame of
reference within which the auditor plans the audit and exercises professional
judgment about assessing risks of material misstatement of the financial
statements and responding to those risks throughout the audit.
c. The auditor's primary consideration is whether the understanding that has
been obtained is sufficient to assess the risks of men misstatement of the
financial statement and to design perform further audit procedures.
d. The depth of the overall understanding that is required by the auditor in
performing the audit is at least equal to that possessed by management in
managing the entity

29. What differentiates fraud from an error?

a. Materiality
b. Intent
c. Effect on financial statements
d. Frequency of occurrence

30. Which statement is incorrect regarding the auditor's consideration of laws and
regulations in an audit of financial statements?
a. When the auditor becomes aware of information concern possible instance of
noncompliance, the auditor should evaluate its possible effect on the financial
statements.
b. If the auditor concludes that the noncompliance act has a material effect on
the financial statements, and has not been properly reflected in the financial
statements, the auditor should express a qualified or an adverse opinion.
c. The auditor may withdraw from the engagement when the entity does not
take the remedial action that the auditor considers it necessary in the
circumstances, even when the noncompliance is not material to the financial
statements.
d. In order to plan the audit, the auditor should obtain a specific understanding
of the legal and regulatory framework applicable to the entity and the industry
and how the entity is complying with that framework.

31. Which of the following procedures would an auditor most likelyperform in


planning a financial statement audit?

a. Inquiring of the client's legal counsel concerning pending, litigation.


b. Comparing the financial statements to anticipated-results
c. Examining computer generated exception reports to verify the effectiveness
of internal controls.
d. Searching for unauthorized transactions that may aid in detecting unrecorded
liabilities.

32. Which statement is incorrect regarding analytical procedures?

a. Analytical procedures may be helpful in identifying the existence of unusual


transactions or events, and amounts, ratios, and trends that might indicate
matters that have financial statement and audit implications.
b. in performing analytical procedures as risk assessment procedures. the
auditor develops expectations about plausible relationships that are
reasonably expected to exist.
c. When comparison of those expectations with recorded amounts or ratios
developed from recorded amounts yields unusual or unexpected
relationships, the auditor considers those results in identifying risks of
material misstatement.
d. When such analytical procedures use data aggregated at a high level (which
is often the situation), the results of those analytical procedures provide a
clear-cut indication about whether a material misstatement may exist

33. Which statement is correct regarding business risks?


a. The risk of material misstatements in the financial statements is broader than
business risk, though it includes the latter.
b. The auditor should identify or assess all business risks.
c. All business risks give rise to risks of material misstatement
d. A business risk may have an immediate consequence for the risk of
misstatement for classes of transactions, account balances and disclosures at
the assertion level or the financial statements as a whole.

34. Inquiries directed towards those charged with governance may most likely

a. Relate to their activities concerning the design and effectiveness of the


entity's internal control and whether management has satisfactorily responded
to any findings from these activities
b. Help the auditor understand the environment in which the financial statements
are prepared.
c. Relate to changes in the entity's marketing strategies, sales trends or
contractual arrangements with its customers.
d. Help the auditor in evaluating the appropriateness of the selection and
application of certain accounting policies

35. Which statement is incorrect regarding significant risks that require special audit
consideration?

a. The auditor should determine which of the identified risks are in the auditor's
judgment, require special audit consideration
b. The auditor excludes the effect of identified controls relatedto the risk to
determine whether the nature of the risk, the likely magnitude of the potential
misstatement including the possibility that the risk may give rise to multiple
misstatements and the likelihood of the risk occurring are such that require
audit consideration.
c. Routine, non-complex transactions that are subject to systematic processing
are more likely to give rise to significant risks they have higher inherent risks
d. Significant risks are often derived from business risks that may result in a
material misstatement.

36. Some account balances, such as those for pensions or leases are the results of
complex calculations. The susceptibility to material misstatements in these types
of accounts is defined as
a. Audit risk
b. Sampling risk
c. Detection risk
d. Inherent risk

37. The risk that an auditor's procedures will lead to the conclusion that material
misstatement does not exist in an account balance when in fact, such
misstatement does exist is

a. Audit risk
b. Control risk
c. inherent risk
d. Detection risk

38. Audit risk consists of inherent risk, control risk; and detection risk. Which of the
following statements is true?

a. Cash is more susceptible to theft than an inventory of coal because it has a


greater inherent risk.
b. The risk that material misstatement will not be prevented or detected on a
timely basis by internal control can be reduced to zero by effective controls
c. Detection risk is a function of the efficiency of an auditing procedure.
d. The existing levels of inherent risk, control risk, and detection risk can be
changed at the discretion of the auditor.

39. The acceptable level of detection risk is inversely related to the

a. Assurance provided by substantive tests.


b. Risk of misapplying auditing procedures.
c. Preliminary judgment about materiality levels.
d. Risk of failing to discover material misstatements.

40. Which of the following audit risk components may be assessed in non-
quantitative terms?

Control Risk Detection Risk Inherent Risk

a. Yes Yes Yes


b. No Yes Yes
c. Yes Yes No
d. Yes No Yes
41. The auditor's considerations of materiality level relate to:
A B C D
Individual account balances YES YES YES NO
Classes of transactions YES YES NO NO
Disclosures YES NO NO YES

42. The assessment of materiality level in relation to specific account balances and
classes of transactions will enable the auditor to
a. Omit certain necessary audit tests.
b. Assess whether the audit opinion will be modified.
c. Select audit procedures that are appropriate based on acceptable detection
risk
d. Primarily determine whether tests of controls would be performed.

43. The diagram below depicts the auditor's estimated maximum deviation rate
compared with the tolerable rate, and also depic7 the true population deviation
rate compared with the tolerable rate, and also depicts the true population
deviation rate compared with the tolerable rate.

True State of Population


Auditor‘s estimate based Deviation rate is less Deviation rate exceeds
on sample results than tolerable rate tolerable rate

Maximum deviation rate I III


is less than tolerable
rate
Maximum deviation rate II IV
exceeds tolerable rate

As a result of tests of controls, the auditor assesses control risk lower than
necessary and thereby decreases substantive testing. This is illustrated by
situation.
a. I
b. III
c. II
d. IV

44. Which of the following statements concerning audit evidence is correct?

a. To be competent, audit evidence should be either persuasive or relevant, but


need not be both
b. The measure of the validity of audit evidence lies in the auditor‘s judgment
c. The difficulty and expense of obtaining audit evidence concerning an account
balance is a valid basis for omitting the test
d. A client's accounting data can be sufficient audit evidence support the
financial statements

45. Which of the following is the best explanation of the difference, if any, between
audit objectives and audit procedures?

a. Audit procedures establish broad general goals, audit objectives specify the
detailed work to be performed
b. Audit objective are tailor-made for each assignment; audit procedures are
generic in application
c. Audit objectives decline specific desired accomplishments; audit procedures
provide the means of achieving audit
d. Audit procedures and audit objectives are essentially the same

46. In gathering audit evidence in the performance of substantive tests, the auditor

a. Should use-the-test-month approach


b. Relies on persuasive rather than conclusive evidence in the majority of cases
c. Would consider the client's documentary evidence more competent than
evidence gathered from observation and physical inspection
d. Would express an adverse opinion if he has substantial doubt as to any
assertion of material significance

47. Which of the following should a prudent auditor do?

a. Determineoverall responses to address risks of material misstatement at the


financial statement level.
b. Design and perform further audit procedures, including tests of the operating
effectiveness of controls when relevant or required, and substantive
procedures, whose nature timing, and extent are responsive to the assessed
risks of material misstatement at the assertion level
c. Evaluate whether the risk assessment remain appropriate and to conclude
whether sufficient appropriate audit evidence has been obtained
d. All of the given choices.

48. The assessment at the risks of material misstatement atthe financial statement
level is affected by the auditor‘s understanding of the control environment.
Weaknesses in the control environment ordinarily will lead the auditor to
a. Have more confidence in internal control and the reliability of audit evidence
generated internally within the entity.
b. Conduct some audit procedures at an interim date rather than at period end.
c. Modify the nature of audit procedures to obtain more persuasive audit
evidence.
d. Decrease the number of locations to be included in the audit scope

49. The most reliable forms of documentary evidence are those documents that are
a. Prenumbered
b. Easily duplicated
c. internally generated
d. Authorized by a responsible official

50. Which of the following auditing procedures is ordinarily performed last?

a. Reading of the minutes of the directors' meetings


b. Confirming accounts payable
c. Obtaining a management representation letter
d. Testing of the purchasing function

51. Which result of an analytical procedure suggests the existence of obsolete


merchandise?

a. decrease in the inventory turnover rate


b. decrease in the ratio of gross profit to sales
c. decrease in the ratio of inventory to accounts payable
d. decrease in the ratio of inventory to accounts receivable

52. In the audit of which of the following general ledger accounts will tests of controls
be particularly appropriate?

a. Equipment
b. Bank charges
c. Bonds payable
d. Sales
53. The auditor should determine overall responses to address the risks of material
misstatement at the financial statement level. Such responses least likely include
a. Emphasizing to the audit team the need to maintain professional skepticism in
gathering and evaluating audit evidence.
b. Assigning more experienced staff or those with special skills or using experts
c. incorporating additional elements of unpredictability in the selection of further
audit procedures to be performed.
d. Performing substantive procedures at an interim date instead of at period.
end.

54. Which statement is incorrect regarding the nature of further audit procedures?

a. The nature of further audit procedures refers to their purpose and their type
b. Certain audit procedures may be more appropriate for some assertions than
others.
c. The higher the auditor's assessment of risk, the less reliable and relevant is
the audit evidence sought by the auditor from substantive procedures
d. The auditor is required to obtain audit evidence about the accuracy and
completeness of information produced by the entity's information system
when that information is used in performing audit procedures

55. Which statement is incorrect regarding the extent of further audit procedures?

a. Extent includes the quantity of a specific audit procedure to be performed.


b. The extent of an audit procedure is determined by the judgment of the auditor
after considering the materiality, the assessed risk, and the degree of
assurance the auditor plans to obtain
c. The auditor ordinarily decreases the extent of audit procedures as the risk of
material misstatement increases
d. Increasing the extent of an audit procedures is effective only if the audit
procedures itself is relevant to the specific risk

56. Which of the following most would give the most assurance concerning the
valuation assertion of accounts receivable?
a. vouching the amounts in the subsidiary ledger to details on shipping
documents.
b. comparing the receivable turnover ratios with industry statistics for
reasonableness
c. inquiring about receivables pledged under loan agreements.
d. Assessing the allowance for uncollectible accounts for reasonableness.
57. In auditing accounts payable, an auditor's procedures most likely will focus
primarily on management's assertion of

a. Existence or occurrence
b. Completeness
c. Presentation and disclosure
d. Valuation or allocation

58. Which of the following procedures is least likely to be performed before the
balance sheet date?

a. Observation of Inventory
b. Search for unrecorded liabilities
c. Testing internal control over cash
d. Confirmation of receivables

59. An auditor is most likely to inspect loan' agreements under which an entity's
inventories are pledged to support management‘s financial statement assertion
of

a. Existence or occurrence
b. Presentation and disclosure
c. Completeness
d. Valuation or allocation

60. The auditor should design and perform further audit procedures whose nature,
timing, and extent are responsive to the assessed risks of material misstatement
at the assertion level. Which of the following is the most important consideration
in responding to the assessed risks?

a. The nature of the audit procedures.


b. The timing of audit procedures
c. The extent of the audit procedures.
d. All of these are equally important.

61. An auditor selected items for test counts while observing a client's physical
inventory. The auditor then traced the test counts to the client's inventory listing.
This procedure most likely obtained evidence concerning
a. Rights and obligations
b. Existence or occurrence
c. Completeness
d. Valuation

62. Which of the following would represent the most rational response by the auditor
to an assessment of higher than average risk of fraud in an upcoming audit
engagement?

a. Assign more experienced auditors to the engagement.


b. Increase the audit fee to compensate for the added risk and potential
insurance cost associated with the high-risk environment.
c. Assign more auditors to the engagement
d. Increase the amount of control testing performed.

63. A test of an asset for overstatement provides corresponding evidence on


expenses, revenues, and liabilities as follows:

a. Expense overstatement, revenue overstatement, and liability understatement


b. Expense understatement, revenue overstatement, and liability overstatement
c. Expense understatement, revenue understatement, and liability
understatement.
d. Expense overstatement, revenue overstatement, and liability overstatement.

64. Which statement is incorrect regarding the timing of further audit procedures?

a. Timing refers to when audit procedures are performed or the period or date to
which the audit evidence applies.
b. The auditor may perform tests of controls or substantive procedures at an
interim date or at period end.
c. If the auditor performs tests of controls or substantive procedure prior to
period end, the auditor considers the additional evidence required for the
remaining period.
d. All audit procedures can be performed prior to period end

65. Dual-purpose tests are audit tests designed to:

a. test more than one control with single procedure.


b. test controls that are relevant to multiple assertions
c. provide evidence for more than a single accounting period
d. test for monetary errors while testing for compliance with controls
66. Confirmation is most likely to be a relevant form of evidence with regard to
assertions about accounts receivable when the auditor has concern about the
receivables

a. Valuation
b. Classification
c. Existence
d. Completeness

67. The auditor most likely performs extensive tests for possible understatement of

a. Revenues
b. Assets.
c. Capital
d. Liabilities

68. The auditor generally makes a decision not to test the effectiveness of controls in
operation when

a. the preliminary assessment of control risk is at the maximum


b. it is more cost efficient to directly test ending account balances than to test
control procedures
c. the auditor believes that control procedures are not functioning as described.
d. all of the given choices are correct.

69. Which of the following is a. correct response of the auditor when he requires a
loweracceptable level of detections risk?

Substantive Testing Procedures


Nature Timing Extent
a. Less effective Year-end More extensive
b. Less effective Interim Less extensive
c. More effective Year-end More extensive
d. More effective Year-end Less extensive

70. An advantage of using statistical sampling techniques is that such techniques

a. mathematically measure risk.


b. eliminate theneed for judgmental decisions
c. eliminate nonsampling risk
d. have been establishedin the courts to be superior to judgmentalsampling
71. The risk of incorrect acceptance and the risk assessing control risk too low relate
to the

a. Preliminary estimates of materially levels.


b. Allowable risk of tolerable-misstatement
c. Efficiency of the audit
d. Effectiveness of the audit.

72. An internal auditor is testing purchase order to detect possible instances of


fraudulent activity by an employee. Believing the occurrence rate of the
fraudulent purchase orders to be quite low (almost zero) the internal auditor
would like to specify the probabilityof observing at least one irregularity if its true
rate of occurrence is greater than expected. The most appropriate sampling
technique for this situation is

a. Sequential sampling
b. Variable estimation Sampling
c. Attribute estimation sampling
d. Discovery sampling

73. In conducting a substantive test of an account balance, an auditor hypothesizes


that no material misstatements exists. The risk that sample results will support
the auditor‘s hypothesis when a material misstatement exist is the risk of

a. Incorrect rejection
b. Incorrect acceptance
c. Alpha error
d. Type 1 error

74. The sample size of a test of controls varies inversely with

Expected population deviation rate Tolerable rate


a. Yes Yes
b. No No
c. Yes No
d. No Yes

75. In performing tests of controls over authorization of cash disbursements, which of


the following statistical sampling methods would be most appropriate?
a. Variables
b. Stratified
c. Ratio
d. Attributes

76. When performing a financial statement audit, auditors are required to explicitly
assess the risk of material misstatement due to:

a. Errors
b. Fraud
c. Illegal acts
d. Business risk.

77. Generally, the auditor prefers external evidence to internal evidence. This is a
measure of

a. Relevance.
b. Appropriateness.
c. Analysis.
d. Evidence gathering.

78. "A transaction or event is recorded at the proper amount and revenue or
expense is allocated at the proper period" is a financial assertion of:

a. Occurrence
b. Completeness.
c. Valuation.
d. Measurement

79. Which of the following is an invalid statement about an audit evidence?

a. Ordinarily, audit evidence regarding one assertion will compensate for failure
to obtain audit evidence regarding another assertion
b. Ordinarily, audit evidence is obtained regarding each financial assertion
c. The nature, timing and extent of substantive tests will vary depending on the
assertions
d. Audit tests can provide audit evidence about more than one assertion
80. With respect to audit objectives, the term validity relates to which of the following
assertions?

a. Existence and occurrence.


b. Completeness
c. Valuation or allocation
d. Presentation and disclosure

81. The rights and obligations assertion applies to:

a. Current liability items.


b. Both balance sheet and income statement accounts.
c. Assets that are not owned by the company.
d. Balance sheet accounts only.

82. Which of the following would not be a factor in determining the competency of
evidential matter?

a. the source of the evidence


b. timeliness of the evidence
c. the degree of objectivity of the evidence
d. the cost of gathering the evidence

83. For the initial audit engagement, the auditor needs not obtain sufficient
appropriate audit evidence that:

a. Accounting procedure are consistently


b. The opening balances do not contain material misstatements.
c. The prior period's real account balances have been brought forward to the
current period.
d. Appropriate accounting policies are consistently observed or changes in
accounting policies have been properly accounted for and adequately
disclosed.

84. The use of the phrase "present fairly, In all material respects" in the opinion
paragraph is most closely associated with which of the following concepts from
the scope - responsibility of the auditor paragraph?

a. significant estimates
b. substantial guarantee
c. positive conclusion
d. basis for an opinion

85. Identify the appropriate type of opinion to issue when the auditor believes that
there is a minimal loss resulting from the resolution of an uncertain

a. Unqualified opinion
b. Unqualified opinion with a separate explanatory paragraph.
c. Qualified opinion or disclaimer of opinion, depending on whether the
uncertainty is adequately disclosed
d. Qualified opinion or disclaimer of opinion, depending upon the materiality of
the loss

86. Which one of the following statements is incorrect?

a. The auditor's report must state whether the financial statements conform with
the Philippine financial reporting standards.
b. The auditor's report must not state whether the applicable Philippine financial
reporting standards were consistently followed from the prior period to the
current period.
c. The auditor's report must state whether the client has provided adequate
disclosure on the finance statements and in the accompanying notes to
financial statements.
d. the auditor's report must express an opinion on the financial statements taken
as a whole, or explain why an opinion cannot be provided

87. Which of the following statements is not appropriately described in the


responsibility of the auditor paragraph of the independent auditor's report?

a. The audit was planned and performed to obtain reasonable assurance about
whether the financial statements are free of material misstatements.
b. The audit was conducted in accordance with generally accepted auditing
standards.
c. The auditor makes the significant estimates in the preparation of the financial
statements.
d. A statement by the auditor that the audit provides a reasonable basis for the
opinion.

88. Which of the following, modification by adding a paragraph to otherwise a


standardaudit report does not constitute an "emphasis of matter"?
a. A paragraph that highlights a material matter regarding a going concern
problem.
b. A paragraph that discloses the significant information that should have been
included in the notes to financial statements.
c. A paragraph that discusses a significant uncertainty.
d. A description of material inconsistency if an amendment to other information
in a document containing audited financial statements is necessary and the
entity refuses to make the amendment.

89. When there are extreme uncertainties that are significant to the financial
statements, the auditor may consider it appropriate to:

a. Withdraw from the engagement


b. Issue an adverse opinion
c. Disclaim an opinion
d. Issue a qualified opinion

90. Which of the following may not potentially result to an issuance of either qualified
or disclaimer of opinion?

a. The timing of auditor's appointment is too late which results to inability of the
auditor‘s to perform prescribed procedures.
b. The entity's accounting records are inadequate.
c. The auditor is unable to carry out an audit procedure believed to be desirable.
d. The audit engagement requires an audit of balance sheet only

91. Where a limitation on the scope of the auditor's work requires modification of an
unqualified opinion, the auditor's report should describe the limitation and:

a. Indicate that the auditor is no longer responsible to his opinion.


b. Indicate the possible adjustments to the financial statements that might have
been determined to be necessary had the limitation not existed.
c. Refer the users to the particular note to financial statements that adequately
discusses the limitation
d. Indicate that the auditor is not satisfied of the results of the alternative
procedures that he had performed

92. The audit report of the incoming auditor least likely include an indication
a. That the financial statements of the prior period were audited by another
CPA.
b. The type of report issued by the predecessor auditor
c. The division of responsibility between the successor and the predecessor
auditor.
d. The date of the predecessor auditor's report.

93. Which of the following is a true statement?

a. The extent of the audit procedures performed on the corresponding figures is


significantly less than scope of the audit for the current figures.
b. When the financial statements of the prior period have been audited by
another auditor, the current auditor must insist that there would be division of
responsibility with respect to audit
c. When the financial statements of the prior period have been audited by
another auditor, the successor auditor must insist that audit of the financial
statements of the prior periods be performed by the successor.
d. When the comparatives are presented as corresponding figures, the auditor
must specifically refer to the predecessor in the introductory paragraph of the
auditor's report.

94. What are the concerns of an auditor when assessing whether comparative
financial statements meet the requirements of the relevant financial reporting
framework?

Concern 1. That prior period figures presented agree with the amounts and
other disclosures presented in the prior period or if necessary, appropriate
adjustments and/or disclosures have been made
Concern 2. That accounting policies of the prior period are consistent with
those of the current period.

a. Yes, Yes
b. Yes, No
c. No, Yes
d. No, No

95. When the comparatives are presented as comparative financial statements:

a. The auditor should issue a report to which the comparatives are referred to
when the comparative's are materially misstated
b. The auditor is not required to identify the comparative in his report because
his opinion applies only to the current year's financial statements.
c. The auditor should issue a report in which the comparatives are specifically
identified because the auditor's opinion is expressed individually on the
financial statements of each period presented.
d. The auditor is only required to specifically identified the comparatives when
his opinion on the prior year's financial statements is other than unqualified.

96. Which of the following actions by an incoming auditor with respect to the financial
statements of prior year which were audited by another CPA is inappropriate?

a. The incoming auditor report only on the current period and the predecessor
auditor to reissue the audit report on the prior period.
b. The incoming auditor modifies the opening paragraph by stating that the prior
period's statements were audited by another auditor, the type of report and
the appropriate reasons for a modification if the report was modified and the
date of the report
c. The incoming auditor should assess whether the
comparativefinancialstatements meet the requirements of the relevant
financial accounting framework.
d. Review the working papers prepared by the predecessor auditor and
appropriately assess whether he can assume responsibility with respect to
the comparatives that are presented as comparative financial statements.

97. Which of the following methods of testing application controls' utilizes a


generalized audit software package prepared by the auditors?

a. Parallel simulation.
b. Integrated testing facility approach.
c. Test data approach.
d. Exception report tests.

98. A hot site is most frequently associated with:

a. Online relational database design.


b. Disaster recovery
c. Source program.
d. Temperature control for computer.
99. Output controls ensue that the results of computer processing are accurate,
complete, and properly distributed. Which of the following is not a typical output
control?
a. Reviewing the computer processing logs to determine that all of the correct
computer jobs executed properly
b. Periodically reconciling outputs reports to make sure that totals, formats, and
critical details ore correct and agree with input.
c. Maintaining formal procedures and documentation specifying authorized
recipients of output reports, checks, or other critical documents
d. Matching input data with information on master files and placing unmatched
items in a suspense file

100. Which of the following procedures would on auditor most likely perform in
planning a financial statement audit?

a. Inquiring of the client's legal counsel concerning pending litigation.


b. Examining computer generated exception reports to verify the effectiveness
of internal control
c. Searching for unauthorized transactions that may aid in detecting unrecorded
liabilities
d. Comparing the financial statements to anticipated results.

SIMULATED EXAMINATION 5

1. The need for assurance services arises because;


a. there is closeness between a user and the organization or trading partner.
b. there is a potential bias in providing information.
c. economic transactions are less complex than they were a decade ago.
d. most financial statement users today have direct knowledge of a company‘s
operations.
2. Assurance services depict;
a. a wider spectrum of services.
b. a more diverse group of users.
c. greater potential users
d. all of the given choices.
3. A review of any part of an organization‘s procedures and methods for the pupose
of evaluating efficiency and effectiveness is classified as a(n
a. audit of financial statements
b. compliance audit
c. operation audit
d. production audit
4. The primary purpose of an independent audit of financial statements is to
a. provide a basis of assessing management‘s performance
b. comply with laws and regulations
c. assure management that the financial statements are unbiased and free from
materials misstatements
d. provide users with as unbiased opinion about the fairness of information
presented in the financial statements
5. which of the following control procedure may prevent the failure to bill customer for
some shipment?
a. each shipment should be supported by pre numbered sales invoice that is
accounted for
b. each sales order should be approved by authorized personnel
c. sales journal entries should be reconciled to daily sales summaries
d. each sales invoice should be supported by a shipping document
6. an objective of an operation audit is to determine whether as entity‘s;
a. internal control is adequate
b. operation are in accordance with laws and regulations
c. financial statements are presented fairly
d. operation are functioning effectively and efficiently

7. Which of the following is a part of the attest process?


a. communicating the conclusion reached
b. providing the accuracy of the book as records
c. compiling the financial information needed for presentation as financial
statements
d. providing guidance in management decisions
8. Auditing services may include which of the following
a. attesting to financial statements
b. examination of the economy and efficiency of governmental operations
c. evaluation of a division performance for management
d. all the responses are correct
9. Which of the following is responsible for an entity financial statements
a. management
b. audit committee
c. internal auditors
d. board of directors
10. the objective of a reviw of financial statements is;
a. to enable the auditor to express an opinion whether the financial statements
are prepared, in all material respects, in accordance with generally accepted
accounting principles in the Philippines
b. for the auditor to carry out procedures of an audit nature to which the auditor
and the entity and any appropriate third parties have agreed and to report on
factual findings
c. for the accountant to use accounting expertise, as opposed of auditing
expertise, to collect, classify and summarize financial information
d. to enable an auditor to state whether, on the basis of procedures which do not
provide all the evidence that would be required in an audit, anything has come to
the auditor‘s attention that causes the auditor to believe that the financial
statements are not prepared in all material respect, in accordance with generally
accepted accounting principles in the Philippine (negative assurance)
11. Which of the following standards requires a critical review of the work done and the
judgment exercised by those assisting in an audit at every level of supervision?
a. proficiency
b. audit risk
c. inspection
d. Due care
12. In conducting a fraud investigation the auditor shoul first;
a. identify the perpetrators.
b. Obtain the facts
c. obtain confession
d Notify a law enforcement agency
13. Which of the following is an element of a CPA firm‘s quality control system that
should be considered in establishing its quality control policies and procedures?
a. complying with laws and regulations
b. using statistical sampling techniques
c. assigning personnel to engagements
d. considering audit risk and materiality
14. The nature and extent of CPA firm‘s quality control policies and procedures depend
on:
a. the CPA firms size=Yes; The Nature of the CPA firms practice=Yes; Cost-
benefit. Consideration=Yes
b the CPA firms size=Yes; The Nature of the CPA firms practice=Yes; Cost-
benefit. Consideration=No
c. the CPA firms size=Yes; The Nature of the CPA firms practice=No; Cost-
benefit Consideration=YES
d. the CPA firms size=No; The Nature of the CPA firms practice=Yes; Cost-
benefit. Consideration=Yes

15. The primary purpose of establishing quality control policies and procedures in
deciding whether to accept a new client is to
a. enable the CPA firm to attest to the reliability of the client
b. satisfy the CPA firms duty to the public
c. minimize the likelihood of association with clients whose management lacks
integrity
d. anticipate before performing any fieldwork whether an unqualified opinion can
be expressed
16. Which of the following will least create a threat to independence?
a. a member of the assurance team, partner or former partner of the firm has
joined the assurance client
b. deposit made by, or brokerage accounts of, a firm or a member of the
assurance team with an assurance client that is a bank broker or similar
institution, provided the deposit or account is held under normal commercial
terms
c. arrangements to combine one or more services or products of the firm with
one or more services or products of the assurance client and to market the
package with reference of both parties
d. family and personal relationship between a member of the assurance team
and director, an officer or certain employees.
17. Before accepting an engagement to audit a new client a CPA is required to obtain
a. an understanding of the prospective clients industry and business
b. the inability to review the predecessor auditor‘s working papers
c. the CPA lack of understanding of the prospective clients operations and
industry
d. the indications that management has not investigated employees in key
positions before hiring them
18. which of the following factors most likey would cause a CPA not to accept a new
audit engagement?
a. the prospective client‘s unwillingness to permit inquiry of its legal council
b. the inability to review the predecessor auditors working papers
c. the CPA lack of understanding of the prospective clients operation industry
d. the indications that management has not investigated employees in key
positions before hiring them
19. Which of the following characteristics would most likely heighten an auditors
concern about the risk of material misstatements in an entity‘s financial statements?
a. the entity industry is experiencing declining customer demand
b. employees who handle cash receipts are not bonded
c. bank reconciliations usually include in- transit deposits.
d. equipment is often sold at a loss before being fully depreciated
20. an account may accept an engagement to apply agreed-upon procedures to
prospective financial statements provided that;
a. distribution of the report is limited to the specified parties involved
b. the prospective financial statements are also examined
c. the responsibility for the adequacy of the procedures performed is taken by the
accountant
d. negative assurance is expressed on the prospective financial statements taken
as a whole
21. An audit obtain knowledge about a new clients business and its industry to
a. make constructive suggestion concerning improvements to the clients internal
control
b. develop an attitude of professional scepticism concerning management
financial statements assertions
c. evaluate whether the aggregation of know misstatements causes the financial
statements taken as a whole to be materially misstated
d. understand the events and transactions that may have an effect on the clients
financial statements
22. Which of the following factor would least influence an auditor‘s consideration of the
reliability of data for purposes of analytical procedures?
a. whether the data were processed in a computer system or in a manual
accounting system
b. whether sources within the entity were independent of those who are
responsible for the amount being audited
c. whether the date were subjected to audit testing in the current or prior year
d. whether the date were obtained from independent sources outside the entity or
from sources within the entity
23. in evaluating the reasonable of an entity accounting estimate an auditors normally
would be concerned about assumptions that are;
a susceptible to bias
b Consistent with prior periods
c insensitive to variations
d similar ro industry guidelines
24. the primary objective of the procedures performed t obtain an understanding of
internal control is to provide an auditor with
a. knowledge necessary for audit planning
b evidential matter to use in assessing inherent risk
c.a basis for modifying test of controls
d. an evaluation of the consistency in the application of management‘s policies
25. Management philosophy and operating style most likely would have a significant
influence on an entity‘s control environment when:
a. the internal auditor reports directly to management
b. management is dominated by one individual
c. accurate management job descriptions delineate specific duties.
d. the audit committee actively oversees the financial reporting process
26 the control environment includes which of the following?
a. control activities
b. management philosophy and operating style
c. assessing activity level risks
d application level controls
27. Which of the following would an audit of a company‘s internal control include?
a. An engagement to perform internal audit procedures.
b. Implementation of key financial controls
c. concluding on the accuracy of the statements of cash flows
d. Testing of managements risk assessment procedures
28. which one of the following is considered an adequate document in an internal
control system?
a. the accounting clerk initials the banks reconciliation
b. the mailroom mails commissions checks
c. invoices are prepares without sequential numbering
d. fixed assets are assigned useful lives at random
29. A material weakness in the design of the operation of controls that is discovered in
an audit of internal controls results in;
a. not submitting a management letter
b. an unfavourable opinion
c. the firing of the auditors
d. adjusting audit journal entries
30.as the acceptable level of detection risk decreases, an auditor may:
a. reduce substantive testing by relying on the assessment of inherent risk and
control risk
b. postpone the planned timing of substantive test from interim dates to the year-
end
c. eliminate the assessed level of inherent risk form consideration as a planning
factor
d. lower the assessed level of control risk form the maximum level to below the
maximum
31. in an audit of financial statements, an auditors primary considerations regarding
internal control is whether the control:
a. reflects management‘s philosophy and operating style
b. affects managements financial statements assertions
c. provides adequate safe guards over access to assets
d. enhances management‘s decision-making processes
32. Assessing control risk at below he maximum level most likely would involve:
a. performing more extensive substantive test with large sample sizes that
originally planned
b. reducing inherent risk for most of the assertions relevant to significant account
balances
c. changing the timing of substantive test by omitting interim-date tsting and
performing the test year-end
d. identifying specific internal controls relevant to specific assertions

33.in planning an audit, the auditors knowledge about the design of relevant internal
controls should be used to:
a. identify the types of potential misstatements that could occur
b. assess the operational efficiency of internal control
c. determine whether controls have been circumvented by collusion
d. document the assessed level of control risk
34. To determine whether the client‘s system of internal control is operating effectively
in minimizing an error of failure to invoice a shipment, the auditor would select of
transactions from the population represented by the
a. customer order file
b. bill of loading file
c. open invoice file
d. sales invoice file
35. Tracing copies of sales invoices to shipping document will provide evidence that all
a. Shipments to customer were recorded as receivables
b. billed sales were shipped
c. debits to subsidiary accounts receivable ledger are for sales shipped
d. shipments to customers were billed
36. Controls over approving credit relate to the:
a. completeness assertion
b. rights and obligations
c. valuation or allocation
d. occurrence
37. The following are the four steps that an auditor undertakes in assessing control risk

A. Determine what control procedures are used by the entity


B. Identify the system's control objectives
C. Design tests-of controls
D. Consider the potential errors or irregularities that could result

In what order would an auditor perform these steps?

A. DBAC
B. BCDA
C. BDAC
D. DCAB

38. Which of the following statements is correct concerning an auditor's assessment of


control risk?
a. Assessing control risk may be performed concurrently during an audit while
obtaining an understanding of the entity's internal control.
b. Evidence about the operation of controls in prior audits may not be considered
during the current year's assessment of control risk.
c. The basis for an auditor‘s conclusion about the assessed level of control risk
should be documented if the control risk is assessed at the maximum level.
d The lower the assessed level of control risk, the less assurance the evidence
must provide that the controls are operating effectively.

39. After assessing control risk at below the maximum level, an auditor desires to seek
a further reduction in the assessed level of control risk. At this time, the auditor would
consider whether:
a. it would be efficient to obtain an understanding of the entity's information
system relevant to financial reporting.
b. the entity's internal controls have been placed in operation.
c. the entity's internal controls pertain to any financial statement assertions.
d. additional evidential matter sufficient to support a further reduction is likely to
be available.

40. Proper authorization of write-offs of uncollectible accounts should be approved in


which of the following departments?
a. Accounts receivable
b. Credit
c. Accounts payable
d. Treasury

41. Which of the following controls most likely could prevent EDP personnel from
modifying programs to bypass programmed controls?
a. Periodic management review of computer utilization reports and systems
documentation
b. Segregation of duties within EDP for computer programming and computer
operations
c.. Participation of user department personnel in designing and approving new
systems
d. Physical security of EDP equipment

42. Misstatements in a batch computer system caused by incorrect programs or data


may not be detected immediately because:

a. errors in some transactions may caused rejection of other transactions in the batch.
b. the identification of errors in input data typically is not part of the program.
c. there are time delays in processing transactions in a batch system.
d. the processing of transactions in a batch system is not uniform.

43. Which of the following input controls is a numeric Value computed to provide
assurance that the original value has not been altered in construction of transmission?
a. Hash total
b. Parity checks
c. Encryption
d. Check digit
44. Which of the following tasks could not be performed by generctli7ed auditsoftware?
a. Choosing a sample of accounts receivable for confirmation
b. Footing the bank reconciliation
c. Summarizing data
d. Reading the minutes of the board of directors' meeting

45. Which of the following tasks may be performed by generalized audit software?
a. Selection of a sample
b. Extraction of information
c. Obtaining file statistics
d. All of the given tasks may be performed

46. It involves the application of audit procedures to less than 100 percent of items
within an account balance or class of transactions.
a. Analytical procedures
b. Substantive testing
c. Audit sampling
d. Tests of controls

47. The likelihood of an account balance or class of transactions to misstatements that


could be material, 'individually or when aggregated with misstatements in other
balances or transaction classes, assuming there were no related internal controls.
a. Audit risk
b. Inherent risk
c. Control risk
d Detection risk

48. If the auditor is concerned that a population may.Cantain exceptions, the


determination of a sample size sufficient to include at least one such critical exception is
a characteristic of:
a. Random sampling
b. Variable sampling
c. Discovery sampling
d. Probability-proportionalelo-size sampling

.49. Uncertainties that still exist even if the auditor examines 100 percent of an account
balance is •
a. Sampling risk
b. Nonsampling risk
c. Inherent risk
d. Control risk

50. As the expected population deviation rate increases (all other factors remaining the
same)
a. The sample size should increase.
b. The sample size should decrease.
c. The sample size should remain constant,
d. The change in the sample size cannot be determined.
51. The risk of incorrect acceptance and the likelihood of assessing control risk too low
relate to the:

A. allowable risk of tolerable misstatement


B. preliminary estimates of materiality levels
C. efficiency of the audit
D. effectiveness of the audit

52. The maximum misstatements that may exist in a population that an auditor is willing
to accept
A. Materiality level
B. Tolerable error
C. Tolerable deviations
D. Likely misstatements

53. Which of the following is not an element of nonsampling risk?

A. The auditor uses inappropriate procedures in auditing accounts receivables. B. The


use of unreasonably small sample size.
C. Misinterpretations of audit evidence.
D. Auditor fails to recognize the error in the sample.
54. While performing test of details during on audit, an auditor determined that the
sample results supported the conclusion that the recorded account balance was
materially misstated. It was, in fact, not materially misstated: This situation illustrates the
risk of:

A. assessing control risk too high


B. assessing control risk too low
C. incorrect rejection
D. incorrect acceptance

55. Which of the following would most likely be an advantage in using classical
variables sampling rather than probability-proportional-to-size (PPS) sampling?

A. An estimate of the standard deviation of the population's recorded amounts is not


required.
B. The auditor rarely needs the assistance of a computer program to design an efficient
sample.
C. inclusion of zero and negative balances generally does not require special design
considerations.
D. Any amount that is individually significant is automatically identified and selected.

56. An auditor may achieve audit objectives to particular assertions by:

A. 100 percent performing analytical procedures.


B. adhering to a system of quality control
C. preparing auditor working papers
D. increasing the level of detection risk

57. Which of the following is not an information source for developing analytical
procedures used in an audit?
A. Relationships among financial statement elements.
B. Relationships between financial and relevant nonfinancial data
C. Comparison of financial data with anticipated results (e.g., budgets and forecasts).
D. Comparison of current year financial data with projections for next year's financial
results.

58. If, when performing analytical procedures, an auditor observes that operating
income has declined significantly between the preceding year and, the current year, the
auditor should next:

A. require that the decline be disclosed in the financial statements.


B. consider the possibility that the financial statements may be materially misstated.
C. inform management that a qualified opinion on the financial statements will be
necessary
D. determine management's responsibility for the decline and discuss The issue with
the audit committee
59. Which of the following is an invalid statement about the relationship of accounting
arid internal control to the persuasiveness of audit evidence?
A. The effectiveness of the client's accounting and internal control has a significant
impact an. the competence of most types of evidence
B. Both physical examination and reperformance are likely to be highly reliable if the
accounting internal controls are effective.
C. The effectiveness of accounting and internal control system is inversely related to the
sufficiency of audit evidence required.
D. The effectiveness of accounting policies and procedures on revenue and cash
receipts significantly affects the extent of substantive procedures on cash
disbursements and expenditures.

60. It refers to the measure of the quantity of audit evidence considered necessary to
achieve the predetermined acceptable level of detection risk:

A. Relevance.
B. Validity.
C. Sufficiency.
D. Appropriateness.
61. The manner of obtaining audit evidence that consists of examining records,
documents or tangible assets:

A. Inspection.
B. inquiry
C. Analytical procedures.
D. Observation.

62. Which of the following least likely affect the persuasiveness of audit evidence that
the auditor obtains?

A. The source of the audit evidence.


B. The consistency of evidence obtained from different sources.
C. The number of audit evidence from a group of sources.
D. The effectiveness of accounting and internal control of the client entity.

63. The following statements relate to audit evidence, except:

A. The competence of audit is likely improved by selecting a larger sample size or


different population items.
B. The evidence obtained from a source outside the client entity is more persuasive
than that one obtained from within.
C. When a client's accounting and internal controls are effective, the internal evidence
obtained is more reliable.
D. The evidence obtained directly by the auditor through physical examination,
computation, observation, or confirmation is more competent than the information that
is obtained indirectly.
64. An objective evidence is more reliable than evidence that requires considerable
judgment to determine whether an assertion is correct. The following are examples of
objective evidence, except:
A. Confirmation replies.
B. Physical count of cash and securities.
C. Replies from client's legal counsel.
D. Adding a list of accounts payable.

65. The auditor gathers evidential matter in order to

A. detect fraud and irregularity


B. evaluate management's stewardship
C. eliminate detection risk
D. form an opinion with respect to financial assertions

66. The decision as to how much evidence to be accumulated for a given set of
circumstances

A. is provided by following the Philippine financial reporting standards.


B. is one requiring professional judgment
C. is determined by statistical analysis
D. is provided in the Philippines Standards on Auditing.

67. Which of the "following is least considered in determining the sufficiency and
appropriateness of the audit evidence that the auditor will obtain regarding opening
balances?
A. The length of years in operations of the entity.
B. The materiality of the opening balances relative to the current period's financial
statements.
C. The accounting policies adopted by the entity.
D. The risk of misstatements of accounts.

68. In determining the sufficiency of evidential matter, which of the following would not
normally be a factor?

A. Cost/benefit considerations.
B. The sampling technique used.
C Materiality of the account balance.
D. Acceptable level of audit risk.

69. The aggregate uncorrected misstatements include:


A. Specific misstatements during the current audit year identified by the auditor. B. Net
effect of uncorrected misstatements identified during the audit of previous year.
C. The auditors best estimate of other misstatements which cannot be specifically
identified.
D. The sum of the three misstatements given.

70. If based on the aggregate of uncorrected misstatements the auditor believes there
may be material misstatements, the auditor should perform additional procedures. If the
client refuses to adjust the financial statements and the auditor is able to conclude that
the aggregate of uncorrected misstatements is material, the auditor should:

A. Issue a standard opinion.


B. Consider resigning from the engagement.
C. Appropriately modify the audit report.
D. Obtain additional representation letter covering uncorrected misstatements.

71. When a client auditor uses a report of the auditor of a service organization, the client
auditor:

A. Should refer the matter in a separate emphasis of matter paragraph of his auditor's
report.
B. Should refer the matter by modifying the scope and opinion paragraphs of the
auditor's report.
C. Should attach the copy of the service organization auditor's report to his audit report.
D. Makes no reference in his auditor's report oh the service organization.

72. Which of the following is incorrect regarding unaudited opening balances?


A. The collection of opening accounts receivable balances during the current period will
provide some audit evidence of their existence.
B. The verification of opening balances for accounts payable and inventories are
equally difficult to do.
C. The verification of opening inventory balance may include analytical procedures.
D. The auditor will ordinarily examine the records underlying the opening balances for
noncurrent assets.

73. Opening balances are based upon the closing balances of the prior period and
reflect the effect of

I. Current transactions (e.g., stock dividends) that will be given retroactive effect
recognition.
II. Transactions of prior periods.
III. Accounting policies applied in the prior periods.
A. All of these
B. I only
C. I and II only
D. II and Ill only

74. The confirmation of customers‘ accounts receivable rarely provides reliable


evidence about the completeness assertion because:

A. many customers merely sign and return the confirmation without verifying their
details.
B. recipients usually respond only if they disagree with the information on the request
C. customers may not be inclined to report understatement errors in their accounts.
D. auditors typically select many accounts with low recorded balances to be confirmed

75. Auditors try to identify predictable relationships when using analytical procedures.
Relationships involving transactions from which of the following accounts most rely or
yield the highest level of evidence?

A. Accounts receivable
B. Interest expense
C. Accounts payable.
D. Travel and entertainment expense

76. To test whether all sates transactions have been recorded, an auditor should test a
sample drawn from an entity's file of:

A. Receiving reports
B. Sales orders
C. Bills of lading
D. Sales invoices

77. An auditor ordinarily sends a standard confirmation request to all banks with which
the client has done business during the year under audit, regardless of the year-end
balance. A purpose of this procedure is to

A. Provide the data necessary to prepare a proof of cash.


B. Request a cutoff bank statement and related checks be sent to the auditor.
C. Detect kiting activities that may otherwise not be discovered.
D. Gather evidence regarding the completeness assertion.

78. The type of test used to check that customer sales are not entered tin some
unreasonable amount is known as a in):

A. output control test


B. batch control test
C. parallel testing
D. edit test
79. The auditor notices significant fluctuations in key elements of flu company's financial
statements. it management is unable to provide an acceptable explanation, the auditor
should

A. consider the matter a scope limitation.


B. perform additional audit procedures to investigate the matter further.
C. intensify the examination with the expectation of management fraud.
D. withdraw from the engagement.

80. The criteria against which the auditor measures the fairness of financial statement
presentation are known as:
A. generally accepted auditing standards.
B. Philippine financial reporting standards.
C. generally used accounting principles.
D. generally accepted governmental accounting principles.

81. A CPA, engaged to examine financial statements: observes that the accounting for
a certain material item is pot in conformity wit Philippine financial reporting standards,
and that ibis fact permanently disclosed in a footnote to the financial statements. The
CPA does not agree with this departure from PERS and should

A. Not allow the accounting treatment for this it-'n to affect the type of opinion because
the deviation from Philippine financial reporting standards was disclosed.
B. Express an unqualified opinion and add an explanatory paragraph emphasizing the
matter by reference to the footnote.
C. Qualify the opinion because of the deviation from Philippine financial reporting
standards.
D. Disclaim an opinion.

82. When the financial statements of the prior period were not audited the incoming
auditor should:

A. not allow the inclusion of the corresponding figures in the financial statements of the
current period.
B. obtain sufficient appropriate audit evidence that the corresponding figures meet the
requirements of the relevant financial reporting framework.'
C. Disclaim his opinion and treat the unaudited corresponding figures us 'oasis ul scope
limitation
D. Insist that an a prior year's financial statements most be made.

83. Which of the following is an incorrect statement about comparatives

A. The auditor may express a qualified, adverse of disclaimer at opinion with respect to
one or more financial statements for one-or more periods, while issuing a different
report oh other financial statements.
B. When reporting on the prior period financial statements in connection with the current
year's audit, if the opinion on such prior period financial statements is different from the
opinion previously expressed, the auditor should disclose the substantive reasons for
the different opinion in the opening paragraph.
C. The auditor may consider expressing an opinion on prior period financial statement
which is different from the opinion that he had previously expressed on such financial
statements.
D. When the prior period financial statements are not audited, the incoming auditor
should carry on appropriate procedures of verifying the opening balances.

84. The following explanatory paragraph accompanies the auditor's report for financial
statements as of and for period ending December 31 2010:
"Because we were appointed auditors of the Company during 2009, we were not able to
observe the counting of the physical inventories at the beginning of 2009 or satisfy
ourselves concerning those inventory quantities by alternative procedures. Since -
opening inventories.... Our audit report on the financial statements for that, year ended
December 31, 2009 was modified accordingly."
The foregoing paragraph is included in connection with a report in which the auditor.
A. Expresses an unmodified opinion regarding the current financial statements but a
modified report regarding comparatives
B. Expresses unmodified opinion regarding the current period figure but a modified
report regarding the corresponding figures.
C. Expresses modified report on both the current financial statement and comparatives.
D. Expresses modified report regarding the current period arid corresponding figures.

85. For what purpose does the following explanatory paragraph in (in audit report that
accompanies the financial statements of Grey Company as of and for the year ended
December 31, 2008 serve?
"As discussed in Note Na. 9 to the financial statements no depreciation has been
provided in the financial statements which practice, in our opinion, is not in accordance
with the Philippine financial reporting standards in the Philippines. This is a result of a
decision taken by management at the start of 2007 and caused us to qualify our audit
opinion on the financial statements relating to the year. Based on the straight-line
depreciation the loss for the year should be increased by P1.2 Million in 2008 and
P800,000 in 20074...,"
A. An explanatory paragraph for a modification of the auditor's report regarding the
current period and the corresponding figures.
B. An explanatory paragraph for a modification of the corresponding figures but not for
modification of current period figures
C. An explanatory paragraph for a modification of the auditor's report regarding both the
current financial statements and prior year's financial statements.
D. An explanatory paragraph regarding a modification of the auditor's report regarding
prior year's financial statements only.
86. The following modification is made on the opening paragraph at the audit report that
accompanies the financial statements of the Gold, Inc.

We have audited the accompanying balance sheet as of December 31. 2008, and the
related ... for the year then ended. These financial statements ... "The financial
statements of the company as of and for the year ended December 31, 2007, were
audited by another auditor whose report dated April 5, 2008 expressed an unqualified
opinion on those statements."
The Modification is made in connection to:
A. Prior period financial statements were audited by other auditor and the incoming
auditor decided to share responsibilities whir
B. Prior period financial statements were audited by another auditor and such financial
statements of prior year are used as comparatives.
C. Reference to the predecessor auditor's report oh the corresponding figures in the
incoming auditor's report for the current period
D. A modified report regarding the current period figures but unmodified report
regarding the corresponding figures.

87. When the prior year's financial statements, which are used as comparatives, were
audited by other auditor, the incoming auditor should modify:

A. The opening paragraph of the audit report


B. The scope - responsibility of the auditor paragraph of the audit report
C. The opinion paragraph of the audit report
D. All the paragraphs of the audit report

88. Which of the following is appropriate when material inconsistency exist in the other
information and the entity refuses to make the amendment?

A. Issue a qualification opinion due to inconsistent data.


B. issue a qualified opinion because material inconsistency may raise doubt about the
audit conclusion drawn from audit evidence.
C. Include an emphasis of matter paragraph describing the material inconsistency.
D. Attach a separate statement that reconciles the inconsistency.

89. They are not presented as complete financial statements capable of standing atone,
but are an integral part of the current period financial statements intended to be read
only in relationship to the current period figures.

A. Corresponding figures
B. Prior period figures
C. Comparative financial statements
D. Comparatives
90. When the auditor was unable to satisfy himself as to the appropriateness of the
unaudited opening balances for both, current and noncurrent assets, the auditor should
express a(n):

Balance Sheet Income Statement


A. Qualified Unqualified
B. Qualified Qualified
C. Unqualified Unqualified
D. Unqualified Unqualified
.
91. If there is an inconsistent application of accounting policies during the current period
in relation to opening balances and the change has not for or disclosed, the auditor
should express a (n):
A. Unqualified opinion with explanatory paragraph.
B. Qualified opinion or adverse opinion.
C. Unqualified opinion with explanatory paragraph or qualified opinion.
D. Qualified or disclaimer of opinion.

92. It exists when other information, not related to matters appearing in the audited
financial statements, is incorrectly stated or presented.

A. Material inconsistency
B. Material misstatement of fact
C. Material weaknesses
D. Misstatement

93. Which of the following is descriptive of related services as provided by the


framework of Philippine Standards on Auditing and other related PSAs?

A. Related services comprise compilation, agreed-upon procedures and management


advisory consultation.
B. Both audits and reviews are intended to provide moderate level of assurance. C. The
auditor who renders an engagement to undertake agreed-upon procedures intends to
issue factual findings.
D. In a consultancy engagement, the accountant is engaged to use accounting
expertise as opposed to auditing expertise to collect, classify and summarize financial
information.

94. According to Philippine Standard on Auditing, the procedures employed in doing


compilation are:

A. Designed to enable the accountant to express a limited assurance.


B. Designed to enable the accountant to express a negative assurance.
C. Not designed to enable the accountant to express any form of assurance.
D. Less extensive than review procedures but mare extensive than agreed-upon
procedures.
95. What type of assurance would you expect to see for an audit on the fairness of
financial statements?
A. No assurance.
B. Limited assurance.
C. Negative assurance.
D. Positive assurance.

96. What type of assurance would you expect to see for a compilation of financial
statement information?
A. No assurance.
B. Limited assurance.
C. Negative assurance.
D. Positive assurance.

97. Which one of the following consists primarily of inquiries of on entity's management
and comparative analyses of financial information?

A. audit.
B: compilation.
C. agreed-upon procedures.
D. review.

98. The internal audit function

A. can be applied to operations, computer systems, and business processes.


B. is not designed to add value to an organization.
C. is not designed to improve an organization's operations.
D. should not be involved with risk management.

99. Which of the following is not a typical internal audit?

A. Risk management and control evaluations


B. Operational audits
C. Performance audits
D. Compliance audits

100. In order for auditors to be able to recognize potential fraud, they must be aware of
the basic characteristics of fraud. Which of the following is a characteristic of fraud?

A. Unintentional deception.
B. Taking unfair or dishonest advantage of uninformed individuals.
C. Lack of training.
D. Negligence on the part of executive management.

Das könnte Ihnen auch gefallen